Sie sind auf Seite 1von 149

ORARIO marted Aula A22 ore [10.30 - 12.30] Mercoled Aula C11 ore [8.30-9.

30] - Gioved Aula


F5 ore [10.30 - 12.30]
27/10/10-Richiamo sui campi: il campo scalare, il concetto di gradiente
28/10/10 Loperatore nabla; il teorema dArchimede, il teorema della divergenza, il flusso di un campo
attraverso una superficie
1/11/10 Def di divergenza, divergenza di campo colombiano, esercizi
??teorema della divergenza, applicazioni
9/11/10 Significato fisico del rotore; rotore di un campo di velocit rigido; angolo solido ed angolo solido
con segno. Il flusso di campo gravitazionale attraverso superficie chiusa tramite langolo solido.
Lequazione: G 4 . Esercizio: la somma dei vettori rappresentanti una superficie chiusa nullo.
Introduzione al teorema di Stokes
10/11/10 Lequazione di Laplace nel caso della temperatura, soluzione dellequazione di Poisson in regione
illimitata.
?? Il teorema di Stokes, nuova definizione del rotore; la carica elettrica
16/11/10 La legge di Coulomb, sistema MKS e CGS
18/11/10 Applicazioni del teorema di Gauss al calcolo dei campi
??? il potenziale
2/12/10 Il dipolo elettrico
14/12/14 coeff. di mutua capacita e mutua influenza, capacit di condensatore piano e sferico; energia di un
condensatore
16/12/10 I limiti dellelettromagnetismo, il problema della stabilit dellatomo, equazione di continuit
FISICA II
Sono arcinote le esperienze che mostrano che una bacchetta di vetro strofinata con una pelle di gatto ( non
ditelo alla societ protettrice degli animali! ) attira piccoli pezzi di carta ecc. ecc., come pure noto il fatto
che le foglioline di un elettroscopio divergono se viene avvicinata una bacchetta di vetro dopo essere stata
strofinata ( elettrizzata). Tali e mille altre esperienze possono essere spiegate se si ammettono i seguenti fatti:
le particelle che costituiscono la materia possiedono una propriet che si chiama: carica elettrica. Tale
propriet pu essere di due tipi: positiva e negativa. Particelle con cariche dello stesso tipo se ravvicinate si
respingono, mentre se sono di tipo diverso, si attirano.
Comparazione delle cariche
Si dir che due particelle hanno cariche uguali se, poste alternativamente nella medesima posizione
rispetto ad un terzo corpo carico, subiscono la medesima forza. Risulta che due particelle che hanno carica
uguale se testate con un corpo C, hanno carica uguale anche se testate con un qualunque corpo C,
quindi quella che si data una buona definizione di uguaglianza di carica, intrinseca alla coppia, in
quanto indipendente da C. Risulta sperimentalmente che fondendo tra loro corpi con carica positiva e
negativa, si possono ottenere corpi che con sono n attratti n respinti da corpi carichi. Si dice quindi che le
cariche positive hanno compensato le negative ed ecco dunque il motivo per cui le cariche vengono detto
positive e negative e non, ad esempio, alte e basse. Si dir che un corpo A ha carica doppia, tripla, di
un secondo B, se posta di fronte ad un terzo corpo carico C, risente di una forza doppia, tripla, di quella
che risentirebbe B e che ha una carica -2 volte, -3 volte quella di B, se risente di una forza doppia, tripla,
di quella di B, ma diretta in senso opposto. Anche ora, come prima, la carica non risulta dipendere dal corpo
di prova C.
Quanto detto sopra ha chiaramente carattere discorsivo; il caso ora di passare ad aspetti quantitativi.
Si fissi in modo preciso ed univoco un modo di caricamento di un dato corpo. (ad esempio: si strofini per un
assegnato periodo di tempo il corpo con una pelle di gatto dalle caratteristiche specificate, esercitando una
forza tal dei tali per un ben determinato tempo, movendo il corpo con velocit assegnata.) Alla fine di tale
caricamento sul corpo vi sar presente una carica che verr detta unitaria. Dalla misura delle forze elettriche
che si esercitano allora su altri corpi, sar allora possibile associare ad ognuno di questi un valore per la
carica elettrica posseduta.
Legge di Coulomb (1785).
1
Consideriamo due corpi, di dimensioni trascurabili (rispetto alla reciproca distanza), quindi considerati
puntiformi e siano fermi in un sistema di riferimento inerziale. Possiedano poi carica q
1
e q
2
. Il primo sia
collocato in un punto individuato dal vettore r
1
ed il secondo dal vettore r
2
Le cariche siano immerse nel
vuoto e siano a distanza enorme dalle altre eventuali cariche. Risulta allora sperimentalmente che sul primo
agisce una forza data da:
3
2 1
2 1
2 1 1
r r
r r
f

q q K
e sul secondo una forza:
f
2
= f
1
Osservazione: siccome corpi puntiformi non esistono, la legge di sopra significa che, tanto pi due corpi
sono piccoli e tanto meglio la loro forza dinterazione quella di sopra
Osservazione: quasi nessun testo riferisce di un lavoro di Gauss del 1835 che corregge la formula di
Coulomb quando le cariche sono in moto con velocit relativa v (ma senza accelerazione!). Anche se tale
formula non verr probabilmente usata in seguito, la riporto per curiosit:
)] cos
2
3
1 ( ) ( 1 [
2 2
3
2 1
2 1
2 1 1
+

c
v
q q K
r r
r r
f
In questa formula c la velocit della luce nel vuoto (circa 300000 km/h) e langolo tra v e
2 1
r r . Il
termine correttivo alla legge di Coulomb dunque proporzionale al rapporto al quadrato della velocit
relativa diviso quella della luce: nelle normali esperienze dunque un termine estremamente piccolo e questo
giustifica il fatto che viene sempre trascurato. Nel caso in cui ci siano presenti pure delle accelerazioni,
lespressione della forza diventa poi molto pi complicata e si presentano gravi problemi per la
determinazione dei moti delle cariche perch, fino ad ora, la prima equazione cardinale diceva: ma = f, dove
f era una funzione nota di r e v, f = f(r,v,t), ma ora, se dipende dallaccelerazione, f = f(r,v,a,t) e quindi: f =
ma = f(r,v,a,t) e le cose si complicano (Vedi comunque Feynmann, vol I, formula 28.3)
La formula. 3
2 1
2 1
2 1 1
r r
r r
f

q q K
esprime in termini matematici il fatto che tale forza diretta lungo la
retta che congiunge i due punti (r
2
- r
1
) infatti un vettore che ha direzione della retta che passa per i due
punti e che, al variare della distanza reciproca, decresce come il quadrato di questa distanza. Che dipenda
poi dal prodotto delle due cariche, deriva dalla definizione che si data sopra di multiplo di una data
carica. La quantit K una costante, nel senso che non varia al variare delle posizioni, al variare della carica,
al variare del tempo nel quale si esegue lesperienza (Se gli antiche Romani avessero eseguito lesperienza,
avendo caricato i punti materiali con il medesimo procedimento seguito da noi ed avessero posto i due punti
alla medesima distanza reciproca alla quale li mettiamo noi, avrebbero trovato la medesima forza che
troviamo noi e questo vuol dire che il K il medesimo e cio non dipende dal tempo). Dipende per
ovviamente dalla definizione di unit di carica. Se infatti assumiamo come unit di carica una che dieci
volte superiore, mentre prima due cariche unitarie a distanza unitaria si respingevano con una forza
numericamente data da K, ora due cariche unitarie ( cio dieci volte pi grosse di prima) si respingono con
una forza che 100 volte la precedente. Se vogliamo scrivere linterazione come: 3
2 1
2 1
2 1 1
~
r r
r r
f

q q K
,
occorrer prendere
K K 100
~

.
Fatte dunque queste considerazioni, risulta dunque che, se si gi definita lunit di carica prima della
determinazione sperimentale della legge di Coulomb, la costante K da determinarsi sperimentalmente.
Viceversa, si pu assegnare un valore arbitrario alla costante K e definire in tal modo lunit di carica. ( si
fissi, ad esempio, la costante K pari a 17,81. Lunit di carica allora sar quella che, posta ad una distanza
unitaria da unaltra uguale, risente di una forza pari a 17,81 newton). Se si segue questa seconda strada, non
si sceglier ovviamente per K il numero a caso 17,81, ma quello che semplifica maggiormente la formula di
sopra, cio il numero 1. In tale sistema una carica sar unitaria se, posta di fronte ad una distanza unitaria
da una uguale, risente di una forza unitaria. In tale modo lunit di carica viene ad essere non ununit
fondamentale, ma dipendente dalle unit di lunghezza, massa e tempo, nel senso che , se noi mutiamo
queste, muta lunit di carica. Per ragioni storiche, chi usa per la meccanica il sistema CGS ( centimetro per
le lunghezze, grammo per la massa e secondo per il tempo), pone K = 1 [ quindi per lui lunit di carica
quella che, posta ad un cm da una uguale, risente di una forza di una dine e prende il nome di Franklin,
2
(simbolo: Fr) o anche statcoulomb (simbolo: statC)], mentre chi usa il sistema MKS adotta una definizione
per lunit di carica indipendente dalle unit di lunghezza, massa e tempo ( quale sia questa unit di carica lo
si vedr in seguito) e la costante K viene allora ad essere soggetta a misura. A questo punto i seguaci del
sistema MKS si dividono in due schiere: c chi preferisce scrivere la costante K come:
0
1

` K
e chi la
scrive invece come:
0
4
1

` K
. Si dice che i primi seguono il sistema MKS, mentre i secondi, il sistema
MKS razionalizzato. Sarebbe quindi pi chiaro scrivere allora:
zato razionaliz zato razionaliz non
K
0 0
4
1 1

`
Noi seguiremo il sistema MKS razionalizzato e quindi dovrei ora dare la definizione di unit di carica, ma
questa si preferisce darla in seguito. Comunque, definita eattamente pi avanti questa unit di carica, la
costante e
0
assoggettabile a misura sperimentale. Tale costante prende il nome di : costante dielettrica del
vuoto. La sua misura migliore fornisce il valore: 8.854187817 10
-12
nel sistema MKS. Si ha quindi: K =
8.987551785 10
9
9 10
9
Alla carica unitaria si d il nome di Coulomb, simbolo C. Una carica unitaria
dunque tale che, posta di fronte ad una uguale, alla distanza di 1 metro, sottoposta ad una forza di (circa) 9
10
9
newton ( una forza enorme! Mai in pratica si potr caricare un corpo di un Coulomb! )
Esercizio: determinare a quanti Franklin corrisponde 1 Coulomb
Ci si pu chiedere ora: la carica che un corpo pu avere una quantit variabile con continuit o varia a
scatti, come, ad esempio il numero di persone in una stanza? La risposta si e no! mi spiego meglio:
lacqua contenuta in un bicchiere variabile con continuit o no? Se assumiamo un punto di vista
macroscopico, la quantit dacqua chiaramente una quantit che pu variare di poco quanto si vuole, ma a
rigore, se si assume un punto di vista microscopico, siccome lacqua composta da molecole, qualunque
quantit dacqua sar espressa da un numero intero di molecole e quindi una quantit che varia in modo
discreto. Lestrema piccolezza della molecola, sia per quanto riguarda il suo peso che il suo volume, fa s che
nella vita quotidiana la quantit dacqua possa considerarsi una grandezza continua. Analogo discorso vale
per la carica elettrica. Ogni carica elettrica positiva (negativa) un multiplo intero della carica del protone
(elettrone). La carica poi del protone , con incredibile precisione, pari al valore assoluto di quella
dellelettrone. Pi precisamente si ha:
20
10

<

protone
elettrone protone
q
q q
.
lestrema piccolezza poi della carica del protone permette, nella fisica macroscopica, di considerare la carica
elettrica di un corpo come variabile continua. La carica del protone risulta essere, secondo le ultime misure:
q
protone
= 1.60217733x10
-19
4.9x10
-26
C. Anche se il Coulomb una carica enorme, tale frazione una
quantit piccolissima
Esercizio: stimare la carica elettrica contenuta in 32 grammi di ossigeno.
Risposta: la carica totale 0 perch, come tutta la materia nello stato normale scarica: non c attrazione
n repulsione sensibile tra due corpi nelle condizioni normali, se non la debolissima attrazione
gravitazionale. Ma questa neutralit dovuta al fatto che ogni porzione di materia comprende una grande
carica positiva ed una uguale negativa. Grande quanto? Calcoliamolo: sar una sorpresa!. Sappiamo che 32
gr. di ossigeno sono una mole, quindi contengono il numero di Avogadro di molecole, cio circa 6 10
23
molecole. Il numero atomico dellossigneo 8, quindi la molecola, che biatomica, contiene 16 protoni. La
carica positiva dunque contenuta in 32 gr di ossigeno sar allora:
1.6 10
-19
x 16 x 6 10
23
= 1.5 10
6
C
Una quantit fantasticamente enorme!
Esercizio: Si sospendono a due fili lunghi 30 cm e fissati ad un chiodo comune, due massettine di circa 0.1 g
luna ugualmente caricate e si vede che i loro centri distano allequilibrio, di 6 cm. Trovare la carica su di
ognuna.
3
Risposta: circa 6.3 10
-9
C. Tale valore desunto da unesperienza realistica; di tale ordini di grandezza
sono le cariche coinvolte nei pi semplici esperimenti di elettrostatica, come sfregamenti di bacchette
Osserviamo per che tale carica anche, se piccolissima, pur sempre costituita da circa6.3 10
-9
/1.6 10
-19
= 4 10
-10
cariche elementari!
Esercizio: Il nucleo del ferro ha un raggio di circa 4 10
-15
m e contiene 26 protoni. Stimare la forza
repulsiva tra due di questi.
Risposta: N 14
10 4
) 10 6 . 1 ( 10 9
15
2 19 9

Questa, essendo il nucleo in equilibrio,deve essere controbilanciata


da forze nucleari, cio forze che diminuiscono rapidissimamente con la distanza perch fuori dal nucleo non
si fanno praticamente sentire. Non ci occuperemo di tali forze.
Legge di Newton e legge di Coulomb
Come si vede, comparando la legge di Coulomb alla legge di Newton sullattrazione gravitazionale, le due
leggi coincidono se noi facciamo la sostituzione:

'


2 2
1 1
q m
q m
K G
La differenza tra la forza gravitazionale e la elettrica ( che pi esattamente dovremmo dire elettrostatica
perch, non dimentichiamolo, le cariche devono essere ferme) che, mentre la forza gravitazionale sempre
attrattiva (la massa m una quantit positiva), quella elettrica pu essere sia attrattiva (quando le cariche
hanno segni opposti) che repulsiva (quando le cariche hanno lo stesso segno), in quanto la carica q pu
essere sia positiva che negativa. La legge di Coulomb cade poi se le cariche sono in moto mentre quella di
Newton susssiste con migliore precisione (anche se lei per non rigorosamente vera per corpi in moto. Non
pu esserlo, per un motivo semplicissimo del quale si fatto cenno nel corso di Fisica 1. Riuscite a
vederlo?). Unaltra differenza importante tra il campo elettrico ed il gravitazionale che per il campo
elettrico vale rigorosamente il principio di sovrapposizione, (affermazione da prendersi con le molle, vedi
tentativi di modifica non lineari di Sommerfel, Bopp) mentre per il campo gravitazionale vera soltanto se
la massa che lo produce non troppo grossa
Se dunque si fissata lunit di carica elettrica, questa viene ad essere una grandezza fisica la cui unit
indipendente dalle unit scelte in meccanica (la carica del proton sempre la stessa anche se noi misuriamo
le lunghezze in km anzich in metri ed il tempo in anni anzich in secondi!) mentre dipende dalle unit scelte
la costante K. Precisamente si ha, indicando con [C] lunit di carica, con [M] lunit di massa e con [T]
lunit di tempo:
[ ] [ ][ ][ ] [ ]

[ ]
2
tan
2 2
C L T L M K
quadrato al za dis
forza
_
Cio:
[ ] [ ][ ] [ ] [ ]
2 2 3
C T L M K
In tal modo il sistema di unit di misura un sistema a quattro unit.
Viceversa, se non si gi fissata lunit di carica elettrica, la legge della forza pu servire per definire lunit
di carica procedendo cos: si fissa un valore arbitrario alla costante K ( e conviene assumerla pure
dimensionale) e la scelta pi semplice ed ovvia di porla uguale ad 1. Lunit di carica allora, per
definizione, quella carica che, posta a distanza unitaria da unaltra uguale (ricordiamoci che stato visto pi
sopra come poter stabilire luguaglianza tra due cariche), risente di una forza pari a K.
chiaro allora che, procedendo cos, lunit di carica dipende dalle unit fissate nella Meccanica. In tale
sistema le unit fondamentali sono soltanto 3 e le dimensioni della carica vengono dunque ad essere:
[ ] [ ] [ ] [ ]
1 2 / 3 2 / 1
T L M C
K poi un numero puro, senza dimensioni, e cio
[ ] [ ] [ ] [ ]
0 0 0
T L M K
Il sistema MKSA
4
Risulta che chi ha scelto di usare nella meccanica il sistema MKS, ha pure scelto nellelettrologia il primo
modo di procedere, ma naturalmente le cose non sono cos liscie. Infatti, anzich definire direttamente
lunit di carica, si preferito, per motivi di precisione delle esperienze, definire prima lunit di corrente
elettrica e tale unit ha ricevuto il nome di Ampre (simbolo: A). Anticipando un po le cose, si pu dire che
si scoperto che due fili elettrici rettilinei indefiniti percorsi da corrente si attirano (o si respingono se le
correnti sono in verso opposto). Lesperienza dimostra che la forza che si esercita sullunit di lunghezza del
filo proporzionale al prodotto delle intensit di corrente ed inversamente proporzionale alla distanza tra i
due fili. Daltra parte lintensit di corrente altro non che la carica elettrica che passa nellunit di tempo
attraverso la sezione del filo. Si dunque proceduto cos per determinare lunit di corrente:
diremo, per definizione di Ampre, che due fili rettilinei indefiniti a distanza reciproca di un metro sono
percorsi entrambi dalla medesima corrente di 1 Ampre (simbolo: A) se sulla loro unit di lunghezza agisce
una forza di 210
-7
newton ( il valore 210
-7
stato scelto arbitrariamente)
A questo punto si definisce lunit di carica elettrica, unit che prende il nome di coulomb (simbolo: C) cosi:
il coulomb la carica trasportata in un secondo attraverso una sezione di un filo elettrico percorso dalla
corrente di 1 ampre
Stando cos le cose, si pu eseguire questa esperienza ideale: si pongano due coulomb puntiformi a distanza
reciproca unitaria ( 1 metro) e se ne misuri la forza elettrica. Tale valore sar quello della costante K
Lesperienza mostra allora che il valore della costante K :
2 2 3 9
10 987551795 . 8

s C m Kg K
( intuitivamente chiaro che una tale misura diretta non pu, per forza di cose, fornire un risultato con cos
tante cifre decimali! Se non altro per il fatto che dicendo puntiformi si fa unapprossimazione. Valori cos
precisi sono stati ottenuti con altri sperimenti, ma concettualmente questa la strada da seguire)
Una cosa importante: Le equazioni di Maxwell, che sono un set di equazioni che contengono tutto
lelettromagnetismo, ivi compresa la legge di Coulomb, e che vedremo in seguito, permettono di affermare,
su base teorica, che vale limportante uguaglianza sorprendente:
2 7
0
10
4
1
c


dove con c si indica la velocit della luce nel vuoto.
La costante
0
usata da noi, come dai non razionalizzati, si chiama costante dielettrica del vuoto ed il
miglior valore, in unit MKSA, :
12
0
10 854187817 . 8


(I pi attenti di voi potranno chiedersi: come mai non stato fornito lerrore con cui noto il valore di una
tale grandezza? La risposta che tale valore esatto, dato che la velocit della luce nel vuoto ha un valore
esatto. Vedi a tal proposito infatti la definizione di metro data pi sopra. Inoltre la formula di sopra ritenuta
esatta, cio non si ammette che sia, ad esempio:

2 7
0
10
4
1
c
se non ammettendo = 0!).
Osservazione 1 Coulomb, posto di fronte ad 1 Coulomb ad una distanza di 1 metro viene respinto con una
forza di circa 9x10
9
N, cio da una forza enorme! Il Coulomb dunque una unit enorme! Se voi strofinate
una bacchetta di vetro, quale dunque lordine di grandezza della carica che viene a depositarsi su di lei?
La carica positiva pi piccola, o carica elementare, che si trovata in natura la carica del protone. Le
misure pi precise forniscono per lei:
q
protone
= 1.602176461x10
-19
C con un errore quadratico medio di 3.9X10
-8
La carica negativa pi piccola che si trovata in natura la carica dellelettrone. Questa risulta, con
sorprendente precisione, in valore assoluto pari a quella del protone. Per lesattezza (q
elettrone
negativa!):
20
10

<
+
protone
elettrone protone
q
q q
.
risultato poi che ogni carica un multiplo intero della carica del protone (o dellelettrone). A causa
per della piccolezza della carica elementare, in molti casi si potr considerare la carica come una grandezza
che varia con continuit.
noto poi con grande certezza un fatto fondamentale, che va sotto il nome di
Principio di conservazione della carica:
la somma algebrica delle cariche contenuta in un sistema chiuso (cio che non abbia scambi di materia
con il resto del mondo) si mantiene costante nel tempo, checch avvenga al suo interno
5
Esercizio: La massa dellelettrone risulta essere di circa 9.109x10
-31
Kg. Qual il rapporto tra la forza
elettrica e la forza gravitazionale tra due elettroni? E tra due protoni? ( massa protone ~ 1840 volte superiore
a quella dellelettrone).
Risposta: Tra due elettroni tale rapporto vale circa:4.16810
42
. Il quadrato di tale numero, cio circa 1.710
85
corrisponde al numero stimato di particelle delluniverso. Come mai? Credo che nessuno lo sappia!
La grandezza di tale rapporto spiega perch spessissimo, nei problemi di elettrostatica, le forze
gravitazionali, che pure dipendono anche loro dal quadrato della distanza, si trascurino
Il sistema CGS
Risulta che chi ha scelto di usare nella meccanica il sistema CGS, ha pure scelto nellelettrologia il secondo
modo di procedere ed ha posto K = 1. In tale sistema lunit di carica quella che, posta ad una distanza
unitaria (= 1 cm) da una uguale, risente di una forza unitaria (1 dine). Tale unit detta Franklin.
Esercizio A quanti Franklin corrisponde 1 Coulomb?
Risposta:
1C =
9
10 98755 . 8
Franklin
Ulteriori osservazioni sulla legge di Coulomb
Per quanto riguarda lesponente della legge di Coulomb, scrittala come:
+

3
2 1
2 1
2 1 1
r r
r r
f q q K
si potuto stabilire nel 1971 che :
16
10 7 . 2

<
Quindi, a parole, la forza decresce proprio con il quadrato della distanza!
Ci si pu chiedere ora quali siano i limiti di validit della legge di Coulomb, dato che evidentemente non
pu valere se le due cariche sono coincidenti. Si scoperto che vale per distanze tra le cariche d comprese
tra:
< < <

d cm
13
10
migliaia di chilometri
Al di l di questi limiti non si sa se vale o no.
Esercizio Uno dei primi modelli dellatomo latomo di Bohr (anni intorno al 1910). Secondo Bohr
latomo didrogeno costituito da un protone, il nucleo che si pu considerare puntiforme (sappiamo che i
nuclei sono dellordine di 10
-13
- 10
-12
cm) e fermo ( dato che la sua massa molto maggiore di quella
dellelettrone) e da un elettrone che gli gira intorno su di un orbita circolare ed trattenuto da una forza
che, anche se lelettrone in moto, riteniamo espressa dalla formula di Coulomb. Sapendo da misure
sperimentali che le dimensioni di tale atomo sono di circa 0.5x10
-8
cm, determinare 1) la forza di attrazione
tra protone ed elettrone. 2) la velocit con cui lelettrone si muove nella sua orbita.
Risposta:
1) per quanto riguarda la forza, si ha subito:
N f
8 2 11 2 19 9
10 2 . 9 ) 10 5 /( ) 10 6 . 1 ( 10 9


Questa forza sembra piccolissima, ma in realt, rispetto ad esempio alla forza peso che agisce su di un
elettrone, enorme! La forza peso infatti soltanto: (9.109x10
-31
x 9.81) ~ 9x10
-30
N. Fare quindi attenzione
in Fisica a non dire che un qualcosa grande o piccolo se non rispetto ad un qualcosa daltro! Molte volte
siamo portati inconsciamente a dire che qualcosa grande o piccolo se grande o piccolo rispetto a ci che
cade direttamente sotto i nostri sensi, ma non detto che ci che grande o piccolo rispetto a noi sia pure
grande o piccolo rispetto al problema che si sta trattando!. La terra grande rispetto a noi, ma se
consideriamo il suo moto sullorbita, essendo questa di molti milioni di km, si pu considerare puntiforme.
2) percorrendo lelettrone unorbita circolare di raggio r con velocit di modulo v, sottoposto ad
unaccelerazione centripeta data da v
2
/r e quindi ad una forza centripeta pari a m v
2
/r. Tale forza dovuta al
protone che esercita una forza data dalla legge di Coulomb. Dovr quindi aversi:
r m
q v
r
m
r
q
0
2
2
2
0
4
1 v
4
1


Sostituendo i dati, si ottiene:
6
v ~ 2.2*10
6
m/s ~ c/137
Veniamo ora ad esaminare unaltra propriet molto importante del campo elettrostatico:
Principio di additivit delle forze elettrostatiche e campo elettrostatico
Supponiamo ora che le cariche presenti siano ora tre, non pi due: q
1
posta in r
1
, q
2
posta in r
2
, e q posta in r.
Quanto varr la forza sulla carica q? La risposta non si pu dedurre dal puro ragionamento, occorre ricorrere
allesperienza e questa dice che, con ottima approssimazione, la forza che agisce su q la somma vettoriale
delle forze che si avrebbero se questa fosse in presenza della sola 1 e della sola 3. In formule:
1
1
]
1

3
2
2
2
3
1
1
1
r r
r r
r r
r r
f q q q K
Generalizzando poi al caso in cui una carica q in presenza di n cariche q
1
, q
2
,q
n
, la forza su di lei si
scrive:

n
i
i
i
i
q qK
1
3
r r
r r
f
Forza elettrostatica su una carica q posta in r prodotta da n cariche in r
i
di valore q
i
Nel caso in cui le cariche che determinano la forza su q possano essere considerate come un continuo , la
forza agente si potr scrivere come:

'
'
'
3
dq q K
r r
r r
f
(attenzione ai simboli! r il punto dove si trova la carica q, mentre r il generico punto dove si trova la
generica carichetta dq)
od anche, introducendo lo scalare (r), funzione del punto e detto densit di carica e definito come:
dV
dq
dV 0
lim

`
si scriver(le doppie sbarre sono sostituite dalla singola):

spazio lo tutto
dV q K ) ' (
'
'
3
r
r r
r r
f
Con questultima formula (o con quella con cariche discrete) sorge un fatto nuovo rispetto alla legge di
Coulomb tra due cariche. Nel caso di due, si era detto che valeva nel vuoto, mentre quella di sopra vale
anche nella materia. Come mai? Il fatto che la materia altro non che cariche e quindi, se nella

n
i i
i
i
q qK
1
r r
r r
f
noi sommiamo su tutti i protoni ed elettroni presenti, possiamo a buon diritto dire di
essere nel vuoto e quindi la formula esatta. Si potrebbe fare a ci unobiezione: la legge di Coulomb vale se
le cariche sono ferme, ma nella materia gli atomi sono sottoposti ad un moto di agitazione termica e poi gli
elettroni, nel loro moto intorno al nucleo non possono essere certo considerati fermi muovendosi con una
velocit dellordine del centesimo di quella della luce! Come stanno dunque le cose? Senza addentrarci in
difficili dimostrazioni, deve essere chiaro che i moti macroscopici, che mediamente sono nulli, non alterano
lespressione della forza. Se infatti non fosse cos, come avrebbe potuto Coulomb trovare la sua formula? I
dispositivi che ha usato erano anche loro fatti di atomi e soggetti allagitazione termica!
Il rapporto tra la forza elettrostatica e la carica su cui tale forza agisce, , per definizione, il campo
elettrostatico nel punto dove posta la carica q, campo che si indica con E
elettrostatico
(r). si ha cio nel caso
discreto:


n
i i
i
i tico elettrosta
q K q
1
/ ) (
r r
r r
f r E `
mentre nel caso continuo sar (d
3
r altro non che un nome per indicare un volumetto infinitesimo centrato
intorno al punto individuato dal vettore r) :

spazio lo tutto
tico elettrosta
d K dq K ' ) ' (
'
'
'
'
) (
3
3 3
r r
r r
r r
r r
r r
r E
7
Formula fondamentale che permette il calcolo del campo elettrostatico in un punto generico r dalla
conoscenza della densit di carica.
Quindi si ha che la forza elettrostatica agente su di una carica q posta nel punto individuato dal vettore r
data da:
f = q E
elettrostatico
(r)
(Il pi delle volte il pedante pedice elettrostatico si omette)
Esercizio: Un anello circolare di raggio R porta una carica elettrica Q uniformemente ripartita. Determinare
lintensit del campo elettrico in un punto P dellasse z a quota D.
Risposta: Per motivi di simmetria il campo E avr solo componente z( per assurdo: se avesse una
componente perpendicolare a z, ruotando lanello di 180 intorno al suo asse, tale componente cambierebbe
di segno. Ma la distribuzione di carica presente dopo questa rotazione esattamente quella di prima ed il
campo E non deve quindi essere mutato. La componente in questione dunque dovrebbe contemporaneamente
cambiare di segno e non mutare. Deve quindi essere nulla!). Detto questo, un elementino di lunghezza dl
dellanello porta una carica dq = dl e dista:
2 2
D R +
dal punto P, sicch lintensit del campo E da lui
generato in P data da:
dl
D R
E
2 2
0
4
1
+



. Per quanto detto sopra, lunica componente del campo
quella lungo z che vale:
2 2
2 2
0
4
1
D R
D
dl
D R
E
z
+
+



. Sommare i contributi di tutti i tratini, vuol dire
raccogliere a fattore la quantit:
2 2
2 2
0
4
1
D R
D
D R
+
+


e sommare su tutti i dl ottenendo cor la
lunghezza della circonferenza, cio 2R. Ottengo allora:
3 2 2
0
2 2
2 2
0 ) (
4 4
2
D R
D Q
D R
D
D R
R
E
z
+

+
+

Esercizio: Un disco di raggio R porta uniformemente distribuita una carica Q. Determinare il campo elettrico
in un punto P dellasse distante D dal disco.
Risposta: il disco pu essere visto come costituito da tantissime corone circolari concentriche, di spessore
infinitesimo dr e raggio generico r . Ma ognuna di queste corone pu essere considerata un anello carico.
Dato che la sua superficie di questo anello : 2rdr, la carica infinitesima dQ che porta, : dQ = 2 rdr,
dove la densit superficiale di carica, cio Q/(R
2
). Il campo generato da questo anello riportato
nellesercizio di sopra e quindi il campo di tutto il disco sar:
) 1 (
2
) 1 (
2
) (
2
) (
4
2
2 2
0
2
2 2
0
2
2
0
3 2 2
0 0
3 2 2
0 D R
D
R
Q
D R
D
R
R
dr
D r
r D
dr
D r
D r
E
R R
z
+

+

+



Osserviamo che per D molto grande, si ha[ osservando che per piccino . (1+)
n
1+n]:
2
2
2
2
2
2
2 2
2
1
]
2
1
1 [ 1
) (
2
1
1
1
1
) ( 1
1 1
D
R
D
R
D
R
D
R
D
D
D R
D

+

+

+

e cio il campo decresce,


come deve essere, con il quadrato della distanza dal disco.
Se ora, nella formula del campo, si tiene costante e si fa crescere R allinfinito, si ottiene il campo generato
da uno strato piano infinito di carica. Si ha dunque:
0
2 2
0
2
) 1 (
2
lim

+



D R
D
E
R
strato
si ottiene quindi il fatto, forse un po sconcertante, che il campo elettrico generato da uno starto omogeneo
piano ed infinito non dipenda dalla distanza dallo strato.
Chi trovasse un po difficile il calcolo dellintegrale sopra riportato, pu evitarlo ragionando cos:
8
Con riferimento alla figura, la componente di E perpendicolare al disco dovuta allelementino di superficie A
a distanza r da P dato da:
) cos( ) / (
4
1
2
0


r A E
. Come risulta chiaro dalla figura, :
A A ) cos( , sicch ho:
) / (
4
) / (
4
2
1
0
2
0
D A r A dE


Il campo totale sar la somma di tutti gli A
1
che sono in corrispondenza con i punti del disco, cio quelli
compresi tra:
-arctan(R/D) < < arctan(R/D)
Ma questa somma fornisce larea della calotta sferica , che ben nota dalle scuole: una calotta sferica
equivalente alla superficie laterale del cilindro di uguale altezza, che ha per raggio quello della
corrispondente sfera. Quindi, nel nostro caso, questarea : 2Rh, dove h laltezza della calotta, altezza che
:
] 1 [ )] cos( 1 [
2 2
D R
D
D D
Max
+

. E quindi:
] 1 [
2
] 1 [ 2
4 4
) / (
4
2 2
0
2 2
2
2
0
1 2
0
2
0 D R
D
D R
D
D
D
A
D
r A dE E
+

+


Esercizio: sia dato un segmento di lunghezza L uniformemente carico con densit . (Lo si pensi adagiato
lungo lasse x con il suo centro nellorigine). Trovare la direzione del campo elettrico generato da lui in un
punto P di coordinate x,y.
Per risolvere il problema basterebbe applicare la formula fondamentale di sopra, tenendo presente che,
siccome la carica si suppone unidimensionale, occorrer sostituire il dq con dx, ma il calcolo dellintegrale
pu essere difficile e comunque il procedimento sarebbe pesante ed opaco. Meglio ragionare nel modo
seguente:
Tenendo presente la figura di sotto, il campo elettrico infinitesimo dE dovuto al trattino infinitesimo dx (che
va da C ad A) ha direzione ovvia di figura ed intensit data da:
2
0
4
1
r
dx
dE

. Dalla figura risulta:


9
CA cos() = AB e cio dx = CA = AB/ cos(). Ma dalla similitudine dei triangolo ABP e ABP risulta :
AB/r = AB/R. Quindi
R
B A
R R
B A
r r
AB
r
AB
r r
dx
dE
figura vedi
' '
4
1 ' '
) cos( 4
1
) cos( 4
1
) cos( 4
1
4
1
0 0 0
2
0
2
0



In altri termini: il campo elettrico infinitesimo dovuto al trattino dx lo stesso di quello dovuto dallarchetto
infinitesimo AB (supposto dotato di medesima densit ). Il campo elettrico totale sar la somma vettoriale
di tutti i vetturini dovuti a tutti i trattini infinitesimi che compongono il nostro segmento o, il ch lo stesso
per quanto visto sopra, al campo elettrico dovuto allarco compreso tra i segmenti tratteggiati. Ma allora la
direzione del campo subito trovata: per motivi di simmetria sar sulla bisettrice dellangolo sotto il quale
visto da P il nostro segmento.
Il campo elettrico
Supponiamo ora di porre in un punto r una carica q, ferma e di osservare sperimentalmente che sottoposta
ad una forza che tanto maggiore quanto maggiore la sua carica. Diremo quindi che sottoposta ad una
forza elettrica (non dir pi per elettrostatica perch ora non so se questa forza dovuta a cariche
elettriche ferme, in moto o ad altro!). Ebbene, il rapporto tra questa forza e la carica medesima , per
definizione, il campo elettrico che vige nel punto dove posta la carica, in quel determinato istante. Ho
cio:
q t t / ) ( ) , ( f r E `
(sarebbe opportuna una precisazione: lintroduzione della carica q per testare il campo pu, esercitando
delle forze sulle altre che generano il campo, disturbare questo: un problema generale relativo alla misura
di un grandezza in Fisica: lo strumento di misura deve essere scelto da perturbare il meno possibile la
grandezza che si vuol misurare! Nel caso in questione, lattenuazione del disturbo si ha prendendo piccola la
carica q o meglio, con un processo ideale, si definir il campo elettrico come:
q t t
q
/ ) ( lim ) , (
0
f r E

`
Il concetto di flusso di un campo
Si considerino due vettori infinitesimi con la coda in comune, dr
1
e dr
2
Essi individuano un
parallelogrammo la cui superficie :
2 1
r r d d dS
La relazione scritta suggerisce che opportuno introdurre un vettore che rappresenta la nostra superficie,
precisamente il vettore:
2 1
r r S d d d
Notate che se si scambiano i due vettori, il prodotto vettore cambia di verso. [Una persona che si muove
lungo il bordo della superficie spostandosi secondo il vettore dr
1
se vede larea alla sua sinistra allora
disposta come il vettore dS (cio i suoi piedi sono la coda e la sua testa la punta del vettore) ] Questo fatto
suggerisce allora di distinguere, cosa del resto ovvia, sulla superficie due facce, convenzionalmente indicate
con - e con + ed il vettore dS diretto dalla faccia alla +. La superficietta quindi orientata.
Consideriamo ora una superficietta immersa in un campo vettoriale A(r). Nel punto dove disposta, il suo
vettore dS formi un angolo con il vettore del campo A. Si definisce:
10
flusso elementare del campo attraverso la superficie dS lo scalare:
d = A dS
Notate che il segno di tale flusso positivo se il campo attraversa la superficie passando dalla faccia negativa
alla positiva, negativa se avviene il contrario. Il flusso nullo se il campo lambisce la superficie senza
attraversarla (dS ortogonale ad A)
Si consideri ora una superficie finita S immersa nel campo e si tracci su di essa una rete a maglie finissime,
a forma di parallelogramma. Ad ognuno di questi parallelogrammini associato un vettorino come risulta dal
disegno sottostante:
(Mi rendo conto che poco visibile! Aggiustatelo voi!)
Ebbene, il flusso del campo attraverso la superficie sar definito come la somma algebrica dei flussi
attraverso i vari parallelogrammini che la compongono e, se questi diventano infiniti, si avr:


S
S
dS r A A ) ( )) ( `
La parola flusso per tale prodotto scalare suggerita dalla seguente considerazione: Il campo A sia il
campo di velocit di un fluido. Immergiamo nel fluido una superficie tanto piccola che il campo A si possa
ritenere costante in ogni punto della superficie. Chiediamoci: qual il volume di fluido che passa attraverso
la superficie in questione nellunit di tempo? molto facile rendersi conto che tale volume proprio dato
da AdS . Se ne deduce quindi che il nome flusso scelto appropriatamente
Esercizio: Si consideri il seguente campo E = r/r
3
e se ne calcoli il flusso attraverso un rettangolo di vertici
A = (X,Y,Z); B = (X+a,Y,Z); C = (X+a,Y+b,Z); D = (X,Y+b,Z) orientato in modo che il suo vettore
rappresentativo sia diretto lungo z
Risposta considero un elementino di superficie dS = dx dy k. Ora : EdS =
( )
dxdy
z y x
z
3
2 2 2
+ +
E quindi subito:
( ) ( )

+ +
+ +

+ +

a X
X
b Y
Y
dx
z y x
z
dy dxdy
z y x
z
3
2 2 2
3
2 2 2
Ora risulta:
( )
2 2 2
2 2 3
2 2 2
1
z y x
z y
zx
dx
z y x
z
+ +
+

+ +
;

+ +
+ +

+ +
+
]
) (
2 arctan[
2
1 1
2 2 2 2 2 2
2 2 2
2 2 2
2 2
z y x z y x
z y x xyz
dy
z y x
z y
zx
Il teorema della divergenza.
Legge di Archimede: un corpo immerso in un fluido riceve dal basso verso lalto una forza opposta al peso
del fluido spostato.(Am. J. Phys. 1991, pag 761, dove usa il principio dei lavori virtuali) In formule, la
spinta dellacqua esterna al volume si scrive come:

S
pdS
Lopposto del peso del fluido spostato si scrive come:

V
dV g
Archimede dice:
11


V S
dV pd g S
Ma essendo
g p
, si ha:


V S
dV p d p S
Relazione che deve valere per ogni funzione p(x,y,z)
Esercizio: Consideriamo una superficie chiusa S. orientiamola cho la sua normale diretta verso lesterno.
Dimostrare che :
0

S
dS
Soluzione: basta porre nella relazione

V S
dV p d p S
, p = 1
Esercizio: Consideriamo un fluido immeso in un assegnato campo di forza h: la massa dm contenuta nel
volume dV sottoposta alla forza del campo data da: df = dV h. Pu questo fluido essere e restare fermo
sotto tale campo?
Risposta: la condizione di equilibrio , lo si visto sopra, che la pressione sia legata al campo di forza ed alla
densit dalla relazione: h p . Se ne deduce che perch ci sia la possibilit di avere equilibrio, la h
deve essere un gradiente, ovverosia che h sia una campo conservativo, e quindi che il suo rotore sia nullo. E
quindi: ) ( ) ( 0 h h h + . Moltiplicando scalarmene per h, si ha che per la possibilit di
equilibrio deve essere:
0 ) ( h h
(condizione sul campo per poter avere lequilibrio di un fluido immerso nel campo). Osservazione: si
potrebbe essere indotti a pensare che la relazione di sopra sia soddisfatta per qualunque campo vettoriale h,
indotti in errore dalla considerazione che il prodotto vettore ortognonale ai fattori e che quindi rot h sia
ortogonale a h, ma non cos!
Esercizio: trovare un campo vettoriale non ortogonale al suo rotore.
Risposta. Consideriamo il campo di velocit di un moto rotatorio intorno ad asse z con velocit angolare .
Si visto che il rotore di v 2. Se ora aggiungiamo al nostro campo un campo costante il rotore non
cambia, coinvolgendo le derivate. Prendiamo allora come campo costante un campo diretto come z. ecco
allora che il rotore non ortogonale al campo
Esercizio: Mostrare che il gradiente di una funzione, cio f, si pu scrivere come:
V
fd
V f
S


S
0
lim
dove S la superficie che racchiude il volume V
Soluzione: basta considerare nella

V S
dV p d p S
un volume V infinitesimo.
La relazione ottenuta pu intendersi come una definizione alternativa del gradiente di uno scalare
Sia ora a(x,y,z) un generico campo vettoriale definito nella regione . La quantit:
z
a
y
a
x
a
z
y
x

una
funzione scalare definita in .Questa funzione ha un significato fisico ed unimportanza fondamentale e
prende il nome di: divergenza di a. Si pu osservare, dalla definizione data di divergenza, che questa si pu
rappresentare formalmente come il prodotto scalare effettuato componente per componente, tra
loperatore vettoriale definito come:
z y x

k j i `
Ed il campo vettoriale a = a
x
i + a
y
j + a
z
k
Ricordiamo poi che loperatore era gi stato introdotto nel formare il gradiente di una funzione scalare.
L, applicato ad un campo scalare produceva un campo vettoriale, qui, applicato scalarmente ad un campo
vettoriale, produce un campo scalare: la divergenza del campo vettoriale in questione.
12
Scrivo ora la relazione integrale di sopra, tre volte, cambiando i nomi: una volta scrivendo al posto di
p(x,y,z), B
x
(x,y,z) unaltra volta scrivendo al posto di p(x,y,z), B
y
(x,y,z) ed una terza volta scrivendo al posto
di p(x,y,z), B
z
(x,y,z). Ho:

'







V
z
S
z
V
y
S
y
V
x
S
x
dV B d B
dV B d B
dV B d B
S
S
S
Moltiplico ora scalarmene ambo i membri della prima relazione per i, quelli della seconda per j e quelli
della terza per k. Osservo poi che : , , ,
z
B
B
y
B
B
x
B
B
z
z
y
y
x
x

k j i Ho dunque:

'





V V
z
z
S
z
V V
y
y
S
y
V
x
V
x
S
x
dV
z
B
dV B d B
dV
y
B
dV B d B
dV
x
B
dV B d B
k S k
j S j
i S i
Sommando tra loro i primi membri e tra loro i secondi, ottengo subito:


V S
dV d ) ( B S B
Che leggo quindi nel modo seguente: dato il generico campo vettoriale B(x,y,z) definito in una certa regione
dello spazio, se si considera un arbitrario volume V nella regione, delimitato dalla superficie chiusa S, si ha
limportantissimo teorema di Gauss ( o teorema della divergenza):


V S
dV d ) (B S B
Che si legge: il flusso attraverso un superficie chiusa S di un generico campo vettoriale B (Flusso del campo
B infatti il mome che prende il primo membro delluguaglianza) uguale allintegrale nel volume
delimitato dalla superficie in questione, della divergenza del campo. (In fondo, come si visto sopra, questo
teorema di Gauss risale ad Archimede! Mica male il siracusano!).
Osservazione. Chiaramente il campo vettoriale B non completamente generico; il membro di destra ci dice
che deve essere derivabile e poi deve essere continuo (ricordiamo che in pi dimensioni, la derivabilit non
implica la continuit, come invece capita nel caso unidimensionale)
Osserviamo che, se il volume V nel quale si integra nella relazione di sopra diventa infinitesimo, allora si ha
subito la relazione:
V
d
V
S



S B
B 0
lim
relazione che pu intendersi come nuova definizione della divergenza di un campo vettoriale
A parole: la divergenza di un campo vettoriale calcolata in un punto rappresenta il flusso specifico (cio il
flusso diviso per il volume racchiuso dalla superficie attraverso la quale si calcola il flusso o anche, il flusso
nellunit di volume) del campo attraverso una superficie che racchiude un volumetto infinitesimo
contenente il punto o anche, pi precisamente ancora, si pu definire la
Divergenza di un campo vettoriale calcolata in un punto P del campo come il limite (se esiste!) del
flusso attraverso una superficie chiusa che racchiude un volume contenete P diviso il volume stesso,
quando il volume tende a zero
stato sottolineato il se esiste, perch tale limite potrebbe benissimo non esistere. Vediamo un caso molto
importante: calcoliamo la divergenza del campo: r/r
3
scegliendo il punto P nellorigine degli assi.
Calcoliamo a tal fine il flusso attraverso una superficie sferica con centro lorigine e raggio r, con la
convenzione di orientare la normale alla superficie verso lesterno. Sezionata su questa sup. sferica una
13
superficietta dS, si ha:
2 3 3
r
dS
r
rdS
d
r
S
r
e quindi, sommando su tutte le superficiette che compongono la
mia superficie sferica, ho:

4
4
2
2

r
r
e dividendo quindi per il volume della sfera, ho:
3
3
4
r

.per
quanto detto, il limite di tale espressione, per r tendente a 0, dovrebbe dare la divergenza in P, ma tale limite
non esiste! (sarebbe ). Si potrebbe pensare che tale limite non esiste perch il campo stesso non definito
nellorigine e tende ad infinito se il punto si avvicina allorigine, ma si possono dare dei casi in cui il limite
non esiste perch diverso a seconda della forma del volume dV che si contrae a zero. Consideriamo, ad
esempio, il campo che ha solo componente x e che questa nulla per le x negative, mentre Ax per le x 0 e
calcoliamo il flusso attraverso le facce di un parallelepipedo orientato come gli assi che si estende nella x tra
h ed h, mentre nella y e z, si estenda tra h e h.si vede allora subito che si ha flusso solo attraverso la faccia
ad x positiva e che vale: Ah
3
. essendo il volume (+1)h
3
, la divergenza in O dovrebbe essere:
3
3
) 1 (
0 lim
h
Ah
h
+

ma tale limite non esiste in quanto dipendente da (al cui variare varia la forma del
volumetto)
Avendo definito la divergenza in questultimo modo, si pu dare una dimostrazione del teorema della
divergenza nel seguente agile modo: per un volumetto infinitesimo dV, la definizione di divergenza fornisce:

A
= dV A
Consideriamo ora un volume V generico, delimitato da una superficie S e dividiamolo idealmente in due parti
V
1
e V
2
mediante una superficie arbitraria. Questa superficie dividente comune alla prima come alla
seconda parte, ma il vettore che la rappresenta ha un verso se pensata come delimitante la prima ed il verso
opposto se pensata come delimitante la seconda e quindi la somma dei flussi attraverso le superfici S
1
e S
2
che delimitano i due volumi il flusso attraverso S. Quindi si ha: (S) = (S
1
)+ (S
2
) Procedendo nel
processo di suddivisione di un volume in due sue parti un numero indefinito di volte, otteniamo la
suddivisione di V in un numero enorme di volumetti, ognuno infinitesimo: per ognuno allora, essendo
infinitesimo, varr la :
A
= dV A . L somma di tali flussi , per il discorso delle superfici comuni fatto
sopra, pari a (S) e la somma dei mebri di destra altro non che lintegrale nel volume della divergenza del
campo, c.v.d.
Esercizio: qualcuno pu pensare di mostrare il teorema della divergenza mediante il seguente ragionamento
pi breve di quello sopra esposto:
un semplice esercizio dimostrare eseguendo le derivazioni, che, se (x,y,z) una funzione scalare ea
a(x,y,z) un campo vettoriale, vale:
a a a + ) (
In particolare, se a un vettore costante, si ha: a a) ( . Stando cos le cose, riprendiamo la
nostra legge di Archimede:

V S
dV p d p S
e moltiplichiamo ambo i membri scalarmente per un vettore
arbitrario e costante a. Ho allora:

V S
dV p d p a S a
Essendo a costante, lo porto sotto il segno dintegrale, e quindi:


V S
dV p d p a S a
Per la relazione scritta sopra, essendo a costante, ) ( a a p p . Chiamando B il vettore pa, la relazione
integrale di sopra si scrive:


V S
dV d B S B
Come volevasi dimostrare. Che cosa c che non va nel ragionamento esposto?
Risposta il ragionamento corretto, ma non ha validit generale, essendo B il prodotto di una funzione
arbitraria (la p) per un vettore fisso (a), ha, in ogni punto della regione V sulla quale si integra, direzione
costante)
14
Osservazione: si parlato di divergenza di un campo in un determinato punto. Perch questo modo di dire
abbia senso, occorre che la divergenza dipenda soltanto dal campo vettoriale ( e dal punto). Ma nella
definizione di divergenza:
z
a
y
a
x
a
z
y
x

, viene tirato in ballo anche il sistema di assi cartesiano x,y,z


che estraneo al campo; una mia scelta considerare una terna cartesiana od unaltra e viene quindi il
sospetto che scegliendo una terna lepressione di sopra fornisca in un dato punto un certo valore e che
prendondo invece nel medesimo punto unaltra terna, si possa ottenere un altro valore. In altri termini,
indicando con un soprassegno le grandezze relative alla seconda terna, chi mi assicura che sia:

z
a
y
a
x
a
z
y
x
z
a
y
a
x
a
z
y
x

?
Risposta: stai tranquillo, il tuo dubbio si dissipa se tieni presente che la divergenza di un campo si pu
scrivere anche come:
V
d
V
S



S a
a 0
lim dove non si tira assolutamente in ballo nessuna terna
cartesiana!
Osservazione: Nel teorema di Gauss, B sia un campo di velocit di un fluido e dS una superficietta
trascinata dalla corrente., allora
dt dS B
rappresenta il volumetto spazzato dallareola dS nel tempuscolo
dt e quindi il primo membro di Gauss altro non che lincremento, nellunit di tempo del volume che
allistante iniziale era delimitato dalla superficie S e quindi si ha:
la divergenza di un campo di velocit lincremento nellunit di tempo del volume unitario
Questultima osservazione dovrebbe spiegare il nome di divergenza dato a
z
B
y
B
x
B
z
y
x

: un volume
si dilata nel tempo se le linee di velocit divergono da un suo punto interno.
Esercizio: Calcolare la divergenza del campo r/r
3
.
Risposta: Risulta:
5 2 2 2
2 2 2
3 2 2 2
) (
2
) ( z y x
z y x
z y x
x
x
+ +
+ +

+ +

ed analogo risultato per le altre analoghe


derivate. Sommando quindi, se r diverso da 0 si ottiene 0. Si ha quindi che la divergenza nulla in tutti i
punti nei quali definita, cio in tutto lo spazio tranne lorigine e nellorigine non definita.
Esercizio: Calcolare il flusso del campo di sopra attraverso una superficie sferica di raggio R con centro
nellorigine. (banale!). Si pu ottenere il risultato in modo diretto e passando attraverso la nozione di
angolo solido. Farlo nei due modi!
Risposta: 4.
Esercizio: calcolare il flusso del campo di sopra attraverso una superficie S che racchiude un volume V
contenente lorigine.
Risposta: racchiudiamo la superficie S in una grande sfera di raggio R con centro lorigine. Nel volume V
interno alla sfera ed esterno ad S , per quanto visto sopra, 0
3

r
r
. Applichiamo ora il teorema della
divergenza a tale volume. Per questo teorema, si ha che il flusso attraverso la superficie che delimita tale
volume 0. Ma questa superficie in questione data dalla superficie della grande sfera ed il flusso attraverso
questa , come si visto, 4 e dalla superficie S in questione, o meglio, dalla superficie -S, in quanto questa
superficie, vista come delimitante V ha segno opposto alla medesima ma vista come delimitante V. Si ha
dunque
+
S Sfera
0 4 4
S S S Sfera
. Ovviamente, se moltiplico il campo per un
dato fattore, il flusso moltiplicato per lo stesso fattore. Se il fattore in questione Q/4
0
, il campo
risultante quello di una carica puntiforme Q posta nellorigine. Se ora poi, al posto di una carica puntiforme
ne ho un numero arbitrario, tutte contenute nel volume V delimitato da S, il flusso del campo da esse
generato sar, per ladditivit dei campi, la somma dei flussi dei campi generati dalle singole cariche.
Esercizio: calcolare il flusso del campo di sopra attraverso una superficie S che racchiude un volume V non
contenente lorigine.
Risposta: (seguite il discorso facendo dei disegni: capirete subito tutto!) Immaginiamo di tagliare la
superficie S con un piano; questo taglio d luogo a due calotte, che chiamo S
1
e S
2
. Posso dunque scrivere: S
15
= S
1
+ S
2
. Considero ora una superficie S
*
che saldata con S
2
. dia luogo ad una superficie chiusa che
racchiude al suo interno lorigine. Scriver: = S
*
+ S
2
. Sembrerebbe pure che S
*
saldata con S
1
dia luogo ad
una superficie chiusa contenente lorigine, ma non cos, perch per ottenere una siffatta superficie occorre
mutare il verso di orientazione di S
1
e quindi scriver:
*
= S
*
- S
1
. Stando cos le cose, e * sono due
superfici chiuse che racchiudono lorigine e quindi, per lesercizio precedente, il flusso del campo attraverso
di loro 4. Scrivo allora:

'

4
4
1
* *
2
*
S
S
S
S
Sottraendo membro a membro, si ha si ha:
0
1 2
+
S S
, cio, a parole, il flusso del campo r/r
3
attraverso una superficie chiusa non contenente lorigine nullo. Se ora poi, al posto di una carica
puntiforme ne ho un numero arbitrario, tutte esterne al volume V delimitato da S, il flusso del campo da
esse generato sar, per ladditivit dei campi, la somma dei flussi dei campi generati dalle singole cariche.
Si dimostrato quindi limportante teorema di Gauss:
Il flusso del campo elettrostatico attraverso una generica superficie chiusa pari alla somma algebrica di
tutte e sole le cariche elettriche contenute nel volume racchiuso dalla superficie/
0
. In formule:
0


i
i
S
tico elettrosta
Q
dS E
Da notare che per lottenimento di tale risultato entrato pesantemente in gioco il fatto che il campo elettrico
di una carica diminuisse con il quadrato della distanza. Si pu allora verosimilmente supporre che, nel caso
non statico, tale risultato non valga. Ma un miracolo della Natura ci pemette di dire sempre, ovunque, in ogni
caso, che:
0


i
i
S
Q
dS E
A parole dunque possiamo enunciare limportantissimo teorema di Gauss ( o prima equazione di Maxwell
in forma integrale) nel modo seguente:
Ad ogni istante il flusso del campo elettrico attraverso una generica superficie chiusa pari alla
somma algebrica delle cariche elettriche contenute nel medesimo istante nel volume racchiuso dalla
superficie/
0
.
Se le cariche sono distribuite con continuit secondo una densit di carica (R), la somma delle cariche
contenute nel volume V si scrive:


V
i
i
dV Q ) (r
. Ho quindi:


V
i
i
S
dV
Q
d ) (
1
0 0


S E
. Applicando il teorema della divergenza la primo membro, ho:


V V S V
dV dV d dV 0 ) ( ) (
1
0 0

E r S E E
Essendo il volume V in gioco completamente arbitrario, se ne deduce che deve essere in ogni punto (oltre
che in ogni istante):
0

E
Il campo elettrostatico generato da una carica, essendo centrale, conservativo, come lo sar pure quello
generato da 2,3n cariche. Quindi potr scrivere:
0
statico
E
Questa equazione vale solo nel caso stazionario (cio quando ogni grandezza non dipende dal tempo), ma in
generale non valida. In realt lultima relazione scritta costituita da 3 equazioni: una per ogni componente
del vettore: rotore di E. queste tre equazioni, unitamente a quella di sopra della divergenza di E costituiscono
un sistema di quattro equazioni differenziali alle derivate parziali nelle tre incognite E
x
, E
y
e E
z
la cui
soluzione che, come deve fisicamente essere, si annulla allinfinito, , ovviamente:
16

spazio lo tutto
tico elettrosta
d K dq K ' ) ' (
'
'
'
'
) (
3
3 3
r r
r r
r r
r r
r r
r E
Difficolt: Quattro equazioni in tre incognite? Non dovrebbe essere sovra-determinato e quindi impossibile?
Il fatto che se due componenti del rotore di un campo sono nulle, nulla pure la terza e quindi delle tre
equazioni omogenee una si pu omettre, ottenendo cos un sistema di tre equazioni nelle tre incognite E
x
, E
y
ed E
z
. Giustifico ora laffermazione. Sia dunque:
0
0
0

'

z
E
x
E
y
Ex
x
Ey
z
E
y
E
x z Tesi
y
z
Dalla prima e seconda condizione si ha:

dy
x
E
E dy
z
E
E
y
x
y
z
) ( ) ( . Derivando la prima rispetto
a x e la seconda rispetto ad z, si ha:

dy
x z
E
E
y
dy
z x
E
E
x
y
x
y
z
) ( ) (
2 2
. Il teorema di
Schwartz giustifica quindi la tesi.
Altra giustificazione: siano c
1
,c
2
,c
3
le tre componenti del rotore di un campo, di cui due nulle. allora:
c = (0, 0, c
3
). Calcolo il flusso di c attraverso la superficie chiusa di un parallelepipedo disposto
parallelamente agli assi e di dimensioni: h, dy, dz. Per il teorema della divergenza, essendo la divergenza di
un rotore identicamente nulla, tale flusso nullo. Daltra parte questo flusso pure: - c
3
(x,y,z)dydz + c
3
(x+h,y,z)dydz =[- c
3
(x,y,z) + c
3
(x+h,y,z)]dydz. Dalla nullit di tale flusso e dallarbitrariet di h, se ne
deduce che c
3
(x,y,z) costante al variare della x e quindi deve essere uguale a c
3
(,y,z) e quindi nullo
Applicazioni:
Vediamo ora come il teorema di Gauss permetta di ottenere con grande semplicit dei risultati gi ottenuti
nel corso di Fisica 1 ma con molta fatica.
Si consideri una superficie sferica di raggio R e centro O uniformemente ricoperta da una carica elettrica di
densit . Mostrare che il campo elettrico al suo interno nullo e che allesterno il medesimo di quello che
avremmo se la carica fosse concentrata nel suo centro. Consideriamo a tal fine una superficie sferica
concentrica di raggio r >R (<R). per motivi di simmetrica il campo E radiale ed in ogni punto di questa
superficie lintensit la medesima. Il flusso dunque di E attraverso questa superficie allora molto
semplicemente dato da E 4r
2
. Ma per il teorema di Gauss questo flusso vale 0 ( 4r
2
= Q
totale
) e quindi si
deduce che il campo elettrico nullo (e quindi si deduce che lintensit del campo elettrico :
E = Q
totale
/4
0
r
2
, cio la stessa che avremmo per una carica puntiforme concentrata in O).
Applicando il medesimo tipo di ragionamento ad una retta portante una densit di carica costante e
considerando come superficie di Gauss quella di un cilindro di raggio r ed altezza h, coassiale con la retta, si
ottiene per lintensit del campo:
E = /2
0
r
Cio il campo decrescesolo con la distanza e non con il quadrato, come invece avviene se le cariche
occupano una regione finita di spazio.
(Prima equazione di Maxwell in forma differenziale)
Obiezione: Ma non si era visto sopra che la divergenza del campo generato da una carica puntiforme era 0?
Allora 0 anche la divergenza del campo generato da un qualsivoglia numero di cariche e come allora che
la relazione di sopra uguaglia la divergenza di E ad un qualcosa di diverso da 0?
Esercizio: consideriamo un filo di lunghezza infinitesima ds immerso e trascinato da un fluido caratterizzato
da un campo di velocit v(x,y,z). al tempo t=0 il filo rappresentato dal vettorino infinitesimo ds. Quale sar
il vettorino che lo rappresenter al tempo dt?
Risposta: al tempo dt la coda del vettore si sar spostata dalla sua posizione originale di un tratto:
v(coda) dt, mentre la punta si sar spostata di: v(punta) dt. Il vettore ds dunque che rappresentar il nostro
vettorino nella sua posizione finale avr la coda in coda+ v(coda) dt e la punta in punta+ v(punta) dt sar
allora:ds =[punta+v(punta)dt (coda+v(coda)dt)] =[punta-coda+v(punta)dt v(coda)dt)]= ds+v(punta)dt
v(coda)dt) . Daltra parte : punta=coda+ds e quindi: ds = ds + [v(coda+ds) v(coda)] dt ~ ds +
[(v/s)ds] dt.
17
Attenzione: la scritta: s
s
v
d

non altro che una scrittura zippata per indicare il vettore la cui i-esima
componente :

3
1 j
j
j
i
ds
s
v
. Riassumendo, quindi, possiamo scrivere:
dt ds
s
v
ds ds dt d d d
j
j
j
i
i i

+
' '
s
s
v
s s
In particolare, se ds era diretto lungo lasse x e lungo dx, dt dx
x
d d

+
v
s s
'
, se ds era diretto lungo lasse
y e lungo dy,
dt dy
y
d d

+
v
s s
'
, se ds era diretto lungo lasse z e lungo dz, dt dz
z
d d

+
v
s s
'
Esercizio consideriamo una particella di un fluido che l tempo t = 0 abbia la forma di un parallelepipedino di
spigoli infinitesimi da, db, dc. diretti lungo i tre assi x,y,z ortogonali. Il fluido sia caratterizzato da un un
campo di velocit v(x,y,z). Il parallelepipedino durante il suo moto verr sia spostato (ovviamente!) che
deformato. Quale sar il volume assunto dalla particella allistante t+dt?
Risposta: Il volume iniziale , come noto, dato da:
z
y
x
dc
db
da
d d d dV
0 0
0 0
0 0
) ( c b a
Dallesercizio di sopra si ha allora immediatamente:
) 1 (
) 1 (
) 1 (
) (
' ' ' '
dt
z
v
dc dt dc
z
v
dt dc
z
v
dt db
y
v
dt
y
v
db dt db
y
v
dt da
x
v
dt da
x
v
dt
x
v
da
d d d dV
z
y
x
z
y
x
z
y
x

+
c b a
Sviluppiamo ora il determinante avendo per cura di trascurare, per la loro piccolezza, tutti i termini che
contengono i dt
2
, e i dt
3
. si ottiene allora:
) 1 ( ) ] [ 1 (
'
dt dV dt
z
v
y
v
x
v
dc db da dV
z
y
x
dV
v +

+

E quindi subito:
v

dt
dV dV
dV
'
1
Da qui subito uninterpretazione della divergenza di un campo di velocit:
La divergenza di un campo di velocit rappresenta la velocit di crescita di un volume del fluido, per unit di
volume medesimo(questultima affermazione proviene dallaver diviso la derivata per il volumetto dV in
questione) Con altre parole: La divergenza di un campo di velocit rappresenta la velocit di crescita di un
volume unitario del fluido. Con altre parole: La divergenza di un campo di velocit rappresenta la velocit
specifica di crescita del volume. Non ci si deve comunque dimenticare che i volumi di cui si parla sono
volumi infinitesimi, sicch risulta, giustamente, la divergenza una funzione di punto e non di volume
Altro modo di giungere alla medesima interpretazione della divergenza di un campo di velocit
Partiamo dal teorema della divergenzache riscrivo dopo aver moltiplicato per dt:

V S
dV dt d dt ) (v S v
. Che cosa rappresenta lintegrando del primo integrale? Ma chiaro! Non altro che il volumetto spazzato
dalla porzioncina infinitesima dS appartenente alla grande superficie chiusa S nel tempuscolo dt. Ma allora,
se io sommo (segno di integrale) tutti questi volumetti ottengo lincremento di volume realizzato nel
tempuscolo dt, volume variabile nel tempo che la superficie mobile S racchiude. Se si considera il caso in cui
questo volume racchiuso infinitesimo, nel secondo membro non compare pi il simbolo di somma e
diventa semplicemente: dt div(v) dV, da cui subito la tesi
18
Data lestrema importanza del teorema della divergenza, pu essere utile ed interessante mostrare anche
unaltra derivazione, di natura pi analitica
Consideriamo dapprima un tetraedro con un vertice nellorigine degli assi e tre lati orientati come gli assi,
che saranno dunque rappresentati dai tre vettori infinitesimi: dxi, dyj, dzk Allora il vettore che rappresenta
la faccia perpendicolare allasse x sar:
dS
1
= - dy dz i/2
la faccia perpendicolare allasse y sar:
dS
2
= - dx dz j/2
la faccia perpendicolare allasse z sar:
dS
3
= - dx dy k/2
la quarta faccia :
dS
4
= - (S
1
+ S
2
+S
3
) (perch?)
Per calcolare il flusso con sufficiente precisione attraverso ognuna di queste face, prender per il campo la
media dei valori che assume ai vertici di ogni faccia interessata. Detti P (=origine assi), Q (=sullasse x), R
(=sullasse y), S )=sullasse z), i quattro vertici, avr:
d
1
= (A
P
+A
R
+A
S
) dS
1
/3 = -(A
P
+A
R
+A
S
) (dy dz i) /6
d
2
= (A
P
+A
Q
+A
S
) dS
2
/3 = -(A
P
+A
Q
+A
S
) (dx dz j) /6
d
3
= (A
P
+A
R
+A
Q
) dS
3
/3 = -(A
P
+A
R
+A
Q
) (dx dy k) /6
d
4
= (A
Q
+A
R
+A
S
) dS
4
/3 = (A
Q
+A
R
+A
S
) [(dy dz i) +(dx dz j) +(dx dy k)] /6
Facendo la soma, per il calcolo del flusso totale, ho:
d =
d
1
+ d
2
+ d
3
+ d
4
= [(A
Q
+A
R
+A
S
) -(A
P
+A
R
+A
S
) ] (dy dz i) /3 + [(A
Q
+A
R
+A
S
) -(A
P
+A
Q
+A
S
)]
(dx dz j) /3 + [(A
Q
+A
R
+A
S
) -(A
P
+A
R
+A
Q
)] (dx dy k) /6
=[A
Q
- A
P
] (dy dz i) /3 + [A
R
- A
P
] (dx dz j) /3 + [A
S
- A
P
] (dx dy k) /6 =
=[A
Q
- A
P
]
x
(dy dz) /6 + [A
R
- A
P
]
y
(dx dz) /6 + [A
S
- A
P
]
k
(dx dy) /6
Con uno sviluppo in serie di Taylor arrestato al primo ordine, ho ( a meno di infinitesimo dordine
superiore) (ricordare che il volume di un tetraedro dato dallarea di base X laltezza /3):
dV dz dy dx
z
A
dz dy dx
y
A
dz dy dx
x
A
d
z
y
x
A

6 / 6 / 6 /
La quantit:

,
_

P
z
P
y
P
x
z
A
y
A
x
A
una quantit scalare ed denominata divergenza del campo vettoriale A (nel punto P). Osserviamo, per
inciso, che questa quantit pu scriversi formalmente come un prodotto scalare tra loperatore vettoriale
(leggasi: nabla) definito come:
z y x

k j i `
ed il campo A. Scriver dunque indifferentemente:
dV dV div d A A
Leggiamo quanto ottenuto:
Il flusso di un campo vettoriale attraverso le facce di un tetraedro retto infinitesimo pari al prodotto
del volume del tetraedro per la divergenza del campo calcolato la vertice ( o in un punto qualunque del
tetraedro con differenze allora di infinitesimi dordine superiore al volume)
incollando opportunamente le facce di uninfinit di tali piccoli tetraedri, si pu ottenere praticamente
qualunque volume finito. La superficie S che delimita questo volume V sar determinata dallinsieme delle
facce che restano scollate, cio non comuni a due tetraedri. Se faccio la somma dei flussi del campo
attraverso tutti i tetraedri, sommo i flussi su tutte le facce scollate ottenendo cos il flusso attraverso S. gli
altri addendi si annullano due a due perch ogni faccia comune, vista appartenente ad un tetraedro ha un
verso e vista appartenente al tetraedro contiguo ha verso opposto ed il flusso attraverso le due facce nullo.
Si ottenuto cos il teorema della divergenza:
Teorema della divergenza:
19
Per un campo vettoriale A [che deve essere continuo e derivabile( ricordate che in pi dimensioni la
derivabilit non implica la continuit, contrariamente al caso unidimensionale), visto che compaiono le
derivate nel teorema. Inoltre queste derivate devono essere continue ) vale luguaglianza:


S V
d dV S A A
Dove V un volume generico (contenuto nel campo, ovviamente) e S la superficie che lo racchiude
( purch sufficientemente regolare)
Riespongo con altre parole quanto dovrebbe essere gi chiaro: se avete capito saltate la lettura. chiaro che
incollando fra loro faccia a faccia un numero opportuno di parallelepipedi infinitesimi, si pu
approssimare il corpo con precisione arbitrariamente grande. Ora, la somma dei flussi del campo A
attraverso le facce dei singoli parallelepipedi eguaglia la somma dei flussi attraverso le facce scoperte
( cio non in contatto con le altre) di tali parallelepipedi [infatti i flussi attraverso le altre facce si annullano a
vicenda dato che i vettori di due facce incollate hanno la medesima lunghezza e direzione, ma versi
opposti] vale cio:



,
_


k
k
N
i j
i
j
N
i
i
d d d S A S A
1
4
1
) (
1

con k variabile nellinsieme delle facce scoperte.
d
i
il flusso uscente dal parallelepipedo i-esimo, ed in tutto ce ne sono N;
) (i
j
dS
la faccia j-esima del
parallelepipedo i-esimo.

j
dS la j-esima faccia scoperta. Di facce scoperte ce ne sono 0 per i
parallelepipedi interni e possono essere 1, o 2 o 3 4 o 5 per quelli di confine.
Il teorema dunque dimostrato ( alla buona)
Altra propriet delloperatore : il rotore di un campo
Ricordiamo che, dato un campo vettoriale f, una condizione che garantiva la conservativit del campo (sotto
opportune ipotesi della regione) era che valesse in ogni punto:

'

0
0
0
x
f
y
f
z
f
x
f
z
f
y
f
y
x
x z
y
z
Ricordando la definizione di prodotto vettore, si vede che queste tre condizioni si posso scrivere in forma
vettoriale come:
0 f
Ebbene, dato un campo vettoriale A, se in ogni punto calcoliamo il vettore A , abbiamo costruito un
nuovo campo vettoriale. Il vettore A prende il nome di rotore di A
Ci si pu chiedere: qual il significato fisico del rotore di un campo? A tal proposito conviene considerare il
campo di velocit v di un fluido: in un generico punto P del campo il vettore dato dalla velocit del punto
materiale che, in quellistante, transita per il punto P. Ebbene, immaginiamo di mettere nel fluido un
piccolissimo semino che verr trascinato dalla corrente. Allora, in ogni istante, il v il doppio della
velocit angolare con cui il semino ruota su s stesso.
Altro significato fisico del rotore di un campo vettoriale A. Consideriamo un punto P in un generico campo
vettoriale A e consideriamo poi una piccolissima superficie orientata (che essendo piccolissima pu essere
considerata piana), ad esempio un cerchio con centro in P. Detto questo, calcoliamo la circuitazione del
campo lungo il bordo (orientato) di tale superficie e, per ottenere qualcosa indipendente dallestensione di
questa superficie, (estensione che dipende da una nostra scelta e che non ha nulla quindi a che vedere con
propriet intrinseche del campo) dividiamo tale circuitazione per la superficie del cerchio. Il risultato
ottenuto, facendo tendere la superficietta a zero, fornir un valore che indico con r. r dipende, ovviamente,
dal campo, dal punto P e dallorientazione della superficietta (orientazione che non viene persa assumendo
che questa sia infinitesima). Ceteris paribus, variando lorientazione varier r e quindi ci sar
unorientazione per la quale questo rapporto massimo. Ma unorientazione individuata da un versore u
20
perpendicolare al piano che individua lorientazione. Ebbene, u il versore del rotore del campo nel punto P
ed r la lunghezza del rotore.Per la giustificazione di queste affermazioni, vedere pi avanti.
Esercizio: consideriamo il campo di velocit di un corpo rigido che ruota con velocit angolare costante
intorno allasse z . Trovare il rotore di tale campo
Esercizio (importante)
Dimostrare che:
0 ) ( f
Risoluzione: la componente x di rot(f) : f
z
/y - f
y
/z e quindi calcolando la divergenza devo calcolare:
(f
z
/y - f
y
/z)/ x =f
z;x,y
- f
y;x,z
e quindi per le altre derivate avr: f
x;y,z
- f
z;y,x
e f
y,z,x
- f
x;z,y
e la loro somma
nulla, per il teorema sullinvertibilit delle derivate seconde miste
I campi solenoidali
Ricordo ora la definizione di campo conservativo per istituire ora un parallelismo:
data una curva immersa in un campo A che ha come estremi (potremmo dire: i cui bordi sono) due punti A e
B [=> data una superficie immersa in un campo B che ha come bordo una certa linea (che ovviamente
chiusa)], consideriamo

l A d lungo la nostra curva [=> consideriamo

S B d attraverso la nostra
superficie]. Ebbene, se tale integrale non dipende dalla curva [=>ebbene, se tale integrale non dipende dalla
superficie], ma solo dai punti A e B che ne costituiscono il bordo, gli estremi [=>ma solo dalla linea che ne
costituisce il bordo], diremo che il campo A conservativo [=>diremo che il campo B solenoidale]
Quale una condizione che, verificata da un campo A, ci permette di dire che il campo conservativo?
Risposta: che sia 0 A , come si visto. Analogamente, quale una condizione che, verificata da un
campo B, ci permette di dire che il campo solenoidale? che la sua divergenza sia 0 . E perch?
Consideriamo una superficie chiusa S (normale orientata verso lesterno) immersa in un campo B. Tagliamo
in due questa superficie mediante una linea chiusa orientata (che, per fissare le idee, ma non
necessariamente, potr pensarsi piana e giacente sul piano x-y) {Orientare una linea vuol dire scegliere su
di lei un verso di percorrenza. Una orientazione su di una linea chiusa induce poi unorientazione della
superficie che racchiude. Precisamente: consideriamo una persona che percorra la linea movendosi nel
verso di percorrenza fissato su questa. Un vettore diretto come piedi-testa dellomino rappresenter la nostra
superficie se questa viene a trovarsi alla sinistra del viaggiatore } Questa linea chiusa, col suo verso di
percorrenza scelto, determina due superfici orientate: S
1
e S
2
che hanno questa linea chiusa come bordo
comune e che, insieme, formano la nostra S di partenza. Lorientazione di partenza della superficie S, come
viene ad essere rispetto allorientazione di S
1
ed S
2
? Per rispondere basta vedere con la mente che, se
lorientazione di S
1
concorde con S , quella di S
2
opposta!
Ma allora subito si pu dire: se div B =0, per il teorema della divergenza , il flusso di B attraverso S
nullo. Ma questo flusso altro non che il flusso attraverso S
1
(il segno meno per quanto detto sopra a
proposito delle orientazioni!) il flusso attraverso S
2
. Il ch quanto dire:
) ( ) (
2 1
B B
S S

c.v.d
Interludio: il concetto di angolo solido (orientato)
Consideriamo una superficietta infinitesima a forma di parallelogramma costruita sui vettori infinitesimi da e
db e consideriamo il vettore dS che la rappresenta. Sar: dS = da db. Il suo centro poi sia individuato nello
spazio dal vettore r. Consideriamo lorigine degli assi, O, e tracciamo idealmente tutte le semirette che
partendo da O arrivano al bordo della superficietta. Otteniamo cos un cono di vertice O . Diremo,
analogamente al caso piano, che questo cono individua nello spazio un angolo solido (infinitesimo), che
altro non che linsieme dei punti contenuti nel cono. Consideriamo pi sfere, di centro O e di raggi R
1
, R
2
,
R
n
esse intersecheranno col cono delle superficiette infinitesime dS
1
, dS
2
dS
n
. La geometria insegna che:
K
R
dS
i
i

2
con K costante. Tale rapporto, non dipendendo dal raggio, dipende dunque solo da quanto
aperto langolo solido ed quindi idoneo ad essere assunto quale misura dellangolo solido (il
procedimento lanalogo, nello spazio, della definizione di radiante nel caso dellangolo piano). Diremo
quindi che un angolo solido ha ampiezza di uno steradiante se intercetta su di una sfera di raggio unitario
con centro nel vertice, una superficie unitaria. Essendo la superficie della sfera pari a 4R
2
, ne deriva che
langolo solido totale di 4 steradianti.
Riconsideriamo ora la nostra superficietta dS che avevamo trascurato. dS (r/r) altro non che la
proiezione, con segno, della superficietta sulla superficie della sfera passante per r. Dividendo il prodotto
21
scalare per r
2
ottengo dunque la misura (con segno!) dellangolo solido individuato dalla superficietta e
dallorigine O. Potendo il prodotto essere positivo o negativo, di conseguenza diremo che langolo solido
positivo o negativo a seconda che da O si veda la faccia negativa o la positiva della superficietta dS.
Scriveremo dunque per langolo solido infinitesimo d in valore e segno:
3
r
d
d
r S

Esercizio
Consideriamo un cono di apertura . Quanti steradianti misura langolo solido individuato dal cono in
questione?
Risposta: tracciamo una sfera di raggio R con centro nel vertice. Su questa sfera il cono individua una
calotta sferica. Il rapporto tra larea di questa calotta ed il quadrato del raggio misurer lampiezza
dellangolo solido. Ora noto dalla Geometria elementare che larea di una calotta sferica e pari
allarea laterale di un cilindro che ha come base il cerchio massimo ed altezza la saetta della calotta.
Indicando dunque con h questa saetta, larea della calotta allora :
S = R
2
h
E quindi subito:
) cos 1 ( R h
con ovvio significto dellangolo
Esercizio
Consideriamo una terna di assi cartesiana ortogonale. I punti di coordinate positive giacciono
allinterno di un angolo solido. Quanti steradianti misura questo angolo solido?
Risposta. I segni delle coordinate dei punti possono essere: (+++),(++-),(+-+);(-++),(---) Ognuno
individua un angolo solido che, per motivi di simmetria ha ampiezza pari agli altri. Quindi langolo solido
totale ripartito in 8 angoli solidi uguali. La risposta allora: 4/8 = /2 steradianti
Il teorema di Gauss o, anche, prima equazione di Maxwell in forma integrata
Vengo ora ad enunciare e dimostrare un importantissimo teorema, dovuto a Gauss. Si consideri una
generica superficie chiusa S con la normale orientata verso lesterno e siano q
1
, q
2
, q
n
le cariche
contenute al suo interno (cio allinterno del volume delimitato dalla superficie in questione). Ebbene,
vale la relazione:


n
i
i
S
q Q
Q
d
1
0
`

S E
Prima equazione di Maxwell in forma integrata o teorema di Gauss
A parole: Il flusso del campo elettrico (sintende valutato ad un generico istante t se il campo varia nel
tempo) attraverso una generica superficie chiusa, uguale alla carica racchiusa dalla superficie in
questione al medesimo istante t , divisa per
0
Nel sistema MKS
Dimostrazione Supponiamo di avere una sola carica q contenuta allinterno in r
1
. allora:
22
3
1
1
0
4
) (
r r
r r
r E


q
e quindi
'
'
1
4 '
'
'
1
4
'
'
4
2
0
2
0
3
0
dS
q
dS
q
dS
q
d
S S S S


r r
r r
r r
r r r r
r r
S E

dove, ovviamente, S r d dS ' ' ` e r il versore di r che il vettore che va dal punto dove posta la carica
q al punto variabile dintegrazione sulla superficie. Quindi dS la proiezione sul piano perpendicolare ad r
dellelemento dS di superficie. Ma, Essendo dS infinitesimo, proiettare sul piano in questione e proiettare
sulla superficie sferica di centro q e raggio r , a meno di infinitesimi dordine superiori, la medesima cosa.
Ma allora 2
'
'
r r
dS
altro non che langolo solido infinitesimo d di centro q e passante per il bordo della
superficie dS (e dS). Scriver quindi:
0 0
2
0
4
'
'
1
4
q
d
q
dS
q
d
S S



r r
S E
Se ho pi cariche allinterno, dato che il campo la somma dei campi ed il flusso la somma dei flussi, si
dimostrato il teorema di Gauss. Per il campo gravitazionale si avr, per quanto visto sopra:
M G d
S
4

S g
Se ora applichiamo il teorema della divergenza a quanto ottenuto e la definizione di densit, si ha:
0
1
0 0 0

,
_



V V V S
dV dV
q
dV d


E E S E
Se un integrale nullo, non detto che lintegrando sia nullo, ma se lintegrale calcolato su di un arbitrario
volume ( quale V perch S arbitraria) nullo, allora deve essere identicamente nullo lintegrando, cio
deve aversi:
0

E
Prima equazione di Maxwell (circa met del XIX secolo) in forma differenziale
Nel sistema MKSA
Nel sistema CGS lequazione di Maxwell assume una forma un po diversa. Potremo formalmente dire che
nel nostro sistema, per lunit scelta per la carica, la costante
0
vale 8.85 10
-12
, mentre nel sistema CGS, per
la diversa scelta dellunit di carica, la costante della legge di Coulomb si scrive sempre come 1/4
0
, ma ora
la costante
0
vale 1/4. Quindi per trasformare le equazioni dal sistema MKSA nel sistema CGS,
occorrer sostituire al posto di
0
il numero: 1/4.
Quindi:
4 E
Prima equazione di Maxwell (circa met del XIX secolo) in forma differenziale
Nel sistema CGS
Il campo gravitazionale conservativo. Tale dunque deve essere il campo elettrostatico, dato che dato
dalla medesima formula. Ma sappiamo che la conservativit di un campo si pu esprimere dicendo che il suo
rotore nullo. Quindi per il campo elettrostatico sar:
0 E
Se ora consideriamo il campo elettrico, non pi il campo elettrostatico, che cosa possiamo dire? Ebbene,
un dato di fatto che la prima equazione di Maxwell formulata come sopra, sia in forma integrale che
differenziale, vale rigorosamente sempre, avendo cura di precisare il fatto che il campo E e la densit
devono essere calcolati al medesimo istante come pure il flusso del campo ad un dato istante deve essere
uguagliato alla carica contenuta al medesimo istante /
0
.Laltra relazione, quella che esprime la nullit del
rotore invece non vale in generale, ma solo in presenza di fenomeni stazionari (dove in ogni punto dello
spazio ogni grandezza non dipende dal tempo, da non confondere con statico, dove ogni velocit nulla. Se
una ruota omogenea circolare ruota uniformemente intorno ad un dato asse, in ogni punto dello spazio la
velocit della particella, come ogni altra sua propriet, che transita per di l, non varia nel tempo e quindi
23
siamo in presenza di un fenomeno stazionario. M se la ruota non pi omogenea o anche se non pi
circolare, il fenomeno non pi stazionario).
Queste formule che forniscono il rotore e la divergenza del campo elettrostatico sono molto importanti
perch permettono di determinarlo. Infatti esiste un teorema importante dellanalisi (vedi Panofsky -Classical
Electricity and Magnetism, pag. 2 e pi avanti)che afferma: Se di un campo si conosce il suo rotore e la
sua divergenza (in ogni punto, sintende!) allora il campo univocamente determinato se simpone che
sia nullo allinfinito.
Possiamo quindi porre formalmente questo problema: sia una funzione assegnata di x, y, z, determinare il
campo E tale che: 1) il suo rotore sia identicamente nullo. 2) la sua divergenza sia la (x,y,z)/
0
3) sia nullo
allinfinito. Qual la risposta? Semplice, sulla base di quanto si visto, sar:
[ ]

+ +
+ +
' ' '
) ' ( ) ' ( ) ' (
] ) ' ( ) ' ( ) ' [(
) ' , ' , ' (
4
1
) , , (
3
2 2 2
0
dz dy dx
z z y y x x
z z y y x x
z y x z y x
k j i
E

che si scrive anche, pi sinteticamente:

'
'
] ' [
) ' (
4
1
) (
3
0
dV
r r
r r
r r E

dove lintegrale esteso a tutto lo spazio
Si detto sopra che il campo elettrostatico conservativo. Quale sar allora il suo potenziale? Cominciamo
con il caso in cui si abbia una sola carica Q puntiforme che per comodit considero allorigine degli assi. Il
potenziale , per definizione, una qualunque funzione il cui gradiente fornisca il campo elettrico
1
.
facile vedere (basta fare le drivate parziali!) che se della funzione:
C
z y x
Q
C
r
Q
z y x +
+ +
+
2 2 2
0 0
1
4 4
1
) , , (

Con C costante arbitraria, facciamo il gradiente, otteniamo proprio:


3
0
4 r
Q r

cio il campo elettrostatico. Per quanto riguarda la costante C, il suo valore inessenziale e la porremo per
semplicit nulla. Con tale scelta, il potenziale allinfinito 0. E se la carica non posta nellorigine, m nel
punto di coordinate x, y, z, quale sar il potenziale in x,y,z? Semplice ed ovvio, sar:
2 2 2
0 ) ' ( ) ' ( ) ' (
1
4
) , , (
z z y y x x
Q
z y x
+ +

E se abbiamo ora pi cariche che generano il campo? Semplice, il potenziale sar la somma dei potenziali e
questo deriva dal fatto che la derivata ( e quindi il gradiente) di una somma di funzione (potenziali dovuti
alle singole cariche) pari alla somma dei gradienti delle singole funzioni (che sono, a parte il segno, i campi
generati dalle singole cariche ed il campo totale ne la somma). Generalizzando al caso di una distribuzione
continua avremo allora subito:
'
'
) ' (
4
1
) (
0



zio tuttolospa
dV
r r
r
r

Esercizio: provate, facendo il gradiente della scritta sopra, che questo gradiente risulta proprio essere

'
'
] ' [
) ' (
4
1
3
0
dV
r r
r r
r

Osservazione Il calcolo di E dallintegrale di sopra esige il conto di tre ( tante sono le componenti di E )
integrali ed ognuno di questi triplo. Se non ci sono particolari simmetrie della distribuzione di carica, fare
1
Attenzione! Molto spesso faccio un po di confusione tra potenziale ed energia potenziale. Fate dunque attenzione! Il
potenziale una funzione il cui gradiente il campo. Di tali funzioni ne esistono infinite: una differisce dallaltra per
una costante. Questo fatto permette di definire come nullo il potenziale in un punto arbitrario. Lenergia potenziale di
una carica q che si trova in un punto del campo dove il potenziale : U = -q . Quindi, se la carica unitaria,
lenergia potenziale ed il potenziale differiscono per il segno. Lenergia potenziale poi, di una carica posizionata in un
dato punto, rappresenta il lavoro fatto da un operatore esterno per portare la carica dal punto dove il potenziale stato
scelto come nullo, alla posizione in questione
24
questi integrali tripli pu essere veramente pesante! Ma noi potremmo risparmiarci gran parte del lavoro
calcolando dapprima (che richiede solo il calcolo di un integrale triplo) e poi, ottenutolo, derivarlo rispetto
ad x ( ottenendo E
x
), derivarlo rispetto ad y (ottenendo E
y
) e derivarlo rispetto a z (ottenendo E
z
). essendo
molto pi facile derivare che integrare, il vantaggio evidente!)
interessante ora vedere come la prima equazione di Maxwell, unitamente al principio di Einstein, che
afferma che
qualunque fatto avvenga in un punto P, non pu influire in alcun modo su ci che posto in un punto
Q se non dopo un tempo maggiore (o uguale) della distanza tra P e Q divisa per c (=velocit della luce
nel vuoto)
permette di dimostrare che la carica di un corpo non dipende dalla velocit di questo (mentre invece la
massa, se la velocit di questo comparabile con c, si, come ha previsto lo stesso Einstein). Infatti, si
consideri una carica q ferma al centro di una sfera di raggio R. Il flusso di E q/
0
.
.
Si metta ora
istantaneamente in moto con una velocit V. Il flusso del campo elettrico calcolato ad un istante t < R/c deve
dare sempre il medesimo valore di prima perch i punti della superficie non si sono ancora accorti della
messa in moto di q. ma allora la carica allistante t, pur essendo in moto con velocit V, deve avere sempre il
valore q, per il teorema di Gauss. Quindi la carica non dipende dalla velocit c.v.d.
Il dipolo elettrico
Consideriamo un insieme di cariche puntiformi. Vogliamo approssimare il campo elettrico da loro generato,
a grande distanza, con ununica carica puntiforme di valore opportuno (Q) posta in un punto opportuno (R).
Siccome il campo elettrostatico generato da cariche puntiformi esattamente uguale al campo gravitazionale
generato da masse puntiformi, si pu applicare il risultato di un esercizio degli appunti dellanno scorso in
cui si mostra che deve essere:

'

i
i
i
i i
i
i
q
q
q Q
r
R
Questo risultato non fa una grinza sela carica totale non nulla, (le cariche, a differenza delle masse,
possono anche essere negative!). se la carica totale nulla, la R perde di senso ed il problema permane.
Consideriamo, per semplicit allora di avere due cariche puntiformi: +q, posta in r
2
e -q posta in r
1
. Si
chiama momento elettrico di dipolo o momento dipolare il vettore:
) (
1 2
r r p q `
Lintensit del dipolo dunque data dal prodotto della carica per la reciproca distanza d ed diretto dal - al
+. Consideriamo ora il campo elettrostatico in un punto dellasse del dipolo distante r dal centro e molto
distante (r>>d). :
3 3
2 2 2 2 2 2
2 2
)] / 1 ( ) / 1 [(
1
)
) 2 / 1 (
1
) 2 / 1 (
1
(
1
)
) 2 / (
1
) 2 / (
1
(
r
p
K
r
Kqd
r d r d
r
Kq
r d r d r
Kq
d r d r
Kq E

+
+

In termini vettoriali posso scrivere:


3
2
r
K
p
E
Consideriamo ora il campo in un punto sempre distante r, ma sullasse del dipolo. Con laiuto del disegno di
sotto facile vedere che in questo caso :
3
r
K
p
E

Il caso generale, cio quando il vettore r forma un angolo con p, si riconduce ai primi due. Basta vedere p
come somma di due dipoli: p
1
diretto dalla carica negativa al punto dossrvazione e p
2
perpendicolare ad r e
diretto verso la carica positiva (vedi disegno). Sar allora:
2 1
p p p +
25
e:
) 2 (
1
2 1
3
p p E
r
K
usando la relazione di sopra:
) 3 (
1
1 3
p p E
r
K
Daltra parte :
r r
r r p
p

1
e quindi subito:
) 3 (
1
2 3
p r
r p
E

r r
K
Se ora vogliamo trovare il potenziale di dipolo, cio trovare una funzione il cui gradiente (col segno
cambiato) fornisce il campo E di sopra, la ricerca diretta pu presentare qualche difficolt, allora:
Con tutto rigore (in r
1
c la carica negativa):
1
1
]
1


2 1 0
4
1
) (
r r r r
r
q q

Ora : ] 2 1 [ ]
2
1 [ 2 ) ( ) (
2
1 2
2
2
1
2
1 2 2
1 1
2
1 1
2
1
r
r
r
r
r
r r r
r r r r
r r r r r r r r

+

+ .
Analogamente:
] 2 1 [
2
2 2
2
2
r
r
r r
r r

e quindi:
3
0
1 2
3
0
2
2
2
1
0
2
2
2
1
0
4
1
) (
4
) 1 ( ) 1 (
4
2 1 2 1
4
1
) (
r r
q
r r r
q
r
q
r
q
r
r p
r r r
r r r r
r r r r
r

1
]
1


+ +

+
1
1
1
1
]
1

Dove si sono tralasciati gli infinitesimi dordine superiore e si usata lapprossimazione:


2 / 1 1 + +
Valida per piccino. Per il potenziale di dipolo possiamo dunque scrivere
3
0
4
1
) (
r
dipolo
r p
r

Facendo il gradiente, col segno cambiato, si ottiene nuovamente per il campo elettrico di dipolo:
]
) ( 3
[
4
1
3 5
0
r r
dipolo
p r r p
E


26
Torniamo ora al caso generale in cui si hanno tante cariche puntiformi, di carica totale 0. il campo da loro
generato, a grande distanza, sar allora approssimato da un dipolo di carica positiva pari alla somma delle
positive, posizionata nel centro di carica delle cariche positive e di carica negativa pari alla somma delle
negative, posizionata nel centro di carica delle cariche negative. Tale campo, essendo un campo di dipolo,
decrescer col cubo della distanza. Se il centro di carica delle positive coincide con quello delle negative,
(come nel caso di cariche uguali, ma di segno alterno, poste ai verici di un quadrato) il termine di dipolo
sparir ed il campo decrescer con la distanza ancora pi rapidamente.
Esercizio due dipoli, di momento p
1
e p
2
solo allineati ad una distanza reciproca z. trovare la forza
dinterazione.
Risposta: Se sono messi: -,+,-,+ oppure +,-,+,- la forza sar di attrazione, altrimenti di repulsione. Per quanto
riguarda lintensit, si ha che il campo E generato dal dipolo 1 esercita, sulle due cariche del dipolo 2 una
forza la cui intensit data da:
2 ) 1 ( 2 ) 1 (
) ( ) (
dipolo dipolo dipolo dipolo
q d z q z f + E E
Dove con d si indicata la distanza reciproca delle cariche del dipolo 2.
Ottengo quindi:
4
0
2 1
2
3 3
0
1
6
4
]
) (
2 2
[
4 z
p p
q
d z z
p
f
dipolo


+

Esercizio: Un dipolo p si trova in un campo elettrico E. Determinare la forza agente su di lui
Risposta: non essendo altro un dipolo che due cariche di segno opposto, la forza cercata sar la somma delle
forze agenti sulle cariche. Sia r il vettore che individua la carica negativa; p = q. allora:
f = -qE(r) +q E(r+), cio: p r r +
i i i i i
E E q E E q f ] [ )] ( ) ( [
, ,
e quindi:
forza agente su un dipolo: p
i i
E f
Esercizio: Un dipolo p si trova in un campo elettrico E. Determinare il momento rispetto ad un punto O
agente su di lui.
Risposta: non essendo altro un dipolo che due cariche di segno opposto, il momento cercato sar la somma
dei momenti agenti sulle cariche. Sia r il vettore che individua la carica negativa; p = q. allora:
E p r f
r E r E r E r r E r E r r E r
r E r r E r r E r r E r M
+
+ + + + + + +
+ + + + + +
)] ( )] ( ) ( [ )] ( ) ( ) ( [
)] ( ) ( ) ( [ )] ( [ ) ( )] ( [


, , , , , ,
, , , ,
q q q
q q q
e quindi:
momento agente su un dipolo:
E p r f M +
Esercizio (non proprio facile): Si consideri un dipolo p nellorigine di un piano x-y orientato lungo y. Si
abbandoni una carica q di massa m da ferma in un punto dellasse x distante r dallorigine. Mostrare che
sotto lazione del campo elettrico del dipolo descriver una semicirconferenza di centro lorigine con
diametro lasse x.
Risposta: Lenergia potenziale della carica :
) cos(
4 4
3
0
3
0


r
r
qp
r
q
U

r p
e
lenergia cinetica : ] ) ( [
2
1
2 2

`
` r r m E
cin
+ e lenergia totale dunque:
] ) ( [
2
) cos(
4
2 2
3
0


`
` r r
m
r
r
qp
U E E
cin
totale
+ + +
Se, ripeto: se, vincoliamo la carica a muoversi su di una circonferenza di raggio r, lenergia della carica
risulta:
2
3
0
) (
2
) cos(
4


`
r
m
r
r
qp
U E E
cin
totale
+ +
Se la poniamo ferma allistante iniziale ( = /2), il principio di conservazione dellenergia permette di
scrivere:
27
0 ) (
2
) cos(
4
2
3
0
+ +

`
r
m
r
r
qp
U E E
cin
totale
che pari pari lespressione dellenergia di un pendolo di lunghezza r che si muove in un campo di gravit:
3
0
4 mr
qp
g

quando abbandonato inizialmente in posizione orizzontale con velocit nulla e quindi il


moto della carica, cos vincolata, sar esattamente quello del pendolo. E se non presente il vincolo? Nulla
cambia se la componente centripeta della forza elettrica esercitata dal dipolo giusto pari alla forza
centrifuga della carica. Questa forza centrifuga, quando la massa nella posizione , data da:
r
v
m
) (
2

.
Daltra parte la velocit si ricava dal principio di conservazione dellenergia scrivendo:
) cos(
2
) (
2
) cos(
4
3
0
2 2
3
0


r
mr
qp
v v
m
r
r
qp

E quindi la forza centrifuga :
) cos(
2
/
4
0
2


r
r
qp
r mv F
centrifuga

Calcoliamo ora la componente centripeta della forza elettrica. :
[ ]
centrifuga dipolo centripeta elettrica
F
r
q
r r r r
q
r r r
q
r
q f
1
]
1

r p r p
r p r r r p r p r r p r
E ) ( 3
4
) ( 3
4
]
) ( 3
[
4
4
0
3 5
0
3 5
0

Se la carica per inizialmente non ha velocit nulla, il suo moto pi complicato
Esercizio (per nulla semplice): Determinare tre costanti del moto di una carica attratta da un dipolo posto
nellorigine ed orientato lungo lasse z.
Risposta: Una costante del moto L
z
(ovvio!); unaltra lenergia meccanica (ovvio!); la terza si pu trovare
cos: partiamo da:
p r
p r r p
r f r M
p r r p
E f


3
0
3 5
0
3 5
0
4
]
) ( 3
[
4
]
) ( 3
[
4 r
q
r r
q
r r
q
q
dipolo

La seconda equazione cardinale allora dice:
L p r
p r r p
r f r M
dt
d
r
q
r r
q


3
0
3 5
0
4
]
) ( 3
[
4
Moltiplicando ambo i membri scalarmente per L, si ha:
) ( ) (
4 4
3
0
3
0
v r p r L p r m
r
q
r
q


Ricordo ora la seguente formula di calcolo vettoriale:
d b c a d c b a ) ( ) ( e quindi dalla formula di sopra si ha subito:
)] )( ( ) ( [
4
)] )( ( ) ( [
4
) ( ) (
4 4
2
3
0
2
3
0
3
0
3
0
p r
r
r v p p r v r v p v r p r L p r
dt
d
r
r
qm
r
r
qm
m
r
q
r
q

Ora : rdr/dt = r dr/dt e quindi:
] [
4
] [
4
)] ( ) ( [
4
) (
4
2
0
2
3
0
2
3
0
3
0
dt
dr
r r
qm
dt
dr
r r
r
qm
dt
dr
r r
r
qm
r
q r v
p r v p p r v p L p r

Ora :
dt
dr
r r r
dt
dr
dt
d
r
r dt
d
2 2
r v
r
r
r

e quindi subito:
)
4
(
4
) (
4
0 0
3
0
r
qm
dt
d
r dt
d qm
r
q r
p
r
p L p r

Daltra parte :
28
2
3
0
3
0
2
1
)
2
( )
4
(
4
L
dt
d
dt
d
dt
d
r
q
r
q


L L
L L M L p r L L p r

e quindi, uguagliando le due espressioni, ho subito:
0 ]
4 2
[
2
1
)
4
(
0
2
2
0
+
r
qm L
dt
d
L
dt
d
r
qm
dt
d r
p
r
p

Si conclude quindi che:
r
qm
L
r
p +
0
2
2
la costante del moto cercata
Linee di forza e superfici equipotenziali di un dipolo
Voglio ora determinare le linee di forza del campo elettrico di un dipolo disposto al centro degli assi e lungo
lasse x. Anzitutto, godendo il problema di evidente simmetria cilindrica, baster analizzare le linee di forza
che giaccione nel piano x-y. Ricordo che una linea di forza una linea la cui tangente in ogni punto diretta
come il campo in quel punto. Dovr unque essere:
x
y
E
E
dx
dy

Ora :
2 2 2 3
2
2 3
2
3
3 ) 3 (
y x
xy
E
E
r
xy
p
r
K
E p x
r
p
r
K
E
x
y
y x


Occorre dunque risolvere lequazione
differenziale:
2 2
2
3
y x
xy
dx
dy

Unimpresa non da poco! Come fare? Unidea di provare a passare da coordinate cartesiane a coordinate
polari
Dalla formula di trasformazione delle coordinate ho subito:

'

+



d R dR dy R y
d R dR dx R x
) cos( ) sin( ) sin(
) sin( ) cos( ) cos(
E quindi:
[ ]


d R dR d R dR ) sin( ) cos(
) ( sin ) ( cos 2
) sin( ) cos(
3 ) cos( ) sin(
2 2

1
]
1

+
e quindi:



d R dR
1
]
1

+
1
]
1

) ( sin ) ( cos 2
) ( sin ) cos(
3 ) cos(
) ( sin ) ( cos 2
) sin( ) ( cos
3 ) sin(
2 2
2
2 2
2
da cui:

d d
R
dR
) sin(
) cos(
2
) ( sin ) ( cos 2
) sin( ) ( cos
3 ) sin(
) ( sin ) ( cos 2
) ( sin ) cos(
3 ) cos(
2 2
2
2 2
2

1
]
1

1
]
1

+

Integrando ottengo:
) ( sin ) (
2
C R
Si ottenuta una notevole semplificazione passando a coordinate polari!
29
Una interessante identit (di Gauss-Green)
Consideriamo n cariche q
1 ,
q
2
,

q
n
nei punti: r
1 ,
r
2
,

r
n
ed altre m cariche Q
1 ,
Q
2
,

Q
m
nei punti: R
1 ,
R
2
,
R
n
e consideriamo la grandezza:


,
_

,
_

j i
n
i
m
j
j i
j
i
n
i
m
j
j i
j i
j i
j i
Q
q
Q q Q q
, 1 1 1 1
R r R r R r
Il termine tra parentesi tonde altro non che il potenziale( a meno di un inessenziale fattore
moltiplicativo)generato dalle cariche Q
j
nel punto r
i
dove posta la carica q
i
potenziale che opportunamente
sar indicato con (r
i
). Quindi scriver:


,
_

,
_

n
i
i i
j i
n
i
m
j
j i
j
i
n
i
m
j
j i
j i
j i
j i
q
Q
q
Q q Q q
1 , 1 1 1 1
) (r
R r R r R r
Siccome le cariche Q non sono n pi belle n pi brutte delle q, potr anche scrivere:

m
j
j j
j i
j i
j i
Q
Q q
1 ,
) (R
R r

Dove il potenziale generato dalle cariche q.
E quindi subito la banale relazione:



m
j
j j
n
i
j i
Q q
1 1
) ( ) ( R r
Nel corso di Fisica 1 si dimostrato, in modo piuttosto pesante, che che una sfera omogenea produce al suo
interno un campo gravitazionale che cresce linearmente con il raggio e si annulla (evidentemente)
nellorigine.Allesterno invece la sfera crea un campo identico a quello che produrrebbe se fosse concentrata
nel suo centro e quindi anche il potenziale esterno non muta concentrando la massa nel centro. Siccome il
campo gravitazionale e quello elettrostatico seguono la medesima legge di dipendenza dalla distanza, le
propriet di sopra varranno anche per il campo elettrico di una sfera omogenea di carica. Il potenziale
dunque di uno strato sferico omogeneo, di densit di carica superficiale costante , sar lo stesso di quello
che si avrebbe concentrando lo stato nel suo centro. Consideriamo ora lidentit di Gauss-Green, ponendo n
= 1 e le cariche Q siano quelle che formano uno strato sferico omogeneo di raggio R centrato nellorigine.
Chiamo q lunica carica delle n ed r la sua posizione. Per quanto detto sopra, il potenziale quello
generato da una crosta sferica di raggio R nel punto distante r dal suo centro. Ho dunque:

m
j
j
j
totale
q
Q
r
Q
q
1
r R
30
Ma le cariche Q costituiscono un continuo superficiale e quindi sar opportuno sostituire la sommatoria con
un integrale e la generica carica Q con dS, dove dS la superficietta elementare della sfera di raggio R che
porta la carica Q
totale
. Ho quindi:
2 2
4 4 R
dS
q
R
dS q
Q
dS q
r
q
dS q
r
Q
q
S S
totale
S
S
totale

r R r R r R
r R
Leggiamo quanto si scritto: q/r altro non che il potenziale (a parte un inutile fattore) generato da una
carica q nel centro O di una sfera di raggio R. e molto opportunamente sar indicato con v(O). Nel secondo
membro lintegrando altro non che il potenziale generato da q nel punto generico R della superficie sferica
ed allora la frazione sinterpreta come il valore medio del potenziale sulla superficie della sfera di raggio R.
Ho quindi limportante risultato:
Il valor medio del potenziale generato da una carica esterna su di una superficie sferica pari al valore
del potenziale nel centro della sfera
Per linearit la relazione di sopra si estende ad un qualunque sistema di cariche esterne e quindi:
Il valor medio del potenziale su di una superficie sferica non contenete cariche, pari al valore del
potenziale al centro della sfera
Ne consegue subito allora che:
In una regione priva di cariche il potenziale non pu avere un massimo od un minimo. Infatti, se per
assurdo in P il potenziale fosse massimo (o minimo) in P lui supererebbe (sarebbe inferiore alla) la media dei
valori su di una superficie sferica centrata in P. Ma questo fatto ha unimportante conseguenza: nel campo
elettrostatico (non ad immediato ridosso di una carica), una carica pu essere in equilibrio (E=0) ma
lequilibrio non pu essere stabile. Questo fatto apre allora il problema di come la materia possa esistere,
cio come le cariche che la compongano possano essere in equilibrio. Ma per questo punto, vedi pi avanti
Lequazione di Poisson e Laplace
Far ora due passaggi banali ottenendo un celebre ed importante risultato valido in elettrostatica:
se sostituisco in
0
/ E il campo E con , ottengo subito:
( )
0


. Sviluppando le derivate,
si ha:
0
2
2
2
2
2
2
) (


z y x
`
Il primo membro, cio la somma delle tre derivate seconde, si chiama: laplaciano di (in onore di Laplace).
E quella che si scritta prende il nome di equazione di Poisson ed una condizione a cui deve soddisfare,
punto per punto, la funzione potenziale. dove c il vuoto, cio dove non ci sono cariche, lequazione prende
il nome di: equazione di Laplace (che dunque un caso particolare dellequazione di Poisson) e suona cos:
0
2
2
2
2
2
2


z y x

`
Limportanza dellequazione di Poisson risiede nel fatto che molti altri campi scalari, oltre al campo
potenziale, soddisfano allequazione in questione. Tra questi, uno molto importante che imparerete presto a
conoscere, il campo della temperatura allinterno di un corpo isotropo e omogeneo, in condizioni
stazionarie ( cio quando la temperatura in ogni punto non varia nel tempo). Vedrete infatti nei corsi
successivi che la temperatura in un tale corpo soddisfa allequazione:
K
Q
z y x T
`
) , , (
Senza entrare troppo nei dettagli, il membro di destra nullo se non c produzione di calore allinterno del
corpo, quindi la temperatura soddisfa allequazione di Laplace allinterno del corpo. Spesso si presenta il
problema di determinare la temperatura nel caso in cui la si conosca sulla superficie che lo delimita.
Quindi, dal punto di vista matematico, si tratta di determinare la soluzione dellequazione di Laplace con la
condizione che questa coincida sul bordo della regione con una funzione assegnata. La formula trovata
sopra, cio
'
'
) ' (
4
1
) (
0



spazio lo tutto
dV
r r
r
r

, soddisfa allequazione di Poisson in una regione che


31
coincide con tutto lo spazio ed alla condizione aggiuntiva che sul bordo di tale regione, cio in punti
infinitamente distanti a dove si trova la nuvola di carica, la sia nulla ( infatti se r nella formula infinito,
lintegrale nullo in quanto lo diventa lintegrando). In genere la risoluzione dellequazione di Laplace con
determinate condizioni sul boro riesce impossibile nel 99.99% dei casi. Solo in casi particolarissimi
risolubile analiticamente ed ora voglio mostrarne uno di questi, interessante per il metodo risolutivo usato,
anche se ben difficile che un tal caso si presenti in pratica.
Esercizio Fornire una seconda prova del fatto che se una funzione ubbidisce allequazione di Laplace,
(si dice che armonica) allora il suo valo medio sulla superficie di una sfera, coincide con il suo valore
al centro Teorema della media. (Vale anche il teorema inverso, ma non ne dar la dimostrazione)
Dimostrazione: Sia V ed S volume superficie di una sfera.. Da 0 ) ( , applicando il teorema della
divergenza si ha:

V S
d dV S ) ( 0
.Stando cos le cose, valutiamo la differenza fra il valor
medio della relativo ad una superficie sferica di raggio r+dr e quello relativo ad una di raggio r. A tal fine,
dal centro comune delle due sfere, tracciamo uninfinit di conetti, ognuno di ampiezza infinitesima d, che
intercettano sulle due sup. sferiche due superficiette infinitesime dS
1
= r
2
d e dS
2
= (r+dr)
2
d.La differenza
in questione :

'

'

+
+
+
+
) (
2
) (
2
) (
2
2
) (
) ( ) (
) (
2
) (
2
) (
0
) , , ( ] ) , , ( [
4
1
] ) , , ( ) , , ( [
4
1
4 ) ( 4
r S r S r S r S
r S r S
r S
r S dr r S
d
r
dr
dS
r r
dr
d r
r r
dr
d
r
dr
d dr
r
d r dr
r
r
d r dr r
r
dS
dr r
dS
S

Quindi la media la medesima se compiuta su di una sfera di un certo raggio o di un altro. Se il raggio
allora infinitamente piccolo, questa media coincide ovviamente con il valore della al centro, c.v.d.
Esercizio Un punto si muove in modo del tutto casuale sulla superficie di una sfera di raggio R. determinare
il valor medio nel tempo del quadrato della componente x del vettore posizione, cio determinare:
dt t x
T
T
T


0
2
) (
1
lim
Risposta: Ricordo che x
2
+y
2
+z
2
=R
2
. e quindi: <x
2
> + <y
2
> + <z
2
> = R
2
Daltra parte ovvio che il valor
medio di x
2
uguale a quello di y
2
ed a quello di z
2
e quindi si ottiene subito:
3
2
2
R
x
Vediamo ora un secondo metodo per ottenere il teorema della media
Sviluppiamo in serie la funzione potenziale nellintorno di un punto che chiamo origine degli assi. Ho allora:
0
0
) (
) , , (
!
1
) , , (

,
_

k
k
z y x
z
z
y
y
x
x
k
z y x
Arrestando lo sviluppo al secondo ordine ottengo:

,
_

+
2
2
2
2
2
2
2 2 2
2
2
2
0
0
0
2 2 2
2
1
) 0 ( ) , , (
z
z
y
y
z x
xz
y x
xy
y x
xy
x
x
z
z
y
y
x
x z y x


Se calcolo il valor medio della su di una sfera con centro lorigine e raggio R, i termini contenenti potenze
dispari di x,y,z danno contributo nullo ed il valor medio di x
2
ovviamente diverso da zero e pari al valor
medio di y
2
ed al valor medio di z
2
. ho quindi:

,
_

,
_

+
2
2
2
2
2
2 2
2
2
2
2
2
2
2
6
) 0 (
2
1
) 0 ( ) , , (
z y x
R
z y x
x z y x



32
Se ubbidisce allequazione di Laplace, si dimostrata la tesi (almeno se R sufficientemente piccolo per
giustificare laver trascurato i termini successivi), c.v.d.
Esercizio (importante) Si consideri una lastra rettangolare di lati L e b, di cui il primo disposto lungo lasse
x ed il secondo lungo y. Si supponga inoltre che lungo lasse z la temperatura non varii, ma che dipenda solo
da x ed y. Sui bordi si impongano le seguenti condizioni:Bordo y=b, la T sia nulla. In formule: T(x,0)
=0Bordo x=0, la T sia nulla. In formule: T(0,y)=0Bordo x=L, la T sia nulla. In formule: T(L,y)=0Bordo y=0,
la T sia: sin(px/L) In formule: T(x,b) = sin(px/L),Con p intero >0
Occorre dunque risolvere lequazione:
0 ) , , (
2
2
2
2


y
T
x
T
z y x T
con le condizioni sopra elencate. A tal proposito bene dire che si dimostrata in matematica, sotto larghe
ipotesi sulla forma della regione e sulla funzione al bordo, lunicit della soluzione di tali problemi e quindi
questo fatto ci conforta ne sapere che, quale che sia il metodo da noi seguito per trovare la soluzione, anche
se pu suscitare delle perplessit, questo metodo certamente corretto, in quanto fornisce la soluzione. Per
procedere assumo arbitrariamente che la nostra incognita si possa scrivere come il prodotto di una funzione
della sola x e di una funzione della sola y. Cio pongo tentativamente:
T(x) = X(x) Y(y)
Con questa posizione, lequazione di Laplace diventa:
X(x)Y(y) + X(x) Y(y) = 0
Dividendo ambo i membri per T ottengo:
0
) (
' )' (
) (
' )' (
+
y Y
y Y
x X
x X
Ora si ha che il primo addendo del primo membro una funzione della x, mentre il secondo addendo una
funzione della y. In genere, non pu avvenire che una funzione della variabile x pi una della variabile y sia
sempre nulla: variando la x e tenendo fissa la y la nullit si distrugge, a meno che, pur variando la x la
funzione della x non varii e cio che la funzione della x sia la funzione costante e quindi anche che sia
costante (la medesima col segno cambiato) la funzione della y. Porr quindi:
2 2
) (
' )' (
,
) (
' )' (

y Y
y Y
x X
x X
( per pura comodit successiva che indico tale costante come un quadrato)
Ma la soluzione generale di queste due equazioni immediata: si ha cio:
X(x) = Acos(x)+Bsin(x) Y(y) = C exp(y) + D exp(-y)
Con A, B, C, D costanti per ora arbitrarie.
Ho dunque scritto la soluzione dellequazione di Laplace fino a questo punto come:
T(x,y) = [ Acos(x)+Bsin(x)] [C exp(y) + D exp(-y)]
Ma occorre anche soddisfare le condizioni sul bordo. La prima condizione impone:
T(x,b) = [ Acos(x)+Bsin(x)] [C exp(b) + D exp(-b)]=0
E quindi [C exp(b) + D exp(-b)] = 0 (non si pu imporre A=0 e B=0, altrimenti la soluzione sarebbe
identicamente nulla). E quindi: C exp(b) =- D exp(-b) C=-D exp(-2b). allora:
T(x,y) = exp(-b) [ Acos(x)+Bsin(x)] [-D exp(y) exp(-2b) + D exp(-y)] exp(b)=
=D exp(-b) [ Acos(x)+Bsin(x)] [- exp((y-b)) + exp(-(y-b))]
E quindi:
D [ Acos(x)+Bsin(x)]sinh((y-b))
La seconda condizione impone :
T(0,y) = D [ Acos(0)+Bsin(0)]sinh((y-b))=0
E quindi A = 0 e la soluzione allora assume la forma:
T(x,y) =C sin(x) sinh((y-b))
La terza condizione impone :
T(L,y) = C sin(L) sinh((y-b)) = 0
Questa condizione si soddisfa, senza annullare identicamente la soluzione come verrebbe se si assumesse C
= 0, ponendo :
= n/L
con n intero qualunque diverso da 0 (altrimenti si annula la soluzione!). Quindi deve essere:
33
T(x,y) = C sin(nx/L) sinh(n(y-b)/L)
Lultima condizione impone :
T(x,0) = C sin(nx/L) sinh(-nb/L) = sin (px/L)
Che comporta che sia n = p e che sia C sinh(-nb/L)=, cio C = -/ sinh(nb/L)
La soluzione dunque del problema in questione :
T(x,y) =
) sinh(
L
b
p

sin(px/L) sinh(p(y-b)/L)
Se b, ) exp( ) sin( ) , ( y
L
p
x
L
p
y x T

. Se poi = 1/p
2
, allora :
), exp( ) sin(
1
) , (
2
y
L
p
x
L
p
p
y x T


e :
) exp( ) sin( ) , ( y
L
p
x
L
p
pL
y x T
y

La temperatura e la sua derivata normale al bordo calcolate sul bordo sono allora :
) sin(
) , (
), sin(
1
) 0 , (
0
2
x
L
p
pL y
y x T
x
L
p
p
x T
y

Si ha allora il fatto molto interessante: se prendiamo p negativo e molto molto grande, al limite infinito, sia la
temperatura sul bordo che il flusso di calore (che non altro che la derivata normale, fatta la conducibilit 1)
diventano piccini piccini, al limite nulli e quindi la temperatura in ogni punto di questa lastra infinita
dovrebbero tendere a 0, ma la formula di sopra che fornisce la T in un generico punto, dice che nei punti la
cui ascissa x rende il seno non nullo, lesponenziale rende la T grandissima, al limite infinita! In altri termini:
se eseguiamo mediante una misurazione rigorosissima la determinazione della temperatura sul bordo e la
troviamo nulla, non possiamo essere sicuri che sia matematicamente nulla: potrebbe essere data dalla nostra
funzione seno con un p sufficientemente alto. Lo stesso dicasi per la sua derivata. Ci aspetteremmo allora la
temperatura della lastra in ogni punto estremamente prossima allo zero, ma la formula non dice cos!
(Problema mal posto). Per c un per che ci salva (un po)! Dal punto di vista pratico, eseguire una misura
della temperatura in un punto, vuol dire calcolare la media della temperatura in un intorno piccolo a piacere
(ma mai rigorosamente nullo! La larghezza del bulbo del termometro pu essere piccola a piacere ma non
nulla) del punto scelto. Se ora si esegue la media della T trovata sopra in un intervallinodi larghezza dx (che
la larghezza del bulbo) piccola a piacere, essendo p destinato a crescere allinfinito, la media del seno
nulla ed il paradosso cade.(discorso da prendersi un po con le molle)
Il teorema del rotore ( o di Stokes)
Si visto che, sotto lipotesi di connessione semplice di una regione, la conservatitivit di un campo A
definito nella regione implica il fatto che il rotore di A sia nullo e viceversa. In formule:

0 0 A r A d
per ogni curva
Questo fatto pu indirci a pensare che la circuitazione di un vettore sia una funzione del suo rotore,
funzione che si annulla con questo. Ma che funzione potr mai essere? Osserviamo che dal punto di vista
dimensionale la circuitazione il prodotto del campo per una lunghezza ed il rotore il quoziente del campo
per una lunghezza, quindi, sempre dal punto di vista dimensionale, la circuitazione di un campo sar il
prodotto del rotore per una superficie, ma di quale superficie? Verosimilmente della superficie racchiusa
dalla curva . Ma sappiamo che la superficie rappresentata da un vettore, come pure un vettore il rotore di
un campo, mentre la circuitazione uno scalare. Come ottenere da due vettori uno scalare? Facendone il
prodotto scalare, cio il flusso! Tentativamente potremmo allora pensare che la circuitazione di un campo sia
una funzione del flusso di questo attraverso la superficie racchiusa dalla linea. Ma di quale funzione si
tratterebbe? A questa domanda risponde il sig. Stokes
34
Consideriamo una linea chiusa orientata, arbitraria immersa in un campo vettoriale A(x,y,z) e la
circuitazione del campo lungo la linea chiusa, cio

r A d
. La curva chiusa in questione pu essere vista
come il bordo di una superficie limitata da questa curva. Attraverso questa superficie, che si trova immersa
nel campo A, vi sar un flusso di questo stesso campo, cio si potr calcolare

S
dS A
. Ebbene, sotto larghe
ipotesi, vale la relazione:
Teorema di Stokes


S
d d S A l A

A parole: la circuitazione di un vettore lungo una generica curva chiusa orientata ugule al flusso
del rotore del campo attraverso la superficie cha ha come bordo la curva
Dimostrazione:
Considero dapprima il caso in cui la linea chiusa un triangolino rettangolo infinitesimo PQR. Fissiamo
allora un sistema dassi cartesiano ortogonale con lorigine nellangolo retto (punto P) e disposto in modo
che il lato PQ giaccia lungo lasse x ed il lato PR lungo lasse y. Sar allora: PQ = dx i; RP = -dy j; QR = (dy
j dx i) Orientiamo il triangolo percorrendo il bordo cos: PQR. allora il vettore che rappresenta il
nostro triangolo sar il vettore: dS = (dx dy )k/2.
Calcolo ora la circuitazione del campo (circuitazione che chiamo lavoro anche se il campo non un campo
di forza).
Per calcolare con sufficiente precisione i lavori nei tre tratti, per ognuno di questi converr assumere come
campo il valore intermedio tra gli estremi del segmento. Scriver quindi per la circuitazione:
nel tratto PQ:
[A(P)+ A(Q)] (dx i)
Nel tratto QR
[ A(Q) +A(R) ](dy j - dx i)
Nel tratto RP
[ A(R) +A(P) ]( -dy j )
Sommando, ottengo subito:
{- [A
x
(R) - A
x
(P)] dx + [A
y
(Q) A
y
(P)] dy}
Ora osservo che :
+

dy
y
A
P A R A
x
x x
) ( ) (
con infinitesimo dordine superiore a dy
ed pure:
+

dx
x
A
P A Q A
y
y y
) ( ) (
con infinitesimo dordine superiore a dx
La circuitazione risulta allora essere, a meno di infinitesimi superiore a dS ( infatti, siccome dx dy un
infinitesimo dello stesso ordine dellarea del triangolino dS, infinitesimo dordine superiore a dS):
) (
2
1
2
dy dx
dy dx
y
A
x
A
d
x
y
+ +

,
_

l A
Calcolo ora il flusso del rotore di A attraverso il triangolo
Devo dunque fare:
( ) ( ) ( ) ( ) ( ) ( ) ( ) ( ) ( )
2
0 0
dy dx
dS dS dS d
z y x z z y y x x
A A A A A A S A + + + +
e quindi:
dS
dy dx
y
A
x
A
d
dS
x
y
) (
2
A S A

,
_


Essendo in questo caso il vettore dS diretto lungo z, la parentesi tonda dunque la componente z del rotore
del campo. Analogamente si ottengono le altre componenti. Supponiamo ora che la superficie S sia di
35
estensione fissa, ma di orientazione variabile. Al variare di questa orientazione varia il prodotto scalare
scritto sopra e diventa massimo quando il dS ed il rotore hanno la medesima direzione. In questo caso, il
rapporto tra la circuitazione e la superficie in questione fornisce proprio la lunghezza del vettore rotore, come
era stato preannunciato pi sopra.
Si dunque dimostrato che, a meno di infinitesimi superiori allarea, il flusso del rotore di un campo
attraverso la sup. di un triangolo rettangolo infinitesimo pari alla circuitazione del campo lungo il
perimetro.
Vengo ora al caso in cui il circuito non sia pi infinitesimo, ma che sia costituito da una generica linea chiusa
orientata . A tal fine imagino di tagliare il circuito con una arbitraria linea l. con questo taglio si sono
formati due circuiti con la linea l in comune, ma con diverso verso di percorrenza se pensata come facente
parte dellun circuito o dellaltro e quindi la somma delle circuitazioni lungo i due circuiti quella lungo
loriginale. Procedendo con ulteriori tagli, ci si riduce a considerare circuiti infinitesimi, per i quali si
dimostrato il teorema del rotore. Con altre parole ed allungando il discorso, potr anche dire: considero una
rete, a maglie strettissime (al limite infinitesime) e la maglia sia costituita da triangolini rettangoli. Immagino
ora di tendere questa maglia sul circito e di tagliare via con una forbice la rete debordante oltre . Ora, se
ci immaginiamo loperazione, dovrebbe risultare chiaro che il circuito pu essere immaginato come
costituito da alcuni rami di maglie della rete, precisamente da quei rami che non sono comuni a due maglie.
Ciamer tali rami, rami scoperti. La circuitazione del campo A lungo allora altro non che la somma dei
lavori lungo i rami scoperti. Ora avviene che la somma dei lavori lungo i soli rami scoperti pari, a sua
volta, alla somma dei lavori lungo tutti i rami, sia scoperti che a comune con altre maglie. La cosa dovuta
al fatto che nel calcolare il lavoro lungo un tratto comune a due maglie, una volta questo tratto percorso in
un verso (considerato il tratto come appartenente ad una maglia) ed una volta lo stesso tratto percorso in
senso inverso (considerato tale tratto come appartenente ala maglia confinante con la precedente) e quindi il
contributo al lavoro lungo i tratti comuni nullo.
In formule si ha dunque:



tratti i tutti scoperti tratti
d d d r A r A r A

Ma la somma lungo tutti i tratti altro non che la somma delle circuitazioni lungo tutte le maglie. Essendo
ognuna infinitesima, per quanto si dimostrato sopra, ogni circuitazione pari al flusso del rotore attraverso
la maglia e la somma di questi flussi altro non che, per definizione di flusso, il flusso del rotore attraverso
la superficie che ha come bordo la linea
Il teorema dunque dimostrato
Osservazione: Ma, si pu osservare, di superfici che hanno un assegnato bordo ne esistono a iosa!
Pensiamo, ad esempio, ad una superficie sferica tagliata da un piano per il suo centro. Lintersezione tra
piano e superficie una circonferenza. Orientiamola. Questa bordo sia di un cerchio, sia di una semisfera,
diciamo quella di sinistra. Ora uguaglieremo la circuitazione di A al flusso del rotore di A attraverso il
cerchio od attraverso la semisfera? Capite bene che il fatto che deve capitare per toglierci dall imbarazzo di
dover precisare attraverso quale superficie si deve calcolare il flusso, che tale flusso sia il medesimo
attraverso tutte le superfici che hanno quella data linea come bordo! Se fosse cos allora non sarebbe
necessario specificare attraverso quale superficie va calcolato tale flusso. forse troppo sperare che avvenga
un simile fatto? Ora considerate bene questa cosa: dire che il flusso indipendente dalla superficie (sempre
che il bordo sia comune!) la stessa cosa che dire che il flusso attraverso la superficie chiusa (con la normale
orientata verso lesterno) costituita dallunione di due superfici con medesimo bordo, nullo. E che cosa
garantisce che il flusso di un vettore F attraverso una superficie chiusa arbitraria sia nullo? Ma chiaramente,
per il teorema della divergenza, che sia: div F = 0 ! Quindi, nel nostro caso, non sorgerebbe nessuna
difficolt nel teorema di Stokes se fosse:
0 ) ( A
Ma cos proprio! Basta applicare la definizione di rotore e poi la definizione di divergenza e si ottiene che
il primo membro identicamente nullo (tenere presente il teorema sullinvertibilit delle derivate seconde
miste!)! ( osservate poi questo fatto assai curioso! Il rotore di A formalmente un prodotto vettore tra
loperatore vettoriale nabla ed il vettore A. Ma il prodotto vettore tra due, ortogonale alluno come
allaltro. Quindi questo prodotto vettore dovrebbe dunque essere ortogonale al vettore nabla. Ma fare la
divergenza di un vettore, altro non che, formalmente, fare il prodotto scalare del vettore con il vettore
nabla. Quindi, se faccio il prodotto scalare tra nabla ed un vettore a lui perpendicolare ( A ), dovrei
36
ottenere 0 ed proprio cos! Il punto debole per nel ragionamento che nabla non un vettore!; se lo fosse,
avrei che ) ( ) ( A A , per la commutativit del prodotto scalare, ma mentre il primo termine
si visto essere nullo, il secondo un operatore non nullo! Altra motivazione alla buona che la divergenza
di un rotore nulla: si sa che il prodotto misto di tre vettori ottenuto mediante il determinante di una
matrice le cui righe sono costituite dalla componenti dei tre vettori. Ma scrivendo la matrice tali componenti,
comparendo le componenti di nabla due volte, il determinante nullo)
Si detto dunque che la relazione: 0 ) ( A si pu dimostrare molto semplicemente svolgendo i
calcoli indicati dalloperatore nabla. Alternativamente la si pu dimostrare in modo intuitivo, senza svolgere
calcoli opachi, deducendola dal teorema di Stokes e quello della divergenza. Infatti, consideriamo una
arbitraria superficie chiusa immersa in un campo vettoriale A. Sulla superficie tracciamo una piccolissima
circonferenza che viene a dividere S in due parti: un cerchietto, che linterno della circonferenza, che indico
con S
1
ed il resto della superficie S, che chiamo S
2
. Calcoliamo la circuitazione di A lungo tale
circonferenza. Per il teorema di Stokes, del quale si ammessa la validit, tale circuitazione sar pari al
flusso di rot A attraverso S
2
. Facendo tendere a 0 il raggio della circonferenzina, la circuitazione tende a zero;
il flusso del rotore, che gli uguale, tende dunque anche lui a 0 e la superficie attraverso la quale si calcola il
flusso tende alla superficie chiusa S. Si ottiene quindi che il flusso del rot A attraverso una generica
superficie chiusa 0. Ma tale flusso, per il teorema della divergenza, altro non che lintegrale nel volume
racchiuso da S della divergenza del rotore di A, integrale che dunque nullo. Per larbitrariet del volume, si
ha la tesi.
Un interludio vettoriale
Siano p ed a due vettori dati. a si potra scomporre in due componenti, indicati con a
||
e con a

rispettivamente paralleli e perpendicolari a p. Dovrebbe essere chiaro a tutti che vale:
a
||
= 1/p
2
p(pa)
e che poi :
a

= a - a
||
= a - 1/p
2
p(pa)
Stando cos le cose, voglio trovare unespressione che mi fornisca p(pa).
Ora : pa = p( a
||
+ a

) = p a

= p (a

rotato di 90
) e quindi: p (pa) = p
2
(a

rotato di 180
) = - p
2
(a

) e quindi,
dalla relazione di sopra:
p (pa) = p(pa) - p
2
a
Sia ora p loperatore vettoriale nabla. Con laiuto della relazione di sopra, si pu supporre ora che sia:
A A A ) ( ) (
Relazione che ci torner molto utile in seguito. Preciso che con la scritta: A intendo semplicemente un
vettore la cui componente i-esima altro non che
i
A . Da notare infini che la relazione ottenuta
faticosamente con metodi geometrici poteva anche ottenersi pi facilmente ma in modo meno trasparente
calcolando semplicemente le componenti del membro di sinistra e quelle di destra e verificandone
materialmente luguaglianza. Da notare per che la dimostrazione di sopra non rigorosa: le propriet del
triplo prodotto vettoriale semplicemente la suggerisce. Anche qui, come precedentemente proceduto, la non
rigorosit deriva dal fatto che nabla non un vettore, per cui, mentre
p(pa) = (pa) p
non invece
) ( ) ( A A
perch a sinistra c un vettore ed a destr un operatore
Un teorema di calcolo vettoriale
Si visto sopra che il campo elettrostatico caratterizzato da: 0 E e
0

E
. Nel caso non statico
la conservativit del campo cade ed il rotore di E non sar pi nullo, ma un vettore che al momento non ci
interessa di specificare. Le altre equazioni fondamentali deelelettromagnetismo preciseranno il rotore di E e
la divergenza ed il rotore di un altro campo, del campo B che si visto nellespressione della forza di
Lorentz (tale formula in effetti definisce i campi E e B). Ci si pu chiedere come mai sia tanto importante
dare la divergenza ed il rotore di un campo vettoriale. Il fatto che, come accennato precedentemente, esiste
un teorema che afferma:
Teorema:
37
Se di un campo vettoriale conosciamo il suo rotore e la sua divergenza, allora il campo vettoriale in
questione perfettamente definito (se simpone che sia nullo a distanza infinita dalla regione limitata
in cui sono diverse da zero divergenza e rotore)
Vediamone linteressante dimostrazione.
Cominciamo ad esaminare il caso in cui un campo abbia assegnata divergenza ma rotore nullo. Ma questo
il caso del campo elettrostatico. Detto E tale campo e /
0
la sua divergenza, si hanno le equazioni:

'



0
) , , (
0
E
E

z y x
La cui soluzione, come si visto sopra, nulla allinfinito, :




spazio
d '
'
) ' (
4
1
) (
3
0
r
r r
r
r
E

Considero ora il caso in cui il campo, che indico con B abbia divergenza nulla e rotore assegnato ( che per
motivi di opportunit futura indico com
0
j. Ho dunque per tale campo B:

'



j B
B
0
0

Ovviamente, essendo la divergenza di un rotore identicamente nulla, la funzione vettoriale j non pu essere
completamente arbitraria, ma dovr essere div(j) =0. La prima equazione, quella che impone la nullit della
divergenza di B pu essere automaticamente soddisfatta ponendo;
A B
(infatti, quale che sia il campo vettoriale A, la divergenza di un rotore sempre nulla). Sostituendo tale
espressione nelle altre equazioni, si ha, per lesercizio di calcolo vettoriale sopra riportato:
j A A A
0
) ( ) (
Se, per miracolo, il vettore A a divergenza nulla, la relazione di sopra si semplifica enormemente
diventando:
j A
0

La relazione vettoriale non altro che una scrittura condensata delle sue tre componenti, cio di:

'




z z
y y
x x
j A
j A
j A
0
0
0

Ma ognuna di queste equazioni formalmente identica allequazione di Poisson per il potenziale


elettrostatico e la soluzione sar allora la medesima, mutati i simboli. E cio:

spazio lo tutto
d '
'
) ' (
4
) (
3
0
r
r r
r j
r A

2
Quindi le equazioni

'



j B
B
0
0

hanno come soluzioni, nulle allinfinito:


spazio lo tutto
d '
'
) ' (
4
) (
3 0
r
r r
r j
r A
A B

Resta per da verificare se capita il fortunatissimo caso riportato sopra, se cio la divergenza di A nulla.
Vediamo:
2
La scritta implica delle derivazioni rispetto ad x, y, z, mentre ' implica delle derivazioni rispetto ad x', y', z' che
sono le variabili che rappresentano r' che il vettore destinato a spazzare tutta la regione dintegrazione
38



spazio lo tutto spazio lo stutto
d d '
'
) ' (
4
'
'
) ' (
4
) (
3
0
3
0
r
r r
r j
r
r r
r j
r A

Loperatore di divergenza si portato sotto il segno di integrale perch questo fatto sulle varabili primate
mentre nabla opera sulle non primate che, nellintegrale, figurano come parametri. Operando ora col teorema
della divergenza, si ha:


S spazio lo tutto
d d S
r r
r j
r
r r
r j
r A
'
) ' (
4
'
'
) ' (
4
) (
0
3
0

S una superficie immensa che racchiude tutto lo spazio. Ma siccome su tale superficie, immensamente
distante, la j nulla, il miracolo si avverato!
Notiamo che come E si scrive come il gradiente di uno scalare detto potenziale, cos B si scrive come il
rotore di un vettore A che verr allora chiamato potenziale vettore. ora interessante dare a B, che il rotore
di A, unespressione pi opportuna di:



spazio
d '
'
) ' (
4
) (
3
0
r
r r
r j
r B

. Ho dunque:



spazio
d '
'
) ' (
4
) (
3
0
r
r r
r j
r B

Loperatore di rotore si portato sotto il segno di integrale perch questo


calcolato integrando sulle varabili primate mentre rotore opera sulle non primate che, nellintegrale, figurano
come parametri. Per questo motivo, j, che funzione di variabili primate costante rispetto alloperatore
nabla e quindi valendo a a f f ) ( per a costante, si ha:


spazio
spazio spazio spazio
d
d d d
'
'
'
) ' (
4
' ) ' (
'
'
4
' ) ' (
'
1
4
'
'
) ' (
4
) (
3 0
3 0 3 0 3 0
r
r r
r r
r j
r r j
r r
r r
r r j
r r
r
r r
r j
r B

Da
quanto si visto sopra, se di un campo M si conosce divergenza e rotore, il campo si pu scindere nei due E
e B e scrivere cio:
M=E+B
Esercizio. Dimostrare che se a un vettore costante, ) ( ) ( B a B a . Risposta: sostituiamo al
posto di nabla un generico vettore p. : ) ( ) ( ) ( B p a p B a B a p . risostituendo al posto di p il
nabla, si ha una forte indicazione sulla validit della relazione, validit che un noioso computo pu
confermare
Esercizio
Mostrare che vale:


V S
d dV S B B
Dove B un generico campo vettoriale; V un generico volume ed S la superficie che lo racchiude
Risposta: si tratta di dimostrare luguaglianza di due vettori. Saremo sicuri delluguaglianza di due vettori P
e Q se per ogni vettore costante ed arbitrario a si ha Q a P a . Moltiplico dunque il primo membro
delluguaglianza scalarmene per il vettore costante arbitrario a:


V
sopra di esercizio
V V S
divergenza teor
S S
dV dV d d d dV ) ( ) ( ) (
.
B a B a S B a a S B S B a B a
come dovevasi dimostrare.
Applichiamo la relazione di sopra al caso in cui il volume V sia infinitesimo, si ha allora subito una nuova
interpretazione geometrica del rotore di un campo vettoriale scrivendo:
V
d
V
S



S B
B 0
lim
La relazione ottenuta pu pure essere considerata una definizione alternativa del rotore di un campo
vettoriale
39
I conduttori in elettrostatica
Come lo dice la parola stessa, conduttore elettrico sar un corpo che conduce la corrente elettrica, cio
che permette il passaggio della corrente attraverso di lui, mentre isolante elettrico sar un corpo che non lo
permette. In realt non esiste una netta distinzione tra conduttori ed isolanti; tutti i materiali permettono un
certo passaggio di corrente, quelli che la conducono meglio sono detti conduttori e quelli che la conducono
peggio, isolanti. Questa distinzione che, malgrado quanto detto sopra, si fa dei materiali come conduttori
ed isolanti, dovuta al fatto che la conducibilit elettrica si estende su di una scala che spazza su ben 24
ordini di grandezza! Il miglior conduttore, in condizioni ambientali, largento ed il miglior isolante il
quarzo. Vari materiali poi, che in condizioni ordinari sono isolanti o cattivi conduttori, a temperature
bassissime diventano poi superconduttori. Questa propriet della superconduzione, scoperta da
Kammerling ed Onnes nel 1911 circa, stata spiegata soltanto nel 1956 da Bardeen, Cooper e Schrieffer
con un lavoro che ha fruttato loro il Nobel
In Elettrostatica si pu per dare una definizione un po formale di conduttore dicendo che: Un conduttore
un corpo allinterno del quale , in ogni punto, E = 0
3
. Come mai tale definizione? Il fatto che un
conduttore, diciamo il rame per appoggiare il discorso, a causa della regolarit della disposizioni dei nuclei
nellinterno del cristalli, gli elettroni pi esterni dellatomo (elettroni di conduzione), sono attratti tanto dal
nucleo a cui appartengono quanto dai nuclei vicini. Ne risulta quindi che la forza totale che si esercita su di
loro praticamente nulla e quindi sono liberi di spostarsi nel materiale. Quando lelettrone poi arriva in
prossimit della superficie, la situazione non pi simmetrica: gli elettroni sono attratti solo dai nuclei che
sono al di quadella superficie, dato che al di l della superficie non c nulla o c una materia diversa.
Ebbene, questa asimmetria impedisce allelettrone di lasciare il conduttore, a meno che una forza
particolarmente intensa non lo strappi, compiendo un lavoro di estrazione
3
Punto qui da intendersi in senso macroscopico, cio come una regione piccolissima rispetto alle dimensioni
macroscopiche in gioco (ad esempio le parti dimensioni delle parti sensibili degli strumenti di misura), ma pur sempre
contenenti moltissimi atomi perch chiaro che se siamo vicinissimi ad un nucleo o ad un elettrone di un atomo, il
campo elettrico sar elevatissimo!
40
41
42
43
Esercizio: calcolare la densit degli elettroni di conduzione nel rame sapendo che la densit del rame di
8.92 g/cm
3
e che 1 atomo contribuisce con 1 elettrone agli elettroni di conduzione e che il peso atomico del
rame 63.54
Risposta Essendo il peso atomico pari a 63.54 ed essendo il rame monoatomico, si ha che 63.54g di rame,
cio 63.54 /8.92 = 7.12 cm
3
contengono un numero di atomi pari al numero di Avogadro, cio 6.02 10
23
atomi. Quindi 1 cm
3
di rame contiene 6.02 10
23
/7.12 = 0.845 10
23
atomi di rame e quindi altrettanti elettroni
di conduzione che vengono dunque a possedere una carica di 13500 C. come vedete, questa una carica
enorme!
Riprendiamo il discorso: da quanto visto nellesercizio, un campo elettrico pure piccolissimo, data lenorme
densit di carica libera di muoversi, darebbe luogo ad un ingentissimo moto delle cariche, con effetti
macroscopici di riscaldamento del materiale. Non verificandosi questo ovviamente in caso stazionario, se ne
deduce che deve essere E = 0. Vediamo ora alcune propriet che discendono dalla definizione di conduttore.
1) allinterno di un materiale conduttore ovunque = 0
Infatti, ci sia per assurdo una regione con diverso da 0. Considerata una sferetta in questa regione, tutta
contenuta nel conduttore, per il teorema di Gauss il flusso attraverso la superficie di questa sferetta dovrebbe
essere la carica contenuta, quindi diverso da 0. ma poich ogni punto della superficie nel conduttore, dove
E = 0, il flusso nullo. Dallassurdo segue la tesi
Ma che succede allora se io deposito nellinterno di un conduttore della carica? Dovendo essere nulla la
carica allinterno, una volta raggiunto lequilibrio elettrico, la carica non pu trovarsi altro che sulla
superficie (in realt si trover su uno strato dellordine di qualche atomo sulla superficie). Questa carica si
disporr sulla superficie in modo tale da far s che il campo elettrico da lei generato in punti interni al
conduttore sia nullo Qualitativamente si potr dire ( vedi esercizio pi avanti) che la carica si addensa tanto
pi quanto pi la superficie curva ed in modo particolare quindi sulle punte ( se il conduttore le
presenta), dove la curvatura grandissima.
Che cosa si pu dire del campo elettrico sulla superficie di un conduttore?
Che perpendicolare alla superficie. Infatti, se avesse una componente tangente alla superficie, questa
metterebbe in moto gli elettroni di conduzione sottostanti generando una corrente elettrica superficiale con
effetti macroscopici di riscaldamento che non possono verificarsi in condizioni statiche. Oppure, altra
dimostrazione pi formale: La superficie di un conduttore in elettrostatica una superficie equipotenziale
(infatti la d.d.p. tra due suoi punti pu essere calcolata lungo un percorso tutto immerso nel conduttore, dove
E nullo). Il campo elettrico, che ne il gradiente, ne dunque ortogonale
Quanto vale lintensit del campo elettrico in un punto P nelle immediate vicinanze di un conduttore?
Consideriamo un cilindretto di altezza e base infinitesime a cavallo della superficie del conduttore, (scatola
di Gauss) con il suo asse perpendicolare a questa. Il flusso del campo elettrico diverso da 0 solo attraverso
la base esterna al conduttore ed il teorema di Gauss applicato alla scatola fornisce subito: E = /
0
e quindi
vettorialmente:
n E
0

fuori appena
con n versore alla superficie diretto verso lesterno e la densit di carica del conduttore nel punto P del
conduttore pi vicino a P
E il campo E proprio sulla superficie del conduttore come sar? Osserviamo che E passa dallessere nullo
appena allinterno del conduttore, allessere espresso da:
n E
0

appena fuori dal conduttore. Sar quindi


ragionevole assumere che sulla superficie sia una media tra i due valori, cio sia:
n E
0
sup
2

erficiale
Di questo fatto se ne pu dare anche una giustificazione pi rigorosa ragionando cos: sia O un punto della
superficie, dS unareola infinitesima centrata i O Siano poi P e P due punti simmetrici rispetto ad O, posti
sulla verticale, a distanza da P un infinitesimo dordine superiore a
dS
( questa condizione vuol dire che,
benche larea dS sia infinitesima, il punto P cos vicino a lei, che un osservatore in P la vede infinita e
quindi il campo generato da lei le sar ortogonale). Il campo in P, che sappiamo essere pari a /
0
n dato
dalla somma di due campi: il primo quello dovuto alle cariche sullareola, campo che sar diretto come n e
44
varr , il secondo quello dovuto alle cariche restanti sul conduttore, campo che chiamo
,
. Ho dunque in
P:

,
+ n n
0
nel punto Pinvece, interno al conduttore, sappiamo che il campo nullo. Ora in P il campo dovuto
allareola sar, ovviamente, -n, lopposto di quello prodotto in P, mentre quello prodotto dalle cariche
restanti si potr considerare sempre pari a
,
, dato che, nello spostamento ideale dallun punto allaltro, le
restanti cariche del conduttore si mantengono sempre, rispetto alla distanza reciproca di questi due punti, che
infinitesimo del secondo ordine, a distanza infinita (un infinitesimo del primo ordine infinitamente
grande rispetto ad uno del secondo ordine!) e quindi in campo da loro generato in P il medesimo di quello
in P
Ho dunque in P

,
+ n 0
Sommando ottengo:
n
0
2


,
In O le cariche del disco non contribuiscono evidentemente al campo; resta quello creato dalle altre che
n
0
2


,
Supponiamo ora che il conduttore abbia una cavit interna vuota. La sua superficie del conduttore viene ad
essere sia quella esterna che quella interna che delimita la cavit. Una carica elettrica si pu distribuire su
questa superficie interna? No, perch se ci fosse una densit superficiale > 0 in qualche punto di questa
superficie, in altri punti dovrebbe aversi < 0 (altrimenti il teorema di Gauss applicato ad una superficie
chiusa contenente la cavit e tutta interna al conduttore condurrebbe ad un assurdo). Consideriamo allora una
linea di campo che parte dalla zona con >0 per arrivare a <0 e prolunghiamola idealmente con una linea
allinterno del conduttore che ci riporta al punto di partenza. Si avrebbe allora

0 l E d , assurdo, essendo
il campo elettrostatico conservativo.
Da quanto si visto sopra, il campo elettrico passa dal valore 0 allinterno del conduttore al valore E = /
0
appena fuori. Subisce quindi una discontinuit passando attraverso la superficie. In realt passer con
continuit da un valore allaltro nello spessore atomico entro il quale si distribuisce la carica. Questa carica, -
per la presenza del campo elettrico che in lei si trova, viene spinta in fuoridal campo e, con ottima
approssimazione, potremmo dire che il campo effettivo che agisce su di lei sia la media del campo elettrico
interno ed esterno, cio E = /2
0
. Se il campo elettrico sufficientemente intenso, riuscir ad estrarre le
cariche dal conduttore, provocando una scintilla di scarica e lequilibrio elettrostatico non sar possibile.
Comunque sulla carica che si trova sullunit di superficie agisce una forza diretta verso lesterno pari a:
E =
2
/2
0
che risulta dunque una pressione verso lesterno.
0
2
2

tica elettrosta
p
Notare che, giustamente, questa pressione non dipende dal segno della densit
Esercizio una superficie sferica di raggio R porta una densit superficiale di carica costante . Calcolare la
forza con la quale, a causa della carica, una semisfera respinge laltra.
Risposta Consideriamo un anello individuato tra langolo e + d, di superficie quindi 2R
2
sin d. La
componente lungo lasse z della forza elettrostatica allora:
df
z
=(2R
2
sin d)
2
/2
0
cos
la forza totale agente sulla semisfera sar allora la somma di tutte le forze trovate, al variare di tra 0 e /2,
cio:
0
2 2
2 /
0
0
2 2
2 / cos sin 2 / 2

R d R f

Esercizio. Sia abbia una sfera di carica omogenea di raggio R e densit . Calcolare la forza con la quale una
semisfera respinge laltra
45
Il metodo delle immagini
Problema Supponiamo di avere un semispazio riempito di materiale conduttore e sopra di lui, ad una
distanza h una carica puntiforme Q. vogliamo determinare il campo elettrico in tutto lo spazio.
Risposta ovviamente dove c il conduttore, sar E = 0. resta quindi da determinarlo nel semispazio dove c
la carica Q. importante capire che il campo non lo stesso di quello determinato dalla sola carica Q. Infatti
questa richiama, dalle regioni lontane del conduttore, cariche di segno opposto a Q che andranno quindi ad
addensarsi intorno al piede H della perpendicolare condotta da Q al piano (cariche di influenza o di
induzione). Il campo richiesto sar quello generato dalla carica Q, ma anche da queste cariche che
formeranno una densit di carica superficiale dipendente dalla distanza da H e questa densit ci ignota,
ma vogliamo calcolarla.
Per risolvere il problema giova fare una lunga digressione e considerare il seguente importantissimo
Teorema di esistenza ed unicit:
Si abbiano n conduttori. Supponiamo che dei primi m sia noto il potenziale:
1
,
2
,
m
(valori che possono
essere scelti ad arbitrio) mentre dei restanti sia nota la carica depositata su di ognuno: Q
m+1
, Q
m+2
,Q
n
(anche
queste cariche sono arbitrarie) ( eventualmente m = n, se di tutti conosciamo il potenziale, ed m =0 se di
tutti conosciamo le cariche) Questi conduttori siano poi contenuti allinterno di una cavit scavata allinterno
di un conduttore ed oltre a loro nella cavit ci sia una nuvola di carica di densit (r). Il teorema di esistenza
ed unicit afferma che si viene a creare nello spazio tra i conduttori e la cavit, a causa delle cariche che
portano i vari conduttori, un potenziale ( esistenza) e che questo potenziale (e quindi anche il campo
elettrico che ne il gradiente) unico (unicit). Lunicit vuol dire che il potenziale non pu variare senza
che varino le cariche o il valore del potenziale su un conduttore o la .
Le ipotesi che stanno alla base del teorema sono che le forme dei vari conduttori sono che i conduttori
abbiano una forma sufficientemente regolare, cosa che accade sempre in pratica.
Agli occhi del fisico questo problema pu sembrare una cosa lapalissiana: se ho dei conduttori carichi si
former pure nello spazio circostante un campo! Dal punto di vista puramente matematico, se lo formuliamo
in termini precisi, si capisce che non banale ed io fornir una dimostrazione soltanto per quanto riguarda
lunicit della soluzione
Vediamo la formulazione in termini matematici:
si consideri una regione V racchiusa da una superficie S (superficie di un grande conduttore cavo che
contiene tutti gli altri). Allinterno di V si considerino n regioni, di volumi V
1
, V
2
, V
n
, delimitate dalle
superfici chiuse S
1
, S
2
,S
n
(sarebbero le superfici dei vari conduttori contenute nella cavit). Sotto opportune
ipotesi di regolarit delle superfici, si ha che esiste ed unica una funzione che in V* = V (V
1
V
2
V
n
)
soddisfa allequazione di Laplace:
0
2
2
2
2
2
2
) (


z y x
`
e che inoltre assume il valore nullo in tutti i punti di S (sul grande conduttore), il valore assegnato
1
in tutti i
punti della superficie S
1
, il valore assegnato
2
in tutti i punti della superficie S
2
il valore assegnato
m
in
tutti i punti della superficie S
m
e che su ognuna delle superfici dei rimanenti conduttori assuma un valore
costante (anche se incognito e diverso da conduttore a conduttore) ed inoltre soddisfi alle seguenti
condizioni:
0
1
1

+
m
S
Q
d
m
S
,
0
2
2

+
m
S
Q
d
m
S

0
1

n
S
Q
d
n

S
(queste ultime condizioni esprimono semplicemente, in termini della nostra funzione , il fatto che i
conduttori m+1, m+2n portano le cariche assegnate ed essendo conduttori il potenziale su ognuno di essi
costante)
Vediamo la dimostrazione dellunicit: Supponiamo, per assurdo, che ci possano essere due potenziali (r) e
(r) che differiscono in qualche punto dello spazio ma che coincidano sui primi m conduttori dove valgono
proprio gli m valori
1
,
2
,
m
e che siano compatibili col fatto che i restanti conduttori portino le cariche
assegnate. Consideriamo il seguente integrale
46
[ ] [ ]



n
S S S S S
d

2 1
*
) ( ) ( ) ( ) ( S r r r r
immediato vedere che questo integrale nullo. Infatti, quando integro sui primi m conduttori i cui
potenziali sono noti, e hanno il medesimo valore, sicch la loro differenza nulla, quindi lo pure
lintegrando e dunque pure il risultato dellintegrazione. Quando integro sui restanti conduttori, dei quali
conosco la carica, sia che sono costanti e quindi posso portare fuori dal segno di integrale e quindi scrivo
per il generico i-esimo di questi conduttori:
[ ] [ ] [ ] [ ] 0 ) ( ) ( ) ( ) ( ) ( ) ( ) ( ) ( ) ( ) (
0 0
1

'

'





i i
S Si S
Q Q
d d d
i i
r r S r S r r r S r r r r
Il contributo poi sulla S nullo essendo questa allinfinito
Tenendo conto dunque di quanto si visto e del teorema della divergenza, posso scrivere:
[ ] [ ] [ ] [ ] { }dV d
V S S S S S
n


*
) ( ) ( ) ( ) ( ) ( ) ( ) ( ) ( 0
2 1
*
r r r r S r r r r

tenendo conto che , come si visto precedentemente: a a a + f f f ) ( , si ha:


[ ] [ ] { } [ ] [ ] [ ][ ] { } dV dV
V V

+
* *
) ( ) ( ) ( ) ( ) ( ) ( ) ( ) ( ) ( ) ( ) ( ) ( 0 r r r r r r r r r r r r
Ma osservando che nello spazio =/
0
=, si ha:
[ ] [ ] { } [ ] dV dV
V V


*
2
*
) ( ) ( ) ( ) ( ) ( ) ( 0 r r r r r r
essendo lintegrando una quantit sempre non negativa, se per qualche valore di r fosse positiva, lintegrale
sarebbe positivo. Essendo invece nullo, si deduce che in ogni punto r deve essere:
0
e cio che il campo elettrico univocamente definito. Inoltre, le due funzioni e , avendo medesimo
gradiente, differiscono al pi per una costante additiva, ma questa costante nulla dovendo essere ambedue
le funzioni nulle allinfinito.
Dopo questa ampia digressione, torniamo al problema iniziale. La regione V nella quale vogliamo calcolare
il campo pu essere vista come una semisfera, di centro H e raggio R enorme. A questa regione poi
sottraiamo pure una sferetta centrata in Q. Quindi, pi propriamente, questa la regione V
*
cui si fa
riferimento nel teorema e nella quale vogliamo il campo. Se quindi riusciamo a trovare, in qualunque modo,
una disposizione di cariche (dette cariche immagine), esterne alla regione V tali che, insieme alla carica Q
diano luogo ad un potenziale nullo sulla superficie di tale regione, per il teorema di unicit sopra mostrato, si
ha che il campo E generato da Q e da queste cariche immagine esattamente uguale a quello generato da Q e
dalle cariche indotte da Q stessa sulla superficie del conduttore. Ma dove saranno poste e quanto varranno
queste cariche immagine? immediatamente chiaro che basta, per porre pari a 0 il potenziale sulla superficie
in questione, considerare una sola carica pari a Q e posta in un punto simmetrico a Q rispetto al piano
conduttore. Infatti il potenziale di questa carica in un punto del piano uguale ed opposto al potenziale
generato da Q ed inoltre. La superficie della sfera, essendo infinitamente lontana dalle due, pure a
potenziale 0. Queste due cariche poi generano, in un punto del piano distante r da H un campo la cui
intensit
3 2 2
0 ) (
2
r h
h Q
E
+


e perpendicolare al piano e diretto verso linterno. Ma per E, appena fuori dalla superficie di un conduttore,
si ha: E = /
0
.n con n volta verso lesterno. Confrontando le due espressioni, si ha subito:
3 2 2
) (
2
r h
h Q
+

Problema Quanto varr la carica che si trova su tutta la superficie del semispazio conduttore? (rispondete o
facendo unintegrazione, oppure)
47
Problema: Con quale forza la carica Q attratta dalle cariche indotte che si trovano sul piano? (NON fare
conti!)
Un po di geometria
Determinare il luogo dei punti le cui distanze r
1
ed r
2
da due punti dati A e B sono tali per cui r
1
/ r
2
=
Costante. Questo luogo la cosiddetta circonferenza di Apollonio (da Perge, in Panfilia, da non confondere
con Apollonio Rodio, autore delle Argonautiche. E noto specialmente per un trattato sulle coniche) (relativa
ai due dati punti ed al dato rapporto) con centro sulla retta AB che contiene al suo interno uno dei due punti.
[provate a dimostrarlo. Pu essere utile ricordare che in un triangolo la bisettrice di ciascun angolo divide il
lato opposto in parti proporzionali agli altri due; e, viceversa, se un lato di un triangolo diviso in parti
proporzionali agli altri due, la congiungente del punto di divisione col vertice dellangolo opposto la
bisettrice di questangolo. Questa affermazione si estendono alle bisettrici degli angoli esterni di un
qualunque triangolo (non isoscele)]. Nello spazio il luogo di tali punti sar allora una superficie sferica.
Bene. Consideriamo ora due cariche di segno opposto: Q e Q poste ad una certa distanza tra loro. Qual il
luogo dei punti dello spazio che si trova a potenziale 0? Per quanto visto sopra, una superficie sferica S.
Che racchiude una carica, diciamo Q. Se ora noi, leviamo Q e poniamo dove c S un conduttore sferico,
per il teorema di unicit, nulla cambia nel campo elettrico fuori dalla sfera. Si cos risolto il problema della
determinazione del campo elettrico generato da una carica puntiforme in presenza di una sfera conduttrice
posta a potenziale nullo. ( detto a il raggio della sfera, b la distanza di Q dal centro della sfera, la carica Q
deve essere posta a distanza a
2
/b dal centro ed avere valore Qa/b)E sa la sfera invece di essere messa a terra
isolata, qual il campo?
Una interessante propriet del campo elettrico
Supponiamo di avere un certo numero di conduttori a vari potenziali:
1
,
2
,
m
Nello spazio a loro esterno
si creer un campo elettrico che sar il gradiente della funzione potenziale (r). Ebbene, lenergia del
campo elettrico che si instaura minima. (per la definizione di energia del campo elettrico, vedi pi
avanti) Preciso il senso dellaffermazione: consideriamo uno pseudo campo elettrico, derivante come il
gradiente di una qualunque funzione scalare soddisfacente per alle condizioni al contorno. Ebbene,
lenergia derivante da questo pseudo campo superiore a quella del vero campo elettrico. In formule:

<
Spazio Spazio
dV dV
2 2

Vediamone il motivo. Sia che assumono sui conduttori il medesimo valore, e quindi valgono le relazioni
di sotto:

+
conduttori dei erfici
d
sup
0 ) ( ) ( S
Applichiamo il teorema della divergenza:
[ ] dV d
conduttori dei erfici Spazio

+ +
sup
) ( ) ( ) ( ) ( 0 S
Ora : a a a + f f f ) ( e quindi:
[ ]

dV
Spazio
nullo

'

+ + + ) )( ( ) ( ) ( 0
Essendo = 0, posso scrivere lultima relazione anche come:
[ ]

dV
Spazio
nullo

'

+ ) )( ( ) ( ) ( 0
Da cui:
[ ] [ ] dV dV dV
Spazio Spazio Spazio

) ( ) ( ( ) )( ( 0
2 2 2 2

Quindi, applicando ancora il teorema della divergenza:
48
[ ]
0 ) ( )] ( ) [(
) ( )] ( ) [( ) ( ) ( (
2
0
2 2 2
<


dV d
dV dV dV
Spazio S
Spazio Spazio Spazio



S
e quindi:
dV dV
Spazio Spazio

<
2 2

da quanto detto sopra segue anche subito questo fatto:carichiamo dei conduttori inizialmente scariche ed
isolati. Ebbene, le cariche si dispongono sulle loro superfici in modo che la loro energia elettrostatica
sia la minima possibile (rispetto ad altre distribuzioni di carica sulle superfici).
Infatti, per quanto visto sopra, affinch uno pseudo campo elettrico dia luogo ad unenergia elettrostatica
minore di quella effettiva, occorre che lo pseudopotenziale da cui deriva abbia sulle superfici dei
conduttori un valore diverso da quello reale, ma che sinstauri un simile pseudopotenziale non possibile
in virt dellunicit della soluzione dellequazione di Poisson.
Applicazione del principio esposto sopra
Supponiamo di voler calcolare il campo elettrico tra due conduttori cilindrici, concentrici e carichi, di raggi a
e b. Non sapendo n leggere n scrivere, (in realt, noi, che siamo dotti, sappiamo che il potenziale varia
come 1/r) supporr che il potenziali vari quadraticamente con r, assumendolo nullo sul cilindro esterno e
0
sull interno. Ipotizzo cio:
] ) )( 1 ( 1 [ ) (
2
0
a b
a r
a b
a r
r

+
Lintensit del campo altro non che
0
2
0
) (
) 1 ( 2

a b
a r
a b r

Calcolo lenergia del campo che ( a parte inutili fattori) (H laltezza del cilindro):
H dV
r
V

b
a
dr
a b
a r
a b
r
2
2
) (
) 1 ( 2 2

Ebbene, anche i ciechi vedono che questa quantit dipende dal parametro . Che valore prendere per questo
parametro? Siccome so che questa energia cos calcolata, comunque scelgo sempre maggiore del valore
reale, la migliore approssimazione si otterr prendendo per il valore che minimizza tale energia!. Fatti i
calcoli, viene:
a b
b
+
2 . Quindi il nostro potenziale sar approssimativamente dato da:
] ) )( 2 1 ( 2 1 [ ) (
2
0
a b
a r
a b
b
a b
a r
a b
b
r

+
+ +

+

Due parole (ma solo due) sulle funzioni di variabile complessa
Consideriamo una variabile complessa (w) funzione di unaltra variabile complessa (z). Scriver: w = f(z).
Ponendo w = u+iv e z= x+iy risulta allora u = u(x,y), v = v(x,y). Sia, ad esempio w = z
2
. allora:u+iv = (x+iy)
2
= x
2
-y
2
+2ixy e quindi: u(x,y) = x
2
- y
2
, v(x,y) = xy. Calcoliamo la derivata della f prima muovendoci lungo
lasse x e poi lungo lasse y. C da aspettarsi che le due derivate siano diverse perch dovrebbero
rappresentare le pendenze viste muovendosi lungo una direzione e lungo unaltra. :
z z z
z
z z z
z
z z z
z
z f z z f
z f
z z z z
2 ) 2 ( lim
) ( 2
lim
) (
lim
) ( ) (
lim ) ( '
0
2
0
2 2
0 0
+


E questo
limite risultato indipendente da come z ha teso a zero, ci permette di dire che la derivata esiste (ed
unica). Ebbene, una funzione che sia derivabile in tutti i punti di una regione detta olomorfa o analitica
nella regione. La condizione di olomorfia dunque vuol dire che la derivata unica indipendentemente dalla
direzione seguita da z nel suo annullarsi.Il bello che le funzioni che solitamente sincontrano, cio le
funzioni polinomiali, trigonometriche esponenziali, razionali sono generalmente analitiche. Non l
49
invece la funzione: modulo di z ed altre costruite generalmente con metodi non analitici Vediamo questo
che cosa implica per le funzioni u(x,y) e v(x,y). calcolando la derivata con moto lungo lasse x :
x
v
i
x
u
iv u
x
z f

) ( ) ( ' Calcolando invece la derivata con moto lungo lasse y :


y
u
i
y
v
y
v
i
y
u
i
z f

] [
1
) ( '
Lunicit della derivata impone che sia:

'

y
u
x
v
y
v
x
u
Condizioni di Cauchy-Riemann
Vediamo ora lapplicazione di tutto questo alla Fisica: se f una funzione analitica (olomorfa), dalle
condizioni di Cauchy-Riemann segue subito:
x y
v
y
u
y x
v
x
u

2
2
2 2
2
2
E cio si ha:
0
2
2
2
2

y
u
x
u
Cio la parte reale ( e quindi anche la immaginaria!) di una funzione olomorfa armonica ( vuol dire
che soddisfa allequazione di Laplace bidimensionale)
e poi ancora:
le linee ad u costante sono ortogonali a quelle a v costante
Infatti, facciamo il prodotto scalare tra u e v . Tenendo conto delle condizioni di Cauchy-Riemann si ha
subito che il prodotto scalare nullo, c.v.d. Ma allora , una linea ad u costante (a v costante) pu
rappresentare il profilo di un conduttore bidimensionale, che, come sappiamo, a potenziale costante (che
la nostra u) ed allora le linee a v costante rappresentano le linee di forza del campo elettrico che, come
sappiamo, sono perpendicolari alle linee equipotenziali. Presa allora una qualunque funzione olomorfa, si
risolto il problema del calcolo del campo elettrico per un conduttore bidimensionale. Per esempio,
consideriamo la funzione: w = z
2
vista prima. Troviamo le linee equipotenziali di u(x,y). :
A y x
2 2
Questa lequazione di uniperbole equilatera e per A = 0 si hanno due rette diagonali passanti per lorigine.
Se abbiamo una rientranza ad angolo retto in un conduttore, queste linee rappresentano la nostra rientranza e
quella superficie equipotenziale. Le altre linee continue del grafico forniranno allora altre superfici
equipotenziali e le linee tratteggiate saranno allora le linee di forza
50
Schermo elettrostatico
Consideriamo un conduttore con al suo interno una cavit, di superficie S e di volume V. allinterno di questa
cavit siano sistemate certe cariche in certi punti come pure dei conduttori portanti delle cariche. Nei vari
punti di V ci sar un potenziale (r) ed un campo elettrico elettrico. Sulla superficie S il potenziale sar
costante e il valore sia
(0)
. Supponiamo ora di aggiungere, togliere, spostare delle cariche allesterno del
conduttore. Mi chiedo: che cambiamenti subir il potenziale nei punti di V? Chiaramente sui punti di S sar
ancora costante e quindi varr
(0)
+C, con C costante opportuna e nei punti interni quanto varr? Osserviamo
che la funzione (r) + C soddisfa a tutti i requisiti per poter essere chiamata potenziale nella cavit dopo le
operazioni descritte. Infatti il flusso del suo gradiente sulle sup. dei conduttori, flusso, che legato alle
cariche dei conduttori nella cavit, quello di prima, come deve essere, e su S ha il valore giusto. Ma allora
il campo elettrico nella cavit dopo le operazioni descritte, per il teorema di unicit, il medesimo di prima!
In conclusione:
linterno di una cavit scavata in un conduttore elettrostaticamente indipendente dai campi esterni al
conduttore
(il potenziale cambiato in ogni punto di una medesima costante C, ma questo fatto non avvertibile in
alcun modo. Basti tenere presente che, per sua natura il potenziale definito a meno di una costante
additiva!)
Ma anche
il campo elettrico allesterno di un conduttore non risente di mutamenti del campo elettrico avvenuti
allinterno di una sua cavit per spostamenti di carche allinterno!
E perch? Basta passare al complementare! Mi spiego: un corpo divide lo spazio in due parti: V e V che
sono complementari, nel senso che lesterno di V (V) linterno di V (V)
Morale della favola: se volete fare delle esperienze di elettrostatica essendo si curi di non essere perturbati da
eventuali cariche esterne, messe per caso o a bella posta, da un estraneo, che vanificherebbero le vostre
deduzioni, abbiate cura di eseguirle allinterno di una stanza fatta con pareti conduttrici. Il valore delle
cariche che avete nella vostra stanza influenza il campo elettrico esterno, ma se le spostate per nella stanza,
lesterno non si accorge di questi spostamenti.
Problema Supponiamo di avere un conduttore di forma strana sul quale depositiamo una carica Q.
Sappiamo che questa si distribuisce con maggior densit dove il conduttore pi curvo e sappiamo anche,
per il teorema di unicit che il potenziale, e quindi il campo elettrico e quindi la ( che non altro infatti che

0
E, con E appena fuori dal conduttore) sono tutte grandezze univocamente definite, ma come fare per
calcolarle in pratica? Va subito detto che il problema, tranne casi eccezionali, non passibile di trattazione
analitica, ma solo numerica. Il metodo delle immagini, sopra tratteggiato una risposta che vale per solo in
51
pochissimi casi. Anche il metodo della variabile complessa un metodo di risoluzione, ma vale solo per
problemi bidimensionali e poi una soluzione in cerca di un problema e non il viceversa! Vediamo il
problema in generale pi da vicino:
sia (r) la densit di carica nel punto r del conduttore. In un punto generico dello spazio il campo elettrico
allora dato da:

S
dS
3
0
'
'
) ' (
4
1
) (
r r
r r
r r E

ma se r un punto della superficie del conduttore, noi sappiamo che il campo elettrico :
n E
0
2

(media
tra linterno, che nullo, e lesterno). Quindi ho:

S
dS
3
0 0
'
'
) ' (
4
1
2
) (
) (
r r
r r
r
r n
r

Moltiplicando scalarmene per n, ho:


S
dS
2
'
cos
) ' (
2
1
) (
r r
r r

dove langolo tra il vettore r-r ed il versore normale in r (Equazione integrale di Robin)
Dal punto di vista matematico la relazione scritta unequazione nella funzione incognita (r), unequazione
integrale. Ovviamente la soluzione non unica, ma trovatane una, ogni sua multipla ancora soluzione. Per
definire la soluzione in modo univoco occorre accostarle unaltra che specifichi la carica totale e cio:

S
dS Q ) ' (r
Lequazione integrale si presta assai bene ad essere risolta con un calcolatore mediante il metodo iterativo,
che consiste in questo: si parte con una funzione arbitraria ( ad esempio costante) che indico con
(0)
. Con
questa calcolo

S
dS
2
) 0 ( ) 1 (
'
cos
) ' (
2
1
) (
r r
r r

`
e quindi procedo induttivamente calcolando:

+
S
i i
dS
2
) ( ) 1 (
'
cos
) ' (
2
1
) (
r r
r r

`
i=1,2,
Si dimostra che se la superficie del conduttore incontrata da una generica retta in non pi di due punti, il
procedimento converge ad una funzione che chiamo
*
. Per fare s che la densit di carica trovata sia quella
giusta, occorrer alla fine moltiplicare la
*
per

S
dS
Q
) ' (
*
r
Sistema di cariche e conduttori
Supponiamo di avere n conduttori scarichi ed isolati che occupano i volumi V
1
V
n
limitati dalle superfici
S
1
,S
2
,S
n
.
Depositiamo ora sui vari conduttori delle cariche: Q
1
sul conduttore 1;Q
n
sul conduttore n-esimo. Il primo
conduttore allora si porter al potenziale
1
e ln-esimo al potenziale
n
e queste cariche si distribuiranno
sui conduttori formando delle densit di carica superficiale
i
(r).

Ovviamente, se tutte le cariche sono nulle,
questi potenziali saranno nulli (scelta della costante nella definizione di potenziale). Supponiamo ora di
moltiplicare tutte le cariche per un medesimo fattore . Come si distribuiranno le cariche sui conduttori?
Osserviamo che la densit di carica
i
(r) rende ogni conduttore equipotenziale e lo dota di una carica che
volte la precedente e quindi, per lunicit del potenziale, la distribuzione di carica sar quindi determinata
da
i
(r). Ma questa distribuzione di carica d luogo evidentemente a dei potenziale sui vari conduttori che
sono volte i precedenti. In altre parole. I potenziali dei vari conduttori dipendono linearmente dalle
cariche sui vari conduttori. In formule:
52

'

+ +
+ +
+ +
n nn n n n
n n
n n
Q p Q p Q p
Q p Q p Q p
Q p Q p Q p

2 2 1 1
2 2 22 1 21 2
1 2 12 1 11 1

i potenziali di un sistema di conduttori dipendono linearmente dalle cariche.


Dove p
ij
il potenziale sul conduttore i quando sullo j-esimo deposta una carica unitaria e
nessuna sugli altri
I coefficienti p
i,j
non dipendono dalle cariche, ma solo dalla geometria dei conduttori ed :


n
elemento esimo j
j
n
elemento esimo j
S S S
i
Q
j
S S S
i
i j i
dS
Q
dS p
...
) 0 ... 0 , , 0 , 0 (
0
...
) 0 ... 0 , 1 , 0 , 0 (
0
,
2 1 2 1
) ' (
'
) ' (
4
1
) ' (
'
) ' (
4
1
) ( r
r r
r
r
r r
r
r


dove r
i
un punto del conduttori i-esimo ed il simbolo:
) ' (
) 0 ... 0 , 1 , 0 , 0 (
r
elemento esimo j

denota la densit di carica nel


punto r

quando sul conduttore j-esimo depositata una carica unitaria e zero sugli altri.
Apparentemente p
ij
una funzione di r
i
, dato che tale variabile entra nella sua definizione, ma bene
ricordare che, dato che il potenziale costante su ogni conduttore, p
i,j
costante al variare di r
i
allinterno
del conduttore i-esimo, quindi potr scrivere p
i,j
al posto di p
i,j
(r
i
)e quindi:
iesimo conduttore punto dS p
i
S S S
i
j i
n
elemento esimo j

r r
r r
r
, ) ' (
'
) ' (
4
1
...
) 0 ... 0 , 1 , 0 , 0 (
0
,
2 1


Ora moltiplico ambo i membri per
) ' ' (
) 0 ... 0 , 1 , 0 , 0 (
r
elemento esimo i

ed integro su tutte le superfici. Ho:




n
elemento esimo i elemento esimo j
n
elemento esimo i
S S S S S S
j i
dS dS dS p
...
' '
' ' ) 0 ... 0 , 1 , 0 , 0 ( ) 0 ... 0 , 1 , 0 , 0 (
0
...
) 0 ... 0 , 1 , 0 , 0 (
,
2 1 2 1
) ' ' ( ) ' (
'
) ( ) ' (
4
1
) ' ' ( ) ' ' ( r r
r r
r r
r r

Quando integro sulle varie superfici, p


i,j
costante su tutte queste ed allora posso portarlo fuori dal segno
dintegrale. Resta allora sotto il segno del primo integrale solo
) ' ' (
) 0 ... 0 , 1 , 0 , 0 (
r
elemento esimo i

. Orbene,
questo integrale, essendo lintegrando la densit di carica sui vari conduttori, che sono scarichi tranne li-
esimo che porta carica unitaria, sar perci sempre uguale a 0, quando calcolato sui vari conduttori, tranne
che per li-esimo per il quale fornir 1. Ho dunque:


n
elemento esimo i elemento esimo j
S S S
j i
dS dS p
...
' '
' ' ) 0 ... 0 , 1 , 0 , 0 ( ) 0 ... 0 , 1 , 0 , 0 (
0
,
2 1
) ' ' ( ) ' (
'
) ( ) ' (
4
1
r r
r r
r r

Da questa relazione emerge subito che :
53
p
i,j
= p
j,i
La simmetria di cui godono i coefficienti p
i,j
si pu enunciare dicendo:
Si abbiano due conduttori: se sul primo depositiamo una carica, il secondo viene a trovarsi ad un potenziale
che il medesimo al quale verrebbe a trovarsi il primo se la carica fosse depositata sul secondo
Questa proposizione, che ha avuto bisogno di tutti i passaggi di sopra per essere dimostrata, diventa
tremendamente banale nel caso di cariche e punti:
se in un punto deponiamo una carica, un secondo punto viene a trovarsi ad un potenziale che lo stesso che
si avrebbe nel primo punto se la carica fosse posta nel secondo.
Che non altro che il principio di reciprocit di Gauss-Green
Invertendo la relazione trovata che esprime i potenziali in funzione delle cariche, si scrive subito
unaltrettanto importante relazione:

'

+ +
+ +
+ +
) (
,
) 2 (
2 ,
) 1 (
1 ,
) (
, 2
) 2 (
2 , 2
) 1 (
1 , 2 2
) (
, 1
) 2 (
2 , 1
) 1 (
1 , 1 1
n
n n n n n
n
n
n
n
c c c Q
c c c Q
c c c Q


Il coefficiente c
i,i
detto coefficiente di (auto)capacit del conduttore i-esimo, mentre c
r,s
il coefficiente
dinfluenza del conduttore r sul conduttore s (o del conduttore s su r, dato che, essendo i p simmetrici,
lo sono anche i c, cio si ha):
c
i,j
= c
j,
Problema: determinare il coefficiente di auto capacit di una sfera di raggio R.
Risposta: Il potenziale di una siffatta sfera portante una carica Q , come noto:
R
Q 1
4
0


e quindi
subito:
Per una sfera:
C = 4
0
R
Osservazione:La capacit di un conduttore isolato data, per definizione, dal rapporto tra la carica portata
ed il potenziale a cui si trova. Lindipendenza di tale rapporto dal valore della carica permette di parlare di
capacit di un conduttore anche quando questo scarico.
Difficolt: Il potenziale, come sappiamo, definito a meno di una costante additiva e questa
indeterminazione dovrebbe riflettersi pure nella definizione di capacit! Per definire quindi bene i termini,
stabiliamo di assumere, quando facciamo i nostri ragionamenti teorici, di assumere il potenziale nullo
in punti infinitamente distanti dalla cariche. In questo modo i coefficienti di mutua ed autocapacit
vengono ad essere ben definiti.
Daltra parte, dal punto di vista pratico, portare un conduttore allinfinito un po difficilino, quindi che cosa
si pu fare? Sempre per il motivo per cui il potenziale definito a meno di una costante additiva, potremmo
assumere come potenziale di riferimento il potenziale di un corpo che sappiamo essere costante e cio che,
anche se riceve o cede una certa carica, dal punto di vista pratico in suo potenziale non varii. Un tale corpo
deve quindi avere una capacit enorme ( a rigore infinita, ma la perfezione non di questo mondo!) e quale
potrebbe essere questo corpo? Una sfera di raggio enorme! E qual la sfera di raggio enorme che abbiamo a
disposizione? Ma la terra! La sua capacit, data dalla formula di sopra, pari a
4
10 08 . 7


terra
C farad
(il farad, simbolo F, lunit di capacit, definita come la capacit di quel conduttore che si porta al
potenziale di 1 V se caricato con 1 C dunque ununit enorme!). ma la terra un conduttore? Si, se si ha
54
cura di fare un collegamento ben fatto con ci che vogliamo mettere a terra. Stando cos le cose, capiamo
perch la frase: mettere a terra un conduttore ( che vuol dire collegarlo con la terra) lequivalente di dire:
mettiamo il potenziale del conduttore a 0
Problema determinare il coefficiente di autocapacit, per una lunghezza L, di un cilindro indefinito di
sezione circolare con raggio R
Soluzione: Sia Q la carica e la densit di carica sulla superficie di tale cilindro. Il teorema di Gauss fornisce
subito lintensit del campo elettrico in un punto distante r >R. La direzione poi del campo, per motivi di
simmetria, sar radiale. dunque:
r
R
E
0

il potenziale sar una funzione (r) tale per cui d/dr = E e quindi (r) =
) ln(
0
0
r
r R

=
) ln(
2
0
0
r
r
L
Q


(R) =
) ln(
2
0
0
R
r
L
Q

(r
0
il punto dove il potenziale viene messo a 0). Ma qui sorge un fatto piuttosto
imbarazzante: si era detto sopra infatti che il potenziale viene messo nullo allinfinito, quindi r
0
dovrebbe
dunque essere infinito, ma allora il potenziale sulla superficie del cilindro sarebbe infinito! Questo fatto
anomalo deriva dallaver fatto una supposizione irreale: qualunque cilindro reale ha una lunghezza finita,
la sua carica contenuta in una regione finita ed il potenziale si potr sempre definire nullo allinfinito.
Proseguendo comunque formalmente si avrebbe: C = Q/(r) =
) ln(
2
0
0
R
r
L
Quindi, per quanto detto, non ha senso parlare di propriet di oggetti infinitamente lunghi, se ci di cui si
vuol parlare conduce ad assurdit. necessario quindi considerare il cilindro di lunghezza finita (2L). Il
conto esatto del potenziale per presenta grosse difficolt matematiche, per cui sar necessario fare qualche
approssimazione ( purch non conducano ad assurdi!) Osserviamo a tal fine che, se il cilindro infinitamente
lungo, il campo elettrico al suo interno nullo e quindi ogni punto del suo interno si trova allo stesso
potenziale dei punti della superficie laterale. Possiamo supporre allora che anche nel nostro cilindro finito il
potenziale del punto pi distante dai bordi, cio del suo centro, sia allo stesso potenziale di corrispondenti
punti sulla superficie, che il valore che deve dividere la carica per ottenere la capacit. Vediamo ora il
cilindro costituito da tanti anelli carichi e determiniamo il potenziale in un punto dellasse. Lanello, di
altezza dz, porta una carica dq = 2Rdz. Un punto sul suo asse a distanza z dal piano dellanello ha
potenziale:
)
2R
L
arcsinh(
R 4
Q
)
2R
L
arcsinh(
2
1
2
2
4
1
0 0
2 /
0 2 2
0
2 2
0
L
R
dz
z R
R
dz
z R
R
d
L




+

+

e quindi:
)
2
( arcsin
4
0
R
L
h
L R Q
C


[ ricordare che arcsinh(x) = ln(x+1+x
2
) ]
Condensatori
Consideriamo un conduttore 1 cavo, scarico, di superficie esterna S
2
e superficie interna S
1
. nella cavit di
questo conduttore vi un conduttore 2, di superficie S
0
, che non tocca il primo. Su 2 venga depositata una
carica Q. allora immediato vedere, tramite il teorema di Gauss, che su S
1
viene a crearsi una densit di
carica tale per cui la carica totale che si trova su S
1
Q. si consideri infatti una superficie di forma uguale
ad S
1
ma leggermente pi grande, che contenga S
1
e che venga quindi a trovarsi tutta nel metallo del
conduttore 1. in ogni suo punto E nullo, dato che nel conduttore e quindi il flusso del campo elettrico
nullo. Ma al suo interno c sicuramente la carica Q messa da noi e quindi deve esserci anche una carica Q
che deve allora ovviamente essere distribuita su S
1
. Essendo poi il conduttore 1 scarico, sulla S
2
verr a
trovarsi una carica Q. Se colleghiamo ora a terra il conduttore 1, la carica presente su S
2
lascia il conduttore
55
che resta soltanto con la Q sulla sua faccia interna. Diciamo allora che abbiamo costruito un condensatore
ad influenza totale ( perch ogni linea di forza che parte da un conduttore, finisce sullaltro) al quale abbiamo
comunicato una carica Q. La capacit C di un condensatore definita dal rapporto:
) 2 ( ) 1 (

Q
C `
dove
(1)
e
(2)
sono i potenziali dei due conduttori, od armature. Tale differenza di potenziale si chiama
pure voltaggio e si indica con V e si misura, ovviamente in volt. Lunit di capacit ovviamente il farad.
Problema: esprimere la capacit di un condensatore in termine dei coefficienti c
r,s
Risposta: il conduttore 1 porti una carica Q, mentre il 2 una carica Q. allora:

'

+
+
) 2 (
2 , 2
) 1 (
1 , 2
) 2 (
2 , 1
) 1 (
1 , 1


c c Q
c c Q
e quindi:

'

Q
c c c c
c c
Q
c c c c
c c
21 12 22 11
21 11 ) 2 (
21 12 22 11
12 22 ) 1 (

e allora
22 12 21 11
21 12 22 11
) 2 ( ) 1 (
c c c c
c c c c Q
C
+ + +


`
Problema Sia abbiano due superfici conduttrici cilindriche indefinite, di sezione normale circolare, di raggio
r
1
ed r
2
(> r
1
) disposte coassialmente. Determinare la capacit, per unit di lunghezza, di siffatto condensatore
Problema si abbiano due strati conduttori, ognuno delimitato da due superfici piane parallele ed indefinite. I
due conduttori siano disposti parallelamente. Sulla superficie di uno, per fissare le idee, il superiore, viene
depositata, per unit di superficie, una carica
1
, mentre sullaltro, linferiore, una carica, sempre per unit di
superficie,
2
. Determinare il campo elettrico in tutto lo spazio e le densit di carica
1,1
,
1,2
,
2,1
,
2,2
sulla
quattro superfici.
Risposta: per motivi di simmetria, il campo E sar sempre perpendicolare alle lastre. Al di sopra
dellarmatura superiore valga E
1
; nellarmatura sar 0, fra le due armature sar E
2
, nellarmatura inferiore
sar 0 e sotto linferiore sar E
3
( tali valori indicano le componenti lungo lasse z perpendicolare alle
piastre). Viste le lastre ad una grande distanza, queste si comportano, dato che non riusciamo ad individuare
la loro struttura interna, come uno strato di carica con densit =
1
+
2
. il campo generato da uno strato ha
il medesimo valore da una parte come dallaltra (verso opposto) e vale /2
0
. Quindi avr:
0 2 , 2 0 1 , 1 0 2 1 0 3 1
/ / 2 / ) ( 2 / + E E
Dove le ultime due uguaglianze derivano dalla relazione tra campo elettrico e densit di carica nelle
immediate vicinanze di un conduttore. Deve essere poi
2 2 , 2 1 , 2 1 2 , 1 1 , 1
; + +
e quindi:

'

1 , 1 2 , 2
2 , 1 1 , 2
2 1
2 , 1
2 1
1 , 1
2
2


Detta d la distanza tra le due lastre, la differenza di potenziale vale poi: V =


1,2
d /
0
Se ora
1
= -
2
, si
creato un condensatore piano, la cui capacit, riferita ad una superficie S allora data da:
d
S
C
0
capacit di condensatore piano
Se le due lastre di superficie S non sono inserite nel resto del conduttore, la capacit di questo sistema non
esattamente quella trovata, per effetto dei bordi, (infatti la densit di carica non sar uniforme e quindi non
56
sar pi V=Ed) ma lo sar approssimativamente quanto pi piccolo d rispetto a
S
( non si pu dire:
quanto pi piccolo d rispetto ad S perch non hanno le stesse dimensioni!)
Problema: Mostrare che per un condensatore piano i coefficienti c
i,j
, valgono:
S
d
c c S
d
c c
0
1 , 2 2 , 1
0
2 , 2 1 , 1


Risposta: Partiamo dalla relazione fondamentale:

'

+
+
) 2 (
2 , 2
) 1 (
1 , 2
) 2 (
2 , 1
) 1 (
1 , 1


c c Q
c c Q
Si gi visto che i c sono
simmetrici, cio: c
1,2
= c
2,1
= y. Daltra parte, sempre per simmetria, c
1,1
= c
2,2
= x. Quindi la relazione di
sopra si scrive:

'

+
+
Q y x
Q y x
) 1 ( ) 2 (
) 2 ( ) 1 (


e quindi risolvendo:
d
S
c c y
d
S Q
c c x
0 1 , 2 2 , 1 0
2 1
2 , 2 1 , 1
;


Energia di un sistema di cariche
Supponiamo di portare n cariche puntiformi q
1
, q
2
, q
n
, tutte inizialmente a distanza reciproca infinita, fino
alle posizioni r
1
, r
2
,r
n
Per spostare la prima carica non occorre evidentemente alcun lavoro, ma per
spostare la seconda e le successive, s, dato che occorre spostarle in un campo elettrico creato dalle
precedenti gi allocate. Ci si propone ora di valutare tale lavoro.
Osserviamo che, per portare una carica q
2
, inizialmente a distanza infinita da una carica q
1
, ad una distanza
reciproca r
12
, occorre fornire un lavoro
12
2 1
0
12
4
1
r
q q
L

che chiamo: lavoro di assemblaggio della coppia


(1,2). Supponiamo ora di aver gi collocato m cariche al loro posto, avendo compiuto un lavoro L.
Collochiamo ora al suo posto la carica successiva. Per fare ci, occorre compiere del lavoro contro il campo
dovuto alla carica 1, 2, m. Il lavoro totale in questa fase quello di assemblaggio della coppia (1,m+1),
pi il lavoro di assemblaggio della coppia (2,m+1)+ pi il lavoro di assemblaggio della coppia (m,m+1).
Ne consegue allora che il lavoro totale per sistemare tutte le cariche quello di assemblaggio di tutte le
coppie. Se calcoliamo

n
j i
j i
L
1 ,
, conteggiamo effettivamente tutte le coppie, ma ognuna di queste viene
conteggiata du volte( ad esempio compare L
37
e L
73
) mentre dovrebbe essere conteggiata una volta sola. Si
rimedia a ci diveidendo semplicemente al somma per 2. Ancora unosservazione: nella doppia sommatoria
compaiono anche addendi con indici uguali che sarebbero infinito! Occorre allora escluderli dalla
sommatoria. Per il lavoro totale di assemblaggio di n cariche posso dunque scrivere:


n
j i
j i j i
j i
n
j i
j i
j i totale
r
q q
L L
1 , 0 1 ,
8
1
2
1

Osservo ora che la doppia sommatoria pu essere eseguita nel seguente ordine:

,
_


n
i
n
i j
j j i
j
i
n
j i j i
j i
n
j i
j i totale
r
q
q
r
q q
L L
1 1 0 1 , 0 1 ,
8
1
8
1
2
1

Ma il termine tra parentesi altro non che il potenziale generato nel punto dove si trova la carica i da tutte le
altre e quindi ho:

,
_


n
i
i i
n
i
n
i j
j j i
j
i totale
q
r
q
q L
1 0 1 1 0
8
1
8
1

Nel caso in cui le cariche costituiscono un sistema continuo, la formula di sopra diventa:
57

,
_


Spazio Spazio Spazi
totale
d d d continuo sistema L r r r r r
r r
r
r
3 3 3
0
) ( ) (
2
1
'
'
) ' (
4
1
) (
2
1
) (

dove (r) il potenziale nel punto generico r generato dalla distribuzione di carica stessa.Se le cariche, dalla
loro posizione attuale, si riportano allinfinito, in tale processo forniscono il lavoro L. quindi tale espressione
pu a buon diritto ritenersi come una forma di energia (energia elettrostatica) che indico con U, accumulata
nelle cariche.
Si pu dunque scrivere:


spazio lo Tutto
dV u U L ) (r
con u(r)=-(r)(r)/2
Quanto sopra scritto si lascia interpretare dicendo che lenergia totale si pu vedere come una somma ( tale
il significato infatti di un integrale) di tanti termini, ognuno contenuto in un elemento dV il cui valore u.
quindi, a buon diritto u(r) si lascia intendere come densit di energia elettrostatica. Secondo tale
interpretazione, l dove non c carica (e/o non c potenziale) non c neppure energia elettrostatica.
Del resto il ritenere che dove non ci sono cariche non ci sia energia elettrostatica, pu parere naturale.
Sempre secondo linterpretazione di sopra, lenergia che si dovuta spendere per formare la data
distribuzione del campo, risiede dunque nelle varie cariche secondo la formula scritta sopra. Se si hanno
dunque due cariche, Q e q il lavoro compiuto per portarle da distanza reciproca infinita ad una distanza r,
lavoro che sappiamo essere pari a KQq/r e che misura lenergia elettrostatica accumulata, deve essere visto
come:
U = Q (generato dalla carica q nel punto dove c la carica Q)/2+ q (generato dalla carica Q nel punto
dove c la carica q)/2
E cio:
U = KQq/(2r) + KqQ/(2r)
E lespressione trovata giusta!
Il fatto importante per da notare che lespressione:

spazio lo tutto
dV ) ( ) (
2
1
r r
, mentre fornisce il valore esatto
del lavoro compiuto e quindi dellenergia elettrostatica accumulata, non vincola obbligatoriamente a
ritenere u come espressione della densit di energia elettrostatica. In altri termini, possibile ( e presto lo
far!) manipolare matematicamente lespressione ottenendo un integrale, eseguito sempre su tutto lo spazio,
col medesimo valore di prima, il cui integrando per sia diverso (laffermazione ovvia a priori: se due
integrali fatti su una certa regione sono uguali, non detto che gli integrandi siano uguali!)
Cio posso avere:


spazio lo tutto spazio lo tutto
dV u dV u ) (
~
) ( r r
con ) (
~
) ( r r u u
Ho detto: accumulata nelle cariche. Ma questa energia forse posseduta parzialmente dalluna carica e
parzialmente da unaltra? No, una carica sempre la stessa, sia che sia in presenza di altre che sola; non
possibile, esaminandola sia pur attentamente, scorgere in lei alcunch che possa farci subodorare che abbia
talora una maggiore e talora una minore energia. chiaro che questa energia, pari a L, appartiene alla
struttura, delle cariche, alla loro disposizione reciproca, oltre che, ovviamente, dal loro valore. Dovr quindi
essere esprimibile in termini di un qualcosa univocamente associato a questa struttura. E che cosa c di
univocamente associato ad una struttura di cariche? Ma il campo elettrico creato da queste! Ed possibile
esprimere tale lavoro L in termini del campo elettrico generato dalle cariche? Certo! quello che faccio ora.
Essendo infatti: E = /
0
, sostituendo ho:


spazio lo tutto spazio lo tutto
dV dV L E r r r ) (
2
) ( ) (
2
1
0


58
Ora risulta, che se E un generico campo vettoriale e un generico campo scalare, si ha (per provarlo basta
eseguire le derivazioni)
E E E ) (
Ottengo quindi:
[ ]


spazio lo tutto spazio lo tutto
dV dV L E E E r

) (
2
) (
2
0 0
Ma essendo E = -, ho:
[ ]

+
spazio lo tutto
dV L E E E) (
2
0

Applicando il teorema della divergenza, ho:



+
spazio lo tutto inito all erficie
d dV L
inf ' sup
0 0
) (
2 2
S E E E

Dove la superficie allinfinito unimmaginaria superficie che racchiude tutto lo spazio.
Vediamo pi da vicino questultimo integrale. La superficie immaginaria che racchiude tutto lo spazio pu
essere pensata come una sfera di raggio R (che poi si immaginer infinito) che contiene tutta la carica
delluniverso ( indicata con Q) Il potenziale allora varr sulla superficie di tale sfera:
R e
Q
0
4

ed il
campo elettrico:
3
0
4 R
Q R
E

e quindi
4
R
R
E ed allora
R
d
d
R
d

S
R
S E
4

Integrando su tutto langolo solido ottengo un qualcosa di proporzionale a 1/R Ora, perch la mia sfera possa
contenere tutta la carica delluniverso, il suo raggio deve andare allinfinito, ma allora lintegrale zero!
Si ha dunque:


spazio lo tutto
dV L E E
2
0

Che la formula che si voleva ottenere. Lei suggerisce di considerare unenergia associata al campo elettrico
E con una densit di energia pari a:
E E
2
0

u
La formula trovata, associando al campo unenergia, fornisce concretezza fisica al campo elettrico che,
ricordiamolo, fino ad ora era stato introdotto come un puro flatus vocis per evitare la poco accettabile idea di
unazione a distanza.
La formula trovata stata dedotta nel caso del campo elettrostatico, ma, anticipando le cose, si vedr che
vale anche nel caso generale di campo non statico.
Per una deduzione leggermente diversa, vedi Becker pag. 110
Esercizio: Determinare, servendosi della formula per la densit denergia elettrostatica in termini del campo,
la pressione elettrostatica che vige sulla superficie di un conduttore carico, gi ottenuta precedentemente con
altro ragionamento.
Risposta: Supponiamo che una superficietta infinitesima del conduttore,sotto la pressione elettrostatica, si
sposti verso lesterno di uno spostamento infinitesimo dx. Per fare s che questa trasformazione avvenga
passando per stati di equilibrio, eserciteremo dallesterno una pressione verso linterno del conduttore pari
alla pressione elettrostatica (a rigore, di un infinitesimo inferiore perch possa avvenire questa
trasformazione) In questo modo noi, raccogliamo lavoro. Quanto prescisamente? La forza p dS, lo
spostamento dx e quindi il lavoro raccolto dL = pdSdx. Ma dSdx altro non che il volumetto infinitesimo
dV del cilindretto di base dS ed altezza dx che viene creato in questa deformazione del conduttore, quindi
dL=pdV. Ora, in questo volumetto, prima della deformazione, essendo estremamente prossimo alla
superficie del conduttore, era presente un campo E il cui valore dato, come si visto sopra, da: E = /
0
.
Lenergia elettrostatica che era contenuta in lui era dunque data da udV, con u =
0
E
2
/2 =
0
(/
0
)
2
/2 =
2
/2
0
.
Dopo la deformazione il volumetto in questione viene a trovarsi allinterno del conduttore, dove il campo
nullo e quindi lenergia elettrostatica diminuita della quantit
2
/2
0
dV Questa energia stata raccolta
59
dalla mia mano che ha ricevuto, come si visto unenergia pari a pdV. Uguagliando le due quantit e
sopprimendo il dV comune si ottiene: p=
2
/2
0
Esercizio: calcolare il lavoro necessario per costruire una sfera di carica di raggio R e densit costante .
Risposta: chiaro che occorrer compiere un lavoro positivo perch, costruita una sfera di raggio minore, per
accrescerla occorrer vincere la repulsione elettrostatica tra la nuova carica che si porta e la gi costituita. Per
calcolare tale lavoro, potremo usare la formula della densit di energia ed integrare su tutto lo spazio. Il
campo elettrico generato da una sfera carica con carica Q :

'

<
>

R r se
R
r Q
R r se
r
Q
3
0
2
0
4
1
4


E
Quindi:
+

Sfera Esterno Sfera spazio lo tutto
dV
r
Q
dV r
R
Q
dV
4
2
0
0 2 2
3
0
0 0
1
)
4
(
2
)
4
(
2 2



E E
Essendo: dV = 4r
2
dr, ho:
+ +

R
Q R
R
Q
dr
r
Q
dr r
R
Q
R
R
1
4 )
4
(
2 5
4 )
4
(
2
1
4 )
4
(
2
4 )
4
(
2
2
0
0
5
2
3
0
0
2
2
0
0
0
4 2
3
0
0

R
Q
R
Q
R
Q
0
2
0
2
0
2
20
3
8 40
+
Questo lavoro misura dunque anche lenergia accumulata nella nostra distribuzione di carica. Ma, secondo
questa formula, si nota un fatto importante: la carica elementare non pu essere puntiforme! Infatti
lespressione trovata per lenergia diverge se R 0. Beh, direte voi, lidea di un qualcosa di puntiforme
solo unastrazione matematica; in natura nulla puntiforme. Quando noi nei nostri conti supponiamo
qualcosa come puntiforme, solo perch le dimensioni in gioco sono molto maggiori di quelle del corpo.
Benissimo, ma allora sorge questo problema: come fanno le parti dellelettrone a stare insieme? Sorgono
forse delle altre forze, di natura non elettrica? La presenza di tali forze, studiate in dettaglio da Poincar,
solleva problemi a non finire. Forse che le legge dellelettrodinamica non valgono a dimensioni del raggio
dellelettrone? Pare di si. Comunque il raggio dellelettrone si pu stimare con il seguente ragionamento:
come tutti sanno, ad una massa m corrisponde (e viceversa) unenergia data da: E = mc
2
Supponendo che la
massa dellelettrone sia tutta di origine elettromagnetica, il suo raggio (se si immagina lelettrone una sfera
carica con densit uniforme) si pu stimare ponendo:

2
0
2
2
0
2
20
3
20
3
c m
Q
R c m
R
Q
e
e

1.69 10
-13
cm
Questo valore (od un suo prossimo che pu variare a seconda di come si immagini distribuita la carica
allinterno della sfera o se anche la forma stessa non propriamente sferica) detto raggio classico
dellelettrone e determina i limiti dellapplicabilit dellelettrodinamica classica. Bisogna per tener presente
che in realt tali limiti sono ancora pi ristretti per lesistenza dei fenomeni quantistici che entrano in gioco
per distanze dellordine di cm
mc
10
10 4 . 2


h
Altro modo di risolvere il problema: supponiamo di costruire la nostra sfera portando via via sulla sfera che
si gi formata, delle croste sferiche di spessore infinitesimo. Calcoliamo il lavoro per portare dallinfinito
una siffatta crosta e sommiamo su tutte le croste. Procediamo: quando la sfera nel suo accrescersi giunta ad
avere raggio r (< di R), il potenziale da lei generato sulla sua superficie lo stesso di quello che avremmo se
tutta la carica fosse concentrata nel suo centro e quindi :
r
r r
carica
1
3
4
4
1
) (
3
0




60
La carichetta dq che si porta ora, avendo forma di crosta sferica, si scriver: dr r dq
2
4 e quindi il lavoro per
portarla al suo posto, sar:
dr
r
dr r
r
dq r dL
0
4 2
2
0
2
3
4
4
3
) (


Il lavoro totale si otterr integrando tra 0 e R ottenendo cos:
L=
R
Q
R dr
r
dL
R
2
0
5 2
0 0 0
4 2
20
3
15
4
3
4




Altro modo di risolvere il problema: applicando direttamente la formula:

spazio lo tutto
dV L ) ( ) (
2
1
r r
Calcoliamo il potenziale generato dalla sfera in un generico punto distante r dal centro della sfera:
R
Q r R
dr
r
Q
dr
r
r
d d d r
R
R
r R
R
r r
1
4 6
) ( 1
4 4
3
4
) (
0 0
2 2
2
0
2
3
0

+ +


r E r E r E
Scrivendo dQ = 4r
2
dr, ho:

dq r) (
2
1

Problema: Lespressione :


spazio lo tutto
dV E E
2
0

chiaramente sempre maggiore di 0, mentre lespressione:


n
j i
j i
j i
j i
q q
1 , 1
0
8
1
r r
o la sua analoga nel caso continuo, da cui derivata, possono benissimo essere pure
negative! Come mai questa differenza? (vedi Becker pag. 109-110)
Esercizio: Un condensatore di capacit C porta una carica Q (unarmatura Q e laltra Q) Calcolare il lavoro
compiuto per caricare il condensatore.
Risposta: partiamo dalla formula:

spazio lo tutto
dV L ) ( ) (
2
1
r r
Lintegrazione su tutto lo spazio si riduce
allintegrazione sulle due armature, dato che fuori non c carica. Ma su ogni armatura il potenziale
costante e vale
1
sulluna e
2
sullaltra. Quindi :
2
2
2 1
2
2
1
1
2
1
2
1
2
1
) (
2
1
) (
2
1
) ( ) (
2
1
CV
C
Q
QV Q Q dV dV dV L
armatura armatura spazio lo tutto
+

r r
Esercizio: Consideriamo un condensatore a lastre piane e parallele, carico con una carica Q. con che forza si
attirano le armature?
Risposta Trascurando gli effetti ai bordi, la carica si trova tutta sulla faccia interna, dando luogo ad una
densit superficiale . Ma sappiamo allora che presente una pressione elettrostatica p
e
=
2
/2
0
e quindi la
forza sar: f = p
e
S = S
2
/2
0
. Altro ragionamento: le due cariche sono due strati di carica. Il campo generato
da uno strato : E = /2
0
. La carica dellaltra armatura immersa in tale campo e quindi risente di una forza
f = QE = Q /2
0
Macchine elettrostatiche vedere gli appunti scannerizzati a pag. 44
Condensatori in serie ed in parallelo
un semplice esercizio vedere che due condensatori di capacit C
1
e C
2
messi in parallelo ( le due armature
di un condensatore sono collegate elettricamente alle due dellaltro) sono equivalenti ad un unico
condensatore di capacit C data da:
C = C
1
+ C
2
un semplice esercizio vedere che due condensatori di capacit C
1
e C
2
messi in serie ( unarmatura di un
condensatore collegata elettricamente ad una dellaltro e le due restanti portano una carica Q e -Q) sono
equivalenti ad un unico condensatore di capacit C data da:
61
2 1
1 1 1
C C C
+
Esercizio: Supponiamo di avere n cariche puntiformi fisse nello spazio. Il campo da esse generato si
annuller (forse) in certi punti dello spazio i quali certamente non coincidono con la posizione di alcuna
carica. La nullit del campo vuol dire che una carica ferma messa in questi punti resta nel posto, dato che
non c forza che la mette in moto. Ci vogliamo chiedere se qualcuno di questi punti dove sia annulla il
campo e dove non c carica, pu essere punto di equilibrio stabile. Un punto di equilibrio stabile se,
ovviamente di equilibrio ed inoltre se, allontanata di sufficientemente poco in direzione qualsivoglia un
carica, su di lei sorge una forza diretta verso la primitiva posizione che tende dunque a riportare la carica
nella posizione di equilibrio. Pensiamo ad una pallina in fondo ad una buca: comunque si sposti la pallina
( purch per di poco), essa viene sospinta verso il punto pi basso dove era prima e quindi il fondo di una
buca un punto di equilibri stabile. Non cos invece la sommit di un colle: se noi spostiamo la pallina, non
lungo la cresta del colle, ma nella direzione di un versante, anche se la spostiamo di pochissimo, la pallina
cade e si allontana sempre pi dalla sua posizione primitiva. Ebbene, tornando al quesito si pu dire che:
nessun punto di un campo elettrostatico dove il campo nullo di equilibrio stabile
Infatti, per assurdo, ammettiamo O punto di equilibrio stabile per una carica positiva (negativa). Considerata
allora una superficie sferica di centro O e raggio sufficientemente piccolo, in ogni punto di questa il campo
sarebbe diretto verso il centro ( nella direzione di r). ma allora il flusso di E attraverso questa superficie
sarebbe diverso da 0 e la qual cosa implicherebbe, per la prima equazione di Maxwell in forma integrata, la
presenza di una carica allinterno, cosa esclusa.
Stando cos le cose,ci si pu chiedere come mai la materia, che in ultima analisi altro un insieme di cariche,
possa essere stabile.
Il quesito veramente fondamentale, nel senso che per rispondere adeguatamente, non sufficiente la Fisica
che si studiata fino ad ora, ma occorre ricostruirla dalle fondamenta, costruendo cos quella Fisica detta
moderna che si andata costruendo nella prima met del 900 e che si articola nella Meccanica quantistica
e nella teoria della relativit. La Fisica moderna molto pi difficile a comprendersi di quella che si studia in
questo corso, sostanzialmente perch tratta di aspetti della realt che sono estremamente lontani dalle nostre
concezioni, opinioni, intuizioni che ci siamo formati nel nostro mondo che sostanzialmente caratterizzato
da corpi delle dimensioni dal micron in su e da velocit piccole rispetto a quella della luce. Ma quando si
indagano fenomeni del microcosmo, concetti quali quello di traiettoria di una particella che sono cos
chiari alla nostra intuizione, svaniscono e perdono di senso. Al loro posto sorgono nuovi concetti, alla nostra
mente estremamente nebulosi, quali quello di funzione donda complessa in uno spazio di Hilbert ad infinite
dimensioni ecc. ecc. ma allora, si dir la Fisica viene meno al suo compito principale che quello di
spiegare il mondo? Si e no: per far andare bene le cose basta porre come fine della Fisica uno meno
ambizioso: il fine quello di costruire una teoria coerente che possa permettere di fare delle predizioni su
esperimenti che poi siano confermate. Che i suoi concetti base ci siano pi o meno nebulosi non importa.
La Fisica moderna, con le sue astruserie astratte ed antiintuitive, e con le sue splendide previsioni
puntualmente verificate assolve a pieni voti il compito che le stato additato.
Ritornando al problema della stabilit della materia, prima di dichiarare che non dovuta alla nostra
debolezza nel manipolare bene le leggi della meccanica e dellelettromagnetismo lincapacit di spiegare la
stabilit della materia, ma che sono le leggi stesse che usiamo impotenti a comprenderla nella foresta delle
deduzioni che da loro si possono trarre, cercher di abbozzare qualche tentativo di spiegazione.
Come prima cosa si potrebbe giustamente obiettare che la mancanza di stabilit si ha in un campo
elettrostatico, ma le carche dal punto di vista microscopico sono in moto vorticoso: la teoria cinetica della
materia vanta antiche tradizioni!. Ebbene, se consideriamo allora le equazioni pi complete, che trattano le
cariche in moto, si vede che la situazione ancora peggiore! Essendo le cariche in moto, si potr parlare di
equilibrio dinamico. La terra non in equilibrio rispetto al sole, ma la sua orbita si, nel senso che essa non
cambia ( sempre la medesima ellisse). Per le cariche in moto, anche un equilibrio dinamico inaccettabile:
una carica accelerata irraggia energia e quindi un elettrone ruotante intorno ad un nucleo perde energia ed in
una frazione infinitesima di secondo precipita nel nucleo. In conclusione: le leggi complete
dellelettromagnetismo non solo non risolvono il problema, ma anzi lo acuiscono.
In secondo luogo si pu far presente che, oltre alla forza elettrica, possono intervenire altre forze tra la
particelle e che la stabilit dovuta allazione di queste. In effetti, se una carica in moto, su di lei pu agire
anche unaltra forza, oltre allelettrica: la cosiddetta forza magnetica che verr analizzata pi avanti nel
corso, ma si dimostra che lei non pu fornire una spiegazione.
62
Si potrebbe allora avanzare lipotesi che la materia sia effettivamente instabile; se noi la crediamo stabile
solo perch i tempi di un suo decadimento sono talmente lunghi che noi non ce ne accorgiamo. Al
contrario, si risponde, se il mondo fosse retto dalla leggi della meccanica e dellelettromagnetismo
classico, sarebbe sparito dopo il suo sorgere nel volgere di un amen.
Voglio ora limitarmi a dare solo una vaga idea di come la nuova meccanica possa spiegare questo problema
partendo da un suo principio controintuitivo. Si tratta del principio di indeterminazione di Heisenberg che
ora illustro intuitivamente.
Come ben noto, la misura di ogni grandezza suscettibile di variare con continuit, affetta da un errore. La
fisica classica ammette per che, a prezzo di enormi fatiche e spese, magari anche in un tempo futuro,
lerrore che si commette nella misura pu essere reso piccolo a piacere, anche se mai nullo. E su questo fatto
anche la nuova meccanica, cio la meccanica quantistica daccordo. Per lei per sorge un fatto nuovo e
precisamente osserva che esistono coppie di grandezze, dette coniugate che hanno questa propriet: che
anche teoricamente impossibile, oltre che di fatto, escogitare un apparecchio che riesca a misurare
contemporaneamente le due grandezze coniugate in modo tale da rendere piccolo a piacere lerrore di misura
delluna e dellaltra. In altri termini, se vogliamo misurare luna grandezza con un errore stratosfericamente
piccolo, possiamo farlo, ma questo strumento, non importa come fatto, perturba necessariamente laltra
grandezza ed in modo non esattamente prevedibile, sicch una misura di questa non pu essere eseguita con
un errore piccolo a piacere. Esempio tipico di una coppia di grandezze coniugate, sono la coordinata x (o y o
z) di un punto materiale e la componente rispetto ad x (o y o z) della sua quantit di moto. Detto x e p
x
lerrore nella misura della coordinata e della quantit di moto (indicata con p) lungo la coordinata, sia ha
limportantissima relazione, la cui scoperta risale ad Heisenberg, detta: principio dindeterminazione:
2
h
x p
x
La grandezza h una costante fondamentale della fisica. stata introdotta allinizio del 900 dal famoso
fisico Plank ed in suo onore si chiama: costante di Plank ridotta. Nel sistema MKS vale:
-34
10 1.054 h
Lestrema piccolezza di questa costante fa s che, nel caso di fenomeni macroscopici questo vincolo
praticamente non sussista e quindi la meccanica classica (che tratta oggetti macroscopici) procede come se la
costante di Plank fosse nulla e quindi ritiene che una grande precisione nella misura di una grandezza non
limiti la precisione della misura di unaltra.
Naturalmente linterpretazione che si data della formula e cio che i due delta siano i due errori sulle
misure, uninterpretazione molto alla buona;occorrerebbe infatti, se si vuole stabilire una stretta
disuguaglianza, definire esattamente che cosa sintende per x e
x
p perch, bene precisarlo, non sono
esattamente lerrore nel senso che si visto nella teoria degli errori, cio il valore assoluto della differenza
tra la misura di una grandezza ed il suo valore vero. Comunque la definizione precisa non ci interessa per i
nostri scopi.
Veniamo ora alla questione della stabilit della materia ed in particolare dellatomo didrogeno costituito da
un protone e da un elettrone. Il problema : come pu essere stabile questa struttura? Perch lelettrone non
cade sul protone? A tal fine osserviamo che, dato che la massa del protone di oltre 1800 volte maggiore di
quella dellelettrone, si pu immaginare il protone fermo (o animato di moto rettilineo uniforme) e che
intorno a lui si muova lelettrone ( deve essere in moto per non cadere sul protone!) Qualcuno, che conosce
la meccanica ma non lelettrodinamica, potrebbe notare dare la seguente semplice risposta: lelettrone si
muove in un campo di forze centrale, essendo la forza colombiana ed il protone assimilabile ad un punto..
Ma noi sappiamo che in un campo centrale si conserva il momento della quantit di moto rispetto al polo. Se
lelettrone dovesse collidere sul protone, nel momento della collisione si annullerebbe il momento della
quantit di moto, perch il raggio vettore sarebbe nullo (questo ragionamento si presta in realt a qualche
critica). Dovendo il momento della quantit di moto conservarsi, non ci pu essere collasso. Il fatto per
che le equazioni complete dellelettrodinamica prevedono che una carica accelerata irraggi energia nello
spazio sotto forma di onde elettromagnetiche. Irraggiando energia, come se venisse frenata ed allora
chiaro che alla fine cade nel nucleo. Ed i conti mostrano che questa fine avviene in un batter docchio.
Vediamo ora come Heisenberg salvi la stabilit. Se lelettrone ad una distanza r dal nucleo, la sua
energia totale, cio cinetica pi potenziale elettrica :
r
q
m
p
r
q
mv E
totale
2
0
2 2
0
2
4
1
2 4
1
2
1


63
Ora : p p r r , e dalla relazione di Heisenberg
2
h
p r si ha:
r
q
r m r
q
m
p
E
totale
4

,
_

2
0
2
2
0
2
1
2 2
1 1
2
) (

h
La formula implica che lelettrone non pu cadere sul nucleo (vorrebbe dire r=0): infatti la funzione di
destra tende allinfinito se r tende a 0, ma questo assurdo perch lenergia del sistema sicuramente
limitata!
interessante trovare per quale valore di r lenergia totale minima. Derivando rispetto a r e ponendo
uguale a 0 la derivata, si ottiene:
m q
r
2
2
0
4
8
1 h

Sostituendo i valori, si ottiene: . ) 10 5 . 0 (
8
1
10
m r

cio 1/8 del raggio dellatomo didrogeno
Un riesame retrospettivo -------------RIVEDERE------------------
Si detto che lesponente nella legge di Coulomb vale -2 con una precisione meravigliosa. Tale precisione si
ottenuta con esperimenti molto indiretti: sostanzialmente osservando che se un conduttore carico A, si tocca
la parete interna di una cavit di un secondo conduttore B che contiene il primo, A si scarica completamente.
Laffermazione segue dal teorema di Gauss che vale solo se lesponente proprio 2. Se lesponente nella
legge di Coulomb fosse anche solo lievemente diverso da 2, il conduttore interno dopo il contatto non
sarebbe completamente scarico. Siccome la neutralit di un corpo si pu accertare con precisione, ecco una
precisa conferma al valore dellesponente. Che si possa determinare con precisione lesponente con una
misurazione diretta della forza, una pia illusione, anche perch la legge vale per corpi puntiformi e il
ritenere una dato corpo come puntiforme non che unapprossimazione. Ne consegue dunque che la forza
elettrica tra due corpi carichi le cui dimensioni non siano completamente trascurabili rispetto alla loro
distanza, seguir solo approssimativamente la legge di C e tanto meglio quanto pi grande sar la loro
distanza. Siamo quindi indotti a scrivere, per il campo elettrico ( o per il potenziale) in un punto, lontano ma
non troppo da delle cariche che lo creano:
+ + +
3 2
r r r
Q
totale

il problema stimare questi coefficienti. Nella formula il primo termine predomina decisamente sul secondo
al crescere della distanza ed il secondo sul terzo. Quindi il secondo, il terzo e gli altri si potranno trascurare,
ma in un caso per il secondo termine ha unimportanza notevole: precisamente quando la carica totale
nulla! Ma se nulla il campo non dovrebbe essere nullo? A rigore non detto!. Considerate infatti due
cariche puntiformi di valore opposto. Viste da lontano, sono un corpo con carica nulla, ma un campo , anche
se debole, c. Consideriamo un punto P infatti sulla retta passante per le due cariche. sso ha distanza
diversa dai due punti e quindi il campo generato da una carica non annulla rigorosamente il campo generato
dallaltra!. Ebbene, il nostro compito di andare a stimare il potenziale in un punto generico P a distanza
grande (ma non troppo!) rispetto alla distanza che separa le due cariche.
Sia dunque la carica -Q nel punto -dr/2 e la carica +Q nel punto +dr/2 (sicch le due sono separate dal
vettore dr) e vogliamo il potenziale nel punto R.
Con tutto rigore :
1
]
1

2 / 2 / 4
1
) (
0
r R r R
R
d
Q
d
Q

Ora : 4 / ) ( ) 2 / ( ) 2 / ( 2 /
2 2
2
dr d R d d d + + + + + r R r R r R r R . Analogamente:
4 / ) ( ) 2 / ( ) 2 / ( 2 /
2 2
2
dr d R d d d + r R r R r R r R e quindi:
64
R
r
r R
r R r R
r R r R
r R r R
r R r R
R

1
1
1
1
]
1

1
1
1
1
]
1

1
1
]
1

+ +

1
]
1

2
2
2
2 2
2 2
2 2 2 2
0
] ) ( 1 [
1 1
1
1 1
1
4 / ) ( 4 / ) (
2 / 2 /
) ( 4
R
Q
R
d
R
d
Q
R
d
Q
R
R
d
Q
R
d
Q
R
dr d R
Q
dr d R
Q
d
Q
d
Q

Dove si sono tralasciati gli infinitesimi dordine superiore e si usata lapprossimazione:
2 / 1 1 + +
Valida per piccino. Per il potenziale di dipolo possiamo dunque scrivere
3
0
4
1
) (
r
dipolo
r p
r

Facendo il gradiente, col segno cambiato, si ottiene per il campo elettrico di dipolo:
]
) ( 3
[
4
1
3 5
0
r r
dipolo
p r r p
E


Esercizio due dipoli, di momento p
1
e p
2
solo allineati ad una distanza reciproca z. trovare la forza
dinterazione.
Risposta: Se sono messi: -,+,-,+ oppure +,-,+,- la forza sar di attrazione, altrimeti di repulsione. Per quanto
riguarda lintensit, si ha che il campo E generato dal dipolo 1 esercita, sulle due cariche del dipolo 2 una
forza la cui intensit data da:
2 ) 1 ( 2 ) 1 (
) ( ) (
dipolo dipolo dipolo dipolo
q d z q z f + E E
Dove con d si indicata la distanza reciproca delle cariche del dipolo 2.
Ottengo quindi:
4
0
2 1
2
3 3
0
1
6
4
]
) (
2 2
[
4 z
p p
q
d z z
p
f
dipolo


+

Equazione di continuit
Con un analogo ragionamento si pu mostrare che la carica indistruttibile, cio non pu annichilirsi n
crearsi n variare a seconda del suo stato di moto. Infatti, per assurdo, supponiamo che una certa carica Q,
presente al tempo 0 in un certo volume V possa, al tempo t
1
risultare aumentata ( o ridotta) di una quantit q
senza che sia avvenuto del passaggio di carica attraverso la superficie che racchiude il volume (il verificarsi
di questo fatto significherebbe che la carica potrebbe crearsi o distruggersi). Consideriamo una sfera con
centro in un punto del volume V e di raggio R tanto grande che la luce, partita dal centro al tempo 0, non sia
ancora arrivata al tempo t
1
sulla superficie della sfera Consideriamo ora il flusso del campo E al tempo 0
attraverso questa superficie. Esso vale, per il teorema di Gauss, Q/
0
Al tempo t
1
il campo elettrico non pu
essere mutato, per il principio di Einstein, quindi il suo flusso deve essere sempre Q/
0
, ma questo contro
Gauss che vuole il flusso al tempo t
1
pari a: (Q + q)/
0
. Laffermazione della indistruttibilit della carica pu
ricevere una veste matematica operando nel seguente modo: si definisce dapprima il concetto di densit di
corrente mediante la seguente osservazione: si abbia nello spazio una distribuzione di carica caratterizzata
da una densit di carica variabile da punto a punto e supponiamo che questa carica sia in moto. Abbiamo
quindi un campo scalare (r,t) dipendente dalla posizione e dal tempo ed un campo di velocit v(r,t), anche
lui dipendente dalla velocit e dal tempo Ebbene, il vettore densit di corrente in un punto r al tempo t
definito come:
j(r,t) `
(r,t)v(r,t)
( diretto come v se + positivo, altrimenti diretto in senso opposto).
Pu anche darsi, come succede nei conduttori, che in ogni punto (macroscopico, sintende!) la densit di
carica sia nulla e questo perch si ha unaltissima densit di carica positiva ( i nuclei degli atomi) ed una
contemporanea altissima densit di carica negativa ( gli elettroni degli atomi), le due densit essendo pari in
valore assoluto. Ora succede che le cariche positive, che costituiscono il reticolo del cristallo, sono ferme,
65
mentre le cariche negative, od almeno una parte di loro, e precisamente quelle cariche che si chiamano
cariche di conduzione e che sono costituite dagli elettroni pi esterni degli atomi, si muovono sotto lazione,
ad esempio di un campo elettrico, creando quindi una corrente. Si ha allora un fatto apparentemente
paradossale: la presenza di una corrente ( dovuta al moto delle cariche negative) pur essendo in ogni punto
nullo. Per ovviare a tale difficolt, basta allargare leggermente la definizione data di densit di corrente,
introducendo una densit di cariche negative,
-
definita ovviamente come la somma di tutte (e sole) le
cariche negative contenute nellunit di volume e la loro velocit v
-
ed una densit di cariche positive
+
e la
loro velocit v
+
Avremo quindi, per estensione della definizione:
+ +
v v j `
Si abbia ora una generica superficie attraversata da questo flusso j di cariche in moto. Ebbene, la corrente
elettrica I che passa attraverso questa superficie, altro non che, per definizione di corrente elettrica:


S
d I S j `
Una corrente che passa attraverso una determinata superficie sar unitaria se trasporta, attraverso questa
superficie, una carica unitaria ( 1 C) in un tempo unitario (1s). Lunit di corrente si chiama Ampre
(simbolo: A).
Seguendo la strada che abbiamo percorso, si ha che il Coulomb ununit fondamentale, mentre lAmpre
derivata. In realt, siccome pi semplice ed pi preciso misurare correnti anzich cariche, si preferisce
dare la definizione che riporto sotto dellAmpre, definizione che prescinde da quella di carica e definire
quindi il coulomb cos: si consideri un filo percorso da una corrente di 1 ampre. Si faccia idealmente una
sezione di tale filo. La carica allora che viene trasportata dalla corrente attraverso la sezione, nel tempo di un
secondo , per definizione, 1 coulomb. Vengo ora alla definizione dellAmpre. un fatto sperimentale che
due fili paralleli ed indefiniti percorsi da corrente si attraggono o si respingono a seconda dei versi di queste
correnti. Ebbene,
lAmper definito come quella corrente che, percorrendo due fili rettilinei indefiniti e paralleli,
distanti un metro tra di loro, fa s che la forza reciproca, per metro del filo, sia di 2 10
-7
N.
4
Dopo questa lunga digressione, torniamo al problema di dare veste matematica al principio di conservazione
della carica elettrica. La carica Q contenuta un in volume V al tempo t si pu scrivere come:

V
dV t z y x t Q ) , , , ( ) (
La carica che esce nellunit di tempo dal volume attraverso la superficie S che lo delimita :

S
dS j
Se (come ) la carica si conserva, allora la diminuzione nellunit di tempo della carica contenuta in V deve
essere proprio pari a

S
dS j
. Deve cio essere:


S V V
d dV
t
t z y x
dV t z y x
dt
d
dt
dQ
S j
) , , , (
) , , , (

Per il teorema della divergenza sar poi:




V S V
dV d dV
t
t z y x
dt
dQ
) (
) , , , (
j S j

Trasportando tutto sotto il medesimo segno dintegrale, si ha:


0 ) (

dV
t
V

j
Per larbitrariet del volume su cui si integra, si ha allora:
0

+
t

j
Equazione di conservazione della carica od equazione di continuit
4
bene precisare che il diametro dei due fili deve potersi considerare trascurabile rispetto al metro di distanza che li
separa per poter considerare la corrente come distribuita uniformemente attraverso la sezione dei fili
66
Dal punto di vista matematico, la relazione trovata pone un legame tra due grandezze, la e la j che a tutta
prima si sarebbe potuto pensare che potessero essere due funzioni completamente arbitrarie, mentre la
relazione di continuit ci dice che cos non .
Altra osservazione: il principio di Einstein summenzionato ci fa capire anche che la legge di Coulomb non
pu avere valore assoluto. Se fosse rigorosamente valida infatti, una carica in P, molto distante da Q
avvertirebbe immediatamente una messa in moto di Q, da una variazione della distanza tra le due, ma ci
escluso. Ripeto, la legge di Coulomb vale per cariche statiche ( o, approssimativamente per cariche in moto
con basse velocit, nel senso che, quanto pi bassa la velocit, tanto meglio vale la legge)
Il teorema del rotore ( o di Stokes)
Consideriamo una linea chiusa orientata, arbitraria immersa in un campo vettoriale A(x,y,z) e la
circuitazione del campo lungo la linea chiusa, cio

r A d
. La curva chiusa in questione pu essere vista
come il bordo di una superficie limitata da questa curva. Attraverso questa superficie, che si trova immersa
nel campo A, vi sar un flusso di questo stesso campo, cio si potr calcolare

S
dS A
. Ebbene, sotto larghe
ipotesi, vale la relazione:
Teorema di Stokes


S
d d S A l A

Dimostrazione:
Considero dapprima il caso in cui la linea chiusa un triangolino rettangolo infinitesimo PQR. Fissiamo
allora un sistema dassi cartesiano ortogonale con lorigine nellangolo retto (punto P) e disposto in modo
che il lato PQ giaccia lungo lasse x ed il lato PR lungo lasse y. Sar allora: PQ = dx i; RP = -dy j; QR = (dy
j dx i) Orientiamo il triangolo percorrendo il bordo cos: PQR. allora il vettore che rappresenta il
nostro triangolo sar il vettore: dS = (dx dy )k/2.
Calcolo ora la circuitazione del campo (circuitazione che chiamo lavoro anche se il campo non un campo
di forza).
Per calcolare con sufficiente precisione i lavori nei tre tratti, converr assumere per il campo il valore
intermedio tra gli estremi del segmento interessato. Precisamente scriver:
nel tratto PQ:
[A(P)+ A(Q)] (dx i)
nel tratto QR
[ A(Q) +A ](dy j - dx i)
Nel tratto RP
[ A +A(P) ]( -dy j )
Sommando, ottengo subito:
{- [A
x
- A
x
(P)] dx + [A
y
(Q) A
y
(P)] dy}
Ora osservo che :
+

dy
y
A
P A R A
x
x x
) ( ) (
con infinitesimo dordine superiore a dy
ed pure:
+

dx
x
A
P A Q A
y
y y
) ( ) (
con infinitesimo dordine superiore a dx
La circuitazione risulta allora essere, a meno di infinitesimi superiore a dS ( infatti, siccome dx dy un
infinitesimo dello stesso ordine dellarea del triangolino dS, infinitesimo dordine superiore a dS):
) (
2
1
2
dy dx
dy dx
y
A
x
A
d
x
y
+ +

,
_

l A
Calcolo ora il flusso del rotore di A attraverso il triangolo
Devo dunque fare:
67
( ) ( ) ( ) ( ) ( ) ( ) ( ) ( ) ( )
2
0 0
dy dx
dS dS dS d
z y x z z y y x x
A A A A A A S A + + + +
e quindi:
2
dy dx
y
A
x
A
d
x
y

,
_

S A
Si dunque dimostrato che, a meno di infinitesimi superiori allarea, il flusso del rotore di un campo
attraverso la sup. di un triangolo rettangolo infinitesimo pari alla circuitazione del campo lungo il
perimetro.
Vengo ora al caso in cui il circuito non sia pi infinitesimo, ma che sia costituito da una generica linea chiusa
. A tal fine considero una rete, a maglie strettissime (al limite infinitesime) e la maglia sia costituita da
triangolini rettangoli. Immagino ora di tendere questa maglia sul circito e di tagliare via con una forbice
la rete debordante oltre . Ora, se ci immaginiamo loperazione, dovrebbe risultare chiaro che il circuito
pu essere immaginato come costituito da alcuni rami di maglie della rete, precisamente da quei rami che
non sono comuni a due maglie. Ciamer tali rami, rami scoperti. La circuitazione del campo A lungo allora
altro non che la somma dei lavori lungo i rami scoperti. Ora avviene che la somma dei lavori lungo i soli
rami scoperti pari, a sua volta, alla somma dei lavori lungo tutti i rami, sia scoperti che a comune con altre
maglie. La cosa dovuta al fatto che nel calcolare il lavoro lungo un tratto comune a due maglie, una volta
questo tratto percorso in un verso (considerato il tratto come appartenente ad una maglia) ed una volta lo
stesso tratto percorso in senso inverso (considerato tale tratto come appartenente ala maglia confinante con
la precedente) e quindi il contributo al lavoro lungo i tratti comuni nullo.
In formule si ha dunque:



tratti i tutti scoperti tratti
d d d r A r A r A

Ma la somma lungo tutti i tratti altro non che la somma delle circuitazioni lungo tutte le maglie. Essendo
ognuna infinitesima, per quanto si dimostrato sopra, ogni circuitazione pari al flusso del rotore attraverso
la maglia e la somma di questi flussi altro non che, per definizione di flusso, il flusso del rotore attraverso
la superficie che ha come bordo la linea
Il teorema dunque dimostrato
Osservazione: Ma, si pu osservare, di superfici che hanno un assegnato bordo ne esistono a iosa!
Pensiamo, ad esempio, ad una superficie sferica tagliata da un piano per il suo centro. Lintersezione tra
piano e superficie una circonferenza. Orientiamola. Questa bordo sia di un cerchio, sia di una semisfera,
diciamo quella di sinistra. Ora uguaglieremo la circuitazione di A al flusso del rotore di A attraverso il
cerchio od attraverso la semisfera? Capite bene che il fatto che deve capitare per toglierci dall imbarazzo di
dover precisare attraverso quale superficie si deve calcolare il flusso, che tale flusso sia il medesimo
attraverso tutte le superfici che hanno quella data linea come bordo! Se fosse cos allora non sarebbe
necessario specificare attraverso quale superficie va calcolato tale flusso forse troppo sperare che avvenga
un simile fatto? Ora considerate bene questa cosa: dire che il flusso indipendente dalla superficie (sempre
che il bordo sia comune!) la stessa cosa che dire che il flusso attraverso la superficie chiusa (con al normale
orientata verso lesterno) costituita dallunione di due superfici con medesimo bordo, nullo. E che cosa
garantisce che il flusso di un vettore F attraverso una superficie chiusa arbitraria sia nullo? Ma chiaramente,
per il teorema della divergenza, che sia: div F = 0 ! Quindi, nel nostro caso, non sorgerebbe nessuna
difficolt nel teorema di Stokes se fosse:
0 ) ( A
Ma cos proprio! Basta applicare la definizione di rotore e poi la definizione di divergenza e si ottiene che
il primo membro identicamente nullo (tenere presente il teorema sullinvertibilit delle derivate seconde
miste!)! ( osservate poi questo fatto assai curioso! Il rotore di A formalmente un prodotto vettore tra
loperatore vettoriale nabla ed il vettore A. Ma il prodotto vettore tra due, ortogonale alluno come allaltro.
Quindi questo prodotto vettore ortogonale al vettore nabla. Ma fare la divergenza di un vettore, altro
non che, formalmente, fare il prodotto scalare del vettore con il vettore nabla. Quindi, se faccio il prodotto
scalare tra nabla ed un vettore a lui perpendicolare ( A ), dovrei ottenere 0 e proprio cos!)
Si detto dunque che la relazione: 0 ) ( A si pu dimostrare molto semplicemente svolgendo i
calcoli indicati dalloperatore nabla. Viceversa, si pu mostrare che, se vale il teorema di Stokes e quello
della divergenza, allora si pu dimostrare in modo intuitivo, senza svolgere calcoli opachi, che la divergenza
68
di un rotore nulla. Infatti, consideriamo una arbitraria superficie chiusa immersa in un campo vettoriale A.
Sulla superficie tracciamo una piccolissima circonferenza che viene a dividere S in due parti: un cerchietto,
che linterno della circonferenza, che indico con S
1
ed il resto della superficie S, che chiamo S
2
.
Calcoliamo la circuitazione di A lungo tale circonferenza. Per il teorema di Stokes, del quale si ammessa la
validit, tale circuitazione sar pari al flusso di rot A attraverso S
2
. facendo tendere a 0 il raggio della
circonferenzina, la circuitazione tende a zero; il flusso del rotore , che uguale tende dunque anche lui a 0 e
la superficie attraverso la quale si calcola il flusso tende alla superficie chiusa S. Si ottiene quindi che il
flusso del rot A attraverso una generica superficie chiusa 0. Ma tale flusso, per il teorema della divergenza,
altro non che lintegrale nel volume racchiuso da S della divergenza del rotore di A. integrale che dunque
nullo. Per larbitrariet del volume, si ha la tesi.
La corrente elettrica
giunto il momento di abbandonare i fenomeni elettrici statici per considerare ci che avviene quando le
cariche sono in moto. Supponiamo dunque di avere una distribuzione di carica variabile da punto a punto
(una nuvola di carica) e di metterla in moto con una velocit variabile da punto a punto e variabile
eventualmente anche con il tempo t . La quantit:
) , ( ) ( ) , ( t t r v r r j `
prende il mome di densit di corrente elettrica (ma molte volte laggettivo elettrica si sottintende, si
omette). Per capire il significato fisico di tale vettore, consideriamo una superficietta infinitesima dS
collocata nel punto r e facciamo il prodotto scalare di j e dS.
S v S j d d
Ma vdS altro non che il volume che fluitoo, nellunit di tempo, attraverso la superficie dS e quindi jdS
misura la carica che passata nellunit di tempo attraverso la superficie dS. Il prodotto scalare di un vettore
a con uno rappresentante una superficie infiitesima dS viene significativamente denominato flusso del
vettore a attraverso la superficie dS e se il vettore a in questione il vettore densit di corrente elettrica,
tale flusso prende il nome di intensit di corrente elettrica attraverso la superficie dS (ma molto spesso la
specifica: attraverso la superficie dS si omette, se chiara dal contesto). Lunit per lintensit della
corrente elettrica , nel sistema MKS, lAmpre (simbolo A). Diremo che una superfici attraversata da
una corrente elettrica di 1 A se attraverso di lei passa in 1 secondo una carica elettrica di 1 Coulomb.
Analogamente, negli altri sistemi di unit di misura, la corrente attraverso una superficie sar unitaria se
attraverso quella superficie passa una carica unitaria nellunit di tempo. Dunque nel sistema MKS lunit di
corrente , per come si proceduto, ununit derivata dallunit di carica. Si vedr per in seguito che di
fatto si preferisce procedere in senso inverso e cio definire in altro modo lunit di corrente e definire la
carica unitaria ( il Coulomb) come quella trasportata in 1 s attraverso una superficie quando questa
attraversata da una corrente di 1 A (Naturalmente laltro modo di definire lAmpre sar tale da fornire per
lunit di carica ci che si precedentemente definito, cio quella carica che posta ad un metro da una
uguale, risente di una forza di circa 910
9
N)
Spesso e volentieri (allinterno dei conduttori o nelle soluzione elettrolitiche) avviene di avere una
distribuzione di cariche puntiformi fisse nello spazio ( ma non sono fisse le cariche positive in una soluzione
elettrolitica) ( tipico esempio: i nuclei degli atomi di un certo materiale che se anche non sono proprio fermi,
sono per termicamente vibranti intorno a determinate loro posizioni medie fisse), con una densit tale da
poterle considerare come costituenti un continuo di carica. Tale distribuzione quindi sar descitta da una
distribuzione di carica positiva indicata con
+
(r). Finemente miscelata a questa carica positiva ferma,
presente poi una carica negativa di esattamente pari valore, indicata con
-
,ma in moto. Avviene quindi che
in ogni punto la densit di carica totale 0, pur essendo presente una densit di corrente dovuta al moto
delle cariche negative. Una quindi pi generale definizione di densit di corrente j sar allora data da:
) , ( ) ( ) , ( ) ( t t r v r r v r j
+ +
+ `
con
+
+ `
totale
dove
-
una quantit negativa e
+
una positiva. v
+
e v
-
sono poi le velocit delle cariche positive e
negative.
In elettrostatica si visto che il campo E allinterno dei conduttori nullo perch altrimenti il campo
metterebbe in moto le cariche libere di muoversi provocando effetti macroscopici, quali il riscaldamento del
materiale che invece non si osserva. Supponiamo ora di non essere pi in condizioni di statiche,
69
mantenendo forzatamente una certa differenza di potenziale V agli estremi di un lungo e sottile conduttore
(filo elettrico) Essendo presente una d.d.p., vi sar dunque un campo elettrico che metter in noto gli elettroni
di conduzione del metallo. Risulta che per molti materiali, specialmente per i metalli, il vettore densit di
corrente j proporzionale al campo elettrico nel dato punto, secondo un coefficiente di proporzionalit
che dipende dal materiale e dal suo stato. Si scriver pertanto per questi conduttori, che vengono detti ohmici
) ( ) ( r E r j legge di Ohm: non dipende da E
La costante viene detta conducibilit elettrica del materiale ed il suo inverso, resistivit elettrica. Alcuni
valori:
Argento = 0.167 10
-7
unit MKS
Rame = 0.172 10
-7
unit MKS
Alluminio = 0.283 10
-7
unit MKS
Mercurio = 9.600 10
-7
unit MKS
Germanio = 0.45 unit MKS
Silicio = 640 unit MKS
Vetro = 10
12
unit MKS
Quarzo fuso = 7.5 10
17
unit MKS
Notare quanto estesa la gamma dei valori!
Osservazione: sembra un po strano legare il campo elettrico, che una forza (su di una carica unitaria)
mediante un coefficiente di proporzionalit, ad una densit di corrente, che una velocit (per la carica
contenuta nellunit di volume). Sarebbe forse stato pi logico pensare ad un legame tra il campo e la
derivata della densit di corrente! Ma una relazione del genere si era gi vista nel caso di attriti viscosi.
quindi logico pensare che anche qui ci sia una forma di attrito, ma di chi e contro cosa? Vediamo meglio la
questione. Se il reticolo cristallino fosse infinito e i nuclei disposti con assoluta regolarit nello spazio, allora
si potrebbe dimostrare che gli elettroni di conduzione sarebbero completamente liberi di muoversi, non
risentendo di alcuna forza dovuta ai nuclei (la forza esercitata da uno controbilanciata da gli altri intorno).
Manulla perfetto a questo mondo! Il metallo contiene certamente delle impurezze che contribuiscono a
rendere il reticolo non pi ideale. Inoltre i nuclei vibrano, per lagitazione termica, intorno alle loro posizioni
medie ed inoltre, cosa importante, la limitatezza del conduttore interrompe bruscamente la regolarit del
reticolo e proprio questa interruzione fa sorgere delle forze che impediscono, o per lo meno ostacolano,
luscita degli elettroni dal metallo. Questa mancanza di assoluta simmetria si pu dipingere dicendo che
ogni tanto lelettrone di conduzione urta con latomo, cede parte della sua energia cinetica, e rimbalza in
una direzione del tutto casuale. Indichiamo con il tempo intercorso in media tra due successivi urti di un
elettrone. Indicando con V
termica
la velocit che un elettrone possiede per il fatto di essere in un corpo a
temperatura T, questa velocit rappresenter anche la velocit che il nostro generico elettrone possieder
subito dopo un urto. Trovandosi ora nel campo elettrico E, al generico tempo t dopo lurto, avr allora una
velocit (e = carica elettrone, m =sua massa):
v(t) = V
termica
+eEt/m
Lespressione mostra che la velocit composta da una parte, la termica, che mediamente nulla (la media
fatta sui vari elettroni) a causa del comportamento caotico dellagitazione termica. Laltra parte invece ha
la medesima direzione per tutti gli elettroni (nellintorno di un punto macroscopico, sintende, dove E il
medesimo per tutti) e mediamente vale allora: eE/m. Potremo quindi dire che la velocit media, che d
luogo alla corrente elettrica macroscopica :
m e m t e m t e
termica termica
/ 0 / / E E V E V v + + +
la densit di carica dovuta agli elettroni di conduzione, che moltiplicata per la <v> trovata fornisce la j, ,
indicando con n il numero di elettroni di conduzione nellunit di volume: = ne e quindi si ha:
j= ne
2
E/m
Per la conducibilit elettrica si ha dunque:
= ne
2
/m
La legge di Ohm dice che non dipende da E. La formula di sopra mostra allora che la legge vera se (e
solo se) non dipende da E. vero questo? Indichiamo con l la distanza (media) tra due centri di collisione
70
(non coincide con la distanza media di due atomi, ma sar maggiore, per quel discorso sul reticolo quasi
perfetto e si chiama libero cammino medio) Il tempo sar allora dato da: = l / velocit. Di quale velocit
si tratta? Della <v> trovata di sopra? No di certo, perch, se non si ha campo elettrico, non che non ci siano
pi collisioni! Ai fini dellintervallo temporale tra due collisioni fondamentale fare entrare in gioco anche
la velocit termica. Come ordine di grandezza la velocit che entra in gioco sar allora:
V
termica
+ eE/m
Potremo allora scrivere, come ordine di grandezza:
m
eE
V
l
termica

Se nellespressione trovata V
termica
>> eE/m ` velocit di drift (e soltanto in questo caso), potremo
scrivere:
termica
termica
V
l
m
eE
V
l

e quindi per la conducibilit elettrica si ha:


termica
termica
mV
l ne
m
eE
V m
l ne
2 2
) (

+

indipendente dal campo elettrico come vuole Ohm


Esercizio: stimare il valore di V
termica
per lelettrone di conduzione
Risposta anticipando ci che forse studierete in seguito (o che forse non studierete mai), la teoria cinetica
della materia permette di dire che la velocit di una particella che si trova in un bagno a temperatura T ,
per il fatto di trovarsi a quella temperatura, sottoposta ad un moto di agitazione termica, al cui velocit media
data dalla relazione:
kT mV
termica
2
3
2
1
2

dove T la temperatura assoluta del corpo ( semplicemente data da 273+ temperatura centigrada) e k una
famosa costante, detta costante di Boltzman e che vale, nel sistema MKS, 1.38 10
-23
. Si ha allora subito (a
T =300):
s Km
m
kT
V
termica
/ 117
10 1 . 9
300 ) 10 38 . 1 ( 3 3
31
23

decisamente una bella velocit!


Esercizio Stimare il tempo medio che intercorre tra due collisioni di un elettrone col reticolo del rame
Risposta dalla formula che fornisce , si ha subito: = m/(e
2
n) e quindi:
= [1/(0.172 10
-7
)](9.1 10
-31
)/[(1.6 10
-19
)
2
(0.845 10
29
)]=2.44 10
-14
s
Esercizio Stimare il libero percorso medio l dellelettrone di conduzione
Risposta: si ha. L = V
termica
= 117000x2.44 10
-14
= 0.28 10
-8
m
Esercizio Determinare per quali valori del campo elettrico la velocit di drift comparabile con la velocit
termica
Risposta deve aversi: 117000 = 1.6x10
-19
x2.44x10
-14
/(9.1x10
-31
) E e quindi
8
10 27 . 0 E V/m
Esercizio Stimare lordine di grandezza del campo elettrico che si ha nei fili elettrici di casa nostra
Risposta La corrente che circola nei fili di casa , come ordine di grandezza, dellAmpre, e la sezione del
filo dellordine del millimetro. Si ha quindi:
I = Sj = (0.001)
2
E e quindi: E 1.72 10
-7
/10
-6
0.17 V/m
Come si vede, tale campo elettrico enormemente inferiore a quello per il quale la velocit di drift
comparabile con la termica e quindi la legge di Ohm pienamente giustificata
Esercizio Stimare la velocit di drift per i fili della corrente di casa
71
Risposta La velocit di drift : eE/m e quindi (1.6 10
-19
)(0.17)(2.44 10
-14
)/(9.1 10
-31
) 7 10
-2
cm/s
Lavreste immaginata cos piccola?
Esercizio Viene deposta allinterno di un conduttore ohmico una carica Q . Stimare il tempo impiegato per
raggiungere lequilibrio elettrostatico.
Risposta: Dallequazione di continuit, usando la legge di Ohm, si ha:
t
e t
t t t t
0
0
0
) ( 0 0 ) ( 0 ) ( 0

+ E E j
Quindi, dopo un tempo pari a
0
/ la carica si ridotta di un fattore e.
Per il rame, tale tempo vale: (0.172 10
-7
x 8.85 10
-12
) = 0.15 10
-18
secondi
Per il silicio tale tempo vale: (640 x 8.85 10
-12
) = 0.56 10
-8
secondi
Per il vetro tale tempo vale: (10
12
x 8.85 10
-12
) = 8.85 secondi
Per il quarzo tale tempo vale:(7.5 10
17
x 8.85 10
-12
) = 76 giorni
Da notare che la carica non si sposta dalla posizione iniziale alla superficie. Infatti, se cos avvenisse,
dovrebbe succedere che la in un punto del conduttore dovrebbe passare da zero ad un certo valore quanto
passa per di li la carica, ma la formula di sopra mostra che se in un punto ad un istante la nulla, tale resta.
Esercizio Si pu immaginare che un elettrone, in moto lungo lasse x sotto lazione di un campo elettrico
costante subisca un urto ogni secondi con un atomo del reticolo. Si pu ipotizzare che, in seguito a tale urto
ceda parte dellenergia cinetica al reticolo (che quindi si riscalda (effetto Joule)) e con probabilit p inverta la
sua velocit e con probabilit (1-p) non la inverta. Determinare il moto della carica a livello macroscopico
(determinare cio il moto della carica per intervalli di tempo molto superiori a ).
Risposta: sia
) (
n
v e
) (+
n
v la velocit dellelettrone un attimo prima ed un attimo dopo aver compiuto
lennesimo urto contro il reticolo.
) (+
n
v , con probabilit p, pari a f
) (
n
v e, con probabilit (1-p), pari a - f
) (
n
v .
Il fattore f, < 1, tiene conto della perdita di energia cinetica. La velocit media quindi, subito dopo lurto
ennesimo allora:
) ( ) ( ) ( ) (
) 1 2 ( ) ( ) 1 (
+
+
n n n n
v p f v f p pfv v
Confondendo il valore attuale della velocit con il suo valore medio, scriver:
) ( ) (
) 1 2 (
+

n n
v p f v
La velocit un attimo prima del successivo urto, sar data da quella di sopra incrementata, per la presenza del
campo elettrico E, della quantit:
m
E
(m denota la massa dellelettrone). Quindi scriver:

m
E
v v
n n
+
+
+
) ( ) (
1
Con laiuto della relazione precedente, posso allora scrivere:

m
E
v p f v
n n
+

+
) ( ) (
1
) 1 2 (
e quindi subito:
[ ]
m
E
v p f v v
n n n
+

+
) ( ) ( ) (
1
) 1 2 ( 1
Dividendo per ottengo:
m
E
v
m
E
v
p f v v
d
dv
n n
n n n
+ +
1
]
1

) ( ) (
) ( ) (
1
) (
) 1 2 ( 1


Essendo f <1 e 0<p<1, risulta >0. Si ottenuta cos lequazione del moto di un corpo sottoposto ad una
forza costante e ad una frenante di tipo viscosa. Il corpo raggiunger una velocit limite che manterr
costante, pari a E/(m)
Consideriamo ora un conduttore omogeneo, di lunghezza L, sezione A e resistivit e supponiamo che in
esso passi una densit di corrente, nel senso della lunghezza, uniforme e costante nel tempo (corrente
72
stazionaria) j. La differenza di potenziale V che c ai capi del conduttore V = E L = (j) L Ora lintensit
di corrente : I = jS e quindi, sostituendo, ho: RI
S
L
I V `

La grandezza R detta resistenza del conduttore e si misura in Ohm, simbolo


Attenzione! La legge di Ohm spesso viene enunciata in modo erroneo dicendo:
V = RI
Oppure anche, in modo perfettamente equivalente e quindi in modo sempre erroneo, dicendo:
V/I = R
Perch questi enunciati non sono la legge di Ohm? Semplicemente perch questi sono la definizione di
resistenza elettrica e, come ben sappiamo, per dare delle definizioni non occorre assolutamente il contributo
dellesperienza, cosa fondamentale invece per formulare delle leggi. Se vogliamo formulare la legge di Ohm
in termini di R, V ed I, questa si enuncia dicendo che il rapporto tra la differenza di potenziale e la
corrente non dipende dalla differenza di potenziale medesima o, pi sinteticamente, che R una
costante (cio non dipende da V)
Diremo che tra due punti ( o tra due regioni, ognuna delle quali equipotenziale) c una resistenza di 1
se, presentando i due punti (o le due regioni) differenza di potenziale di 1 V, dalluno/a fuoriesce (e
nellaltro/a entra) la corrente di 1 Ampre.
Notare che, per lequazione di continuit, in condizioni stazionarie, quando cio ogni grandezza non dipende
dal tempo, dal fatto che .j =0, si ha che il flusso di j, cio la I, attraverso due piani che separano in due
punti ( o regioni) a differenti potenziali, non dipende dalla sezione.
Ancora unosservazione sulla legge di Ohm:
Consideriamo una batteria( = un oggetto caratterizzato da due poli, uno positivo, in quanto possedente
cariche positive e laltro negativo, in quanto possidente cariche negative e tale che tra questi due poli si
mantenga sempre una differenza di potenziale V costante) e connettiamo i due poli con un conduttore
filiforme caratterizzato da una resistenza R. Nel filo scorrer una corrente costante I = V/R. Quindi nel filo
sar presente un campo elettrico dintensit costante diretto come il filo. Ci si pu chiedere allora come
faccia il campo ad essere costante se le cariche sono posizionate sui poli della batteria e non diminuisca con
la distanza da questi. Il fatto che le cariche non si trovano solo ai poli, ma anche sulla superficie del filo
conduttore e sono queste che permettono al campo E di piegare a seconda di come piega il filo. La
determinazione della distribuzione delle cariche per nel filo molto difficile da determinare
5
e dipende
fortemente dalla disposizione di tutto il circuito. Inoltre, mentre in elettrostatica un problema primario
consisteva nella determinazione delle cariche sul conduttore, in presenza di corrente, il problema che
interessa la determinazione della corrente e la distribuzione delle cariche di secondaria importanza. Per
questi due motivi, tale problematiche non verranno affrontate.
Esercizio: mostrare che la determinazione delle cariche sulla superficie di un filo elettrico percorso da
corrente dipende fortemente dal resto del circuito.
Soluzione.Consideriamo un condensatore piano ideale. Connettiamo le due armature con un filo
perpendicolare a loro. Il potenziale nel filo varier linearmente con la distanza dai piatti come quando non
cera il filo. La corrente sar uniforme e sulla superficie del filo non ci sar alcuna carica. Supponiamo ora
che il filo sia percorso dalla medesima corrente I di prima ( e quindi medesima sar lintensit del campo
allinterno), ma sia piegato a corona circolare (di spessore nullo) con centro nellorigine di un sistema di assi
o assuma magari anche una forma pi complicata. Il potenziale varier allora linearmente allinterno del filo
con langolo = arctan(y/x). In altri termini, il potenziale sar una funzione di x ed y e z ed il gradiente di
questa funzione calcolato sulla superficie esterna del filo non c motivo che sia tangente al filo
6
( e quindi
ci sar una carica superficiale!)
5
La soluzione dipenda da unequazione integrale di primo tipo, cio da unequazione del tipo:

1
0
) , ( ) ( ) ( ds s z h z y s f , con f(s) e h(z,s) funzioni note e incognita la y(z)
6
il campo E sulla superficie interna del filo tangente alla superficie: altrimenti la corrente uscirebbe dal filo!)
73
Scarica di un condensatore su di una resistenza
Il disegno di figura rappresenta un condensatore, con una carica iniziale Q che viene chiuso su di una
resistenza di valore R Determinare come varia nel tempo la corrente che percorre il circuito e la differenza di
potenziale tra le armature
Risposta bench non si sia in condizioni stazionarie, supporremo comunque valide le equazioni per il caso
stazionario, in particolare che il campo E sia sempre conservativo, sicch si possa parlare di potenziale e
supporremo che lintensit della corrente sia la medesima per ogni sezione del filo, ammetteremo cio che
ogni sezione del filo non vada caricandosi nel tempo. La differenza di potenziale ai capi del condensatore
vale, al generico tempo t: V(t) = Q(t)/C. se il filo supposto essere un conduttore perfetto, anche in
condizioni non stazionarie ogni suo punto sar allo stesso potenziale, altrimenti la corrente sarebbe infinita.
Quindi questa medesima differenza di potenziale si trova agli estremi della nostra resistenza R. attraverso di
lei quindi passa una corrente data da: I(t) = V(t)/R. Per definizione poi di intensit di corrente, I pari alla
diminuzione, nellunit di tempo, della carica Q sul condensatore, cio: I(t) = - dQ/dt. Riunendo le formule
scritte si ha subito:
RC
t
RC
t
RC
t
e
R
V
e i t i e V t V
RC
V
dt
dV


0
0 0
) ( ; ) (
dove V
0
la differenza di potenziale iniziale, cio al tempo t = 0 ai capi del condensatore. Secondo la nostra
formula la corrente passerebbe istantaneamente dal valore nullo, un attimo prima dellinizio del processo, al
valore V
0
/R. Siamo ben lontani dal caso stazionario! E c allora da credere che, in prossimit dellorigine dei
tempi, la risposta non sia corretta. La derivata temporale della corrente ancora una funzione a decrescita
esponenziale, sicch, col prosieguo del tempo questa derivata diventa sempre pi piccina e quindi con
sempre maggior ragione la corrente (anche se molto piccola) potr ritenersi stazionaria. Quindi la nostra
formula sar tanto pi esatta quanto pi il tempo grande.
Le leggi di Kirchhoff
Consideriamo la figura di sotto:
Questa rappresenta un circuito elettrico o rete elettrica: sono presenti dei nodi, che sono i punti numerati
dellesempio, e dei rami che sono dei segmenti che collegano coppie di nodi, Se questi rami presentano un
ingrossamento ( come il ramo 3-4), vuol dire che in quel ramo inserita una resistenza. Nel ramo 4-5
inserita una batteria di forza elettromotrice ( f.e.m.) nota. (la forza elettromotrice semplicemente la
differenza di potenziale ai capi della batteria. Pure nel ramo 3-7 inserita una f.e.m. e cos nel ramo 8-9. Nel
ramo 5-7 inserito invece un condensatore C. Il problema che di solito si tratta di risolvere di, conoscendo
la carica iniziale del condensatore e la sua capacit, il valore di tutte le resistenze e delle f.e.m, determinare
lintensit delle correnti che percorrono i vari rami ed i potenziali nei vari nodi. intuitivo che le correnti
varieranno nel tempo, per cui, in prima battuta, limitiamoci al caso che ci non succede e questo si avr
senzaltro se nel circuito sono inserite soltanto batterie e resistenza. Si dimostra allora che il problema
condotto alla risoluzione di un sistema di equazioni lineari. Accanto alle definizioni date, aggiungiamo
quella di maglia: un qualunque circuito chiuso contenuto nella rete, mentre maglia elementare una maglia
74
che non contiene al suo interno alcuna altra maglia. Sia N il numero dei nodi in una rete, R il numero dei
rami ed M il numero delle maglie elementari
Teorema: in una rete elettrica sussiste la relazione: M = R- (N-1)
Dimostrazione: consideriamo gli N nodi. Il minimo numero di rami necessari per connetterli tutti ,
ovviamente N-1. Connettiamoli dunque usando N-1 rami. Dopo aver fatto questa operazione, non si creata
ancora alcuna maglia chiusa ed abbiamo ancora a disposizione R-(N-1) rami. Ogni volta poi che collochiamo
uno di questi rimasti, creiamo una maglia elementare, che saranno dunque, alla fine del collocamento di tutti
i rami, R- (N-1), c.v.d.
Dopo questo simpatico e semplice teoremino di teoria dei grafi, vediamo come risolvere la rete. A tal fine,
orientiamo, a nostro piacimento, ogni ramo e chiamiamo i
r
la corrente che passa per il ramo r-esimo: se alla
fine si otterr per la corrente che passa per quel ramo un valore positivo, vuol dire che quello il verso di
percorrenza, altrimenti che lopposto. Fissiamo ora lattenzione su un generico nodo:
Prima legge di Kirchhoff: la somma algebrica delle correnti che convergono a quel nodo nulla.
Commento: la prima legge non asserisce altro che la conservazione della carica: la somma algebrica delle
correnti che converga ad un nodo altro non infatti che la derivata temporale della carica che si trova in quel
nodo. In condizioni stazionarie, tale variazione deve essere nulla. Noi per supporremo che tale prima legge
sia valida anche nel caso non stazionario, purch i fenomeni non siano variabili troppo rapidamente
(altrimenti una trattazione completa e rigorosa troppo difficile). Un tratto generico di conduttore compreso
tra due punti A e B pu essere considerato come una infinit di nodi e quindi la corrente che entra in A
deve essere uguale a quella che esce da B
Seconda legge di Kirchhoff: La somma algebrica delle cadute di potenziale che si incontra percorrendo una
qualunque maglia chiusa nulla.
Precisiamo: Considero un punto della maglia al quale attribuisco arbitrariamente il potenziale V =0. Se nel
mio percorso, lasciato il punto 0, incontro il polo negativo (positivo) e quindi il positivo (negativo) di un
generatore di f.e.m E, il potenziale si incrementa (decrementa) di E passando quindi a +E (-E). Se incontro
una resistenza R , il potenziale decrementa (incrementa) di i
r
R se il mio movimento concorde (discorde) col
verso della corrente fissato sul ramo. Percorso il primo tratto, passo al secondo ed applico sempre la regola
espressa, e poi al terzo e cos via fino a ritornare al punto di partenza. Ebbene, la legge di Kirchhoff dice che
alla fine, il valore ottenuto da questa serie di somme e sottrazioni, deve essere 0, cio il valore del potenziale
allinizio. In modo figurato: se io faccio unescursione in montagna tornando al punto di partenza e sommo
algebricamente tutti gli incrementi di quota incontrati, devo trovare 0.
Questa seconda legge deve la sua validit semplicemente allesistenza del potenziale! Se un potenziale esiste
(ma il discorso vale per ogni campo scalare), ovvio che la somma delle sue variazioni lungo un percorso
chiuso nulla!
Detto questo, assumiamo arbitrariamente pari a 0 il potenziale di un nodo (nodo 1), Sono incogniti i
potenziali nei restanti N-1 nodi e le correnti negli R rami. Ho quindi N -1+ R incognite . Scriviamo ora la
prima legge di Kirchhoff per il nodo 1,2, N . Ho dunque scritto N equazioni di cui per solo N-1 sono
indipendenti e quindi una la cancello perch superflua. E perch soltanto N-1 sono indipendenti?
Laffermazione si pu giustificare per induzione. Supponiamo dunque laffermazione vera per un circuito di
r rami. Aggiungo ora il circuito un altro ramo, che connette il nodo i con il nodo j. Cos facendo introduco
unaltra incognita: precisamente la corrente attraverso questo ramo, ma ora posso scrivere la prima legge di
Kirchoff per il nodo i e per il j, cio ho aggiunto unincognita e due equazioni: chiaro allora che di queste
due equazione una superflua! Scrivo ora la legge di Ohm per ognuno degli R rami. Ho scritto dunque altre
R equazioni. In totale dunque ho scritto N -1+ R equazioni in N -1+ R incognite : non resta che risolvere il
sistema (lineare). Buona fortuna!
Possibile obiezione: come facciamo ad essere sicuri che, cos facendo, otterremo sempre un sistema di
equazioni indipendenti? Non c il rischio di averne una dipendente dalle altre? No! consideriamo infatti le
prime N-1; ognuna di queste introduce una corrente che non veniva citata nelle precedenti equazioni e quindi
non pu essere dipendente da queste precedenti. Nelle ultime R, ognuna di queste fa intervenire un
potenziale che non contenuto nelle precedenti. Conclusione: le equazioni sono indipendenti.
Osservazione e la seconda legge di Kirchoff? Non labbiamo usata! vero! allora inutile ? No, potremmo
usarla per semplificare le equazioni ed arrivare pi rapidamente alla soluzione se non ci interessano tutti i
75
potenziali e tutte le correnti, ma solo, ad esempio, le correnti. Esistono a tal proposito vari metodi: metodo
delle maglie e metodo dei nodi. Vediamo il metodo delle maglie. Nel nostro circuito siano presenti M
maglie. Attribuiamo ad ognuna di queste maglie una corrente di maglia, di pari valore per tutti i rami della
maglia e di verso arbitrario. Si hanno dunque M incognite Un dato ramo sar comune a pi maglie ed allora
la corrente effettiva che lo percorrer sar la somma algebrica delle correnti delle maglie che hanno questo
ramo in comune. Con lintroduzione di queste correnti di maglie, la prima legge di K. automaticamente
soddisfatta. La seconda legge di K. applicata alle maglie permette dunque di scrivere un sistema di M
equazioni in altrettante incognite.
Vediamo il metodo dei nodi. Scegliamo come incognite il potenziale i N-1 nodi (mettendo pari a 0 il
potenziale del nodo restante. Cos facendo la seconda legge di K. automaticamente soddisfatta, mantre
la prima fornisce N-1 equazioni.
Commento: con il metodo delle maglie occorre risolvere un sistema di M incognite, con quello dei nodi, uno
di N-1 incognite. A seconda di questi valori pi opportuno luno o laltro metodo.
Possono essere utili, nella risoluzione del problema delle reti elettriche, alcuni teoremi. Dei quali enuncio
senza dimostrazione il
teorema di Thvenin:
Consideriamo una rete attiva [cio contenente generatori di tensione (= apparecchi che, qualunque cosa
succeda, mantengono tra due nodi una d.d.p. assegnata) o generatori di corrente(=apparecchi che, qualunque
cosa succeda, mantengono in un ramo una corrente assegnata)], che denominer rete (1) e consideriamo due
nodi A e B di questa rete. A questi due nodi sia poi connessa una seconda rete, rete (2). Ebbene, ai fini delle
correnti e potenziali che si instaureranno nella rete (2), la rete (1) equivalente ad un singolo generatore di
tensione in serie ad una resistenza posti tra A e B . La tensione di tale generatore quella che avremmo tra i
due punti se sconnettessimo la rete (2) e la resistenza quella che avremmo tra A e B se sconnettessimo la
rete (2) ed annullassimo i generatori di tensione e corrente.
Duale del teorema di Thvenin, il
teorema di Norton:
Consideriamo una rete attiva [cio contenente generatori di tensione (= apparecchi che, qualunque cosa
succeda, mantengono tra due nodi una d.d.p. assegnata) o generatori di corrente(=apparecchi che, qualunque
cosa succeda, mantengono in un ramo una corrente assegnata)], che denominer rete (1) e consideriamo due
nodi A e B di questa rete. A questi due nodi sia poi connessa una seconda rete, rete (2). Ebbene, ai fini delle
correnti e potenziali che si instaureranno nella rete (2), la rete (1) equivalente ad un singolo generatore di
corrente in parallelo ad una resistenza posti tra A e B . Lintensit di corrente di tale generatore quella che
avremmo tra i due punti se sconnettessimo la rete (2) ed inserissimo tra i punti una resistenza nulla
(cortocircuitassimo A e B) e la resistenza quella che avremmo tra A e B se sconnettessimo la rete (2) ed
annullassimo i generatori di tensione e corrente.(come in Thvenin)
I campi stazionari
Fino ad ora si trattato del campo elettrico nel caso statico: tutte le cariche erano ferme. Anzi no, mi
correggo, si trattato anche di cariche in moto: scarica di un condensatore, raggiungimento dellequilibrio in
un conduttore Ma riesaminiamo il nostro processo da un punto di vista critico: si parlato di potenziali
variabili nel caso di un condensatore. Noi sappiamo per che di potenziali si pu parlare solo in presenza di
campi conservativi ed il campo elettrico conservativo nel caso statico, quando deriva sostanzialmente dalla
legge di Coulomb e la conservativit si esprime attraverso la relazione: xE = 0. Ma la validit di tale
relazione sussiste in caso non statico? La risposta generale : no. Al pi potremo sperare che tale relazione
possa essere abbastanza approssimata anche quando i fenomeni dipendono dal tempo, purch non siano
troppo rapidi. In effetti proprio tale speranza che ci ha condotto al trattamento della scarica di un
condensatore e quindi c da sperare che la trattazione sia abbastanza corretta. Cos infatti e la trattazione
seguita la trovate in tutti i libri, ma c subito da rendersi conto che questa solo approssimata e c da
aspettarsi che rappresenter la realt sempre meno bene quanto pi rapidamente avverr la scarica. Il cercare
di indagare che cosa succede nella soluzione se R tende a 0 come combattere una guerra con armi che si sa
che non funzionano! Chiediamoci allora: qual il campo massimo di validit della equazione xE = 0 che si
detta vera in elettrostatica? La risposta viene dallesperienza accumulata nel XIX secolo ed : vale
rigorosamente nel caso in cui ci sono anche dei moti di cariche, purch si sia in presenza di fenomeni
76
stazionari, cio fenomeni per i quali tutte le grandezze in gioco possono variare da punto a punto ma,
in un dato punto, non variano nel tempo. Ma allora che cosa cambia rispetto al caso statico, visto che le
equazioni che determinano il campo elettrico sono le medesime nel caso statico e stazionario?. Quello che
cambia che, se abbiamo delle cariche in moto, sia pure stazionario, la forza che agisce su di una carica di
prova non pi data solo da qE, ma c un termine aggiuntivo che dipende dalla velocit della carica di
prova. Esattamente si ha (forza di Lorentz)
[ ] B v E f + q
dove B un nuovo campo vettoriale, detto di induzione magnetica. Ci si pu chiedere: e se siamo in
condizioni del tutto generali, con cariche in moto qualunque, per esprimere la forza su di una carica di prova
occorrer forse aggiungere un nuovo termine? No, lespressione trovata la forma pi generale della forza
su di una carica. Naturalmente se le cariche che generano i campi E e B sono in moto vario, diversi saranno i
valori di questi campi rispetto al caso statico o stazionario
7
, ma non ci sar bisogno di introdurre altri campi o
di modificare la formula di sopra per esprimere la forza agente su di una carica q.
Ed allora la domanda che viene spontanea : e come si determinano il campo elettromagnetico (cio E e B)
nel caso generale, od anche nel caso stazionario? Risposta: si determinano, ad esempio, risolvendo le
equazioni di Maxwell relativa al caso generale o stazionario. Sappiamo infatti che, assegnato il rotore e la
divergenza di un campo vettoriale, questo univocamente determinato, a meno di un campo costante. La
soluzione quindi unica se imponiamo che il campo sia nullo a distanza infinita da cariche e correnti. La
domanda allora : e quali sono le equazioni di Maxwell nel caso stazionario? Risposta: le molteplici
esperienze di elettromagnetismo effettuate nei casi stazionari, hanno permesso di affermare che
lelettrodinamica stazionaria si lascia perfettamente interpretare se si ammettono le seguenti equazioni:
o stazionari caso valida sempre valida
o stazionari caso valida sempre valida
j B B
E E
0
0
, 0
0 ,



Dove
0
una costante che nel sistema MKS pari esattamente a 4 10
-7
( che questa costante sia
esattamente il valore indicato, dipende in ultima analisi dalla scelta che si fatta per lunit di carica, o
meglio, per lunit di corrente. Ricordiamo infatti che nella definizione di ampre cera quel fattore 210
-7
piovuto dal cielo. Tale scelta comporta il valore esatto riportato per
0
)
Quesito Ammettendo valide le equazioni di Maxwell stazionarie e la forza di Lorentz come scritte sopra, si
potrebbe, per semplificare le cose, assumere la costante
0
pari ad 1?
Risposta: Ponendo: B
*
= B/
0
, si avrebbe: div (B
*
)=0; rot(B
*
)= j, ma allora la forza di Lorentz diventerebbe
[ ]
*
0
B v E f + q
Dimensioni di B:
Dallespressione della forza di Lorentz, risulta che, dimensionalmente, il campo dinduzione magnetica ha le
dimensioni di una forza /(carica velocit). Cio:
[ ]
[ ]
[ ]
[ ]
] [ ] [ ] [ ] [
] [ ] [
1
2
T C
kg
T L C
T L kg

B sistema MKSA
Lunit di misura del campo B nel sistema MKSA prende il nome di: tesla (simbolo: T), in onore del fisico
Tesla. E precisamente: un campo B ha valore di un tesla se su di una carica di un coulomb che si muove con
velocit di un metro al secondo perpendicolarmente al campo subisce una forza di un newton.
Va detto pure che il flusso del campo B del valore di un tesla attraverso una superficie di un m
2
ortogonale al
campo, quindi un flusso unitario, prende il nome di weber (in onore di Weber) e quindi spesso si usa dire che
lunit di misura di B il weber/m
2
Il campo di 1 tesla molto intenso; in laboratorio i pi intensi campi raggiungono qualche tesla, sicch si usa
spesso ununit 10000 volte inferiore al tesla, unit che si chiama gauss (in onore, ovviamente, del famoso
Gauss). Questa unit pure ( dimostrarlo!) lunit del campo dinduzione magnetica nel sistema CGS. Tanto
per avere unidea, il campo dinduzione magnetica terrestre di circa 0.5 gauss (Come mai la terra ha un
campo magnetico? Non lo so)
7
Quindi il campo E non sar pi fornito dallaspressione:. Che lo determinava nel cso statico
77
Osservazione importante. (Unit di misura) Quanti adottano in meccanica il sistema CGS, in
elettromagnetismo seguono una via leggermente diversa dalla nostra: loro pongono pari ad 1 ci che noi
poniamo pari a 1/4
0
(e cos facendo loro hanno unespressione semplificata della legge di Coulomb)
Per loro infatti vale:
2
2
1
r
q q
f
CGS CGS

Per noi invece :


2
2
0
1
4
1
r
q q
f
MKS MKS

Se facciamo dunque la sostituzione:


CGS MKS
q q
0
4
Passaggio da MKS a CGS-Gauss
allora la nostra legge di Coulomb si trasforma nella loro legge di Coulomb.
Questo significa anche che se noi prendiamo due cariche uguali, pari ognuna a
5
0
10 1 4

Coulomb
alla distanza reciproca unitaria (di 1 metro quindi), esse si respingono con una forza unitaria (di 1 N quindi)
Ovviamente sar pure:
CGS MKS CGS MKS
e j j
0 0
4 4
Passaggio da MKS a CGS
Il campo E generato da una carica per loro : r E
3
r
q
CGS
CGS
, mentre per noi : r E
3
0
4
1
r
q
MKS
MKS

Cio noi scriviamo:


MKS
MKS
q
f
E
e loro scrivono:
CGS
CGS
q
f
E
. Con la sostituzione in questione allora sia
ha:
0 0
4 4
CGS
CGS MKS
MKS
q q
E f f
E
Quindi:
0
4
CGS
MKS
E
E
Passaggio da MKS a CGS
La nostra equazione di Maxwell:
0
0

MKS
MKS
E diventa allora:
CGS CGS
CGS CGS MKS
MKS

4
4
4
0 0
0
0
0 0
E
E
E
le loro equazioni per il campo elettrico sono allora:
o stazionari caso valida sempre valida
CGS CGS CGS
0 , 4 E E . Nel CGS
Come vedete, in questa equazione compare il che di solito associato al cerchio, alla sfera e qui dunque
suona un po stonato, anche se corretto. Nel sistema MKS non figura.
per quanto riguarda lespressione della forza di Lorentz, loro preferiscono definire il campo B in modo da
scriverla cos:
1
]
1

+
CGS CGS CGS
c
q B
v
E f
Con c =velocit della luce nel vuoto. Preferiscono questa scrittura perch il tal modo i campi E e B hanno le
medesime dimensioni e la cosa ha un buon aspetto. In questo modo, oltre alla differenza di unit tra il
sistema MKSA e CGS, c una differenza pure nella definizione del campo dinduzione magnetica B.
Precisamente si ha che se facciamo la trasformazione:
78
CGS MKS
c
B B
0
4
1

Passaggio da MKS a CGS


si ha:
[ ]
1
]
1

+
1
1
]
1

4
+ +
CGS CGS GGS CGS
CGS
GGS MKS MKS MKS
c
q
c
q q B
v
E B v
E
B v E
0 0
0
1
4
4

Nel
sistema CGS poi le unit di B sono:
[ ] [ ] [ ]
3
2
3
2
] [

s g cm B
sistema CGS
Come si vede, compaiono dimensione frazionarie. Il fatto non conforme al gusto tradizionale dei fisici, ma
questo il prezzo da pagare se si vuole E e B con le setsse dimensioni.
Esercizio. Ricordando quanto si detto precedentemente senza dimostrarlo, che si ha:
0 0
2
1

c
,
dimostrare che i seguaci del sistema CGS scrivono le loro equazioni di Maxwell nel caso stazionario, come:
o stazionari caso valida
c
sempre valida
CGS CGS CGS
j B B
4
, 0 Nel CGS
Una propriet importante del campo B Supponiamo che una carica q si muova sotto lazione di un campo
B. quanto vale il lavoro compiuto dal campo sulla carica nello spostamento elementare dr di questa? Vale,
per definizione di lavoro elementare. r B v r f d q d dL ) ( Ma : dt d
dt
d
v r
r
v ` . Sostituendo
ho: dt q d q d dL v B v r B v r f ) ( ) ( . Ma il prodotto vettore tra v e B, perpendicolare a v e
quindi, se moltiplico scalarmene questo prodotto per v, ottengo 0. Essendo nullo il lavoro elementare, nullo
il lavoro relativo a qualunque spostamento. Quindi: Il campo B non compie lavoro sulle cariche. E allora,
chiederete, qual il suo effetto? Semplicemente quello di deviarle senza per variare la loro energia cinetica
( per il teorema dellenergia cinetica)
Veniamo ora allanalisi delle equazioni:
j B B
0
, 0 (r)
In particolare, supponiamo nota la densit di corrente j punto per punto, cio supponiamo di conoscere la
funzione vettoriale j(r). le equazioni scritte, dal punto di vista matematico, sono un sistema di quattro
equazioni differenziali alle derivate parziali che attende di essere risolto Confrontando con le equazioni del
campo elettrico, si vede che la situazione un po simmetrica: l il rotore era nullo e la divergenza no, qui
nulla la divergenza ma non il rotore. Come fare per risolverle? Basta tener presente una formula di calcolo
vettoriale che, per un generico campo vettoriale B(r) dice:
B B B ) ( ) (
dove il laplaciano di di B altro non che:
k j i B
z y x
B B B + + `
(siete pregati di verificare la formula svolgendo esplicitamente i conti per il doppio rotore di sinistra, dopo
aver introdotto le tre componenti di B. Poi svolgete i conti per il membro di destra ed alla fine confrontate le
due espressioni: se non si sono fatti errori, dovrebbero essere uguali)
Applichiamo la formula ottenuta al nostro campo dinduzione magnetica. Tenendo presente che 0 B ,
si ha subito:
( ) j B j B
0 0

Come si detto sopra, questa equazione vettoriale lespressione compatta delle tre seguenti equazioni
scalari:
z
z
y
y
x
x
B B B j j j
0 0 0

Ognuna di queste equazioni formalmente identica allequazione di Poisson per il potenziale elettrico ed
anche le condizioni allinfinito, sia per che per ogni componente di B sono uguali: sono nulle. Essendo
medesime le equazioni, medesime saranno le soluzioni e quindi subito scrivo:
79

spazio lo tutto
d '
'
) ' ( '
4
) (
3 0
r
r r
r j
r B

(Ho scritto ' per ricordare che loperatore nabla qui opera sulle variabili primate e non su r! ed inoltre d
3
r
altro non che un comodo modo per rappresentare lelemento infinitesimo di volume dV ma preferibile a
questultimo perch viene mostrata quale la variabile dintegrazione, cio la r

e non la r)
Per la mancanza di tempo e per non appesantire troppo le cose, con passaggi puramente matematici e
fisicamente poco significativi, non mostrer che la formula trovata si pu scrivere come:

spazio lo tutto
d '
'
) ' ( ) ' (
4
) (
3
3
0
r
r r
r r r j
r B

Come vedete la formula ricorda quella che fornisce il campo elettrico in funzione della densit di carica, ma
qui, nellintegrando, figura un prodotto vettore tra la densit di corrente, valutata in un punto generica ed il
vettore che parte da questo punto generico (r) e va fino al punto nel quale vogliamo calcolare il nostro
campo.
Esercizio La formula di sopra semplicemente la soluzione dellequazione: j B
0
e non detto
che la divergenza del vettore B ottenuto sia nulla. Mostrare allora, mediante il calcolo diretto, che se B
definito dalla formula di sopra, la sua divergenza (come deve essere!) effettivamente nulla.
Risoluzione Riscrivo B come:



spazio lo tutto
d '
'
) ' ( ) ' (
4
) (
3
3
0
r
r r
r j r r
r B

e quindi subito ( ricordiamo che opera qui sulle componenti di r e quindi r ed ogni sua funzione da
riguardarsi, ai fini della derivazione, come una costante):

1
1
]
1



spazio lo tutto spazio lo tutto
d d ' ) ' (
'
) ' (
4
'
'
) ' ( ) ' (
4
) (
3
3
0 3
3
0
r r j
r r
r r
r
r r
r j r r
r B

Ricordando che a(bc)= c(ab) e che il nabla si comporta come un vettore), applicando la relazione
vettoriale alla formula scritta, si ha:

1
1
]
1


1
1
]
1


spazio lo tutto spazio lo tutto
d d '
'
) ' (
) ' (
4
' ) ' (
'
) ' (
4
) (
3
3
0 3
3
0
r
r r
r r
r j r r j
r r
r r
r B

Ma 3
'
) ' (
r r
r r

un campo centrale ( con centro in r) che sappiamo dunque essere conservativo. Il suo rotore
nullo, cio
0
'
) ' (
3


r r
r r
c.v.d
Esercizio (NON FARE) Mostrare, mediante il calcolo diretto, che se B definito dalla formula di sopra, il
suo rotore (come deve essere!)
0
j
Risoluzione Riscrivo B come:



spazio lo tutto
dV'
'
) ' ( ) ' (
4
) (
3
0
r r
r j r r
r B

e quindi subito ( ricordiamo che opera qui sulle componenti di r e quindi r ed ogni sua funzione da
riguardarsi, ai fini della derivazione, come una costante):

1
1
]
1



spazio lo tutto spazio lo tutto
dV dV ' ) ' (
'
) ' (
4
'
'
) ' ( ) ' (
4
) (
3
0
3
0
r j
r r
r r
r r
r j r r
r B

Applico ora una forma di calcolo vettoriale che si pu trovare sugli appunti scannerizzati o che potete con
pazienza verificare voi stessi: basta solo fare i conti. Ecco la formula:
b a a b a b b a b a ) ( ) ( ) ( ) ( ) ( +
80
( ricordando che opera qui sulle componenti di r e quindi r ed ogni sua funzione da riguardarsi, ai fini
della derivazione, come una costante) si ha:
3 3 3
'
) ' (
] ) ' ( [
'
) ' (
) ' ( ) ' (
'
) ' (
r r
r r
r j
r r
r r
r j r j
r r
r r

+
1
1
]
1


1
1
]
1


Legge di Ampre
La circuitazione del campo dinduzione magnetica B lungo una qualunque linea chiusa pari a
0
I
concatenata
Dove I
concatenata
la corrente concatenata al circuito stesso, cio che attraversa la superficie che ha come
bordo il circuito.
Tale legge si dimostra molto semplicemente partendo dallequazione del rotore di B ed applicando il teorema
di Stokes.
Esercizio Si abbia un cilindro indefinito a sezione circolare, di raggio R, il cui interno percorso da una
densit di corrente j uniforme. Trovare il campo B in ogni punto dello spazio (vedi disegno che non c!)
Risposta Il campo potrebbe benissimo essere calcolato mediante luso diretto della formula di sopra (farlo
per esercizio!), ma preferisco trovare la risposta con mezzi pi elementari.
Consideriamo un punto a distanza r (r > R) dallasse del cilindro. Il vettore B, come tutti i vettori, avr tre
componenti: la componente radiale B
r
, una componente lungo lasse z (z diretto come lasse del cilindro)
B
z
ed una componente, ortogonale alle due, che indico con B. In pratica ho il versore r , il versore k ed il
versore r k

`
Ebbene, risulta : B
z
= 0
Infatti, dalla formula integrale di sopra, che fornisce B, essendo j diretto sempre come lasse z, lintegrando
ortogonale a z e quindi B non ha componente lungo tale asse, c.v.d.
Ma risulta anche: B
r
= 0
Infatti, in caso contrario, il flusso di B attraverso una superficie cilindrica chiusa, coassiale con la corrente, di
raggio r, ed alta h, per motivi di simmetria, sarebbe dato semplicemente da:
B
= 2rhB
r
. per il teorema
della divergenza per tale flusso deve essere nullo, essendo div B = 0 e quindi se ne deduce la tesi
Resta quindi solo da determinare B come funzione, per motivi di simmetria, del solo raggio. Resta cio da
determinare B. A tal fine, consideriamo una circonferenza nel piano x-y , centro sullasse del cilindro, e
raggio r e calcoliamo la circuitazione di B lungo tale circonferenza. Per motivi di simmetria , ovviamente,
B costante in ogni punto e quindi la circuitazione vale: 2r B. Per il teorema di Stokes, noi possiamo
uguagliare tale circuitazione al flusso di
0
j attraverso larea del cerchio corrispondente. Ora tale flusso, altro
non , per definizione, che
0
I, dove I la corrente che attraversa il cerchio o, come si dice, la corrente
concatenata al nostro circuito (=la circonferenza di raggio r). Si ottiene allora subito:
r
I
B

2
0

Come si vede da questa formula, se r > R, il campo B non dipende da R e cio il campo il medesimo di
quello generato da un filo rettilineo indefinito percorso da una corrente I
Si invece si vuole il campo in un punto interno (r < R), si ragiona come prima: la circuitazione di B sarebbe
sempre data da:
B
= 2rB, ma la corrente concatenata sarebbe ora: I = r
2
j. Uguagliando, si ha:
r
j
B
2
0


Osservazione
Supponiamo, come capita sovente, che la corrente sia costretta a fluire entro fili di diametro molto piccolo
rispetto alle distanze dai fili ai punti nei quali vogliamo determinare il campo. Risulta allora che la formula
generale:

spazio lo tutto
d '
'
) ' ( ) ' (
4
) (
3
3
0
r
r r
r r r j
r B

si pu semplificare un po. Infatti, detta dS la sezione


normale del filo, dl la lunghezza di un trattino infinitesimo del filo, si ha che dV = dS dl e quindi j dV = j dS
dl = Idl t dove t un versore diretto come j. Ma t , diretto lungo la tangente al filo e quindi dl t si pu
scrivere pi semplicemente dl. Quindi si ha:
81
j dV = Idl
e quindi si avr:

spazio lo tutto
dV'
'
) ' ( ) ' (
4
) (
3
0
r r
r r r j
r B

<=>

fili i tutti
d
I
3
0
'
) ' ( ) ' (
4
) (
r r
r r r l
r B

Indicando semplicemente con r il vettore che parte dallelementino di integrazione dl ed arriva al punto dove
vogliamo trovare il campo, la formula di sopra viene a scriversi:

fili i tutti
r
d
I
3
0
4
) (
r l
r B

Quindi, in pratica, per determinare la direzione ed il verso generato dalla corrente che percorre un filo
rettilineo, possiamo immaginare di stringere il filo con la mano destra posizionando il pollice nel verso della
corrente, lorientamento delle dita fornisce allora la direzione ed il verso delle linee del campo B.
Consideriamo ora un elementino di filo dl percorso da una corrente I. con un ragionamento analogo al
precedente, si ha che la forza magnetica agente su questo elementino, se immerso in un campo B , data da:
B l f Id d
Esercizio Consideriamo due fili rettilinei indefiniti paralleli percorsi da una corrente I
1
e I
2
. Sia D la loro
distanza. Calcolare la forza che si esercita tra i due, per unit di lunghezza.
Risposta. Il primo genera, dove c il secondo, un campo perpendicolare al secondo di intensit
D
I
B

2
0


Se nel secondo la corrente concorde (discorde) col primo, si avr allora, su di un tratto dl una forza di
attrazione (repulsione) data semplicemente dal prodotto del campo trovato moltiplicato I
2
dl. Quindi ho:
D
dl I I
dl I B df

2
2 1 0
2

allora la forza per unit di lunghezza data da:
D
I I
D
I I
f
2 1
7
2 1 0
10 2
2

Questa formula usata per definire lAmpre


Esercizio Una carica q entra , con velocit v, in una regione dove esiste un campo B, uniforme e costante.
Determinare il moto della carica.
Risposta: Il campo B sia diretto come lasse z. :
a i j k k j i B v f m B v B v q B v v v q q
y x z y x
+ + + ) ( ) (
Proiettando lungo i tre assi ottengo:
x y
y x
z
qBv a
qBv a
a

0
la prima equazione fornisce subito:
z = z
0
+v
0z
t.
Moltiplico ora ambo i membri della terza equazione per lunit immaginaria i e sommo alla seconda. Ho
allora:
a
x
+ ia
y
= qB/m(v
y
iv
x
) = i qB/m(v
y
/i v
x
) = - i qB/m(- v
y
/i + v
x
) = - i qB/m(v
x
+i v
y
)
Ponendo
y x
iv v Z + `
, lequazione scritta risulta essere:
82
m iqBZ
dt
dZ
/
Che fornisce subito:
m iqBt
e Z t Z
/
0
) (

e quindi ancora, integrando una seconda volta:
m iqBt
e
iqB
mZ
t iy t x
/ 0
) ( ) (

+
Tenendo presente la relazione di Eulero:

sin cos i e
i
+
ed uguagliando la parte reale di destra alla
parte reale di sinistra e la parte immaginaria alla parte immaginaria, si ha la soluzione:
m qB
m qBt v m qBt v i m qBt v qBt v
i m qBt i qBt
m qB
iv v
i t iy t x
x y y x y x
/
)] / sin( ) / cos( [ ) / sin( cos
)] / sin( ) [cos(
/
) ( ) (
0 0 0 0 0 0
+ +

+
+
qB
m qBt v m qBt v
m t y
qB
m qBt v m qBt v
m t x
y x
y x
) / sin( ) / cos(
) (
) / cos( ) / sin(
) (
0 0
0 0
+

La traiettoria del moto dunque un elica con lasse lungo z. la proiezione nel piano x-y una circonferenza
percorsa uniformemente e di raggio
qB
mv
r
0

e percorsa con velocit angolare = qB/m


Esercizio: nel piano orizzontale x-y fissato nellorigine O un filo elettrico flessibile e di sezione
trascurabile, che passa poi in un piccolo foro, senza attriti, di coordinate :(L,0). Il filo sostiene poi una massa
M sottoposta al suo peso. Risulta quindi che il filo sottoposto ad una tensione T = Mg. Si consideri ora la
presenza di un campo magnetico uniforme e costante diretto come z di valore B. Si vuol determinare la forma
del filo quando percorso da una corrente I procedente da O verso L
Risposta: su di un elementino dl la forza magnetica df = IdlB ortogonale allelementino stesso. Ne risulta
quindi che la tensione T si mantiene inalterata lungo tutto il filo. Introduco ora un versore n, variabile da
punto a punto, ortogonale al generico trattino di filo dl. Ho allora per la forza magnetica sul tratto dl:
df = (IBdl)n
Daltra parte, indicando con e i versori tangenti al filo agli estremi dellelementino dl del filo, si ha che
la forza df
filo
esercitata del resto del filo su questo elemento data da:
) ' ( '
, , , ,
T T T d
filo
f
Ma , trascurando gli infinitesimi dordine superiore:
dl dl
dl
d


n

,
, ,
'
dove il raggio di curvatura della curva assunta dal filo. Impomendo che la risultante delle forze agenti
sul tratto dl nulla, si ha:
dl
T
IB d d
filo
n f f

,
_

0
Da cui si ottiene subito:
IB
T

Il raggio di curvatura assunto dal filo dunque costante e questo significa che la curva un arco di cerchio.
Supponiamo che la corrente nel filo sia via via crescente partendo dal valore 0. La andr quindi
diminuendo partendo dal valore (filo rettilineo, corrente 0) finch, ad un certo punto, per un valore della
corrente che chiamer I
0
, il filo assumer la forma di un semi-cerchio di diametro OL e quindi sar =
0
=
OL/2. Al crescere ulteriore di I, non pu ulteriormente diminuire, essendo il filo obbligato a passare per O
e L. Ne consegue quindi che una soluzione statica non sar allora pi possibile: la massa M salir fino ad
urtare il piano orizzontale, a meno che non venga aumentata. La configurazione allora del filo sar allora
sempre un arco di cerchio, con >
0
, I > I
0
e T > Mg. Quindi, mentre prima del valore I
0
della corrente il
raggio di curvatura diminuisce al crescere della corrente, poi cresce al crescere di questa (o resta fisso se il
filo non pu pi fuoriuscire dal forellino se ha esaurito la sua lunghezza)
83
Esercizio: calcolare il campo magnetico nei punti dellasse di una spira circolare di raggio R e percorsa da
una corrente I costante.
Risposta: La spira sia nel piano x-y con centro nellorigine. nella formula:

fili i tutti
r
d
I
3
0
4
) (
r l
r B

si ha
(vedi figura) dl = ds (-sin i + cos j); r = z k R(cos i + sin j). Nel prodotto vettore posso tralasciare tutti
i termini lineari in cos ed in sin. Infatti, dovendo poi integrarsi su tutti i valori di tra 0 e 2, il contributo
di tali termini nullo. Resta dunque:
3 2 2
2
3 2 2
2 2
2
3 2 2
) (
1
) (
cos sin
) (
) sin (cos ) cos sin (
R z
d R
R z
d R
R z
Rds
+

+


+
+ +
k k
j i j i


Lintegrale dunque banalissimo: lintegrale di una costante e si ha allora:
k B
3 2 2
2
0
) (
2
) (
R z
R
I z
+


Se z >> R, con ottima approssimazione si ha:
k B
3
2
0
2 z
I R

A questo punto opportuno ricordare lespressione del campo elettrico generato da un dipolo in un punto
dellasse del dipolo medesimo:
3
0
2
1
z
p
,

E
Si vede quindi che, se introduciamo il vettore H, detto: campo magnetico:
H=B/
0
si ha subito, in punti sullasse molto distanti dalla corrente:
3
*
0
3
2
0
0
2
1
2
1
z z
I R



,
k H
dove il vettore
*
detto momento magnetico della spira ed , per definizione,

*
=
0
IS
(S il vettore che rappresenta la spira, quindi ortogonale al suo piano, lungo come la sua area ed orientato in
modo che la corrente I, percorrendo la spira, lasci la superficie di questa alla sinistra). Si dimostra che anche
se la spira non ha forma circolare, il campo B (o H) sul suo asse a grande distanza da questa dato dalla
stessa formula di sopra. Per questo motivo una piccola spira percorsa da corrente prende il nome di dipolo
magnetico
Esercizio: calcolare il campo magnetico nei punti del piano di una spira circolare di raggio R e percorsa da
una corrente I costante.
Risposta: Dal disegno di sotto (dove D la distanza del punto P nel quale si calcola il campo dal centro della
spira) risulta: ) cos( 2
2 2 2
DR R D r + e poi anche:
rR
R DR
2
) cos(
) cos(


,sicch:

+

d
DR R D
R D
RI
r
d
I P
fili i tutti
3
2 2
0
3
0
) cos( 2
) cos(
4
) (
4
) ( k
r l
B
Quindi, facendo uno sviluppo in serie di D, per D che tende ad , si ha, tralasciando i termini in D
5
:
84
+
+
+

+
+

4
2
2
3
3
2 3
2 2
)] ( cos
2
15
2
9
)[ cos(
1 ) ( cos 3 ) cos(
) cos( 2
) cos(
D
R R
D D
DR R D
R D

e quindi integrando ottengo:


3
2
0
4
1
) ( ) (
D
I R
P

k B
E quindi ottengo che se il punto P molto distante dalla spira (D >> R), si ha, tralasciando termine
dellordine di 1/D
5
:
3
*
0
4
1
D


,
H
A grande distanza dalla spira, non sar possibile differenziare la forma della spira da una circolare e quindi
verosimilmente ( e cos risulta) anche se la spira non ha forma circolare, il campo B (o H) a grande distanza
da questa dato dalle stesse formule di sopra.
Si ottenuto quindi che il campo H generato da una spira, a grande distanza da questa, per punti
sulasse o nel piano, il medesimo del campo E di un dipolo elettrico lungo la sua direzione e nel piano
diametrale. Ripetendo allora i ragionamenti fatti per determinare il campo di un dipolo in ogni punto dello
spazio, si ottiene allora che:
a grandi distanza da una spira in un punto individuato dal vettore r che ha origine nella spira ( non importa
precisare esattamente il punto, perch a grandi distanze la spira pu essere considerata puntiforme). Si ha:
1
]
1

3
*
5
*
0
) ( 3
4
1
r r


, ,
r r
H
Ricordo che per il dipolo elettrico :
1
]
1

3 5
0
) ( 3
4
1
r r
p r r p
E

Per questa analogia dei campi, una piccola spira percorsa da corrente prende il nome di dipolo magnetico. Le
due facce della spira, che corrispondono nel caso elettrico alle cariche negativa e positiva, sono sono dette
ora poli; precisamente polo Nord la faccia da cui escono le linee di B e polo Sud la faccia nella quale entrano
le linee di B. Si ha dunque, che se noi facciamo la sostituzione:
*
0 0
,
,
p
lespressione del campo E generato dal dipolo elettrico si trasforma nellespressione del campo H generato
dal dipolo magnetico. Mi chiedo ora se lanalogia si spinge oltre e precisamente se la forza che agisce tra due
dipoli elettrici si trasforma nella forza tra due dipoli magnetici facendo semplicemente le sostituzioni
indicate sopra. A tal fine svolgo il seguente
Esercizio: due piccole spire circolari sono percorse da una corrente I
1
e I
2
esse giacciono su piani paralleli a
distanza z ed i loro assi coincidono con lasse z. Trovare la forza reciproca..
Risposta Se le correnti sono equiverse, ci sar una forza di attrazione, altrimenti di repulsione. Quindi una
forza lungo z. La spira 1 produce dove c la spira 2 un campo magnetico. La componente radiale di tale
campo magnetico produrr sulla corrente I
2
una forza lungo z il cui valore sar dato semplicemente da:
r
B I R f
2 2
2
Per valutare B
r
si pu procedere cos:
consideriamo un cilindretto di base la spira 2 ed altezza infinitesima h. Scriviamo che il flusso di B attraverso
lui nullo. Il flusso attraverso le due basi vale (a parte un segno):
h R
z
R
h z z
R
h z z
2
2
4
*
1 2
2
3 3
*
1 2
2
3
*
1
3
*
1
3
2
]
) (
1 1
[
2
]
) (
[
2
1


+

+

Il flusso attraverso la superficie laterale :


h R B
r 2
2
Uguagliando i due flussi si ha:
85
4
2
*
1
3
4 z
R B
r

e quindi, per la forza, si ha subito:


4
*
2
*
1
0
4
*
1
2
2
2
6
4
1 3
4
2
z z
I R f


Per i dipoli elettrici allineati, ricordo che la forza era:
4
2 1
0
6
4
1
z
p p f

Si visto dunque che lanalogia tra campo di dipolo elettrico e magnetico vale, oltre che per i campi, anche
per le forze dinterazione, almeno se i due dipoli sono allineati, ma si pu mostrare che vale in generale e
cio anche quando i dipoli non sono allineati.
A questo punto, nulla vieta di immaginare il dipolo magnetico costituito in analogia con il dipolo elettrico,
cio da due masse magnetiche infinite, di segno opposto, ad una distanza reciproca infinitesima. Si
dunque indotti a porre:
d S
magnetica
q I
*
0
*
`
,
Le due grandezze: q*
magnetica
e d non sono per ora univocamente determinate dalla relazione di sopra, dato
che solo il loro prodotto risulta fissato e pari a
0
IS; si pu pensare dunque che la q*
magnetica
soffra di una
indeterminazione dovuta ad un fattore moltiplicativo arbitrario.
La forza dinterazione elettrica tra dipoli sorgeva da una forza dinterazione coulombiana tra cariche
inversamente proporzionale al quadrato della distanza. Siccome si visto che linterazione tra dipoli
magnetici ha la medesima espressione dellinterazione tra dipoli elettrici, ne deriva che la forza
dinterazione tra cariche magnetiche pure lei inversamente proporzionale al quadrato della distanza e si
avr dunque fra due cariche magnetiche:
3
2 1
2 1
0
*
2
*
1
1
4
r r
r r
f


magnetica magnetica
q q
Legge di Coulomb per la magnetostatica
Carica magnetica unitaria sar dunque quella che, posta di fronte ad una uguale alla distanza unitaria (cio un
metro), risente di una forza di 1/4
0
= 63325,7 N.
E come una carica elettrica puntiforme generava un campo elettrico E, cos una carica magnetica genera un
campo magnetico H dato da:
3
*
0
4
1
r
q
magnetica
r
H

mentre : H B
0
e quindi il campo dinduzione magnetica generato da carica magnetica puntiforme
dato da:
3
*
4
1
r
q
magnetica
r
B

Osservazione. Questo modo di procedere, cio lintroduzione di cariche magnetiche puntiformi, che
generano un campo magnetico mediante la legge di Coulomb magnetica, un po demod, imperversava
invece tempo addietro. Come mai questo cambio di moda? la ragione pi forte forse nel fatto che cariche
magnetiche isolate non sono mai state trovate. La pi piccola unit di carica elettrica stata trovata (
lelettrone o il protone). La pi piccola unit magnetica trovata non la singola carica, ma il dipolo
(magnetico), cio una corrente che percorre una piccolissima spira. Se si trovasse una carica magnetica
isolata, il flusso di H attraverso una superficie chiusa che la contiene sarebbe q
*magnetica
/
0
(il teorema di Gauss
vale per qualunque campo centrale che dipende dallinverso del quadrato della distanza!) e quindi il flusso di
B sarebbe q
*magnetica
e quindi sarebbe violata lequazione di Maxwell che vuole nulla la divergenza di B. Se
86
per noi ci limitiamo ad una trattazione della magnetostatica macroscopica e cio ci manteniamo sempre a
distanze notevoli dalle spire elementari di corrente che costituiscono i dipoli magnetici (e queste spire
elementari altro non sono che le correnti prodotte negli atomi dal moto degli elettroni che ruotano intorno ai
nuclei, cio le cosiddette correnti ampriane), una descrizione in termini di dipoli magnetici pu essere
molto utile. Cos una barretta magnetica (calamita) altro non che un agglomerato di dipoli magnetici tutti
ugualmente orientati.
Esercizio: trovare il campo sullasse di un solenoide cilindrico nel punto di ascisse z (il solenoide un filo
avvolto uniformemente su di una superficie cilindrica. Le spire sono molto vicine tra loro, sicch ogni spira
pu considerarsi piana) se il solenoide costruito con n
1
spire per unit di lunghezza e nel filo scorre una
corrente I
1
. Il solenoide si estende da: -L
1
a 0
Risposta: Il solenoide pu considerarsi come un insieme infinito di anelli, ognuno di altezza dx e raggio R
1
e
quindi percorso da una corrente dI = n
1
I
1
dx. Il generico anello dista z+L
1
-x dal punto dellasse nel quale
vogliamo trovare il campo, con x che va da 0 a L
1
. Il campo di un siffatto anello in un punto dellasse
distante z+L
1
-x dal suo piano vale (si visto sopra):
3 2
1
2
1
2
1
1 1
0
) ) ((
2
R x L z
R
dx I n dB
+ +


Integrando trovo il campo:
1
1
]
1

+ +
+

+ +


2
1
2 2
1
2
1
1 1 1 0
0
3 2
1
2
1
2
1 1 1
0
) (
2
) ) ((
2
) (
1
R z
z
R L z
z L I n
R x L z
dz
R I n dB z B
L

Se il solenoide infinitamente lungo e se si vuole il campo in un punto interno, basta porre nella formula
precedente: z = - L
1
con 0<<1 e poi calcolare il limite dellespressione di sopra per L
1
-> infinito. Si ottiene
allora il risultato ( giustamente non dipendente da ):
1 1 0
I n B
campo interno sullasse solenoide infinito
Esercizio: Una piccola spira circolare di raggio R
2
percorsa da una corrente I
2
ed il suo centro si trova
allascissa z dellesercizio di sopra. Calcolare la forza agente su di lei.
Risposta: Per trovare il campo B
r
generato dal solenoide agente sulla spira, ragiono come in un esercizio
precedente, imponendo che il flusso di B attraverso un cilindretto sia nullo:
z
B
R z B dz z B R dz
z
B
R
r r

2 2
2
2
2
1
) ( 0 ) ( 2
La forza sulla spira allora:
z
B
R I R z B I R f
r


2
2 2 2 2 2
) , ( 2
Se al posto della spira c un secondo solenoide che si estende da z=Z a z= Z+L
2
che possiede n
2
spire per
unit di lunghezza, la forza agente allora:
[ ]

+
+


2
) ( ) ( ) , ( 2
2
2
2 2 2
2
2 2 2
2
2 2 2 2 2 2 2
L Z
Z
r
Z B L Z B R I n dz
z
B
R I n f dz
z
B
R I n dz R z B I n R df
Sostituendo lespressione per B ottengo:

'

1
1
]
1

+ +
+

1
1
]
1

+ +
+

+ + +
+ +

2
1
2 2
1
2
1
1
2
1
2
2
2
2
1
2
2 1
2 1
2
2 2 2
1 1 0
) ( ) ( ) (
2
R z
z
R L z
z L
R L z
L z
R L L z
L z L
R I n
I n
f

Questa espressione, se L
1
ed L
2
tendono ad infinito ed R
1
tende a 0, tende a ( a parte un segno):
2
2 2 2 0 1 1 1 0
0
) )( (
4
1
z
S I n S I n
f

87
Che la stessa espressione della legge di Coulomb se si ammette che un solenoide porti alla sua estremit
una massa magnetica (o carica magnetica) data da:
nIS q
magnetica
0
*
`
ed allora la forza di Coulomb magnetica data da:
2
*
2
*
1
0
4
1
r
q q
f
magnetica magnetica

Se si vuole scrivere la forza come:


H f
*
q
allora il campo magnetico prodotto da una carica magnetica puntiforme :
3
*
0
4
1
r
q
magnetica
r
H

mentre , essendo:
H B
0

allora.
3
*
4
1
r
q
magnetica
r
B

Esercizio: mostrare che allinterno e allesterno di un solenoide infinitamente lungo il campo B non ha
componente perpendicolare allasse del solenoide ( cio non ha componente radiale)
Risposta, (cenni) se , per assurdo lavesse, considerata una scatola cilindrica chiusa, interna al solenoide e
coassiale ad esso, il flusso di B attraverso la superficie di questa sarebbe semplicemente il prodotto di B
r
per
la superficie laterale della scatola. Ma tale flusso deve essere nullo, quindi B
r
= 0
Esercizio: si consideri un solenoide cilindrico infinitamente lungo, con lasse lungo z. Sia r il versore radiale
uscente da un punto P interno o esterno al solenoide. Mostrare che il campo B nel punto P non ha
componente lungo lasse r k

`
Risposta, (cenni) Se, per assurdo lavesse, chiamatala B

, la circuitazione di B lungo la circonferenza


passante per P e centro sullasse, di raggio r , varrebbe 2rB

. Ma tale circuitazione, che pari al flusso del


rotore di B , cio al flusso di j attraverso il cerchio racchiuso da tale circonferenza nullo perch nullo tale
flusso, cio la corrente concatenata con tale circuito
Esercizio: si consideri un solenoide cilindrico infinitamente lungo, con lasse lungo z e raggio R. Mostrare
che B
z
nullo in ogni punto esterno al solenoide.
Risposta (cenni): si consideri infatti un rettangolo avente un lato (di estremi P e Q) unitario disposto lungo
lasse z del solenoide e laltro (di estremi Q e S) di lunghezza maggiore di R. La circuitazione di B lungo i
lati di tale rettangolo . Per il teorema di Stokes, pari alla corrente concatenata moltiplicata
0
, cio
0
nI Ora
il contributo alla circuitazione che proviene dal tratto PQ posto sullasse il valore di B sullasse, dato che il
tratto dintegrazione unitario, e quindi questo contributo pari a
0
nI. Nei tratti poi perpendicolari a questo
il contributo nullo, dato che si visto che B
r
= 0. ne consegue allora che deve essere nulla la B
z
nei punti
del tratto parallelo a PQ, cio in un punto generico fuori dal cilindro. Dai tre esercizi risulta dunque che il
campo B fuori da un solenoide infinitamente lungo nullo. Un ragionamento analogo a quello ora fatto
permette di mostrare che Il campo B allinterno di un solenoide infinitamente lungo ha solo componente
lungo lasse del solenoide e che tale componente costante e quindi vale
0
nI
Campi variabili
Consideriamo una filo elettrico a forma di spira rettangolare, di lati a e b, vertici A
1
,A
2
,A
3
,A
4
, che si muove
con velocit costante V parallela ad b. al tempo t = 0, entri in una regione dove vi presente un campo B
uniforme e costante perpendicolare al piano della spira. Si osserva che a partire da t = 0 fino a che tutta la
spira si trova nel campo, cio fino al tempo t = b/V (poi cessa) scorre nella spira una corrente I data da: I =
VBa/R, dove R la resistenza dellintero circuito. Se orientiamo la spira in modo che, percorrendola secondo
lorientazione, la superficie resta alla sinistra ed il campo B diretto come lomino che percorre la spira, la
corrente in questione scorre in verso opposto allorientazione scelta (vedi disegno). Ai fini dunque della
88
corrente che si determina, tutto avviene come se nel circuito fosse inserito un generatore di tensione di forza
elettromotrice(=differenza di potenziale): - VBa
Come mai accade ci? facile da spiegare: su ogni elettrone di conduzione che si trova nel tratto di filo
lungo a immerso nel campo, agisce una forza magnetica diretta lungo il filo e di valore VB e , dato che
dotato della medesima velocit V che possiede la spira. Questa forza la medesima che si avrebbe se nel filo
fosse presente un campo elettrico E, diretto lungo il filo di valore VB, ovverosia si avesse agli estremi del
tratto a una differenza di potenziale pari a aVB. Questa differenza di potenziale produce quindi la corrente
sperimentalmente osservata. Osserviamo poi che la forza magnetica agente sugli elettroni di conduzione nei
tratti che portano quello lungo a tende a deformare il circuito, ma non ad ostacolare o favorire la corrente.
Supposto il circuito rigido, tale forze risultano inefficaci. Quando poi tutto il circuito inserito, si vede subito
che sullultimo tratto sorge un campo diretto come nel primo che ferma quindi la corrente. Le cariche del
tratto A
3
-A
4
, muovendosi lungo il tratto medesimo, dato che generano una corrente, avranno una velocit v
diretta come il tratto. Ora, la forza di Lorentz che sorge a causa di questa velocit d luogo ad una forza che
si oppone allinserimento della spira nel campo, sicch, per introdurla, occorre spendere un certo lavoro,
lavoro che si ritrova dissipato per effetto Joule nella resistenza della spira.
Da quanto detto, sembrerebbe tutto spiegatoma come avrebbe dovuto ragionare un osservatore solidale
alla spira? Per lui la spira ferma e dunque non c nessuna forza di Lorentz! E perch allora si mette a
scorrere la corrente? Siccome il campo B si visto che inefficace, deve entrare in gioco per forza il campo
E (visto che la carica risento solo di forze elettriche e magnetiche). Bench non ci siano cariche elettriche in
giro (ogni porzione microscopicamente infinitesima dei conduttori scarica!) deve sorgere un campo E. Di
questo campo per ora sappiamo ben poco, n quanto vale n come diretto in un generico punto, ma
certamente, per dare la corrente giusta, la sua circuitazione lungo la spira deve dare, in valore assoluto,
VBa. In condizioni non stazionarie dunque il campo elettrico non conservativo.
Dallesperienza sopra eseguita e dal teorema di Stokes, segue dunque:


S
d d VBa S E l E ) (
dove S la superficie del circuito. Ma Va non altro che la derivata temporale, per losservatore solidale al
circuito, della superficie della spira attraversata dal campo B e quindi VBa non altro che (a parte
eventualmente un segno) d/dt
B
. cio la derivata temporale del flusso di B attraverso il circuito. Ho dunque
(a parte eventualmente un segno):


S s
d d VBa d
dt
d
S E l E S B ) (
Ora, essendo ogni parte del circuito fisso per losservatore a lui solidale, ho che:


S s
d
t
d
dt
d
S
B
S B
E quindi scrivo la relazione di sopra come:

S S s
d
t
d d
t
S B E S E S B ) ( ) (
Per larbitrariet della superficie S, si deduce:
t


B
E
Equazione di Maxwell, valida sempre
Osservazione importante Il segno nella relazione di sopra assolutamente fondamentale: se ci fosse infatti
il +, sarebbe violato il principio di conservazione dellenergia. Infatti consideriamo una spira nella quale
circoli una corrente I . Si avrebbe:

Spira
r
d
I
3
0
4
) (
r l
r B

. Il flusso di B attraverso la spira sarebbe allora


dato da: KI
B
con K costante positiva (perch?) dipendente dalla forma della spira. Da Stokes e da Ohm
si avrebbe:

I
dt
d
K
dt
d
d RI
B
l E
89
da cui subito: I(t) = I
0
e
(R/K ) t
e quindi una corrente indotta casualmente in un circuito crescerebbe
esponenzialmente con generazione di infinito calore per effetto Joule. Se invece c il segno -, si ha:

I
dt
d
K
dt
d
d RI
B
l E
da cui subito: I(t) = I
0
e
(R/K) t
e quindi una corrente indotta casualmente in un circuito decresce
esponenzialmente estinguendosi, come deve intuitivamente essere.
Siamo dunque arrivati stabilire che:
vaiabili lentamente campi sempre valida
sempre valida
t
sempre valida
j B B
B
E E
0
0
, 0
,


Osservazione di carattere matematico: Prima di questultima correzione al rotore di E, il campo elettrico ed il
campo dinduzione magnetica erano disaccopiati, nel senso che per la risoluzione delluno, non occorreva
conoscere laltro (per E bastava conoscere la e per B la j) ora invece, per determinare E, occorre conoscere
B, (o meglio, la variazione di B nel tempo) mentre B ancora indipendente da E ( ma vedremo che nel caso
generale non pi cos). Quindi, se si vorranno determinare delle correnti in circuiti, cio, per la legge di
Ohm, dei campi elettrici, occorrer tenere conto delle variazioni temporali di B ( vedremo presto come),
mentre il B potr essere dedotto dalle correnti sempre mediante la solita espressione che fornisce B nel caso
non stazionario, cio

dV
r
3
0
4
r
j B

( anche se cos non siamo, ma non ne siamo pure troppo discosti!)


Dunque nei campi stazionari valeva lequazione: 0 E , mentre nel caso generale, lequazione valida ,
come si scritto sopra:
t


B
E . Notate che questa equazione si riduce alla precedente nel caso
stazionario (= nulla dipende dal tempo e quindi le derivate temporali sono nulle).
NON LEGGERE FINO ALLA RIGA DI ASTERISCHI
Voglio ora mostrare che lequazione:
t


B
E pu essere dedotta dal seguente ragionamento. Dalla
legge di trasformazione delle accelerazioni sappiamo che se in un sistema di riferimento inerziale una
particella possiede accelerazione a (e quindi sottoposta ad una forza ma), allora in ogni altro sistema di
riferimento inerziale laccelerazione la medesima ( e quindi agisce su di lei la medesima forza).
Consideriamo dunque un sistema di riferimento inerziale S ed una carica q. Secondo losservatore solidale ad
S sulla carica q agisce un campo elettrico E ed un campo dinduzione magnetica B, per cui la forza risultante
:
f = q(E + v B)
Un osservatore solidale ad un sistema di riferimento S in moto relativo uniforme con velocit V rispetto ad
S, vede agire, per quanto detto sopra, la medesima forza f. Con ci non possiamo allora dedurre che
losservatore S ritenga che il campo elettrico misurato da lui, campo che indicher con E sia uguale al
campo E e che il campo B misurato da lui sia uguale al campo B, ma solo che deve aversi:
q(E + v B) = q(E + v B)
dove v la velocit della carica q misurata da S. Dalle leggi di trasformazione delle velocit daltra parte
sappiamo che : v = v V e quindi :
q(E + v B) = q[E + (v-V) B]
da cui subito:
E E + v (B B) = - V B
Nel secondo membro il campo B il campo dinduzione magnetica che agisce sulla carica q e non dipende
certo da alcunch della carica q ed in particolare non dipende dalla velocit v di questa carica. Quindi il
secondo membro non dipende da v e quindi non pu dipendere da v anche il primo membro. Se ne deduce
quindi che deve essere (B B) ortogonale ad un vettore v che daltra parte arbitrario. Ne consegue quindi
che (B B) deve allora essere il vettore nullo e cio:
90
B = B
E quindi poi subito:
E = E + V B= E + V B
Detto questo, passiamo ora a considerare un altro fatto. Si consideri un generico campo vettoriale A,
variabile con la posizione e con il tempo. Scriveremo quindi A = A(r,t). Immaginiamo ora che in questo
campo vettoriale sia immersa una superficie di bordo che si muova attraverso questo campo ed
eventualmente anche deformandosi durante il suo moto. Variando nel tempo sia il campo A che la superficie,
si avr che il flusso del campo attraverso la superficie varier nel tempo e ci si potr chiedere di valutare la
derivata rispetto al tempo di questo flusso, cio di valutare:


) (
) , (
t Superficie
d t
dt
d
S r A
Ebbene, si pu dimostrare ( ma la dimostrazione ve la risparmio!) che vale la seguente relazione:
[ ] [ ]

'

+ +


) ( ) (
) , ( ) , ( ) , (
) , (
) , (
t Superficie t Superficie
d t t t
t
t
d t
dt
d
S v r A r v r A
r A
S r A
(La formula pu apparire un po scostante, ma non vi preoccupate! La v che compare la velocit del
generico punto r della superficie che si sta muovendo.
Applichiamo la formula di sopra quando A un campo magnetico B variabile da punto a punto ma costante
nel tempo nel sistema di riferimento S. Consideriamo ora una spira (il circuito ) collocata in un sistema di
riferimento inerziale S in moto rigido senza deformarsi,con velocit V rispetto ad S. In S la spira sia ferma,
ma cos non in S. Per un osservatore solidale ad S la formula di sopra diventa: (B costante nel tempo e
0 B )
[ ] [ ] { }

+ +
) ( ) (
) , ( ) , ( 0 0 ) , (
t Superficie t Superficie
d t t d t
dt
d
S V r B r V S r B
Ma per losservatore solidale ad S la superficie non cambia nel tempo e quindi


S t Superficie
d
t
t
d t
dt
d
S
r B
S r B
) , (
) , (
) (
Rinsaldati nella nostra fede dalla relazione vettoriale di sopra, potremo dire:

S S
d d
t
t
S V B S
r B
)] ( [
) , (
Ma : ' E V B dove E il campo misurato dallosservatore in moto. Ma B = B, dato che E = 0 ed
allora il flusso di B attraverso S il medesimo se misurato da terra o da un osservatore solidale al sistema in
moto. Si ha quindi:

S E S V B S r B S r B S
r B
d d d t
dt
d
d t
dt
d
d
t
t
S S S S
' )] ( [ ) , ( ) , ( '
) , ( '
e quindi:
0 ] '
) , ( '
[ +

S
d
t
t
S E
r B
Per larbitrariet di S, lintegrando nullo. Tale relazione stata ottenuta sotto lipotesi che B fosse costante
nel tempo in un altro sistema di riferimento. Ne postuleremo al validit in ogni caso
***************************************************************************************
Con la correzione apportata al rotore di E, si pu ora spiegare la presenza della corrente nel circuito! Infatti
la circuitazione di E lungo il circuito ( che sarebbe poi la forza elettromotrice del generatore fittizio
introdotto) uguale, per Stokes, al flusso del suo rotore attraverso la superficie della spira, cio alla derivata
temporale ( col segno -) del flusso di B attraverso la spira. Ora, al variare del tempo, il flusso di B aumenta
perch sempre maggiore la porzione dellarea della spira coperta dal campo magnetico Precisamente, in
un tempuscolo dt larea coperta aumenta di aV dt e quindi il flusso di : aV Bdt. Quindi la derivata temporale
del flusso -aVB che corrisponde proprio, segno compreso alla forza elettromotrice del generatore fittizio!
91
Ecco dunque in estrema sintesi leffetto della correzione del termine del rotore sulle correnti nei circuiti:
Le correnti nei circuiti elettrici, qualora siano lentamente variabili, possono calcolarsi con i metodi usati nel
caso stazionario purch si aggiungano, in ogni maglia, dei generatori di forza elettromotrice di valore e
segno dati dalla derivata temporale del flusso di B (col segno cambiato)nella maglia in questione
Osservazione: che cosa sintende quando si dice lentamente variabili? Hic sunt leones! Mi limiter a dire
che quanto pi lente sono le variazioni, tanto meglio funziona la ricetta.
Esercizio: un filo elettrico, di resistenza R, forma un circuito chiuso ed posto in un campo B variabile.
Calcolare il lavoro compiuto dal campo elettrico nel far percorrere ad una carica q tutto lanello
Risposta la diversit da 0 di tale lavoro mostrer, ancora una volta, che nel filo scorre una corrente.
Calcoliamo tale lavoro:

dt
d
q d
t
q d
t
q d q d q L
S
fermo circuito
S
Maxwell per
S
Stokes per
) (
) (
B
S B S
B
S E l E

e tale lavoro lo stesso che verrebbe compiuto da un generatore inserito nel circuito di forza elettromotrice :
E = -d/dt
Quindi, anche ai fini del lavoro compiuto sulle carche in un circuito, un campo magnetico variabile
sostituibile con un opportuno generatore
Lautoinduzione
Consideriamo un circuito elettrico costituito per semplicit da una sola maglia di resistenza R. Sia esso, per
chiarire le idee, piano e giaccia nel piano x-y. Si possa chiudere il circuito muovendo un tasto. (vedi
figura). Scegliamo un verso di percorrenza del circuito (solitamente, ma non necessariamente, quello che
lascia a sinistra la superficie) Questa scelta di percorrenza del circuito determina la direzione del vettore che
rappresenta la superficie: quello determinato dalla regola della mano destra( cio, si supponga di stringere
con la mano il circuito, con le dita orientate nel senso della percorrenza; il pollice fornir la direzione ed il
verso del vettore superficie). La corrente sar positiva se lo percorre nello stesso verso, negativa altrimenti.
Nel circuito inseriamo un generatore di tensione di forza elettromotrice E (NON confondete con il
campo elettrico!!!). Allistante t = 0, chiudiamo il tasto. Vogliamo determinare la corrente nel
tempo. Osserviamo a tal fine che, se valessero che valgono nel caso stazionario, la corrente sarebbe molto
semplicemente: zero prima della chiusura del tasto e: +E/R da subito dopo in poi. Quindi la corrente sarebbe
discontinua al tempo 0. ma allora, nellintorno di 0 il fenomeno non per nulla stazionario! Sar quindi
opportuno usare almeno le equazioni valide per i fenomeni lentamente variabili nel tempo. Se circola una
corrente I nel circuito, questa determiner un campo magnetico che, dato che le equazioni per B si assumono
essere uguali a quelle che reggono il fenomeno stazionario, sar dato dalla stessa formula che fornisce il
campo nel caso stazionario, e cio:

fili i tutti
r
d
I
3
0
4
) (
r l
r B

( il circuito si assume filiforme). Questa


formula mostra chiaramente che B proporzionale alla corrente I e quindi anche il flusso di B attraverso il
circuito medesimo proporzionale ad I. (Notate che per definire il flusso di B era necessario definire il
vettore della superficie, cosa che stata fatta. Se si inverte il verso di percorrenza del circuito, cambia pure
verso il vettore della superficie e quindi il segno del flusso, ma i conti finali saranno sempre corretti, cio
coerenti con il verso di percorrenza) Potr dunque scrivere:
I L ) (B
Il coefficiente L di proporzionalit dipende solo dalla geometria del circuito e si chiama: coefficiente di
autoinduzione e la sua unit di misura si chiama henry (simbolo H) in onore di un fisico di tal nome.
Lordine di grandezza di tale coefficiente nei circuiti che si incontrano normalmente dellordine del
millesimo di H, ma pu essere aumentato notevolmente in presenza di materiali magnetici ( quali il ferro)
Sul perch il ferro faccia aumentare L, non dato qui di approfondire. A grandissime linee si pu dire che il
ferro, in presenza di un campo B si magnetizza, cio sorgono sulla sua superficie delle correnti elettriche a
livello atomico molto intense che incrementano il valore di B. Torniamo alla formula dellautoinduzione. Se
I varia, varia di conseguenza il flusso di B attraverso il circuito e quindi la corrente che scorrer nel circuito
sar la stessa che si avrebbe se nel circuito fosse inserito, oltre al generatore reale, uno di f.e.m.:
92
E
*
=
dt
dI
L
Particolare attenzione deve essere data al segno - : inserire un generatore negativo (positivo) nel circuito,
vuol dire inserire il generatore in moto tale che la corrente che ne sorgerebbe, sarebbe negativa (positiva).
ora importante notare che il coefficiente L sempre positivo. Quindi, se abbiamo inserito nel circuito delle
batterie la cui differenza di potenziale (positiva) pu essere variata a piacere dallo sperimentatore e noi
aumentiamo a bella posta nel tempo questa differenza di potenziale, la I nel circuito va ovviamente
crescendo e allora, per il fenomeno dellautoinduzione sopra descritto, avviene che la corrente effettiva che si
instaura cresce s, ma meno di quanto crescerebbe senza il fenomeno dellautoinduzione; infatti a causa di
questo, avviene come se si inserissero nel circuito delle batterie aggiuntive di forza elettromotrice negativa
Quindi:
un incremento (o decremento) della corrente crea una forza elettromotrice che si oppone (ci vuol dire che
genera una corrente) allincremento che lha generata (Legge di Lenz)
Ritorniamo al nostro circuito. La forza elettromotrice efficace inserita dunque: E

+ E
*
. per la legge do
Ohm, vi sar nel circuito una corrente data da: I = (E

+ E
*
)/R. Sostituendo si ottiene:
R I = E

L dI/dt
Questa unequazione differenziale del primo ordine lineare a coefficienti costanti la cui soluzione generale
:
+
t
L
R
Ae t I ) (
E/R
Con A costante arbitraria da determinarsi in base ad una condizione iniziale. La condizione in questione, nel
caso presente di imporre che la corrente allistante iniziale sia 0. deve quindi essere:
0
) 0 (
L
R
Ae I

+ E/R =0 quindi A = - E/R


e quindi :
) 1 ( ) (
t
L
R
e t I


E/R
Come mostra questa soluzione, si ha che la corrente non raggiunge istantaneamente il suo valore di regime
E/R ma, a rigore, non vi arriva mai. In pratica per, dopo un periodo di tempo di qualche ( = L/R), la
differenza indistinguibile. Notare anche che questa relazione implica, per la conservazione dellenergia che
il coefficiente L di autoinduzione sia, come detto sopra, positivo.
Si soliti anche dire che al momento della chiusura del circuito, si crea subito la corrente di regime E/R
ma anche una extracorrente di chiusura, di verso opposto data da
t
L
R
e

E/R che inizialmente tanto forte


da annullare completamente quella di regime, ma che poi si estingue nel tempo.
Osservazione ci si pu chiedere perch imporre che la corrente sia nulla allistante 0. Se non considerassimo
lautoinduzione, la corrente sarebbe discontinua: prima 0 e poi E/R. Perch imporre allora al tempo 0 il
valore 0 e non E/R od un altro ancora? Il fatto che lequazione che determina la corrente contiene una
derivata di I. Ma se una funzione (in una variabile) derivabile, deve essere continua. Quindi, siccome il
limite sinistro di I, per t che tende a 0 ovviamente 0, tale deve essere anche il limite destro ed quindi
chiara la ragionevolezza della condizione iniziale
Osservazione Consideriamo ora il nostro circuito molto tempo dopo la chiusura del tasto. L extracorrente
di chiusura orami svanita e nel circuito vige ora solo quella a regime data da E/R. Ad un certo istante,
che senza perdita di generalit possiamo ancora supporre essere listante 0, si cortocircuita il generatore di
tensione, cio si fa crollare istantaneamente a 0 la sua d.d.p. Determinare la corrente nei tempo seguente.
Risposta: la soluzione generale di sopra va sempre bene, basta porre E = 0 e la condizione iniziale nuova.
Si ottiene cos:
t
L
R
e I t I

0
) (
Questa corrente che residua nel circuito mentre non dovrebbe esserci dato che non c pi il generatore, si
chiama extracorrente di apertura
93
se L, il coefficiente di autoinduzione, fosse negativo, una corrente fortuita iniziale nel circuito andrebbe
crescendo da sola nel tempo e la nostra resistenza elettrica, scaldandosi sempre pi assicurerebbe un
riscaldamento gratuito per tutta la vita! Troppo bello per essere vero. Quindi:
Per ogni circuito elettrico il coefficiente di autoinduzione una grandezza positiva
Supponiamo ora che nel nostro circuito con generatore, autoinduttanza e resistenza, si voglia interrompere
istantaneamente la corrente, per esempio, aprendo il circuito, cio sollevando il tasto che prima si era
premuto. Che cosa succede? Un bel guaio! Infatti, aprire il circuito vuol dire, dal punto di vista circuitale,
inserire dove cera linterruttore, una resistenza enorme, R*, al limite infinita- quindi la corrente che circola
data da:
t
L
R R
e I t I
*
0
) (
+

e la potenza che viene dissipata nella resistenza R* data da : W = R*I


2
=
t
L
R R
e I R W
*
2
2
0
*
+

Si ha dunque una potenza enorme(che dura per un tempo brevissimo) e questa potenza molto probabilmente
brucia linterruttore e la corrente dovendo passare per forza passa attraverso laria che separa i due
conduttori ormai staccati formando una scintilla.
Esercizio: determinare il coefficiente di autoinduzione, per unit di lunghezza, L di un solenoide indefinito
caratterizzato da n spire per unit di lunghezza.
Risposta: essendo indefinito, il campo B allinterno costante. Essendoci n spire per unit di lunghezza,
detta S la sezione, il flusso, in un tratto unitario : nSB. Ma B =
0
nI e quindi il flusso : SI n
B
2
0

E quindi subito:
S n L
2
0

Esercizio consideriamo un circuito elettrico costituito da una resistenza R e che possiede un coefficiente di
autoinduzione L. Nel circuito inseriamo un condensatore di capacit C caricato inizialmente con una carica
+Q. al tempo t = 0, chiudiamo il circuito e permettiamo il fluire della corrente. Determinare la I in funzione
del tempo.
Risposta: se non ci fosse lautoinduzione, la corrente sarebbe determinata da I(t) = V(t)/R , come si visto in
un esercizio precedente. Leffetto autoinduttivo impone in questo caso di inserire nel circuito una ulteriore
( oltre a quella del condensatore) forza elettromotrice data da: E
*
= - L dI/dt. Lequazione che determina
quindi levoluzione della corrente sar allora: I(t) = [V(t) - L dI/dt]/R. Ma : V = Q/C ed inoltre : I = -
dQ/dt ( una corrente positiva scarica il condensatore). In conclusione scrivo:
2
2
dt
Q d
L
C
Q
dt
dQ
R +
Questa unequazione differenziale del secondo ordine, omogenea e lineare, a coefficienti costanti,
nellincognita Q(t). La soluzione generale dipender da due costanti che andranno determinate in base alle
condizioni iniziali. Queste, nel caso presente sono che la carica al tempo 0 ha un valore assegnato e che la
corrente, sempre al tempo 0, cio la derivata di Q al tempo 0, nulla. La soluzione, come insegna lAnalisi,
ha tre forme analitiche leggermente differenti a seconda dei valori delle costanti L,C,R. precisamente :
se 0 < R <
C
L
2
[ ]
2 2
2 2
2
0
4
) cos( 4 ) sin(
) (
C R LC
t C R LC t RC
e Q t Q
t
L
R



dove si posto:
LC
C R LC
2
4
2 2


a parte i dettagli, questa soluzione ci dice che si instaurano nel circuito delle correnti oscillanti smorzate il
cui periodo : T = 2/. La decrescita nel tempo e data dal fattore esponenziale. Quindi, al limite, se la
resistenza completamente trascurabile, la corrente diventa una pura funzione sinusoidale del tempo ed il
fenomeno ha carattere oscillatorio con
94
pulsazione
LC
1

Supponiamo ora di aumentare la resistenza R e di portarla al valore:
se
C
L
R 2
la soluzione dellequazione differenziale allora:

,
_

+

RC
t
e Q t Q
RC
t
2
1 ) (
2
0
cio il condensatore si scarica senza che la corrente inverta il suo corso con landare del tempo
se
C
L
R 2 > la soluzione qualitativamente uguale alla precedente ed analiticamente :
( ) Q t
1
2
Q
0
( ) + C R C
2
R
2
4 L C e

_
,

( ) C R C
2
R
2
4 L C t
2 L C
C
2
R
2
4 L C

Vediamo ora pi da vicino la soluzione nel caso limite in cui sia R = 0. Essa :
) cos( ) (
0
t Q t Q dove
LC
1

Per la corrente (che la derivata col segno cambiato), :
) sin( ) (
0
t Q t I
Allistante t = 0 si ha:
Q = Q
0
I = 0
Mentre allistante t = / 2 si ha:
Q = 0 I = Q
0

Inizialmente il condensatore aveva dunque una carica Q


0
, quindi unenergia
C
Q
2
2
1
che rappresentava il
lavoro che abbiamo speso per caricarlo. Ma allistante t = / 2 il condensatore scarico ed allora ci si pu
chiedere: dove andata a finire lenergia? forse sparita? Ma se sparita, chi compie il lavoro per
ricaricarlo, dato che allistante t = / si ritrova di nuovo carico?
Osserviamo che dalla soluzione per la Q(t) e la I(t) si ha subito:
+
2
0
2 2
)
) (
( ) ( Q
t I
t Q

costante
Dividendo ambo i membri per 2C e ricordando lespressione di , si ha pure:
+ ) (
2
1 ) (
2
1
2
2
t LI
C
t Q
costante
il primo addendo lenergia elettrica del condensatore. Ora, questa energia + il secondo termine fornisce una
quantit costante nel tempo. logico quindi ritenere che il secondo addendo sia una energia della corrente
(dimensionalmente proprio unenergia!). Si salva in questo modo il principio di conservazione dellenergia
per un sistema isolato, cosa che sommamente desiderabile! Sicch quindi siamo portati a concludere che,
associata ad una corrente che percorre un circuito di autoinduttanza L, vi sia unenergia pari a L I
2
. Ma,
come noto ad ognuno, non detto che le cose sommamente desiderabili si avverino sempre. Noi vorremmo
che lenergia in un sistema isolato si conservasse, ma non detto che lo sia e laver trovato un termine che
fa tornar le cose non implica che questo termine sia effettivamente unenergia. Occorrerebbe mostrare che
per creare il termine in questione stato compiuto del lavoro, ed in pari misura. Allora soltanto il termine
avrebbe tutte le caratteristiche dellenergia ed a buon diritto pu essere appellato energia di corrente.
Vediamo meglio:
Supponiamo di avere un filo conduttore chiuso ad anello e supponiamo di far sorgere in lui, in qualche
modo, una corrente che quindi cresce perch passa dal valore iniziale nullo ad uno finale diverso da 0. Ora,
come la corrente cresce, si forma nel filo una forza elettromotrice indotta che, per la legge di Lenz, si oppone
95
ad unulteriore crescita della corrente. Per contrastare questa forza elettromotrice, occorre che loperatore
crei un campo elettrico opposto. In questo campo elettrico si muovono le cariche che costituiscono la
corrente e da qui nasce il lavoro che loperatore deve spendere, lavoro che quindi logico che si trovi alla
fine accumulato nella corrente.
Vediamo pi da vicino la cosa: Riprendiamo il circuito RL dove c inserita una batteria di f.e.m. E
Si era scritto: R I = E

L dI/dt. Moltiplichiamo ambo i membri per I ottenendo:
R I
2
+ L I dI/dt = EI
Il membro di destra altro non che la potenza sviluppata dalla batteria di f.e.m. E e la relazione scritta ci dice
che questa potenza va a finire nei due termini. Il primo lo riconosciamo subito come potenza dissipata per
effetto Joule ed il secondo termine un qualcosa di aggiuntivo. Integriamo la potenza sviluppata dalla pila
dallistante t =0 , quando la corrente nulla al generico istante t in cui la corrente vale i Tale integrale
fornisce lenergia totale erogata dalla pila. :
2
0 0
2
1 ) (
) ( Li LIdI dt
dt
t dI
t LI
i t


Quindi lenergia erogata dalla pila si trova in riscaldamento del filo e nel termine Li
2
che rappresenta
dunque unenergia associata alla corrente. Se un circuito con autoinduzione L percorso da una corrente I,
possiede unenergia LI
2
che restituisce quando la corrente si annulla. Infatti supponiamo di togliere dal
nostro circuito il generatore che ha generato in lui la corrente E/R cortocircuitandolo. Nel circuito la
corrente non crolla subito a 0, ma segue la legge vista sopra:
t
L
R
e I t I

0
) (
(si assunto pari a 0 listante in
cui si cortocircuita il circuito). Quindi da questo istante in poi (teoricamente per sempre!) circola nella
resistenza una corrente e quindi c un riscaldamento Joule. E chi fornisce questa energia per riscaldare la
resistenza? Ma la corrente accumulata nellinduttanza nella misura pari a LI
2
! E come facciamo a sapere
che proprio a tanto ammonta lenergia dissipata? Vediamo: la potenza dissipata per effetto Joule , lo
sappiamo,W = RI
2
. quindi lenergia totale sar:
2
0
0
2
2
0
0
2 / 1 LI dt e RI Wdt Energia
t
L
R

come volevasi
dimostrare.
Quindi, sintetizzando:
una corrente I che scorre un circuito di autoinduttanza L, ha unenergia pari a L I
2
Supponiamo ora che il nostro circuito sia un solenoide e consideriamone lunit di lunghezza. Sostituita nella
formula di sopra linduttanza con la sua espressione precedentemente trovata ed esprimendo la corrente in
termini di B allinterno del solenoide, otteniamo subito: Energia = 1/2SB
2
/
0
Se divido questa energia per il
volume di questo solenoide lungo 1, cio se divido lespressione per S, ottengo lenergia attinente allunit
di volume, quindi una
densit di energia pari : u = B
2
/(2
0
)
In questa espressione ogni riferimento al solenoide sparito, resta soltanto il campo dinduzione magnetica e
la formula trovata ci induce a pensare che l dove c un campo B, l c pure una densit denergia
magnetica data dalla formula di sopra. Pu esserci per una riserva dovuta al fatto che la formula di sopra
stata ottenuta del campo magnetico descritto in modo non corretto al 100%, in quanto si usata lequazione
di Maxwell: rot B =
0
j e si trascurata la corrente di spostamento. Vedremo pi avanti per che tale
formula rigorosamente corretta
La mutua induzione
Supponiamo di avere due circuiti elettrici, di resistenza R
1
e R
2
. nel primo sia inserita una forza
elettromotrice E1 e nel secondo E2 Inizialmente i due circuiti sono aperti, ma allistante iniziale, mediante
due interruttori, si chiudono i due circuiti. Vogliamo determinare le correnti nei due circuiti. A tal fine
notiamo che se nel circuito 1 circola una corrente I
1
, questa produce una campo B e quindi un flusso,
variabile se lo al corrente, sia nel circuito 1 che nel 2 e questi due flussi sono proporzionali alla corrente I
1
mediante due coefficienti che chiamo L
1
e M
2,1
e che si chiamano coefficiente di autoinduzione e
coefficiente di induzione di 1 su 2. Analogo discorso si ripete per il circuito 2, ma si dimostra che vale
sempre:
M
1,2
= M
2,1
= M
Dove il coefficiente M si chiama coefficiente di mutua induzione
sicch la corrente che scorre nel circuito 1 determinata dalla forza elettromotrice totale data da
96
E1 +(-L
1
dI
1
/dt)+ (-M
2,1
dI
2
/dt)
Analogamente per il circuito 2 si avr una forza elettromotrice totale data da:
E2 +(-L
2
dI
2
/dt)+ (-M
1,2
dI
1
/dt)
Si avr dunque per le due correnti:
R
1
I
1
= E1 +(-L
1
dI
1
/dt)+ (-M
2,1
dI
2
/dt)
R
2
I
2
= E2 +(-L
2
dI
2
/dt)+ (-M
1,2
dI
1
/dt)
Le due equazioni scritte rappresentano un sistema di due equazioni differenziali del primo ordine, lineari,
non omogenee ed a coeff. costanti nelle due incognite I
1
I
2
. la soluzione generale dipende da due costanti
iniziali che dovranno essere determinate a partire dalle condizioni iniziali che, nel caso presente, consistono
nellaffermare che le due correnti iniziali sono nulle. Supponiamo ora di avere i due circuiti nei quali scorra
inizialmente della corrente ma che i due generatori siano cortocircuitati. Le equazioni che determinano le
correnti sono sempre quelle di prima, ma con E
1
= E
2
= 0 e con la condizione iniziale che le due correnti
abbiano due valori assegnati. Ebbene, risolviamo questo sistema nellipotesi
ulteriore che sia I
1
(0) = 0, R
1
= R
2
=1. La soluzione allora, ottenuta con la macchina per I
1
(t), :
I coefficienti di t negli esponenziali altro non sono che le radici dellequazione algebrica di secondo grado:
0 4 ) ( ) (
2 1
2 2
2 1
+ + + x L L x M L L
Se il coefficiente di x
2
negativo, lequazione presenta una variazione ed una permanenza, e quindi possiede
una radice positiva ed una negativa. Ma questo assurdo, perch una radice positiva implicherebbe che la
corrente andrebbe crescendo esponenzialmente nel tempo, mentre invece, secondo tutte le logiche ed il buon
senso, le correnti non solo non devono crescere, ma devono estinguersi a causa della resistenza.
Ne consegue quindi che deve essere:
L
1
L
2
> M
2
Cio: il coefficiente di mutua induzione sempre minore della media geometrica dei coefficienti di
autoinduzione. [vedi anche Am.J.Phys.37,23(1969)]
Esercizio: mostrare che vale la relazione: M
1,2
= M
2,1
= M
Soluzione: Nel circuito 2 scorra una corrente I2. Il flusso di B nel circuito 1 allora dato da:


1
S
dS B
Per il teorema di Stokes posso scrivere:


1 1 1
1 1
) ( ) (

r r A S A S B d d d
S S
Ma :

2
3
2 1
2 1 2 0
1
) (
4
) (

r r
r r r
r A
d
e quindi subito:





2 1 2 1 2 1
3
2 1
2 1 2 1
2
0
3
2 1
2 1 2 1
2
0
3
2 1
2 1 2
1 2
0
) ( ) (
4
) (
4
) (
4

r r
r r r r
r r
r r r r
r r
r r r
r
d d
I
d d
I
d
d I
Il coefficiente M
1,2
altro non che il coefficiente de I
2
e cio:

2 1
3
2 1
2 1 2 1 0
2 , 1
) ( ) (
4

r r
r r r r d d
M
il coefficiente M
2,1
si ottiene scambiando 1 con 2, ma si vede che sotto questa operazione lintegrando non
cambia
Esercizio
Si consideri un circuito piano fatto da due maglie: 1 e 2 Ogni maglia un quadrato e le due sono unite per un
lato. Ogni lato presenta una resistenza R. il circuito viaggia con velocit costante v verso un campo B
97
ortogonale al piano del circuito, uniforme e costante in modo che prima entra la maglia 1 e poi la 2.
determinare le correnti nei rami.
Risposta: Siano I
1
e I
2
le correnti nelle due maglie. Finch la 1 non entrata tutta nel campo, c attraverso
di lei un flusso di B variabile dovuto al flusso del campo esterno, della corrente di maglia I
1
che d luogo al
fenomeno dellautoinduzione e della corrente di maglia I
2
che d un flusso per mutua induzione. Detto a il
lato della maglia, avremo per la maglia 1: E = - d/dt (Bavt+LI
1
+MI
2
)
La prima legge di K. applicata alla maglia 1 fornisce:
0 4 ) ( 0 4
2 1 2 1 2 1
+ + + + rI rI MI LI Bavt
dt
d
rI rI E
Applicandola alla seconda maglia si ha
0 4 ) (
1 2 1 2
+ + rI rI MI LI
dt
d
Si osservi che, essendo le due maglie uguali, medesimo il coeff. di autoinduzione. Occorre ora risolvere il
sistema delle due equazioni differenziali ottenute, con la condizione che le correnti iniziali sono nulle. La
soluzione :
( ) I2 t
1
15
+
3
2
e

_
,

5 r t
L M
Bav Bav
5
2
e

_
,

3 r t
+ L M
Bav
r
,

'
( ) I1 t
1
15
+
3
2
e

_
,

5 r t
L M
Bav
5
2
e

_
,

3 r t
+ L M
Bav 4 Bav
r

;
Si sono ottenute le due correnti finch non entra la seconda spira nel campo B
Il campo elettromagnetico nella sua generalit
Ci sarebbe ancora moltissimo da dire nei circuiti in regime quasi-stazionario, specialmente quando la f.e.m.
dei generatori inseriti varia sinusoidalmente nel tempo, come avviene nella normale rete elettrica di casa, ma
fugit irreparabile tempus ed occorre allora esaminare le equazioni di Maxwell nella loro completezza.
Questo signor Maxwell osserv, con acume eccezionale, che lultima equazione scritta, e cio:
j B
0

valida nel caso stazionario, non pu essere vera nel caso generale!
Infatti, argoment, se prendiamo la divergenza di ambo i membri, si ottiene al primo membro:
) ( B
Sviluppiamo i calcoli per vedere che cosa si ottiene.
) ( ) ( ) (
y
B
x
B
z
B
x
B
z
B
y
B
x
y
x z
y
z

k j i B
E quindi 0 ) ( ) ( ) ( ] [


y
B
x
B
z z
B
x
B
y z
B
y
B
x
x
y
x z
y
z
B per il teorema di
Schwartz sullinvertibilit delle derivate seconde miste
Per lequazione di continuit invece, la divergenza del secondo membro fornisce :
t

0
Ora, nel caso stazionario si ha che il primo membro ed il secondo coincidono, ma nel caso generale no!
Che fece allora il nostro Maxwell? Disse: correggiamo lequazione scrivendola:
R j B +
0

chiaro che, prendendo un R opportuno, si riescono a sistemare perfettamente le cose! Ma come prenderlo?
Prendiamo ora la divergenza dei due membri. Si deve avere:
R R j B +

+
t


0 0
) ( 0
ma :
98
0 ) ( 0
0 0 0 0 0
+


R
E
R
E
E
t t

Dal fatto che la divergenza di un vettore, non si deduce che necessariamente quel vettore nullo, ma
Maxwell prov, tentativamente a porre: 0
0 0
+

R
E
t
.
Cio scrisse tentativamente:

'



) (
0
0 0
0
t
t
E
j B
B
B
E
E

FU UN TRIONFO!
Osservazione: Le relazioni sopra scritte si chiamano: Equazioni di Maxwell e come in tutte le equazioni vi
sono dei termini noti e delle incognite.. Ovviamente note sono le due costanti
0
e
0
e le due funzioni (x,t)
e j(x,t) possono considerarsi i termini noti ed assegnati liberamente( eccetto il fatto che deve essere verificata
lequazione di continuit!), mentre le incognite sono i campi E e B. Essendo due vettori, le incognite sono
dunque le 6 componenti di questi. Ma, dal punto di vista numerico, le equazionei di M. sono 8: due per le
divergenze e 6 per i rotori. Dunque impossibili da risolversi? No, il fatto che lequazioni delle due
divergenze possono essere eliminate, nel senso che derivano dalle altre due! Infatti, prendiamo la divergenza
della seconda equazione: ) ( ) ( B
B
E


t t
. Ma noi sappiamo che la divergenza di un
rotore sempre identicamente nulla e quindi si ha che la derivata temporale della divergenza di B nulla,
quindi la divergenza di B non muta nel tempo e quindi sar una funzione della sola posizione. Scriver:
) , , ( z y x f B .Se f diversa da 0, vuol dire che presente B. Si consideri ora una regione con cariche e
correnti lontanissime ed una carica di prova lanciata in direzione perpendicolare a B. Lei allora subisce una
forza, ma ci contrario al fatto che non c forza su di un corpo se non c un secondo che la esercita
(osservando le cose in un sistema di riferimento inerziale). Si conclude quindi che la divergenza di B deve
essere nulla. Bench dunque questa equazione sia contenuta nelle altre, si preferisce enunciarla: melius est
abundare quam deficere. Prendiamo ora la divergenza dellultima equazione. Analogamente a quanto detto
sopra si ha: 0 ) (
0

+
t
E
j . Pongo ora, per definizione di :
t



` j . La relazione di sopra
diventa: 0 ) (
0
+

E
t
.Ne consegue allora che la parentesi pu essere solo una funzione
0
f della
posizione e non del tempo e quindi:
) , , (
0
z y x f +

E
. Consideriamo una zona di spazio dove non c
nulla. L allora: ) , , ( z y x f E . Quindi E diverso da 0, ma allora una carica sarebbe sottoposta ad
una forza ma ci contrario al fatto che non c forza su di un corpo se non c un secondo che la esercita.
Osservazione: Il termine
t
E
0
, che dimensionalmente una densit di corrente, per ragioni storiche si
chiama densit di corrente di spostamento e possiede tutte le propriet della normale densit di corrente j
= v che, per differenziarla, si chiama densit di corrente di conduzioneSi pu fare anche unaltra
osservazione, di carattere pi fisico della precedente, per mettere in dubbio la validit generale della
relazione j B
0
e per suggerire la correzione da apportare. Se tale relazione fosse rigorosamente vera,
potremmo considerare un condensatore a facce piane e parallele che si sta scaricando tramite un filo
conduttore in cui inserita una resistenza. Immaginiamo allora un anello ideale che circonda la resistenza.
La circuitazione di B lungo tale anello sarebbe, per il teorema di Stokes, il flusso del suo rotore, cio di j
0

attraverso una qualunque superficie che ha come bordo lanello in questione. Se prendiamo come superficie
in questione quella del cerchio che ha come circonferenza lanello, la corrente che lattraversa, cio quella
99
concatenata allanello, : I, la corrente che scorre nella resistenza e quindi, come vuole Ampre, tale
circuitazione sarebbe pari a I
0
e tutto andrebbe bene, ma se considerassimo ora una superficie, che ha
sempre come bordo lanello ma che passa tra le due armature del condensatore senza intercettare la
corrente. Il flusso di j attraverso tale superficie sarebbe evidentemente nullo! E che fine farebbe il teorema di
Stokes!? chiaro allora che per salvare le cose bisogner considerare il flusso di qualcosa daltro
attraverso questa superficie per compensare la mancanza di j. Ma quale vettore ha un flusso attraverso
questa superficie? Ma il campo elettrico che sussiste tra le armature del condensatore! Ed il flusso di tale
campo (ritenendolo confinato tra le armature) ES. Si quindi tentati di scrivere: E j B +
0
con
costante per far tornare le dimensioni.Ma cos non pu essere perch questa equazione non si riduce a
quella valida nel caso stazionario. Il termine aggiuntivo dovr dunque annullarsi nel caso stazionario:
conterr dunque una derivata parziale rispetto al tempo. Osserviamo allora che, oltre ad essere il campo E ad
avere flusso attraverso la superficie tra le armature, anche una sua derivata temporale ha flusso! E quindi
possiamo provare a porre:
t

+
E
j B
0
, con costante per far tornare le dimensioni Ora, una
costante cone le dimensioni giustesi trova gi allinterno dellelettromagnetismo che si studiato fino ad
ora: la costante
0 0
. Si allora incoraggiati a porre:
1
]
1

+
t
E
j B
0 0

Se le cose stanno cos, vediamo allora che ne dellequazione di continuit: applichiamo loperatore
divergenza ad ambo i membri:
t t t

+

j E j
E
j B
0 0
) ( 0
Come deve essere! Siamo sempre pi incoraggiati a ritenere che la correzione effettuata al rotore di B sia
quella giusta!
Fortuna iuvat audaces!
Vediamo brevemente una prova sperimentale che mostra che la corrente di spostamento produce gli stessi
effetti della corrente di convezione j. A tal fine consideriamo una spira conduttrice avvoltolata su di una
ciambella (tecnicamente un toro) di raggio interno r
1
ed esterno r
2
. (con r
1
>> r
2
-r
1
) nel suo centro,
perpendicolarmente al suo piano, passi un filo conduttore rettilineo ed indefinito, percorso da una corrente
variabile i(t).Questa genera nello spazio circostante un campo dinduzione magnetica che, se la frequenza
non troppo alta, potr essere calcolato con la nota formula che fornisce il campo B nella magnetostatica.
Questo campo B ha un certo flusso attraverso il solenoide. Infatti la superficie attraversata da B si pu
stimare pari a: [ (r
2
-r
1
)
2
/4]*n * 2 r
1
. Infatti la parentesi quadra fornisce larea della sezione circolare del
solenoide, n il numero di spire per unit di lunghezza, che moltiplicato per la lunghezza del solenoide,
fornisce il numero di spire totali. Il campo B poi, venendo ad essere ad una distanza di circa r
1
dal filo che lo
genera, circa pari a
0
i(t)/(2 r
1
) il flusso poi il prodotto delle quantit trovate, cio
[ (r
2
-r
1
)
2
/4]*n * 2 r
1
*
0
i(t)/(2 r
1
) = [ (r
2
-r
1
)
2
/4]*n *
0
i(t)
Variando nel tempo tale flusso, il filo elettrico che costituisce il solenoide, se forma un circuito chiuso, sar
percorso da una corrente elettrica che lo riscalder. Si riscalder ugualmente se, al posto del filo percorso
dalla corrente i(t) mettiamo il solenoide tra due lastre parallele di un condensatore piano, disponendolo
parallelo a queste e poi forniamo al condensatore una carica variabile. Infatti in questo caso si ha tra le
armature un campo E variabile nel tempo il cui flusso attraverso il buco della ciambella circa pari a:
r1
2
E. essendo variabile nel tempo, si former nel solenoide un campo variabile
Vediamo ora un breve quadro riassuntivo delle equazioni di Maxwell e delle loro semplificazioni:
CASO 1 (caso statico)
Tutto fermo (allinfuori eventualmente della carica q di prova che serve per valutare i campi
elettromagnetici dalla conoscenza della forza di Lorentz.
100

'





0
0
0
0
B
B
E
E

[ ] B v E f + q
La soluzione delle equazioni :

0 '
'
'
) ' (
4
1
3
3
0
B r
r r
r r
r E d

Lunica forza che agisce su una carica allora ( oltre alla forza gravitazionale che solitamente per si
esclude) la forza elettrica che nel caso presente prende ovviamente la denominazione di forza elettrostatica.
Il campo elettrostatico conservativo ed ha senso allora parlare di potenziale elettrostatico
CASO 2 (caso stazionario)
Ci sono presenti ora cariche in moto (c una j) , ma nulla varia con il tempo (fuorch, come prima, la carica
q di prova):

'





j B
B
E
E
0
0
0
0

[ ] B v E f + q
La soluzione delle equazioni :



V V
d d '
'
) ' (
4
) ( ; ) ( '
'
) ' (
4
1
) ( ; ) (
3 0 3
0
r
r r
r j
r A A r B r
r r
r
r r E




che si scrive anche:

V
d '
'
'
) ' (
4
1
) (
3
3
0
r
r r
r r
r r E


V
d '
'
'
) ' (
4
) (
3
3
0
r
r r
r r
r j r B

Se le densit di corrente j sono confinate in n circuiti elettrici, di correnti I


1
, I
2
, I
n
, si pu anche scrivere:

i
d I
n
i
i

3
1
0
'
'
) ' (
4
) (
r r
r r
r l r B
Il vettore infinitesimo dl non altro che un trattino infinitesimo del circuito generico diretto nel verso della
corrente
Il campo elettrico ancora conservativo e, a differenza del caso precedente, ora su una carica in moto
presente anche la forza magnetica.
CASO 3 (caso di campi lentamente variabili nel tempo)

'



j B
B
B
E
E
0
0
0

t
101
Le equazioni che determinano B sono le medesime del caso precedente (con linessenziale differenza che ora
e j variano nel tempo) e medesime allora saranno le soluzioni; e cio:



V
d
t
t t t '
'
) , ' (
4
) , ( ); , ( ) , (
3 0
r
r r
r j
r A r A r B

o anche:


V
d t t '
'
'
) , ' (
4
) , (
3
3
0
r
r r
r r
r j r B

Per risolvere le equazioni per E, scrivo: E = E


(1)
+ E
(2)
dove i due campi ausiliari soddisfano, rispettivamente
a:

'



0
) 1 (
0
) 1 (
E
E

'



t
B
E
E
) 2 (
) 2 (
0
Si vede allora che le equazioni per E
(2)
sono le stesse di quelle per B (dove compare -B/t al posto di
0
j)
ed allora ho:

+
V V
d t
t
d t t t t '
'
'
) , ' (
4
1
'
'
'
) , ' (
4
1
) , ( ) , ( ) , (
3
3
3
3
0
) 2 ( ) 1 (
r
r r
r r
r B r
r r
r r
r r E r E r E


o anche:

+
V V
d
t rot
t
d t t t t '
'
) , ' (
4
1
'
'
'
) , ' (
4
1
) , ( ) , ( ) , (
3
3
3
3
0
) 2 ( ) 1 (
r
r r
r B
r
r r
r r
r r E r E r E


Sostituendo nella soluzione trovata il rotore di B con
0
j , ho allora:

+
V V
d
t
t
d t t t t '
'
) , ' (
4
'
'
'
) , ' (
4
1
) , ( ) , ( ) , (
3
3
0 3
3
0
) 2 ( ) 1 (
r
r r
r j
r
r r
r r
r r E r E r E


Espressione del campo elettrico se lentamente variabile nel tempo
non leggere sotto fino alla riga di ***

'

V V
t
V
d d t
t
d t t '
'
'
' '
' ' '
' ' '
) , ' ' (
4 ) 4 (
1
'
'
'
) , ' (
4
1
) , (
3
3
) , ' (
3
3
0 3
3
0
r
r r
r r
r
r r
r r
r j r
r r
r r
r r E
r B


o anche:

'

V V
t
V
d
d
t rot
t
d t t '
'
' '
' ' '
) , ' ' ( ' '
4
) 4 (
1
'
'
'
) , ' (
4
1
) , (
3
3
) , ' (
3
3
0
3
3
0
r
r r
r
r r
r j
r
r r
r r
r r E
r B

'

V V V
d d t
t
d t t ' ' '
' ' '
' ' '
) , ' ' (
'
'
) 4 (
'
'
'
) , ' (
4
1
) , (
3 3
3 3 2
0 3
3
0
r r
r r
r r
r j
r r
r r
r
r r
r r
r r E

'

,
_

,
_

V V V
d d t t
t
d t t ' ' '
' ' '
' ' '
'
'
) , ' ' (
' ' '
' ' '
'
'
) , ' ' (
) 4 (
'
'
'
) , ' (
4
1
) , (
3 3
3 3 3 3 2
0 3
3
0
r r
r r
r r
r r
r r
r j
r r
r r
r r
r r
r j r
r r
r r
r r E

'

,
_

,
_

V V V
d d t t
t
d t t ' ' '
' ' '
' ' '
'
'
) , ' ' (
' ' '
' ' '
'
'
) , ' ' (
) 4 (
'
'
'
) , ' (
4
1
) , (
3 3
3 3 3 3 2
0 3
3
0
r r
r r
r r
r r
r r
r j
r r
r r
r r
r r
r j r
r r
r r
r r E

'

V V V
d t
t
C d t t ' ' ) ' ' ( ) , ' ' (
) 4 (
'
'
'
) , ' (
4
1
) , (
3
2
0 3
3
0
?
r r r r j r
r r
r r
r r E

V
t
t
C d t t ) , (
) 4 (
'
'
'
) , ' (
4
1
) , (
2
0 3
3
0
?
r j r
r r
r r
r r E


****************
102
Resterebbe ora da trovare la soluzione delle equazioni di Maxwell nel caso generale, cio occorrerebbe
vedere come si determinano i campi E e B dalla conoscenza della densit di carica (x,t) e di corrente j(x,t).
Il procedimento un po complicatino ed allora lo tralascio. Chi interessato, pu vedere le gi citate
Letture di Fisica del sig. Feynmann. Comunque non sar inutile iniziare a percorrere una strada che, se
proseguita, porta alla soluzione. Anche se non si arriva fino in fondo, si potranno cogliere importanti fatti
nuovi.Ritorniamo ora un momento sullequazione:
A B
Stando cos le cose, lequazione:
t


B
E viene a scriversi:
t t


A
A E
Dove si scambiato (cosa lecita) lordine di derivazione e quindi:
0
,
_

+
t
A
E
Ma sappiamo che se il rotore di un campo nullo, questo il gradiente di un opportuno scalare: -(r,t) (il
segno introdotto per pura comodit e detto potenziale, che non per il potenziale introdotto in
elettrostatica, ma coincide con lui nel caso stazionario!). Scrivo dunque:
t


A
E
ma chiediamoci ora: unico il campo vettoriale A il cui rotore fornisce il campo B (che determinabile
sperimentalmente)? facile vedere che la risposta negativa: A determinato a meno del gradiente di una
funzione arbitraria ( r ,t) (infatti il rotore di un gradiente identicamente nullo). Quindi io posso, senza
alterare il campo B, incrementare il campo A del vettore arbitrario: grad . Quindi, in formule, se
+ A A'
ho: rot A = rot A, cio il campo B pu essere ottenuto calcolando il rotore di A come di A. Dalla
relazione di sopra che fornisce E si vede per che E cabia se usiamo A al posto di A. Esattamente si ha:
) (
'
t t t t



E
A A
Se io voglio che questa mia alterazione del potenziale vettore non influenzi n B n E, devo allora mutare
contemporaneamente ad A, anche e pi precisamente devo eseguire le seguenti trasformazioni:

'


+
t

'
' A A A
In questo modo n B n E mutano. In altri termini, se e A sono dei potenziali atti ad esprimere i campi
elettrici e magnetici, allora A e esprimono i medesimi campi. Tutto ci si esprime dicendo che B e E
sono invarianti sotto una trasformazione di gauge.
Tutto quanto detto sopra pu sembrare molto astratto e forse lo sar, daltra parte i potenziali stessi sono
qualcosa di molto astratto tant vero che non sono neppure univocamente definiti. Invece i campi E e B
sono reali in quanto direttamente misurabili dalla forza che esercitano su di una carica.. Malgrado il
carattere astratto delle elucubrazioni di sopra, le trasformazioni di gauge ci serviranno per semplificare
enormemente la ricerca delle soluzioni delle equazioni di Maxwell, come ora vediamo subito
A questo punto, avendo espresso B e E in termini di A e , posso riscrivere le equazioni di Maxwell in termi
di A e . Lequazione:
0

E
diventa:
0

A
t
Lequazione
103
t


B
E
diventa:
A
A

,
_


t t

cio identicamente soddisfatta


Anche lequazione :
0 B
identicamente soddisfatta
Lequazione:
) (
0 0
t

+
E
j B
diventa:
1
]
1

,
_

+
t t
A
j A
0 ' 0
) (
Ma si gi visto che il rotore di un rotore uguale al gradiente della divergenza il laplaciano. Ho dunque:
( )
t t t t


1
]
1

,
_

+


0 0 2
2
0 0 0 0 ' 0
) (
A
j
A
j A A A
Riordinando i temini, scrivo:
j
A
A A
0 2
2
0 0 0 0

+
,
_

+
t t
Le equazioni di Maxwell si sono ridotte dunque ad un sistema di 4 equazioni nelle 4 incognite e A (un
vettore ha tre componenti ed una equazione vettoriale corrisponde a tre equazioni scalari!). Ognuna di queste
equazioni contiene tutte e quattro le incognite e questo rende complicata la loro soluzione. Sarebbe stato
bello se avessimo avuto quattro equazioni ma ognuna contenente unincognita sola! Ebbene, a ci si arriva
mediante un uso appropriato della trasformazione di gauge. A tal fine esprimo A e nella parentesi tonda
dellequazione vettoriale in termini di A e . Ho:
2
2
0 0 0 0 2
2
0 0 0 0 0 0
'
'
'
) ' (
t t t t t

A A A
Osserviamo bene la relazione ottenuta: (r,t) una funzione in nostro potere, A e sono dei potenziali
che ci forniscono il valore corretto dei campi. Allora lidea di scelgliere la funzione (r ,t)in modo da
soddisfare lequazione:
0
'
'
2
2
0 0 0 0

+
t t

A
Gauge di Lorenz
Con questa scelta di (r,t) la parentesi tonda nellequazione vettoriale di sopra nulla e lequazione di sopra
si semplifica in:
j
A
A
0 2
2
0 0


t
che un sistma di tre equazioni che non contiene pi lincognita . Ma c di pi: ognuna di queste
equazioni contiene unincognita sola: la prima equazione contiene la sola incognita A
x
, ed il termine noto j
x
, la seconda equazione la sola incognita A
y
e termine noto j
y
e la terza la sola incognita A
z
e termine noto j
z
.
Per quanto riguarda la prima equazione di Maxwell
0

E
ottengo allora:
0


,
_

A
A
t t

104
Dalla gauge di Lorentz, ottengo:
0
2
2
0 0

t t

A
e quindi subito:
0
2
2
0 0


t
Come si vede chiaramente, le equazioni di Maxwell scritte in termini dei potenziali, anzich dei campi E e B,
si riducono sostanzialmente ad ununica equazione nello scalare :
s
t


2
2
0 0


dove una delle tre componenti del potenziale vettore e s una delle tre componenti di
0
j nel caso
dellequazione del potenziale vettore, mentre il potenziale e /
0
nellequazione relativa al potenziale.
Insomma, in tutti i quattro casi, lequazione da risolvere la medesima!
Vediamo dunque da vicino lequazione per il potenziale scalare :
0
2
2
0 0


t
. Nel caso
stazionario diventa:
0


la cui soluzione si vista allinizio della trattazione dellelettricit:

spazio lo Tutto
dV
'
) ' (
4
1
) (
0
r r
r
r

Proviamo a tradurre in parole, alla buona, la formula di sopra: per calcolare il potenziale in un punto r
bisogna dividere lo spazio in N cellette , ognuna piccolissima. Ognuna di queste cellette porta un suo
contributo al potenziale nel punto in questione. Il potenziale totale nel punto sar la somma di tutti questi
contributi. E qual il contributo al potenziale portato da ogni celletta? Semplicemente la carica in lei
contenuta divisa per la distanza tra la celletta in questione ed il punto r nel quale vogliamo trovare il
potenziale. Ma supponiamo ora che la carica sia in moto, quindi il potenziale varier nel tempo, come pure la
carica contenuta in una data celletta. Voglio il potenziale in r al tempo t. Quale sar il contributo di una
generica celletta? Teniamo presente che, come ha solennemente affermato Einstein, non esistono segnali
che si propagano con velocit superiore a quella della luce. Quindi il contributo nel punto r al tempo t al
potenziale dovuto da una data celletta non pu essere determinato dalla carica presente nella celletta al
medesimo istante t, perch questo implicherebbe che linformazione della presenza della carica nella celletta
arriverebbe in r istantaneamente, contro quanto dice Einstein. Se indico con v la velocit con cui una carica
avvisa un punto r della sua presenza, allora il contributo al potenziale in r al tempo t di una data celletta
posta in r, sar determinato non dalla carica presente al tempo t, ma da quella presente ad un tempo t
anteriore, giusto quanto basta per far arrivare in r al tempo t il messaggio della sua presenza. Il
messaggio deve percorrere la distanza ' r r e viaggia con velocit v. Deve quindi essere emesso
v
' r r

secondi prima del tempo t, quindi al tempo
v
t t
'
'
r r
. Quindi, se si vuol tenere presente della velocit
finita di propagazione del segnale nella formula che determina il potenziale quando le cariche sono in moto,
la verosimile correzione al potenziale statico, dovrebbe essere data dalla formula:

spazio lo Tutto
dV
v
t
t
'
)
'
, ' (
4
1
) , (
0
r r
r r
r
r

A questo punto potreste sperare che la formula di sopra sia soluzione dellequazione:
0
2
2
0 0
) , ( ) , (
) , (



t
t
t
t
r r
r


105
Ebbene, la vostra speranza non vana! Si pu infatti verificare che il vostro supposto potenziale la
soluzione dellequazione se prendete per v la quantit
0 0
1

che, oltretutto ha proprio le dimensioni di
una velocit! E risulta che questa velocit ha il valore di circa 300000 km/s, proprio la velocit della luce!
In base poi al banale principio che stesse eqauzioni hanno medesime soluzioni, scriveremo anche subito:

spazio lo Tutto
dV
v
t
t
'
)
'
, ' (
4
) , (
0
r r
r r
r j
r A

Si, ma la A e la cos ottenuta, soddisferanno la gauge di Lorentz? Non resta che verificarlo, ma la verifica
ve la risparmio. Risulta che, se la j e la negli integrandi delle formule di sopra soddisfano allequazione di
continuit ( e in effetti devono soddisfarla se devono essere interpretati come densit di carica e di corrente!),
allora soddisfatta la gauge di Lorentz.
I potenziali e A vengono detti potenziali ritardati
Siamo arrivati in cima alla montagna! La soluzione stata ottenuta facendoci guidare da concetti di
verosimiglianza, pi che da passaggi rigorosi. Per la correttezza formale pu essere consultato il sig.
Feynmann gi ricordato.
Riassumendo: Le equazioni di Maxwell in termini del campo elettrico E e magnetico B sono:

ale differenzi forma In
t
t

,
_



E
j B
B
B
E
E
0 0
0
0


inegrale forma In
S
a a concatenat
chiusa Superficie
S
S
V in contenuta totale
d
t
I d
d
d
t
d
Q
d

,
_

S E l B
S B
S B l E
S E
0 0
0

In termini dei potenziali, vettore A e scalare :


j
A
A
0
2
2
0 0


t
0
2
2
0 0


t
dove:
A B
t


A
E
e vale la gauge di Lorentz:
0
0 0

+
t

A
Viste le equazioni di Maxwell come un set di equazioni nelle incognite E e B o nelle incognite A e con
(r,t) e j(r,t) termini noti, si ha che le loro soluzioni sono:

spazio lo Tutto
dV
v
t
t
'
)
'
, ' (
4
1
) , (
0
r r
r r
r
r

spazio lo Tutto
dV
v
t
t
'
)
'
, ' (
4
) , (
0
r r
r r
r j
r A

Esercizio: mostrare che in assenza di cariche e correnti i campi E e B soddisfano alle equazioni:
0
2
2
0 0


t
E
E 0
2
2
0 0


t
B
B
Energia del campo elettromagnetico
106
Purtroppo manca il tempo di esaminare tutte le conseguenza che la correzione al termine del rotore di B
comporta. Mi limiter semplicemente a segnalare, a parole alcuni pochissimi fatti. Chi fosse desideroso di
vedere pi da vicino la questione, pu consultare i miei appunti scannerizzati oppure molti bei libri, tra i
quali consiglio: la fisica di Feynman, che veramente avvincente. Vediamo questi fatti: consideriamo della
materia ( altro non che protoni ed elettroni in moto) la quale immersa nel campo elettromagnetico da lei
generata. Immaginiamo di isolare questa materia dentro una grande sfera ed il resto delluniverso sta fuori da
questa sfera a distanza astronomica. Ebbene, il campo elettromagnetico sviluppa su queste cariche in moto
una potenza specifica. Infatti, se, come al solito, indichiamo con la densit di carica, su questa densit di
carica agir una densit di forza (che non altro che la forza che agisce sulla carica contenuta nellunit di
volume) che si scrive direttamente dallespressione della forza di Lorentz come:
] [ B v E f +
La potenza sviluppata da questa forza , come noto , il prodotto scalare di f e v. Ottengo quindi:
j E v E v f ` w
(Il campo B non compare perch la forza che esplica ortogonale a v e quindi non compie lavoro. Questa
che si trovata la potenza sviluppata dal campo elettromagnetico sulla materia che si trova nella generica
unit di volume e quindi la potenza sviluppata su tutta la massa compresa nella nostra sfera sar:

V
materia campo
dV W j E
Ricaviamo ora da Maxwell:
t

E B
j
0
0

e quindi:

+


1
]
1

V V V V V
materia campo
dV dV E
t
dV dV
t
dV
t
W B E B E
E
E
E B
E
0
2 0
0
0 0
0
1
2
1

Una formula di calcolo vettoriale dice che :


( ) ( ) ( ) b a a b b a
Ottengo allora subito:
( ) ( ) [ ]

+ +

V V
materia campo
dV dV E
t
W E B B E
0
2 0
1
2

Sostituendo, con laiuto di Maxwell, il rotore di E con


t

B
, osservando quindi che
2
2
1
B
t t

B
B ed
applicando il teorema della divergenza, ottengo subito:

1
]
1

S V V V
materia campo
d dV dV
t
dV W S
B E
E B j E
0
2 0 2
0
2 2
1

dove S la grande superficie che racchiude la grande sfera che contiene tutta la nostra materia.
Interpretazione del risultato. Supponiamo per il momento che non ci sia lultimo integrale, che altro non
rappresenta che il flusso attraverso la superficie in questione del vettore S ( attenzione: non confondere con
la superficie! Malauguratamente si usa lo stesso simbolo. Potrei cambiarlo, vero, ma la tradizione usa
cos!) definito come:
0

B E
S

`
Vettore di Poynting
La relazione ci dice allora che la potenza sviluppata dal campo sulla materia uguale alla perdita ( c il
segno-) nellunit di tempo della cosa tra le parentesi quadre. Se pensiamo che debba valere il principio di
conservazione dellenergia in un sistema isolato, allora la cosa da intendersi: lenergia del campo
elettromagnetico. Daltra parte, che lespressione tra le quadre sia unenergia, ce lo suggerisce sia un
controllo dimensionale, sia la presenza dellintegrale di volume del quadrato del campo elettrico, espressione
che nel caso statico si visto in gran dettaglio essere proprio lenergia del campo elettrico. Vediamo quindi,
nel caso presente, che del tutto generale, che lespressione dellenergia elettrica la medesima sia nel caso
statico che in quello dinamico, anche se i campi sono molto diversi nei due casi. Laltro integrale allora da
intendersi come lenergia magnetica. Stando cos le cose, il campo elettrico ed il campo magnetico, che
inizialmente erano stati introdotti come puri artifici verbali e comodi strumenti per evitare lazione a
107
distanza, acquistano spessore fisico in quanto vengono ad essere enti che possiedono energia. Veniamo
allinterpretazione del flusso del vettore di Poynting. Supponiamo che nella regione che noi consideriamo
non ci sia materia. Allora ovviamente il primo membro nullo ed allora, la relazione di sopra, con la
interpretazione sopra data, si legge: la perdita, nellunit di tempo, della energia elettromagnetica contenuta
in una certa regione, uguale al flusso del vettore di Poynting attraverso la superficie che racchiude la
regione. Qual allora linterpretazione fisica del vettore di Poynting? Ma chiaro! S rappresenta il vettore
flusso di energia. Il altri termini, consideriamo una superficietta

,
d
. Lenergia elettromagnetica che fluisce
attraverso di lei nellunit di tempo ( ovverosia la potenza elettromagnetica che lattraversa) data da:

,
d S
Quantit di moto e momento della quantit di moto del campo elettromagnetico
Consideriamo ora due cariche q
1
e q
2
. al tempo 0,la prima sia nellorigine degli assi con velocit lungo lasse
y e la seconda in un punto dellasse x con velocit lungo lasse x. Anche se non siamo in presenza di un
fenomeno stazionario, plausibile supporre, per basse velocit, ed unanalisi pi puntuale lo conferma, che
con buona approssimazione i campi elettrici e magnetici generati da queste cariche in moto possono
approssimarsi a quelli che si avrebbero se al posto delle cariche ci fossero dei fili percorsi da corrente
disposti secondo le velocit. Ma allora immediato vedere che la forza che agisce su di una non uguale ed
opposta alla forza che agisce sullaltra! Il terzo principio violato! La cosa non da poco! Come si venire a
capo di questo problema? Il principio di azione e reazione non altro che laffermazione che, per un sistema
isolato di due corpi, la quantit di moto si conserva. Si ha infatti:
0 ) ( 0
2 2 1 1 2 2 1 1 2 2 1 1
+ + Q v v a a a a
dt
d
m m
dt
d
m m m m `
se i due corpi non sono isolati, non ovviamente detto che la forza che agisce sulluno lopposto della
forza che agisce sullaltro. Se i due non sono isolati, ma c un terzo o quarto corpo, il terzo principio della
dinamica si enuncia dicendo che la quantit di moto totale costante. Torniamo alle nostre due cariche. Il
fatto che non sono un sistema isolato! E perch? Che corpo c che interagisce con loro? Ma c il campo
elettromagnetico! Ora, come stato possibile attribuire al campo elettromagnetico una densit di energia tale
da rendere valido ancora il principio di conservazione dellenergia, cos, con unanalisi analoga a quella che
non ho svolto, ma che potete trovare negli altri appunti, possibile attribuire al campo elettromagnetico una
densit di quantit di moto per cui risulta:
quantit di moto di 1 + quantit di moto di 2 + quantit di moto del campo elettromagnetico = costante.
Siamo salvi!
Lespressione per la densit di quantit di moto del campo elettromagnetico la seguente:
S q
0 0
`
Sempre di pi quindi il campo elettromagnetico pu dirsi a buon diritto reale possedendo energia e quantit
di moto.
Ma non finita qui! Altri fatti, che ora non espongo (vedi altri appunti) mostrano che se si vuole mantenere il
principio di conservazione del momento della quantit di moto in presenza di cariche, occorre attribuire al
campo elettromagnetico una densit di momento della quantit di moto.
Domanda qual lespressione della densit del momento della quantit di moto del campo
elettromagnetico?
Risposta stupefacente: la densit del momento della quantit di moto del campo elettromagnetico si
costruisce nel medesimo modo con cui si costruisce il momento della quantit di moto di una particella,
cio facendo il prodotto vettore tra r e q. cio:
) (
' 0 0 0
S r q r l `
Grazie Maxwell!
Il campo elettromagnetico possiede dunque energia, quantit di moto e momento della quantit di moto. Ha
dunque tutte le carte in regola per poter dire che reale! Rispetto alla materia ordinaria soltanto molto
meno denso! ( solo questa propriet che lo differenzia dal resto della materia? Beh, fate ingegneria, non
fisica!)
Ma il termine aggiuntivo introdotto da Maxwell nellequazione del rotore ha solo il merito di far tornare i
principi di conservazione? Noooo!, tuttaltro, ha anche il merito di mostrare che la luce consiste in un
campo elettromagnetico variabile e che la luce emessa da cariche che sono accelerate ed esaminando
attentamente la questione dellirraggiamento di onde elettromagnetiche da cariche accelerate, Maxwell ha
guadagnato pure il grandissimo merito di aver messo in crisi tutto!! Ma qui mi fermo ( anche sugli altri
appunti non ne parlo: largomento oggetto di ricerca attuale. Gli interessati possono consultare un testo di
108
Elettrodinamica avanzato, quale il Jakson: elettrodinamica classica od anche il Panofsky: elettricit e
magnetismo, entrambi reperibili in biblioteca, ma queste parti non sono di lettura agevole)
Le equazioni di Maxwell nel vuoto nei diversi sistemi di unit di misura.
B
v
E B
v
E B v E B v E B v E
E
j
B
E
j
B
E
j
B
E
j
B
E
j
B
B
B
E
E
+ + + + +

'





c c
Forza
t c
t c
t c c
t c
t c
t
c
t c c
t
t
t
Heaviside Gauss
em
es MKSA
:
) (
1
0
1
1 4
0
1
4
1
4
0
4
1 4
0
4
0
2
2
2 0 0 0
0




Le equazioni delle onde elettromagnetiche
Voglio ora mostrare, molto succintamente, che, se nelle equazioni di Maxwell mancasse la corrente di
spostamento, allora non potrebbe esistere il campo elettromagnetico in assenza di cariche e correnti, mentre,
grazie a questo termine in pi, anche se la cosa pu sembrare strana, pu esserci un campo E ed un B anche
senza cariche e correnti, cio nel vuoto. Infatti, se non ci fosse la corrente di spostamento, in mancanza di
correnti il rotore di B sarebbe nullo. Essendo poi la sua divergenza 0, avremmo rotore 0 e divergenza 0 e
quindi, imponendo che sia nullo allinfinito, sarebbe ovunque nullo. Ma allora sarebbe nullo anche il rotore e
la divergenza di E e quindi pure il campo elettrico sarebbe nullo. Vediamo ora invece come vanno le cose
con le equazioni complete. Dalla formula di analisi vettoriale che si pu dimostrare banalmente svolgendo
semplicemente i conti, si ha che, per un campo qualunque:
A A A ) ( ) (
Calcolando allora il rotore del rotore di B si ha allora:

B B B B ) ( ) (
0
Daltra parte, per Maxell, :
E B
E
j B

+
t t
0 0 0 0
0
0
) ( ) (

Ma, sempre per Maxwell, :
B E
t


e quindi subito la celebrata equazione di DAlembert od equazione delle onde, nota anche come equazione
delle corde vibranti:
0
2
2
0 0


t
B
B
Cambiamo momentaneamente le unit di misura, ponendo 1
0 0
; allora le eq. di Maxwell in assenza di
cariche e correnti restano valide se sostituiamo il campo B con E e viceversa; si deduce allora che una
relazione perfettamente analoga soddisfatta dal campo E:
0
2
2
0 0


t
E
E
Mostro ora che esiste una soluzione non nulla di questa equazione ( in realt sarebbero tre le equazioni: una
per ogni componente, ma siccome queste tre sono identiche, posso dire di averne una sola!). Indicando una
generica componente con la lettera u, dimostro che esiste una soluzione non nulla trovando la soluzione
generale delleqazione in questione e mostrando che questa non necessariamente nulla ( chiaro che se la
u nulla, soluzione, ma non tutte le soluzioni saranno nulle!). Al fine della ricerca della soluzione generale
dellequazione di DAlembert, pongo per brevit:
109
0 0
2
1

` c
e osservo che c ha le dimensioni di una velocit.. Considero il caso particolare in cui la u sia funzione solo di
due variabili, diciamo x e t.( sar allora
0

z
u
y
u
)
Ho dunque lequazione:
0
1
2
2
2 2
2

t
u
c x
u
Introduco ora, accantgo alle variabili x e t, altre due, ed , definite da:
c
t x ct x ct x
2
,
2


+
+ ` `
La u funzione di x e t e queste variabili, per quanto scritto sopra, sono a loro volta funzioni di ed , sicch
u risulta funzione di ed . Precisamente:
)
2
,
2
(
c
u u
+

Incremento ora la variabile di una quantit infinitesima d, tenendo fissa la . Di quanto varia la u? Osservo
che un incremento d corrisponde ad un incremento
2

d
d
x
dx

,
_

della variabile x e ad un
incremento:
c
d
d
t
dt
2

,
_

della variabile t. Lincremento della u, dato, daltra parte, da:


dt
t
u
dx
x
u
du
x t

,
_

,
_

Sostituendo il dx ed il dt trovati, ottengo:


c t
u d
x
u
du
x t
2 2

,
_

,
_

Se ora io divido questo incremento infinitesimo della u per linfinitesiimo d, , altro non ottengo che la
derivata parziale della u, fatta ad costante. Ho dunque:
x t
t
u
c x
u u

,
_

,
_

,
_

2
1
2
1

Procedendo in modo perfettamente analogo, ma facendo variare ora solo la , ho:


x t
t
u
c x
u u

,
_

,
_

,
_

2
1
2
1

Da queste due relazioni, sommando e sottraendo, ottengo subito:




,
_

,
_

,
_

u u
x
u
t
1
1
]
1

,
_

,
_

,
_



u u
c
t
u
x
A parole: derivare rispetto ad x vuol dire: derivare rispetto a , derivare rispetto a e poi sommare. Analogo
discorso per la derivat rispetto a t. Quindi derivare una seconda volta rispetto ad x quanto si trovato, vuol
dire: derivare rispetto a , e sommare. Ho dunque:
1
1
]
1

,
_

,
_

+
1
1
]
1

,
_

,
_

,
_



u u u u
x
u
x x
u
t
2
2
=
2
2 2
2
2
2

u u u
110
1
]
1

,
_

,
_

'

1
1
]
1

,
_

,
_

1
1
]
1

,
_

,
_

,
_

2
2 2
2
2
2 2
2
2
2


u u u
c
u u u u
c
t
u
t t
u
x
Quindi lequazione : 0
1
2
2
2 2
2

t
u
c x
u
diventa:
0 ) (
2


u u
Questa equazione ci dice che, essendo la derivata rispetto a nulla, la quantit che si deriva, cio

u

indipendente da , quindi

u
funzione soltanto di ( e non ). Potr allora porre:
) (

f
u

, con f
indeterminata. Integrando rispetto ad ottengo:
G F u + ) (
con G costante rispetto alla variabile nella quale si integrato, cio la , ma potendo dipendere dallaltra,
cio . Otterr quindi:
) ( ) ( G F u +
con F e G funzioni del tutto arbitrarie. Ritornando alle vecchie variabili x e t si ha allora:
) ( ) ( ) , ( ct x G ct x F t x u + +
(Una verifica comunque permette di controllare che, quale che siano F e G, la funzione di sopra soluzione
dellequazione di DAlembert)-. La funzione F(x-ct) non rappresenta altro che la funzione F(x) che si sposta
nel tempo lungo lasse x verso sinistra) (ricordare che il grafico di f(x-a) non altro che quello di f(x)
spostato a sinistra del tratto a) e questo spostamento avviene con velocit c! A buon diritto una siffatta
funzione si potr chiamare onda progressiva lungo lasse x (il profilo dellonda dato dalla funzione F(x)).
Analogamente G(x-ct) prende il nome di onda retrograda. Dovrebbe essere chiaro ora perch lequazione di
DAlembert si chiama anche equazione delle onde
Esercizio Si consideri una corda tesa lungo lasse x e T sia la sua tensione. La si deformi spostando ogni suo
punto di un tratto infinitesimo perpendicolare allasse x ( lo spostamento, pur essendo infinitesimo sia
diverso da punto a punto), Si pizzichi cio la corda. Dopo che la si deformata lievemente la si abbandoni
a s stessa. Per la tensione T essa si metter in moto ( una coda di uno strumento musicale pizzicata si mette
a vibrare). Si vuole determinare la posizione temporale di ogni punto della corda.
Risposta. Sia x-y il piano in cui giace la corda. Lo spostamento, al generico tempo t, dallasse x del punto che
prima della deformazione occupava lascissa x, verr indicato con y(x,t). Il nostro problema di individuare
la funzione y(x,t) sapendo la deformazione iniziale, cio la y(x,0), che indicher con f(x). Il trattino
infinitesimo di corda, che inizialmente si trovava tra x e x+dx, al tempo t si trover tra i punti di coordinate
[x, y(x,t)] e [x+dx, y(x+dx,t)~y(x,t)+(dy/dx)dx]. La componente lungo y della forza che la fune che si trova
oltre lascissa x+dx esercita sul trattino , approssimativamente data da
dx x
x
y
T
+

, mentre la componente
lungo y della forza che la fune che si trova prima dellascissa x esercita sul trattino , approssimativamente,
data da
x
x
y
T

, [la T che compare qui non a rigore la T che compare precedentemente, ma lo con
ottima approssimazione]. La risultante allora delle forze lungo lasse y agenti sul trattino allora data da:
dx
x
y
T
x
y
T
x
y
T
x dx x
2
2

+
Se la densit lineare di massa, la massa contenuta nel trattino in questione data da: dm = dx
Ma allora la prima equazione cardinale fornisce per la porzione di corda in questione:
2
2
2
2
2
2
2
2
) , ( ) , ( ) , ( ) , (
t
t x y
T
m
x
t x y
x
t x y
T
t
t x y
m

Ecco perch lequazione di DAlembert si chiama anche equazione delle corde vibranti!
pippo
111
Alcune unit, che non fanno parte del SI, sono cos ampiamente usate che praticamente impossibile
abbandonarle.
Altre unit, tra cui il miglio marino, il nodo, lngstrm (= 10
-10
m), latmosfera, lettaro e il bar, sono
permesse soltanto per un tempo limitato e soggette a futura revisione.
Osservazioni sulle onde elettromagnetiche
Consideriamo unespressione del tipo:
) (
0 0
) cos( ) , (
t i
e e t t



r k
E r k E r E
con c
k

La quantit kr t si chiama: fase dellonda ed il vettore k vettore donda, mentre detto pulsazione
Essa rappresenta unonda che si propaga nella direzione individuata dal vettore k con velocit c. unonda
piana, perch, ad ogni istante, tutti i punti perpendicolari ad k sono sottoposti alla medesima
perturbazione. Per ogni r fissato, la perturbazione varia sinusoidalmente nel tempo. Siccome il vettore
campo elettrico in assenza di cariche e correnti soddisfa lequazione di DAlembert, lespressione di sopra,
come del resto facilissimo verificare, soddisfa lequazione 0
2
2
0 0


t
E
E
Lequazione: div E = 0 implica subito:
0 E k
Cio:
il vettore E ortogonale alla direzione di propagazione dellonda
Analogamente
0 B k
Cio:
il vettore B ortogonale alla direzione di propagazione dellonda
Dalla relazione che fornisce il rot E , si deduce poi immediatamente che
0 B E
Quindi i campi elettrici e magnetici in unonda e.m. sono tra loro ortogonali ed ortogonali alla direzione di
propagazione (loda e.m. dunque trasversale)
CONSIDERAZIONI VARIE da non leggere
Osservazioni sullenergia del campo elettrico in presenza di dielettrici
Spesso si vede questo procedimento:
Lenergia di un sistema elettrostatico data, sempre da:

r Vd U
3
2
1

ma : D e quindi subito:

r Vd U
3
) (
2
1
D
poi, portando il segno di divergenza ad operare anche su V, usando il teorema della divergenza ed il fatto che
allinfinito i campi sono nulli, si ha:

r d U
3
2
1
D E
Ma io ho da obiettare: la divergenza di D non la , ma la
libera
soltanto, ma nella formula di partenza tutta la
carica che presente entra in gioco, indipendentemente dalla sua origine che sia stata cio trasportata
dallinfinito al suo posto finale o sia sorta per polarizzazione nel punto in questione. Altra obiezione: la
formula sembrerebbe valere sempre mentre invece, giustamente, molti testi dicono che vale solo per
dielettrici lineari, cio per quelli per i quali D proporzionale ad E, cio per i quali P proporzionale ad
E. A questo punto opportuno vedere pi da vicino la questione.
112
Supponiamo di avere m molecole neutre, poste nei punti r
1
r
m
con polarizzabilit moleolare
i
.In presenza
di un campo elettrico queste molecole si polarizzano e si possono assimilare a dipoli che a loro volta
generano un campo elettrico. Per il principio di sovrapposizione, il campo E in un generico punto r, sar:
E = E
Q
+ E
dip
Il campo polarizzante agente sulla generica molecola i-esima, che si trova nel punto r
i
, sar E
Q
+ il campo
dipolare dovuto a tutte le altre molecole. Avremo cio

+
m
i j
i j
j
i j
i j
i j j Q i
1
3 5
0
) (
) ( 3
4
1
) (
r r
p
r r
r r
r r p E r E

Con i = 1,2,m
[VIA:Ora, mediante il modello di Gauthier Am.J.Phys. 52,835? (1984) si ha che un atomo posto in un campo
elettrico esterno E si polaririzza presentando un momento di dipolo p dato da:]
Ma si potr assumere:
p
i
=
i

0
E(r
i
)
con il coefficiente
i
caratteristico della molecola (polarizzabilit molecolare) ed indipendente da E e quindi
si ha subito:


1
1
]
1

+
m
i j
i j
j
i j
i j
i j j
i
i Q i i
1
3 5 0
) (
) ( 3
4
) (
r r
p
r r
r r
r r p r E p


Questo che si scritto un sistema di m equazioni nelle m incognite p.
Se gli sono molto piccoli (risultano, secondo il modello di Gauthier Am.J.Phys. 52,835? (1984) il triplo del
volume della molecola), il sistema si presta ad una soluzione iterativa. Si pu porre inizialmente:
) (
0
) 0 (
i Q i i
r E p
e proseguire scrivendo:

1
1
]
1

+
m
i j
i j
n
j
i j
i j
i j
n
j
i
i Q i
n
i
1
3
) 1 (
5
) 1 (
0
) (
) (
) ( 3
4
) (
r r
p
r r
r r
r r p r E p


Si vede quindi che, al primo ordine in , la polarizzazione di un atomo dipende solo dal campo prodotto dalle
cariche e non da quello prodotto dai dipoli circostanti (questo in realt era stato ammesso per fare il primo
passo del procedimento iterativo, ma se si risolve il sistema con un metodo diretto e si fa quindi uno sviluppo
in serie della soluzione in termini di , al primo ordine si ottiene quanto affermato).
Scriver dunque, al primordine in :
) (
0 i Q i i
r E p
Supponiamo ora di avere 1 atomo con la sua polarizzabilit atomica , posto nellorignie e vogliamo
assemblare n carica puntiformi. Ci chiediamo quanto sia il lavoro necessario per compiere questa operazione,
lavoro che corrisponde allenergia del sistema nel suo stato finale. Anzitutto una parte certamente presente di
questo lavoro sar quella necessaria per assemblare le cariche, una dopo laltra, in assenza dellatomo pi il
lavoro compiuto contro le forze che agiscono quando si sposta una carica dovute alla polarizzazione del
dielettrico, polarizzazione che varia con lo spostamento della carica. Per il primo lavoro non ci sono
problemi; si ripete tale e quale il ragionamento fatto quando si era nel vuoto e si ottiene allora che tale
lavoro, passando al caso continuo, dato da:
r
3
1
2
1
d V L


Per essere pi precisi scriver:
113
r
3
1
2
1
d V L
f f


dove con
f
si intende la carica libera (free), cio quella che muoviamo noi, per contrapporla alla carica che
si forma per polarizzazione ed analogamente V
f
il potenziale elettrostatico generato dalle cariche libere.
Vengo ora al lavoro contro le forze elettriche che si destano in seguito alla polarizzazione. Mentre nel caso
del vuoto, quando sistemavo ldallinfinito alla sua posizione finale la prima carica non compivo alcun lavoro
(ovviamente!), ora la carica polarizza latomo che determina un campo elettrico nel quale viene a trovarsi la
carica Q
1
che si sta muovendo per raggiungere la sua destinazione. Nellipotesi di linearit, latomo
costituisce un dipolo che, come si visto sopra, dato da:
3 1
r
Q
r
p
dove r il vettore posizione della carica Q
1
(inizialmente infinito ed alla fine r
1
). Questo dipolo produce nel
punto dove c la carica Q il classico campo dipolare dato da:
6 1 3
3 1
5
3 1
3 5
2
) ( ) (
3 3
r
Q
r
r
Q
r
r
Q
r r
r
r
r
r
r
p
r
r p
E

Il dipolo esercita allora sulla carica una forza attrattiva data da:
6
2
1 1
2
r
Q Q
r
E f
Il lavoro compiuto da questa forza nello spostamento della carica da r
1
allinfinito, rappresenta lenergia
elettrostatica del sistema legata a questo tipo dinterazione e questa energia anche lunica in gioco e
rappresenta dunque lenergia totale elettrostatica U dopo aver sistemato la prima carica. Scriver allora:



1 1 1 1 1
4
1
2
1
6
2
2
1 6
2
1 6
2
1 6
2
1 1
2
) ( ) ( ) (
2
1
2 2 ) (
r r r r r
r r r r
r
r
r f
r
Q
r
r d
Q
r
d
Q
r
d
Q d
r
Q d Q U


=
1 3
1
1
2
1
Q
r
finale
r p
Chiamer questa energia, anche se un po impropriamente, autoenergia della carica Q
1
.
Colloco ora la carica Q
2
. Come risulta dalla risoluzione del sistema di sopra, la carica Q
2
vede latomo
come un dipolo fisso, dovuto alla presenza della caricaQ
1
gi sistemata + un dipolo variabile indotto dalla
carica Q
2
che si sta posizionando. In altri termini, la carica Q
2
in presenza di un dipolo p dato da:
3 2 3
1
1
1 2 1
r
Q
r
Q
r r
p p p +
dove r pa posizione istantanea della carica Q
2
Il campo elettrico agente su di lei dato allora da:
3
3 2 3
1
1
1
5
3 2 3
1
1
1
3
2 1
5
2 1
3 5
) ( ) (
3
) ( ) (
3 3
r
r
Q
r
Q
r
r
Q
r
Q
r r r r
r
r
r
r
r
r
p p
r
r p p p
r
r p
E

restando fisso p
1
mente Q
2
si avvicina, il contributo allenergia del sistema dovuto a p
1
quando la Q
2
ha
raffiunto la sua posizione finale, sar dato dalla nota formula:
2
3
2
2 1
1
Q
r
U
r p

Mentre il contributo U
2
(autoenergia della carica 2),dovuto al dipolo variabile p
2
sar, come si visto sopra,
2 3
2
2 2
2
2
1
Q
r
U
r p

Quindi alla fine del trasporto di Q


2
, lenergia del sistema sar aumentata di U
1
+ U
2
. Lenergia totale allora
del sistema, avendo collocato le cariche Q
1
e Q
2
sar allora:

,
_

2 3
2
2 2
2 3
2
2 1
3
1
1 1 ) 2 (
2
1
2
1
Q
r
Q
r r
U
r p r p r p
114
Quando si colloca la carica Q
3
, lei vede due dipoli; p
1
e p
2
costanti ed un dipolo p
3
variabile dovuto alla Q
3
che si avvicina. Analogamente a quanto visto sopra, si scriver subito per lenergia del sistema una volta
collocata anche Q
3
:

,
_

,
_

3 3
3
3 3
3 3
3
3 2
3 3
3
3 1
2 3
2
2 2
2 3
2
2 1
3
1
1 1
) 3 (
2
1
2
1
2
1
Q
r
Q
r
Q
r
Q
r
Q
r
r
U
r p r p r p
r p r p
r p
e quando finalmente si assemblata la n-esima carica, si avr:

,
_

,
_

,
_

n
n
n n
n
n
n
n
n
n
n
Q
r
Q
r
Q
r
Q
r
Q
r
Q
r
Q
r
Q
r
r
U
3 3
2
3
1
3
3
3
3 3
3
3
3
3 2
3
3
3
3 1
2
3
2
2 2
2
3
2
2 1
3
1
1 1
) (
2
1
2
1
2
1
2
1
r p r p r p
r p r p r p
r p r p
r p

.
o anche:
n
n
n n
n
n
n n
n
n
n
n
n
n
n
Q
r
Q
r
Q
r
Q
r
Q
r
Q
r
Q
r
Q
r
Q
r
Q
r
Q
r
Q
r r
U
3 3 3
2
3
1
3
3
3
3 3
3
3
3
3 3
3
3
3
3 2
3
3
3
3 1
2
3
2
2 2
2
3
2
2 2
2
3
2
2 1
1 3
1
1 1
3
1
1 1
) (
2
1
2
1
2
1
2
1
r p r p r p r p
r p r p r p r p
r p r p r p
r p r p

,
_

,
_

,
_

.
Lenergia dunque associata allatomo si pu vedere dunque come una somma di autoenergie + una somma
di mutue energie. Si pu osservare che il generico termine della mutua energia dello stesso ordine di
grandezza di un termine di mutua energia, ma che i termini di mutua energia sono n, mentre quelli di
autoenergia sono n(n+1)/2 e che quindi, per n infinitamente grande, i termini di autoenergia possono
trascurarsi completamente. (DISCORSO INUTILE E SBAGLIATO)Potremo allora scrivere:
115

n
i
i
j
i
i
i
j
n
Q
r
U
1 1
3
) (
r
p
e ricordando che :
3
j
j
j j
r
Q
r
p
, sostituendo si ha:



n
i
i
j
i
i
i
j
j
j
n
Q
r r
Q U
1 1
3 3
) (

r
r

Essendo gli addendi simmetrici negli indici i e j, posso scrivere:



+ +
i i
i
i
i i
i
n i
n
i
n
j
i
i
i
j
j
j
n
r
Q
r
Q Q
r r
Q U
3 3 2 1
1 1
3 3
) (
2
1
) (
2
1
2
1

r
p
r
p p p
r
r

Nel caso che le cariche costituiscano un continuo, la relazione di sopra si generalizza immediatamente in:

r
3
2
1
d V
dipolare f

Tenendo presente ora linterazione tra le cariche libere, si ha dunque che lenergia di un sistema costituito da
cariche libere ed un atomo polarizzabile, data da:

+ r r
3 3
1
2
1
) (
2
1
Vd d V V U
f dipolare f f

dove V
dipolare 1
vuol dire, ovviamente, il potenziale dovuto al dipolo che costituisce latomo 1
Passiamo ora al caso in cui siano presenti due atomi polarizzabili. Siccome, per quanto si visto risolvendo
il nostro sistema iniziale, al primo ordine nella polarizzabilit non c interazione tra una atomo ed un
altro ( nel senso che la polarizzazione di uno non contribuisce alla polarizzazione dellaltro, anche per
questaltro potr scrivere:

+ r r
3 3
2
2
1
) (
2
1
Vd d V V U
f dipolare f f

e per i due allora:

+ + r r
3 3
2 1
2
1
) (
2
1
Vd d V V V U
f dipolare dipolare f f

Ma allora dovrebbe essere chiaro che in presenza anche di un continuo polarizzabile si dovr avere (al primo
ordine nella polarizzazione!!!)

r E D r
3 3
2
1
d d V U
f

Il campo elettrico che agisce sulla carica Q


1
allora quello prodotto dai due dipoli e dallaltra carica ed :

carica della Campo dipolare campo
Q
3
4 3
4 3
2 5
2 3
2 3
2 3 2 5
1 3
1 3
1 3 1 3
) (
) (
) ( 3
) (
) (
) ( 3 ) (
r r
r r
r r
r r
r r p
r r
r r
r r p r E


ed ovviamente unanaloga espressione varr per il campo sulla seconda carica
Questo campo elettrico altro non che il gradiente, col segno cambiato del potenziale elettrostatico V, con V
dato da:
4 3
2
3
2 3
2 3 2
3
1 3
1 3 1
3
) (
) (
) (
) (
) (
r r
r r
r r p
r r
r r p
r

Q
V
4 3
2
3
2 3
2 3 3
4 2
4 2
2 3
3 2
3 2
1
3
1 3
1 3 3
4 1
4 1
2 3
3 1
3 1
1
3
) (
) (
) (
) (
) (
r r
r r
r r
r r
r r
r r
r r
r r
r r
r r
r r
r r
r r
r

,
_

,
_

Q
Q Q Q Q
V
116
4 3
2
4 2
3 2 4 2 2
4 3
3 1 4 1 2
4
2 3
4
1 3
1 3
) ( ) ( ) ( ) ( 1 1
) (
r r r r
r r r r
r r
r r r r
r r r r
r

,
_


Q Q Q
Q V
Per calcolare il lavoro di costruzione del sistema, portiamo dapprima la prima carica. Nellespressione del
campo che agisce su di lei, manca il contributo del campo generato dalla carica Q
2
, dato che r
4
infinito. Per
portare questa carica dunque occorre integrare tra r
3
ed infinito

dipolare campo
Q Q
5
2 3
2 3
2 3 2 5
1 3
1 3
1 3 1 1 3 1
) (
) (
) ( 3
) (
) (
) ( 3 ) (
r r
r r
r r p
r r
r r
r r p r E


In questa integrazione p
1
e p
2
variano con r
3
e devono essere sostituiti conn le loro espressioni ottenute sopra
(dove per mancano i termini contenenti r
4
per la ragione di prima) in termin i di r
3
.Si ha dunque:
( ) ( )
5
2 3
2 3
2 3 3
3 2
3 2
1 5
1 3
1 3
1 3 3
3 1
3 1
1 3
) (
) (
) ( 3
) (
) (
) ( 3 ) (
r r
r r
r r
r r
r r
r r
r r
r r
r r
r r
r E

Q Q
6
2 3
2 3
1 6
1 3
1 3
1 3
) (
) (
3
) (
) (
3 ) (
r r
r r
r r
r r
r E

Q Q
e quindi:
Energia del sistema dopo aver portato la prima carica:
Energia =
1
1
]
1

6
2 3
2 3
6
1 3
1 3
3
2
1
) (
) (
) (
) (
3
3
r r
r r
r r
r r
r
r
d Q =

,
_

4
2 3
4
1 3
2
1
1 1
4
3
r r r r
Q
Supponiamo di avere un atomo neutro nellorigine caratterizzato da un nucleo puntiforme di carica +e fisso
nellorigine e da una nuvoladi carica elettrica e Portiamo ora una carica Q dallinfinito ad un punto
distante r dal nucleo Man mano la carica Q si avvicina, la nuvola si deforma(forse, ma probabilmente la
deformazione solo un effetto al secondo ordine, comunque non importante sapere se e quanto vale la
deformazione) ed il centro di carica della carica e si sposta verso la carica Q. Questo spostamento, indicato
con x, una funzione del campo elettrico della carica Q. Con uno sviluppo in serie troncato al primo ordine,
si ha che x = E
carica Q
e quindi , dato che E
carica Q
e inversamente proporzionale ad r
2
, si ha che
x = /r
2
.
Dire di x vuol dire anche dire di P e quindi in questa approssimazione P proporzionale al campo elettrico
(approssimazione lineare) e quindi il parametro sostanzialmente la polarizzabilit atomica. Calcoliamo la
forza che agisce su Q quando ad una distanza r dallorigine. C una forza repulsiva dovuta al nuleo pari a
eQ/r
2
ed una forza attrattiva dovuta alla carica negativa e concentrata nel suo centro di carica che si trova
proprio in x e quindi questa forza : -eQ/(r-x)
2
. La forza totale dunque:
f = eQ/r
2
- eQ/(r-x)
2
sostituendo per x ottengo:
f = eQ/r
2
- eQ/(r-/r
2
)
2
Ora :
r
U
f


Dove U(x) una complicata espressione di r che non il caso di riportare. Ma per molto piccolo, cio
facendo uno sviluppo di U(,r) nellintorno di = 0, si ha (trascurando termini costanti in r):

3
10 7
2
4
5
2
7
3
2
) , (

r
eQ
r
eQ
r
eQ
r U
Il significato ficsico di U che esso rappresenta il lavoro fatto da una forza esterna per portare la carica Q
dallinfinito fino alla distanza r e quindi rappresenta lenergia del sistema
Il potenziale elettrico agente sulla carica Q dovuto al dipolo :
117


7
2
4
2
r
e
r
e
r
r
e
r
e
x r
e
r
e
V


Si vede quindi che, al primo ordine in , :
U = Q V
Osservazioni sullenergia del campo elettrico in presenza di dielettrici
Spesso si vede questo procedimento:
Lenergia di un sistema elettrostatico data, sempre da:

r Vd U
3
2
1

ma : D e quindi subito:

r Vd U
3
) (
2
1
D
poi, portando il segno di divergenza ad operare anche su V, usando il teorema della divergenza ed il fatto che
allinfinito i campi sono nulli, si ha:

r d U
3
2
1
D E
Ma io ho da obiettare: la divergenza di D non la , ma la
libera
soltanto, ma nella formula di partenza tutta la
carica che presente entra in gioco, indipendentemente dalla sua origine che sia stata cio trasportata
dallinfinito al suo posto finale o sia sorta per polarizzazione nel punto in questione. Altra obiezione: la
formula sembrerebbe valere sempre mentre invece, giustamente, molti testi dicono che vale solo per
dielettrici lineari, cio per quelli per i quali D proporzionale ad E, cio per i quali P proporzionale ad
E. A questo punto opportuno vedere pi da vicino la questione.
Supponiamo di avere due molecole neutre con polarizzabilit molecolare (per semplicit, per ambedue la
medesima) nei punti r
1
ed r
2
. Portiamo ora dallinfinito una carica Q
1
nel punto r
3
e poi una carica Q
2
nel
punto r
4
. Voglio calcolare in primo luogo la polarizzazione delle due molecole. Quando le due cariche hanno
raggiunto la loro posizione, le molecole presenteranno, rispettivamente, un momento di dipolo p
1
e p
2
. Il
campo elettrico che agisce sul dipolo 1 dato da:

'

1
1
]
1

3
2 1
2
5
2 1
2 1
2 1 2 3
4 1
4 1
2 3
3 1
3 1
1
0
1
) (
) ( 3
4
1
) (
r r
p
r r
r r
r r p
r r
r r
r r
r r
r E Q Q

ed analogamente:

'

1
1
]
1

3
1 2
1
5
1 2
1 2
1 2 1 3
4 2
4 2
2 3
3 2
3 2
1
0
2
) (
) ( 3
4
1
) (
r r
p
r r
r r
r r p
r r
r r
r r
r r
r E Q Q

Se il campo non troppo intenso (rispetto a quello che vige allinterno della molecola), si soliti ammettere
p
1
=
0
E(r
1
) p
2
=
0
E(r
2
)
con indipendente da E, caratteristico della molecola, dimensionalmente e numericamente dellordine di
grandezza del volume della molecola.[Feynmann Lectures on Physics, formula (11.12) pagina elettronica
641]
e quindi subito:

'

1
1
]
1

3
2 1
2
5
2 1
2 1
2 1 2 3
4 1
4 1
2 3
3 1
3 1
1 1
) (
) ( 3
4
r r
p
r r
r r
r r p
r r
r r
r r
r r
p Q Q

'

1
1
]
1

3
1 2
1
5
1 2
1 2
1 2 1 3
4 2
4 2
2 3
3 2
3 2
1 2
) (
) ( 3
4
r r
p
r r
r r
r r p
r r
r r
r r
r r
p Q Q

118
Questo che si scritto un sistema atto a determinare le incognite p
1
e p
2
Se molto piccolo, il sistema si presta ad una soluzione iterativa. Infatti posso porre inizialmente:

'

3
4 1
4 1
2 3
3 1
3 1
1
) 0 (
1
4
r r
r r
r r
r r
p Q Q

'

3
4 2
4 2
2 3
3 2
3 2
1
) 0 (
2
4
r r
r r
r r
r r
p Q Q

e proseguire scrivendo:

'

1
1
]
1

+
3
2 1
) (
2
5
2 1
2 1
2 1
) (
2 3
4 1
4 1
2 3
3 1
3 1
1
) 1 (
1
) (
) ( 3
4
r r
p
r r
r r
r r p
r r
r r
r r
r r
p
n
n n
Q Q

'

1
1
]
1

+
3
1 2
) (
1
5
1 2
1 2
1 2
) (
1 3
4 2
4 2
2 3
3 2
3 2
1
) 1 (
2
) (
) ( 3
4
r r
p
r r
r r
r r p
r r
r r
r r
r r
p
n
n n
Q Q

Si vede quindi che, al primo ordine in , la polarizzazione di un atomo dipende solo dal campo prodotto dalle
cariche e non da quello prodotto dai dipoli circostanti (questo in realt era stato ammesso per fare il primo
passo del procedimento iterativo, ma se si risolve il sistema con un metodo diretto e si fa quindi uno sviluppo
in serie della soluzione in termini di , al primo ordine si ottiene quanto affermato
Scriver dunque, al primordine in :

'

3
4 1
4 1
2 3
3 1
3 1
1 1
4
r r
r r
r r
r r
p Q Q

'

3
4 2
4 2
2 3
3 2
3 2
1 2
4
r r
r r
r r
r r
p Q Q

Supponiamo ora di avere una molecola con polarizzabilit molecolare , posta nellorigine e vogliamo
assemblare n carica puntiformi. Ci chiediamo quanto sia il lavoro necessario per compiere questa operazione,
lavoro che corrisponde allenergia del sistema nel suo stato finale. [FORSE QUESTA PRECISAZIONE
INUTILE. Tale lavoro sar quello necessario per assemblare le cariche, una dopo laltra, in assenza
dellatomo, pi il lavoro compiuto contro il campo elettrico dipolare generato dallatomo in presenza delle
cariche gi assemblate e di quella che si sta trasportando. Per il primo lavoro non ci sono problemi; si ripete
tale e quale il ragionamento fatto quando si era nel vuoto e si ottiene allora che tale lavoro, passando al caso
continuo, dato da:
r
3
1
2
1
d V L


Per essere pi precisi scriver:
r
3
1
2
1
d V L
f f


dove con
f
si intende la carica libera (free), cio quella che muoviamo noi, per contrapporla alla carica che
si forma per polarizzazione ed analogamente V
f
il potenziale elettrostatico generato dalle cariche libere.]
Per calcolare questo lavoro, supponiamo di aver gi collocato al loro posto, cio nei punti r
1
,r
2
r
i-1
le prime
i cariche, sicch lenergia del sistema data da U(Q
1
,Q
2
,Q
i-1
) e collochiamo ora, spostandola dallinfinito,
la i-esima carica Q
i
. La generica j-esima carica (j = 1,2,i) induce nellatomo un momento di dipolo
119
3
4
j
j
j j
r
Q
a
r
p


, j = 1,2,, sicch il momento di dipolo totale che riassume le propriet elettriche
dellatomo risulta essere pari a:
3
1
1
3
4 4 r
Q
r
Q
i
i
j j
j
j
r
r
p

dove r la posizione, variabile nel tempo,


della carica Q
i
che viene trasportata. Questi dipoli creano, unitamente alle cariche Q
1
,Q
2
,Q
i-1
un campo
elettrico che agisce sulla Q
i
. Il lavoro, col segno cambiato, compiuto dalla forza elettrica nello spostamento
della carica Q
i
, sar lincremento di energia del sistema, e quindi lenergia totale sar data da :
U = U(Q
1
)+ U(Q
1
,Q
2
)+ U(Q
1
,Q
2
,Q
3
)+ U(Q
1
,Q
2
,Q
3
Q
n
)
Vediamo dunque questo incremento di energia. La forza elettrica su Q
i
:

'

1
1
]
1

1
1
]
1

6
0
2
2 1
1
3 3 5 3 3
0
3 5 3
1
1 0
) 4 (
2
4 4
3
4 4
) (
r
Q
r r r r
Q
Q
r r
Q
Q
Q
i
i
j j
j
j
j
j
j
j
i
j
j
i
j
j
i
i
r
r
r
r r
r r
r r
p
r
r p
r r
r r
r E


Integrando tale forza in r tra r
i
e linfinito, si ottiene questo incemento di energia. Eseguendo lintegrazione,
si ha:

'

+
1
1
]
1

3
1
1
3
0
2 1
2
1
4
) , , (
i
i i
i
j j
j j
j i
j
i
i
r r
Q
Q
Q Q Q U
r p
r p
r r

E quindi :

'

+
1
1
]
1

n
i i
i i
i
j j
j j
j i
j
i n
r r
Q
Q Q Q Q U
1
3
1
1
3
0
2 1
2
1
4
1
) , , (
r p
r p
r r

Ma immediato vedere che la doppia sommatoria in questione si pu scrivere, aumentando il campo di


variabilit di j, come:

1
1
]
1

n
i
n
j j
j j
j i
j
i
r
Q
Q U
1 1
3
0
4
1
2
1
r p
r r
Si riconosce ora che

+
1
1
]
1

1
1
]
1

n
j j
j
j i
j
n
j j
j j
j i
j
r
Q
r
Q
1
3
0 0 1
3
0
4
1
4
1
4
1
r p
r r
r p
r r
altro non che il potenziale elettrico, nel punto dove c la generica carica i-esima, generato dalle altre
cariche e dalla molecola. Si ha dunque, con ovvio significato dei termini:

n
i
i i
V Q U
1
2
1
Se ora gli atomi sono pi di uno, ma 2,3,m si riconosce subito che, per linearit, lespressione di sopra vale
sempre, a patto di scrivere:

+
1
1
]
1

n
j
m
i j
j
i
j i
j
i
r
Q
V
1 1
3
) (
0 0
4
1
4
1
r p
r r
dove p
(i)
il momento dipolare delli-esimo atomo indotto dalle n cariche libere Q.
Nel caso di passaggio al continuo, si scriver ovviamente:
120

spazio to tutto
libera
d V U r r r ) ( ) (
2
1

A questo punto non c pi nessuna difficolt a trasformare la formula ottenuta nella:


spazio to tutto
d U r r E r D ) ( ) (
2
1
Considero ora una carica puntiforme Q posta nellorigine e circondata da un dielettrico perfetto ed infinito.
Voglio determinare il campo elettrico in un ounto individuato dal vettore c. A tal fine considero un
elementino dv individuato dal vettore r. Il momento di dipolo di questo elemento sar proporzionale al
campo generato dalla carica Q ed al volumetto in questione. Scriver allora: dv
r
Q
a
dv d
3 0
4
r
E p

.
Questo dipolo produce in c un potenziale dato da:
dv
r
Q d
d
3 3
0
2 3
0
) (
) 4 (
) (
4
1
r c
r c
r
r c
r c p

Esprimendo dv in coordinate polari, scriver:


dr d d
cr r c
r c
Q
d
d


) sin(
)) cos( 2 (
) ) cos( (
) 4 (
) (
4
1
2 / 3 2 2
0
2 3
0
+

r c
r c p
Lintegrazione sullangolo si fa subito e porta un fattore 2. Lintegrazione su porta a: 0 se c > r ed a Q/
(4)/
0
dr/r
2
se c < r. Integrando ora tra r=0 ed infinito ottengo un campo come quello della carica c.v.d.
COMPITI DI ESAMI
COMPITO DEL 3/5/02
1) (molto facile) Un ragazzo di 90 kg siede sulla sommit di una scala a compasso, di massa
trascurabile che poggia su di un pavimento privo di attrito. La scala munita a met della sua altezza
di un tirante (fune) orizzontale. Langolo al vertice = 30. Qual la forza esercitata dal
pavimento su ciascun montante? Qual la tensione del tirante?
2) (facile) Un disco di 10 cm di raggio ruota intorno al suo asse partendo dalla quiete, con
accelerazione angolare costante di 10 rad/s
2
Nellistante t = 5 s qual la velocit angolare del disco e
laccelerazione radiale e tangenziale di un punto del disco?
3) (pu avere pi risposte) Con un goniometro avete misurato i tre angoli C B A

di un triangolo
ottenendo, rispettivamente, le seguenti misure: 62 30; 65 17, 60 01 Volete comunicare i valori
dei tre angoli ad un amico, che valori fornirete?
4) (no comment)Una pallina cade inizialmente da unaltezza di 10 metri, rimbalza, ricade, rimbalza
finch finalmente si ferma. Se ad ogni rimbalzo perde, per attriti interni, il 10% dellenergia cinetica
che aveva prima dellurto, che lunghezza percorrer prima di fermarsi? In quanto tempo?
5) (occorre fare schematizzazione) Un sarto sta tagliando una stoffa con una forbice che si chiude con
velocit angolare costante. Con che velocit procede la forbice nel taglio della stoffa?
6) (riflettere un momento) Una bacchetta lunga 2R viene deposta in una scodella semisferica di raggio
R. tra scodella e bacchetta si trascura lattrito. Determinare la posizione dequilibrio della bacchetta.
7) (conviene spezzarlo in due fasi) Unasta rigida di massa trascurabile e lunghezza L ha un estremo
incernierato in un perno orizzontale senza attrito e laltro porta una massa M. Un proiettile di massa
m e velocit V perpendicolare allasta si conficca in questa nel punto di mezzo mettendola quindi in
moto. Trovare langolo massimo raggiunto nel moto. Quanto vale la forza esercitata dal perno
quando lasta alla sua massima altezza?
121
8) (facile) In un punto di una circonferenza di raggio R posta su di un piano orizzontale senza attrito
attaccato un filo di lunghezza 3R disposto inizialmente radialmente. Allaltro estremo del filo
attaccata una massa puntiforme m avente velocit V perpendicolare la filo stesso. A causa di questa
velocit il filo si arrotoler sulla circonferenza tirando quindi verso di lei la massa m. si domanda la
velocit di questa quando urter la circonferenza e la tensione del filo quando sar avvoltolata la
met. (Pi difficile): sapete determinare quanta strada percorre la massa m prima dellurto?
9) (difficile, da risolversi con astuzia e intelligenza, indicate come procedere): un triangolo equilatero di
lato 1 metro posto con un lato parallelo al suolo ad unaltezza di 20 metri. Si vuole sparare da terra
con un cannoncino un proiettile che possa raggiungere la vetta del triangolo equilatero. Se vogliamo
che la velocit del proiettile alluscita dalla bocca del cannone ( si suppone posta a livello del suolo)
sia la minima possibile, quanto deve essere questa velocit e dove deve essere posto il cannone e con
quale alzo deve sparare?
10) (basta aver studiato) Un punto materiale pu muoversi senza attrito sullasse X. Ed inizialmente
fermo. Se una forza agente su di lui esplica una potenza costante, determinare il moto del punto
11) (facile) Tre masse identiche sono poste ai vertici di un triangolo equilatero e sono inizialmente in
quiete. A causa della forza gravitazionale si muoveranno verso il centro del triangolo. Si vogliono le
velocit delle tre masse quando la loro distanza reciproca sar dimezzata
Esame di Fisica del 10/6/02
(molto facile) Nel disegno di figura le carrucole sono senza massa. Trovare la tensione t del filo che regge m,
quella T del filo che regge M e laccelerazione A di M, dato che il sistema immerso nel campo di gravit
diretto verso il basso
Risposta.
Dato che la carrucola mobile senza massa, la risultante delle forze applicate su di lei deve essere nulla,
cio:
T - 2 t =0
Lequazione del moto di M porta a scrivere:
Mg T = M a
M
Lequazione del moto di m porta a scrivere:
mg t = m a
m
Linestensibilit del filo porta a scrivere:
a
M
= -1/2 a
m
e quindi:
{ } , , , aM
g ( ) M 2 m
+ 4 m M
t
3 m g M
+ 4 m M
am
2 g ( ) M 2 m
+ 4 m M
T
6 m g M
+ 4 m M
122
(facile) Un missile (1) viene lanciato verso lalto con una velocit iniziale V
m
. contemporaneamente, ad una
distanza di L metri, ne viene lanciato un altro (2) con velocit di modulo V e formante con lorizzontale un
angolo . Determinare V affinch i due missili possano scontrarsi.(lievemente pi difficile) Se ora non si
fissa il valore di , quanto il valore minimo di V affinch possa esserci lincontro?
Risposta.
Con ovvio significato di simboli, sar:
x
(1)
(t) = L y
(1)
(t) = -1/2 g t
2
+ V
m
t
x
(2)
(t) = V cos() t y
(2)
(t) = -1/2 g t
2
+ V sin() t
Al momento dello scontro le due ascisse devono essere uguali, cio deve essere
V cos() t
incontro
= L
E quindi
t
incontro
= L / V cos()
Ma anche le due ordinate devono essere uguali ed inoltre maggiori di 0.
La prima condizione si traduce nellequazione:
-1/2 g (L / V cos())
2
+ V
m
L / V cos() = -1/2 g (L / V cos())
2
+ tan() L
e quindi
V = V
m
/ sin()
Se, con tale valore di V e quindi con
t
incontro
= tan() L / V
m

risulta y
(1)
(t
incontro
)= y
(2)
(t
incontro
) >= 0, allora la soluzione proprio il valore trovato, V = V
m
/ sin(), altrimenti
non c soluzione.
Non sia assegnato ora il valore di . Sia
*
il valore dellangolo per il quale c scontro con la velocit V
minima possibile. Le relazioni di sopra dovranno sempre valere, e cio dovr essere:
V sin(
*
)= V
m
V cos(
*
) =L/t
incontro
Affich V sia minimo, dovr essere t
incontro
massimo. Ma t
incontro
non pu essere maggiore del tempo di volo del
missile (1) ,cio di 2 V
m
/g. Il massimo valore dunque di t
incontro
sar dunque dato proprio da 2 V
m
/g e lo
scontro avverr proprio nellistante in cui il primo missile ricade a terra. Quindi
V
minimo
cos(
*
) =Lg/(2 V
m
)
Ottengo poi:
tan(
*
) = 2 V
m
2
/(Lg)
e quindi:
V
minimo
= (L
2
g
2
+4V
m
4
)
1/2
/ (2 V
m
)
(media difficolt) I dodici spigoli di un cubo sono costituiti da 12 resistenze di 1 luna. Calcolare la
resistenza che si presenta tra due vertici diametralmente opposti del cubo.
Risposta:
123
I punti A, B, C saranno allo stesso potenziale V, come pure i punti a,b,c ad un potenziale v. V e v sono
connessi da 6 resistenze (i lati Ac, cC, bC, bB, aA, aB), ognuna di valore 1 , quindi da una resistenza
equivalente di 1/6 di . Ora il potenziale V connesso al punto P da tre resistenza in parallelo da 1 luna,
quindi da una resistenza di 1/3 di ed analogamente connesso Q con v. Tra P e Q si ha dunque una
resistenza pari a 1/3 + 1/6 + 1/3 = 5/6 di
(abbastanza facile) Una spira quadrata giace allistante t = 0 nel piano x-y con il suo centro nel punto (00) e
lati paralleli agli assi. Un campo dinduzione B uniforme e costante nel tempo diretto lungo lasse z.
Allistante 0 viene messa in rotazione lungo lasse x con una velocit angolare costante . Se ogni lato della
spira, come pure il segmento dellasse di rotazione che connette due lati, costituito da una resistenza R,
determinare la corrente che passa in ogni ramo del circuito. (si trascurino fenomeni di autoinduzione).
Risposta.
Sui rami paralleli allasse di rotazione, per simmetria, passeranno due correnti opposte. Quindi nel ramo
centrale, non passer corrente. La f.e.m. nella spira sar la derivata rispetto al tempo del flusso di B
attraverso di lei, cio: B S sin( t) ( S la superficie della spira). Dividendo tale forza elettromotrice per
4R, (resistenza totale del circuito) ottengo lintensit della corrente che circola in ogni ramo
124
Esame di Fisica del 20/6/02
Nel disegno di figura il piano orizzontale che regge la massa m = 1 kg presenta un coefficiente dattrito
statico
s
= 0.1. La massa M vale 2 kg.
(1) Determinare il pi piccolo valore di affinch sussista lequilibrio
(2) Il piano sia ora liscio. Lasciata libera di muoversi la massa m quando ad una distanza x
0
dalla
carrucolina, determinare le velocit di M e di m quando questultima tocca la carrucola
Una sbarretta di massa trascurabile e lunghezza L incernierata senza attrito ad unasta verticale passante
per il suo centro. Alle estremit della sbarra sono saldate due masse puntiformi m e 2m Con un martello che
colpisce la massa m con velocit perpendicolare alla sbarra, questa viene messa in rotazione con velocit
angolare . Determinare limpulso esercitato dal perno sulla sbarretta durante la martellata
Il circuito di figura alimentato da una pila da 1.5 V di forza elettromotrice e con una resistenza interna r =
2.5 . Le cinque resistenze sono tutte di 1 . Quali interruttori si devono chiudere affinch la potenza
trasferita dalla pila al circuito sia massima ( cio voglio che sia massima la somma delle potenze dissipate
dalle cinque resistenze R)
125
Il quarto di circonferenza sulla destra ricoperto uniformemente da una carica Q, mentre la parte sinistra, da
una carica Q. trovare il campo elettrico al centro O
Risposte:
(1) La forza premente : mg + T sin(), dove T la tensione della fune. Se moltiplico la forza premente per
il coefficiente
s
, ottengo il massimo valore della forza che lattrito pu esplicare prima che il corpo si metta
in moto. Il pi piccolo valore di si otterr uguagliando questo valore alla componente della forza della fune
che cerca di mettere in moto il blocco, cio T cos(). Dovr dunque avere:
T cos() = [mg + T sin()]
s
126
Ma allequilibrio, poich pure M lo , sar T = Mg = 2*9.81 = 19.62 N. La relazione di sopra diventa
dunque:
19.62*cos() = [1*9.81 + 19.62*sin()]*0.1 1.42 rad
(3) Detta y la lunghezza della porzione pendente del filo, ed L la sua lunghezza totale, deve essere:
(x
2
+ h
2
)+y = L.
derivando rispetto al tempo la relazione ottenuta si ha la seguente relazione tra le velocit (a parte
eventualmente un segno):
v
M
= 2x / (x
2
+ h
2
)v
m
Dato che quando m tocca la carrucola x = 0 , la relazione di sopra implica che M ferma. Solo m in
moto. Dal teorema della conservazione dellenergia meccanica si determina subito allora la sua velocit.
Disposta la sbarra lungo lasse y, limpulso ceduto dal martello si pu indicare con il vettore: I i. La seconda
equazione cardinale applicata alla sbarretta con polo il centro di questa, porta subito a scrivere:
I L/2 = m (L/2)
2
+ 2m (L/2)
2
Da cui subito: I = 3m L/2
Daltra parte, detto I
p
limpulso sviluppato dal perno, la prima equazione cardinale fornisce la relazione:
I
p
+ I i = (m + 2m) v
centro di massa
Ma il centro di massa ad 1/3 L di distanza dalla massa 2m, cio ad 1/6L dal centro di rotazione e quindi :
v
centro di massa
= L/6. In termini vettoriali poi: v
centro di massa
= - L/6i
Quindi, concludendo, si ha:
I
p
+ 3m L/2 i = - (m + 2m) L/6i I
p
+ 3m L/2 i = - 2m L i
Sia la resistenza che presenta il circuito quando la potenza trasferita dalla pila massima.
La corrente che passa allora attraverso la pila :
i = V/( + r)
La potenza dissipata nel circuito i
2
= V
2
/ (r + )
2
. Il massimo di questa espressione, al variare di si
verifica, come banale verificare, quando = r. La resistenza dunque del circuito deve essere dunque pari a
quella interna, cio 2.5 . Ma questo si realizza, come si vede subito, chiudendo il tasto di sinistra e quello in
basso
127
Il valore di E dato da:

2 /
0
2
0
) cos(
4
2

d
R
R
dove = Q/(R/2)
COMPITO DI FISICA DEL 10-7-02
Una lastra quadrata metallica di lato 10 cm e massa 2 kg poggia su di un piano orizzontale scabro con
coefficiente di attrito statico
S
. (vedi figura). Si applica in C una forza di intensit crescente e formante un
angolo di 30 con lorizzontale finch la lastra non inizia a muoversi. Determinare il minimo coefficiente
S
affinch la lastra inizi il movimento ruotando ( intorno ad B, ovviamente)
Nota: la figura sbagliata: la forza inclinata sullorizzontale!
Soluzione Se inizia a ruotare intorno a B, il momento rispetto a B della forza F deve essere almeno pari al
momento rispetto a B della forza peso, cio deve essere:
F
x
l = Mg l/2
E quindi F
x
= Mg / 2 Poich la forza forma un angolo di 30 con lorizzontale, quando F
x
avr il valore
trovato, la F
y
varr: F
x
tan(30) = Mg3/6
La forza con cui il blocco preme sul piano data dal peso del blocco stesso diminuito dalla componente
verticale della forza F agente sul blocco, componente che tenta di sollevarlo. quindi: N = Mg-F
y
Allora, affinch una tale forza F non faccia scivolare il blocco lungo il piano, deve essere:
(Mg-F
y
)
s
F
x
= Mg/2 e quindi:
s
Mg/[2(Mg-F
y
)] e quindi il minimo coefficiente dattrito varr:

s
=Mg/[2(Mg - Mg3/6 )] = 3/(6- 3)
Uno yo-yo ha raggio esterno di 10 cm , mentre quello interno di 5 cm. E la sua massa, di 1 kg
uniformemente distribuita sulla superficie esterna. Un filo avvolto in senso antiorario per chi guarda
sullasse interno ed collegato , tramite carrucola ideale, ad un filo che regge una massa m di kg e pende
nel vuoto. Determinare laccelerazione di m
128
Soluzione
Sta T la tensione della fune, f la componente lungo sinistra che la forza del piano esercita sul disco. La prima
equazione cardinale applicata ad M fornisce:
(1) Mg T = M a
M
La prima equazione cardinale applicata al disco fornisce:
(2) T f = m a
c.m.
dove a
c.m.
laccelerazione del centro di massa del disco.
La seconda equazione cardinale applicata al centro del disco fornisce:
3b) f R - T R = I ` = I a
c.m.
/R
Dove I il momento dinerzia del disco rispetto as un asse baricentrale
Infatti il primo membro costituisce il momento delle forze rispetto al centro del disco assunto come polo,
momento assunto come positivo se fa ruotare il disco in senso orario. Per la seconda equazione cardinale,
tale momento deve essere uguagliato alla derivata rispetto al tempo del momento della quantit di moto del
sistema, sempre calcolato rispetto al centro del disco.
ora abbastanza facile convincersi che se il disco ruota in senso orario di un giro, la massa M scende di
2(R-r) mentre la massa m si sposta di 2R. quindi il rapporto tra gli spostamenti R/(R-r) e siccome tali
spostamenti sono contemporanei, tale pure il rapporto tra le accelerazioni. Sar allora:
(2) a
c.m.
/ a
M
= R/(R-r)
La (1), (2), (3b), (4) hanno come soluzione ( ho posto: a = a
m
A = a
M
J = I):
f
Mg ( ) + r R m J
+ + + R
2
M 2 R Mr R
2
m J Mr
2
a
g MR ( ) + R r
+ + + R
2
M 2 R Mr R
2
m J Mr
2
, , {
A
g M( ) + R r
2
+ + + R
2
M 2 R Mr R
2
m J Mr
2
T
Mg ( ) + J R
2
m
+ + + R
2
M 2 R Mr R
2
m J Mr
2
, }
Determinare la carica sul condensatore dopo molto tempo che si chiuso il tasto
129
Risoluzione A regime nel circuito grande circola la corrente: I = (E
2
E
1
) / ( R
1
+ R
2
). La differenza di
potenziale ai capi del condensatore sar allora:
V = E
2
I R
2

e quindi :
Q = C V
Determinare il campo B al centro della spira di figura
Risoluzione Per motivi di simmetria nullo
Determinare al campo B nel punto O di figura ( langolo in O di 60)
Risoluzione ricordando che il campo B al centro di una spira di raggio R dato da:
B = I
0
/ 2R
Si ha che il campo generato da un arco che un sesto di circonferenza sar:
B = (I
0
/ 2R) /6
E quindi il campo in O sar:
B = (I
0
/ 2R
1
) /6 - (I
0
/ 2R
2
) /6
130
Il circuito di figura immerso in un campo B perpendicolare al piano, costante ed uniforme. La sbarretta AB
si muove verso destra con velocit costante V. ogni elemento del circuito costituito da un filo conduttore di
resistenza r per unit di lunghezza. Al tempo t = 0 la sbarretta si trova al bordo sinistro del circuito.
Determinare la corrente in funzione del tempo che passa nel circuito.
Soluzione Si visto pi volte che un conduttore filiforme in moto in un campo B ortogonale a lui ed alla sua
velocit presenta agli estremi una d.d.p. pari a: VlB. Quindi, per il calcolo delle correnti, come se la
sbarretta fosse un generatore di tensione E = VlB con una resistenza interna pari alla resistenza r della
sbarretta. Detta R
1
la resistenza della parte sinistra del circuito ed R
2
quella della parte di destra, i
1
ed i
2
le due
correnti, i quella che attraversa la sbarretta, sar allora:
e1:=i=i1+i2;
:= e1 i + i1 i2
e2:=-i*r+E-i1*R1=0;
:= e2 + i r E i1 R1 0
e3:=-i*r+E-i2*R2=0;
:= e3 + i r E i2 R2 0
La cui soluzione :
{ } , , i2
R1 E
+ + r R2 R1 r R1 R2
i1
E R2
+ + r R2 R1 r R1 R2
i
E ( ) + R2 R1
+ + r R2 R1 r R1 R2
R
1
poi, (come R
2
) dato da un termine costante, che la resistenza del lato sinistro (lato destro), pi un
termine variabile linearmente nel tempo che la somma delle resistenza tra i due tratti di conduttore tra la
sbarretta ed il latro sinistro (destro)
COMPITO DI FISICA DEL 3-10-02
Il centro o di un disco di raggio r ruota, con velocit angolare costante, lungo una circonferenza di
raggio R (R r), giacente nel piano x-y con centro lorigine O degli assi. Il disco, a sua
volta, ruota intorno al proprio centro con velocit angolare costante. Sia P un punto
della circonferenza del disco mobile ed allistante t = 0 questo si trovi sullasse x,
distante R + r da O.
1) Quanto vale la velocit di P (rispetto al piano x-y) al tempo t = 0 ?
2) A quale istante la velocit di P minima (rispetto al piano x-y) ?
3) possibile che la velocit di un punto del disco sia sempre nulla? Se si, a quali condizioni devono
soddisfare i dati?
4) Sotto quali condizioni la traiettoria di P unorbita chiusa?
Due masse puntiformi m
1
e m
2
sono ferme ad una distanza reciproca D. determinare le velocit che
raggiungono sotto la reciproca interazione gravitazionale quando la loro distanza reciproca si dimezzata.
131
Unasticciola rettilinea, lunga l, ha una densit di massa che varia linearmente da un estremo all altro,
passando dal valore
1
al valore
2
. Determinare la sua massa e la posizione del centro di massa.
Il potenziale in una regione dello spazio dato da:

,
_

2
2 2
0
a
y x
V V . Trovare i punti nei quali il campo
elettrico diretto verso lasse z
Un filo conduttore conduce una corrente I. Il filo termina in una piastra conduttrice omogenea ed infinita che
raccoglie la corrente I. Determinare lintensit e la direzione della densit di corrente nella piastra ad una
distanza R dal punto di attacco.
Un filo conduttore avvoltolato con spire molto strette su di un toro. I raggi del toro sono r ed R, con R>>r.
calcolare il campo B al centro del toro se il filo conduttore percorso da una corrente I
Compito di Fisica del 29/11/02
Una circonferenza di raggio R = 10 m contiene una circonferenza che le tangente internamente, di raggio r
= 1 m. siano P ( della circonf. grande) e P ( della piccola) i punti di tangenza. La piccola inizia a rotolare
senza strisciare allinterno della grande, finch, dopo 1 s, non viene a toccare la grande nuovamente nel
punto P.
1) Quanto valsa la velocit media del punto P durante il moto?
2) Il punto della circonferenza mobile che alla fine sar a contatto con P sar sempre P o sar diverso?
3) Quanti giri ha compiuto intorno al suo centro la circonferenza piccola?
Nel disegno di figura, lasta orizzontale di massa trascurabile. Quanto deve valere X per lequilibrio?
Se si taglia il filo che unisce la massa di 8 Kg con la massa di X Kg, con quale accelerazione inizia a
muoversi la massa di 10 Kg? E quanto vale la forza esercitata da O?
132
Calcolare il campo magnetico al centro di una spira rettangolare di lati a e b percorsa da una corrente di
intensit I
Nel disegno di figura il lato sinistro della spira mobile ed ha massa m. Se un generatore mantiene nella
spira una corrente I costante, come si muove la spira mobile, se immersa in un campo dinduzione
magnetica B ortogonale al piano della spira uniforme e costante?
133
Calcolare il campo B al centro del cubo di figura di sopra.
COMPITO DEL 28/3/03
12) (vale 6/30)Volete misurare la lunghezza di un dato filo, ma non disponete di un metro, ma soltanto di
un cronometro. Costruite quindi un pendolo con tale filo e misurate il periodo T che risulta essere di
3 s. con unincertezza del 2 per mille. Sapete anche che il valore dellaccelerazione di gravit del
luogo risulta essere di 9.8 m/s
2
, con un errore del 5 per mille. Quanto la lunghezza del filo e con
che errore percentuale la conoscete?
13) (vale 3/30) Un disco di 10 cm di raggio ruota intorno al suo asse partendo dalla quiete, con
accelerazione angolare costante di 10 rad/s
2
Nellistante t = 5 s qual la velocit angolare del disco e
laccelerazione radiale e tangenziale di un punto del disco?
14) (vale 4/30) Una pallina cade inizialmente da unaltezza di 10 metri, rimbalza, ricade, rimbalza
finch finalmente si ferma. Se dopo ogni rimbalzo riesce a raggiungere, a causa di attriti interni, solo
il 90% della quota precedente, che lunghezza totale percorrer prima di fermarsi? E in quanto
tempo?
15) (vale 4/30) Un punto materiale di massa M si trova fermo su di un piano orizzontale senza attriti
allorigine di un sistema di coordinate. Al tempo t = 0 una forza costante f i agisce su di lui per un
tempo t
1
e poi, per un tempo t
2
agisce una forza F j. Dove si trova il punto materiale allistante t
1
+
t
2
?
16) (vale 4/30) Un satellite artificiale ruota di moto uniforme intorno alla terra ad una distanza dalla
superficie di 250 km. Se lorbita che descrive una circonferenza, quanto vale la velocit scalare del
satellite?

17) (vale 5/30) Una massa di 1 kg appesa, tramite una molla di costante elastica K al soffitto di un
igloo che si trova proprio al polo Nord. La stessa massa viene poi appesa alla stessa molla ancorata
134
per al soffitto di una capanna che si trova allequatore. La molla sar allungata come prima o no?
spiegare. E quanto saranno i due allungamenti?
18) (vale 6/30) Un pendolo di lunghezza l e massa m attaccato al soffitto di un autobus che si muove
con accelerazione costante a . calcolare il periodo delle piccole oscillazioni
Compito desame del 6/6/03
(1) Un punto materiale di massa m si trova fermo in un punto P di un piano. Dallistante 0 in poi agisce su di
lui una forza le cui componenti sono date da:
0
) cos(
) sin(
0
0

z
y
x
F
t F F
t F F

Determinare il moto che ne consegue


(2) Un carrellino di massa M pu scorrere su di un piano orizzontale privo di attriti, ma sottoposto ad una
forza viscosa con laria di espressione: f = - av. Al tempo t = 0 un cannoncino solidale al carrello spara
orizzontalmente una massa m e nello sparo la polvere rilascia unenergia E ( cio subito dopo lo sparo
lenergia cinetica del proiettile e del carrellino E)
Trovare la distanza massima percorsa dal carrello prima di fermarsi per attrito con laria.
Se volete che il carrello percorra la maggior distanza possibile, la sua massa dovr essere grande o piccola
rispetto a quella del proiettile?
Soluzione:
:= eq1 + m~ v M~ V 0
eq2:=m*v^2+M*V^2=2*E;
:= eq2 + m~ v
2
M~ V
2
2 E
solve({eq1,eq2},{v,V}):
allvalues(%);

'

;
, v 2
E M~
+ m~ M~ m~
2
V
m~ 2
E M~
+ m~ M~ m~
2
M~
,

'

;
, v 2
E M~
+ m~ M~ m~
2
V
m~ 2
E M~
+ m~ M~ m~
2
M~
sol:=(%);
sol

'

;
, v 2
E M~
+ m~ M~ m~
2
V
m~ 2
E M~
+ m~ M~ m~
2
M~
, :=

'

;
, v 2
E M~
+ m~ M~ m~
2
V
m~ 2
E M~
+ m~ M~ m~
2
M~
135
V0:=simplify(rhs(-sol[1,2]));
:= V0
2 m~ E
+ M~ m~ M~
eq:=diff(x(t),t,t)=-alpha/M*diff(x(t),t);
:= eq
d
d
2
t
2
( ) x t

_
,

d
d
t
( ) x t
M~
dsolve({eq,x(0)=0,D(x)(0)=V0},x(t));
( ) x t
2 m~ E M~
+ M~ m~
2 m~ E M~ e

_
,

t
M~
+ M~ m~
assign(%);
limit(x(t),t=infinity);
2 m~ E M~
+ M~ m~
(3) Due massettine uguali, di valore m sono fissate luna ad un estremo di unasta di massa trascurabile e
lunghezza l e laltra a distanza l dallaltro estremo che fissato senza attriti in un piano verticale. Si sposta
lasticciola lievemente dalla posizione di equilibrio. Trovare il periodo delle piccole oscillazioni
(4) Due cariche uguali di valore +Q sono separate da una distanza D ed ambedue sono disposte ad una quota
h da un piano conduttore infinito. Trovare la forza elettrica che agisce su di ognuna
(5) Nel disegno di figura ogni ramo rappresenta la resistenza di 1 . Trovare la resistenza tra A e B
(6) Una spira di resistenza totale R = 10 costituita da due resistenza circolari concentriche giacenti nel
medesimo piano, di raggi: r = 5 cm. ed R = 10 cm. Le due spire sono connesse da due tratti paralleli di
conduttore a distanza trascurabile (vedi figura). La spira immersa in un campo magnetico uniforme B = 0.1
Weber/m
2
costante perpendicolare al piano della figura. Al tempo t = 0, la spira viene messa in rotazione
intorno allasse di figura (passante per il centro della spira e giacente nel piano di questa con velocit
angolare costante = 1 rad/s. determinare la corrente che scorrer nella spira trascurando ogni fenomeno di
autoinduzione
136
(7) Quattro fili rettilinei ed infiniti sono percorsi ognuno da una corrente di 1 A e sono disposti in modo da
formare un quadrato. Determinare il campo B al centro. Se i fili invece portano una densit lineare di carica
costante l, trovare il campo E al centro.
Compito desame del 27/6/03
1) Un punto materiale di massa m vincolato a muoversi su di un piano liscio. Questo punto, oltre alla
forza esercitata dal piano, sottoposto ad una di attrazione gravitazionale generata da una massa M,
fissa, che si trova ad una distanza D dal piano. Scrivere le equazioni del moto del punto in questione.
Quali grandezze dinamiche si conservano durante il moto del punto? Allistante t = 0, m ha velocit
di modulo v
0
e si trova nel punto P del piano. In un tempo seguente si trova nel punto Q. quanto
varr il modulo della sua velocit? ( aggiungere eventuali dati per precisare la risposta)
2) Su due pali paralleli A e B, distanti l ed ugualmente sporgenti dal suolo, disposta una trave
omogenea di massa m e lunghezza 2l in modo che un estremo di questa trave sporge di 3/4l dal palo
A. Determinare la forza che ognuno di questi pali deve esercitare per sostenere la trave
3) Una sbarretta materiale di massa trascurabile e lunghezza l incernierata senza attrito in un estremo
e porta al suo centro ed allaltro estremo due masse puntiformi uguali m. Viene inizialmente
abbandonata da ferma in posizione orizzontale e quindi, per azione della gravit, inizia a ruotare.
Determinare la forza esercitata dal perno nella posizione iniziale e quando passa per la verticale.
4) Due solenoidi ideali di forma cilindrica sono disposti coassialmente ed uno dentro laltro. Quello
esterno, che presenta 2n
0
spire per unit di lunghezza, chiuso su di una resistenza R. quello interno
presenta n
0
spire per unit di lunghezza ed chiuso su di un generatore di corrente che induce una
corrente variabile nel tempo secondo la legge: i = i
0
sen(t). Determinare la corrente che passa nel
solenoide esterno.
5) Un lungo ( cio la sua lunghezza molto maggiore del raggio del cilindro lungo il quale si
avvoltolano le spire) solenoide, possiede n
0
>>1 spire per unit di lunghezza e viene chiuso in modo
da formare una ciambella ( toro). Se nel filo scorre una corrente I. determinare il campo B al centro
della ciambella e dire come cambia qualitativamente tale valore se si raddoppia n
0
(Attenzione!)
6) Una spira rettangolare di lati a e b costituisce un circuito elettrico di resistenza R che si muove
parallelamente al lato a con velocit costante v
0
. allistante t=0 il lato b incontra un campo magnetico
uniforme costante B perpendicolare al piano della spira. Calcolare la corrente che circola nella spira
da t = - infinito a + infinito e la forza agente su di lei
Non assegnati
7) Si consideri un tronco di cono cilindrico lungo L di sezioni circolari quasi ma non esattamente uguali
tra loro e di raggi R
1
ed R
2
molto pi piccoli di L. Se la resistivit del metallo, determinare la
resistenza del filo
8) Considerate il circuito con le resistenze di figura. La resistenza totale tra A e B aumenter
maggiormente se si incrementa di 1 la resistenza R1 o la R6? E perch?
Risoluzione:
Indico con H il piede della perpendicolare condotta da M al piano, r la distanza di m da H. Lintensit della
forza di attrazione gravitazionale varr:
F = GMm/(r
2
+ D
2
)
137
E la componente lungo il piano di questa ( la componente perpendicolare annullata dalla reazione del
piano, dato che m non abbandona il piano) diretta verso H e vale:
2 2
2 2
1
D r
r
D r
GMm F
+
+

Tale forza centrale ( il centro H)determiner il moto della massa m e lequazione del moto si potr dunque
scrivere in termini vettoriali come:
a
r
m
r
D r
r
D r
GMm
+
+
) (
1
2 2
2 2
dove la quantit (-r/r) il versore della forza. Se si vuole, si potr introdurre un sistema di assi ortogonali X
Y nel piano, che sar opportuno scegliere di centro H e scrivere il moto riferendolo a tali assi. Essendo le
componenti di r date da:
2 2 2 2
,
Y X
Y
Y X
X
+ +
le equazioni si scriveranno
Y m
Y X D Y X
Y
D Y X
GMm
X m
Y X D Y X
X
D Y X
GMm
` `
` `

+ + +
+ +

+ + +
+ +

2 2 2 2 2
2 2 2
2 2 2 2 2
2 2 2
1
1
Essendo il moto determinato da un campo centrale, si conserver lenergia meccanica ed il momento della
quantit di moto rispetto ad H
2) Consideriamo di tagliare idealmente la trave ad un tratto distante 3/4l dal palo A. Sopra il palo A viene
allora a gravare una trave orizzontale lunga 6/8l, centrata su di lui e sul palo B una trave lunga 2/8 l, centrata
su di lui. Esse restano in equilibrio, quindi il nostro taglio non muta le forze che valgono ovviamente 6/8 mg
e 2/8 mg
3) Posizione iniziale: Il momento della forza peso rispetto al perno vale:
mg l/2 + mg l = 3/2 l mg
Il momento della quantit di moto dato da:
-m ((l/2)
2
+ l
2
) d/dt
La seconda equazione cardinale fornisce subito allora:
lmg l
l
m
2
3
)
2
(
2 2

1
]
1

+
` `
e quindi:
l
g
5
6

` `
Il centro di massa, che si trova a l dal perno, subisce allora unaccelerazione data da: g l
10
9
4 / 3
` `
diretta verso il basso. Se non ci fosse stato il perno, laccelerazione sarebbe stata ovviamente pari a g e
diretta verso il basso. Ne segue allora che la forza esplicata dal perno pari ad 1/10 g m e diretta verso lalto.
Posizione finale: il centro di massa descrive una circonferenza di raggio l. la velocit nel punto pi basso
si ricava subito dal principio di conservazione dellenergia meccanica. Quindi laccelerazione centripeta del
centro di massa nota, e quindi la forza centripeta agente sulla sbarra. Questa daltra parte, determinata dal
peso ( noto) e dalla forza del piolo ( incognita ma che si ricava dunque subito)
4) Sia I lintensit di corrente incognita. Questa crea nel solenoide grande un campo uniforme B dato
da I n B
0 0
2 e quindi un flusso pari a IS n S n B
2
0 0 0
4 ) 2 ( . La corrente i crea allinterno del
solenoide grande un campo B dato da ) sin(
0 0 0 0 0
t i n i n B ed il flusso di questo attraverso il
solenoide grande ( se s la sezione di quello piccolo):
) sin( ) 2 ( ) 2 ( ) 2 (
0 0 0 0 0 0 0 0
t i s n n s n i n s n B
Compito desame del 16/7/03
138
1) Il centro di una sfera di carica di densit uniforme e raggio r e massa m si trova ad una distanza di
20 r dallasse di un cilindro indefinito di carica, di densit sempre , e raggio 2 r. La sfera viene
lasciata da ferma. Trovare la velocit che avr raggiunto quando il suo centro dister 23 r dallasse
del cilindro.
2) Una cassa striscia su di un pavimento. La sua velocit iniziale di 1 m/s.Si arresta dopo aver
strisciato per 1 m. Qual il coefficiente dattrito dinamico tra cassa e pavimento?
3) Nel circuito di figura le batterie hanno resistenza interna trascurabile. Si trovi la corrente che passa
nei vari rami. Si trovi lenergia erogata dalla batteria di 12 V in 3 s. Si trovi il calore totale prodotto
in questo tempo nelle resistenze.
Compito che vorrei dare il 22 settembre 2003
Un blocco di massa M poggia su di un piano orizzontale liscio. In esso scavato un piccolo cunicolo a
forma di quarto di cerchio, disposto verticalmente, con un estremo sulla superficie orizzontale libero e laltro
sbocca sulla superficie verticale ( vedi fig.1). Il raggio di tale canalino R. Nella parte superiore del cunicolo
viene lasciata cadere una cassettina m che, sotto lazione della gravit discende senza attrito ed abbandona il
blocco alluscita. Determinare le velocit di M ed m alluscita di questa dal canale.
Il problema presenta 2 incognite quindi si risolve con due equazioni che sono: la conservazione
dellenergia meccanica e della quantit di moto lungo lasse x
Valutare la tensione della fune nel problema rappresentato dalla figura 2
Una piastra conduttrice schematizzabile con un rettangolo di larghezza L e di lunghezza indefinita percorsa
da una corrente uniforme I nel senso della sua lunghezza.
Calcolare il campo magnetico in un punto dellasse della piastra
Calcolare il campo magnetico in un punto distante D ( > L/2) dallasse della piastra e giacente nel piano di
questa ( per questa domanda pu essere utile considerare la piastra come costituita da uninfinit di fili
rettilinei indefiniti)
Una parte di una rete elettrica costituita un triangolo i cui tre lati sono delle resistenze di valori: R
A
, R
B
, R
C
( vedi figura). Si vogliono sostituire queste tre resistenze con altre tre: R
X
, R
Y
, R
Z
disposte a stella( vedi
figura) senza alterare le caratteristiche elettriche della rete. Calcolare che valori devono avere queste tre
resistenze
139
Risposta. Deve essere: resistenza tra A e B: R
x
+ R
y
= // tra R
C
e (R
B
+ R
A
), cio:
+ Rx Ry
1
+
1
Rc
1
+ Rb Ra
Analogamente: R
x
+ R
z
= // tra R
B
e (R
C
+ R
A
), cio:
+ Rx Rz
1
+
1
Rb
1
+ Rc Ra
Ed infine: R
y
+ R
z
= // tra R
A
e (R
C
+ R
B
), cio:
+ Ry Rz
1
+
1
Ra
1
+ Rc Rb
Risolvendo ottengo:
{ } , , Rz
Rb Ra
+ + Rc Ra Rb
Ry
Rc Ra
+ + Rc Ra Rb
Rx
Rb Rc
+ + Rc Ra Rb
Compito del 30-3-04
1) La densit del rame, in unit MKS, pari a 8890. Calcolare la densit in unit CGS.
Risposta: 1 m
3
contiene dunque la massa di 8890 kg di rame, cio 8890x10
3
g. 1 cm
3
contiene allora la massa
di 8890x10
3
/100
3
g = 8.89 g/cm
3
c.v.d
Conoscendo che il peso di un cubo di rame di 7 kg, con un errore dell1%, quanto vale il suo lato e con che
errore noto?
Risposta: Essendo la densit data dalla massa/volume, si ha che il volume la massa/densit e quindi il
volume di 7000g di rame di cm
3
7000/8.89 = 787,4 cm
3
. Il lato allora di cm:
3
4 , 787 = 9,234 cm. Se
lerrore sulla massa dell1%, si potr dire, grosso modo, che questa compresa tra 7000*(1-0.01) = 6930 g
e 7000*(1+0.01) = 7070 g e quindi il lato sar compreso tra 9,265 cm e 9,203 con un errore relativo dato da:
(9,265-9,203)/(2*9,234) = 0,3% .Per trovare lerrore si poteva procedere anche cos, con ovvio significato
dei termini: 3

m
l . Quindi, differenziando, si ha: dm m dl
3
2
3
) (
3
1

. Lerrore relativo sul lato dl/l.


Quindi:
m
dm
m
dm m
l
dl
3
1
) (
3
1
3
3
2
3

. Quindi lerrore relativo sul lato, quindi anche quello percentuale, un terzo
di quello sulla massa, cio circa lo 0.3 % come si trovato sopra.
140
2) Una ruota di raggio R = 10 cm viene messa in rotazione da ferma intorno al proprio asse con una velocit
angolare crescente in modo uniforme nel tempo. Dopo un secondo dal momento della messa in moto, viene a
ruotare con una velocit angolare tale che, se da quel momento in poi ruotasse uniformemente, farebbe un
intero giro in 1 secondo. Calcolare il modulo della velocit di un suo punto del bordo dopo 2 secondi dalla
messa in moto e larco descritto dal punto in questi due secondi.
Risposta: la frase: fare un intero giro in un secondo, vuol dire avere velocit angolare = 2 rad/s. Detto
questo, osserviamo che la velocit angolare data, nel caso in questione, da: = t con incognito. Se t =
1, proprio dato da che quindi 2 radianti/s
2
. Quindi: = 2 t. Al tempo t = 2 la velocit angolare
dunque di 4 radianti/s ed il modulo della velocit, fornito da: v = R, allora di 40 ~ 125.6 cm/s. Il moto
caratterizzato da accelerazione angolare costante = e quindi larco descritto al tempo t sar di t
2


radianti. Se t = 2, larco descritto sar di 4 radianti, cio due giri
3)Un punto materiale P si muove di moto rettilineo uniforme lungo lasse x di un sistema di riferimento con
velocit v
0
pari ad 1 cm/s uscendo al tempo t=0 dallorigine degli assi. Un secondo sistema di riferimento
con origine coincidente col primo, ruota con velocit angolare uniforme pari ad 1 radiante al secondo
intorno al suo asse z. Determinate il modulo della velocit di P nel tempo rispetto ad un osservatore solidale
al sistema ruotante.
Risposta: al tempo t la distanza del punto dallorigine degli assi sar pari a v
0
t. e quindi la sua velocit di
trascinamento sar: (v
0
t) j. Essendo la sua velocit assoluta pari a (v
0
) i, la sua velocit relativa risulta (v
0
)
i - (v
0
t) j e quindi il modulo di tale velocit risulta: v
r
=
2
0
2
0
) ( ) ( t v v +
4) due molle, di costanti elastiche K
1
e K
2
e di lunghezze a riposo l
1
ed l
2
sono legate luna allaltra come in
figura. La due molle sono equivalenti ad ununica molla. Determinare di questa la lunghezza a riposo e la
costante elastica.
Risposta: ovvio che la lunghezza a riposo della molla equivalente la somma delle due lunghezze. Si
allunghi ora la molla equivalente di un tratto unitario. La molla 1 sar allora allungata di un tratto x
1
e la
molla 2 di un tratto x
2
. ovviamente sar:
1) x
1
+ x
2
= 1
Ma la giunzione tra le due molle ferma e questo vuol dire che tanto tira luna quanto tira laltra Si deve
quindi avere:
2) K
1
x
1
= K
2
x
2
La (1) e (2) sono un sistema nelle due incognite x
1
ed x
2
. risolte, si trova subito K
1
x
1
che misura pure la forza
con cui lestremo libero della molla attirato. Dato che lallungamento della molla equivalente
unitario, tale forza misura pure il K
equivalente
che risulta essere dato da:
2 1
1 1 1
K K K
e equivalent
+
5) Due palline hanno massa una doppia dellaltra e procedono libere con velocit costanti ed uguali in
modulo luna verso laltra lungo lasse x. Si urtano quando raggiungono lorigine di un sistema di assi
cartesiano ortogonale e 1 secondo dopo lurto, la pi massiccia si trova nel punto di coordinate (1,2). Dove si
trover laltra? (se mancano dati, aggiungerli; se sono in contrasto, segnalarlo ed eventualmente correggerli)
Risposta: La + massiccia proceda nel verso positivo delle x con velocit v. La velocit del centro di massa
allora: v
m m
mv mv
3
1
1 2
1 2

. Poich sulle due palline non agiscono forze esterne, la loro quantit di moto
prima e dopo lurto sar la medesima e quindi la velocit del centro di massa non varia e quindi dopo 1 s si
trova allascissa (1/3)v. La massa + leggera si trover allora sulla retta passante per il punto (1,2) ed il centro
di massa (di coordinate 1/3v e 0), dalla parte opposta del centro di massa ed ad una distanza da questo doppia
di quella intercorrente tra il centro di massa e la pallina pi massiccia. Avr quindi coordinate: (-2+v, -4) Al
variare di v tra 0 ed infinito, le posizioni vengono a trovarsi su di una semiretta parallela allasse x di origine
(-2,-4)
Compito desame del 16/9/04
Due ruote, di raggio R
1
e R
2
, con massa trascurabile, portano ognuna, incastonato nel rispettivo semi-raggio,
due masse m
1
ed m
2
. La prima in rotazione libera, con velocit angolare
1
, attorno ad un asse passante per
il suo centro. La seconda libera di ruotare intorno al suo asse centrale, parallelo al primo, ma inizialmente
ferma. I due assi vengono avvicinati finch le due ruote entrano in contatto reciproco. Per attrito, quella in
141
moto pone in rotazione laltra ed alla fine ruotano ambedue senza strusciare luna sullaltra. Determinare le
velocit angolari finali
Risoluzione
La componente lungo lasse di rotazione del momento della quantit di moto di ognuna delle due ruote
proporzionale, ovviamente, alla velocit angolare. Quindi si potr scrivere:
2 2
2
1 1
1
J L J L
z z

La forza di attrito generata dalla ruota 2 (1) sulla 1 (2) determina un momento dato da:
F R M F R M
z z 2
1
1
1

Dove la F incognita
La seconda equazione cardinale applicata alla ruota 1 (2) fornisce allora:

'


2 2 2
1 1 1

J
dt
d
F R
J
dt
d
F R
Nelle relazioni di sopra, la F incognita: sar allora opportuno cercare di eliminarla per non avere
unincognita tra i piedi. Leliminazione pu effettuarsi dividendo la prima per R
1
e la seconda per R
2
e
sommando. Si ottiene cos:

,
_

,
_

,
_

+ 0 0
2
2
1
1
1
2
2
2
1
1
1
2
2
2
1
1
1
R
J
R
J
Cost
R
J
R
J
R
J
R
J
dt
d
iniziale

Le equazioni del moto di sopra valgono finch c strusciamento tra le due ruote, strusciamento che fa
sorgere la forza di attrito. Ma quando la velocit del punto di contatto, visto come appartenente alla prima,
uguale alla velocit del punto di contatto, visto come appartenente alla seconda, la medesima, lattrito cessa
e le due ruote continueranno a ruotare, per inerzia, per tutta leternit con le velocit angolari raggiunte. Per
quanto detto, tali velocit angolari dovranno soddisfare la relazione:
2 2 1 1
R R
finale finale

Questa relazione e quella di sopra sono un sistema di due equazioni nelle due incognite
1

2
finali
Nel circuito di figura ( due quadrati uguali uniti per un lato e nello stesso piano) ogni lato presenta la
resistenza di 1 Ohm. Il circuito immerso in un campo magnetico uniforme, perpendicolare al piano della
spira che cresce linearmente nel tempo. Determinare le correnti
Risposta Si hanno due maglie quadrate identiche immerse nel medesimo campo magnetico variabile nel
tempo. Quindi la corrente che, per la legge di Lenz, sorge in una, sorge nellaltra. Ma allora la corrente nella
resistenza comune nulla. Ma allora si pu pensare di togliere tale resistenza senza alterare la corrente negli
altri rami. Ma cos facendo si ha ora una spira rettangolare di resistenza totale 6 Ohm immersa in un campo
magnetico variabile che, per la legge di Lenz, fa sorgere nel circuito una forza elettromotrice pari a:

Cost l
dt
dB
l
spira erficie spira erficie sup
2
sup
2
2 2
La corrente in ogni lato sar allora:

6
2
sup
2
Cost l
spira erficie
ESAME DI FISICA PER ENERGETICI 7-6-07
Tempo a disposizione: 2 ore. No appunti n libri
1) Una lamiera pesa 11 Kg al m
2
Con una bilancia misuro il peso di un quadrato ricavato da tale lamiera, che
risulta essere di 8 Kg, con un errore relativo dell1%. Quanto risulta essere il lato di tale quadrato? E con
quale errore noto?
142
2) Un pendolo di massa m e lunghezza l si trova inizialmente fermo e nella sua posizione di equilibrio
stabile. Unaltra massa m, in moto ascendente, con angolo

rispetto alla verticale e modulo v della velocit,


urta anelasticamente la massa m del pendolo. Determinare langolo massimo formato dal filo con la verticale
(la lunghezza l realizzata con unasta di massa trascurabile)
3) Su di un piano inclinato senza attrito ed inizialmente formante un angolo di 45 con la verticale, giace una
massettina m . linclinazione

del piano diminuisce molto lentamente nel tempo con la legge:


t + 4 /
determinare il moto della massettina
4) Due linee cariche eguali con densit lineare di carica t sono disposte parallelamente allasse y ed
intersecano il piano x-z in (tL,0). Determinare la funzione potenziale V(x,z) con V(0,0)=0
5) Quattro cariche uguali, ciascuna di 1 coulomb, sono ai vertici di un quadrato di lato 1 metro. Qual la
forza su di ognuna? Quanto vale il campo elettrico a met del lato?
6) Se assumo come unit di carica quella che ad una distanza di cinque metri da una uguale risente di una
forza di cinque N, quanto vale la carica di 1 coulomb in questa nuova unit?
In Alternativa ai problemi sopra esposti, potete cercare di risolvere il seguente problema, che giudico
abbastanza facile ma per nulla banale:
Unautomobile, di massa m, percorre lasse x, nel verso crescente, con velocit di modulo v
0
. Giunta al punto
x=0, lautomobilista scorge dinnanzi a s, a distanza L, un muro rettilineo infinito parallelo allasse y. Il
guidatore pu frenare, o sterzare o fare luna e laltra cosa insieme onde evitare lurto. Quale effettiva
manovra gli conviene compiere? Eviter lurto con la manovra migliore? ( si supponga che,
indipendentemente dalla manovra, la strada possa esercitare sullauto, grazie allattrito, una forza di modulo
massimo pari a F
0
)
Suggerimento: cercare di manovrare in modo che lavanzamento (a partire da x=0) lungo lasse x sia minimo
Schizzo di soluzione: lasfalto, mediante il meccanismo dellattrito, esercita sullautomobile la forza che, a
seconda della manovra intrapresa dal guidatore (mediante il freno e/o lo sterzo), la rallenta o la devia o
ambedue le cose insieme. allora intuitivo che, nella manovra migliore, il modulo della forza esercitata
dallasfalto debba essere sempre pari al valore massimo F
0.
Dovr dunque allora essere:
(1)
2
0
2 2
F F F
y x
+
ma quale delle due componenti della forza governa lo spostamento lungo lasse x? Ovviamente la F
x
! E
che relazione c tra tale componente e lo spostamento? Imitando il ragionamento che ha permesso di
stabilire il teorema dellenergia cinetica, scriver:
2
) (
2
1
x x x x
x
x x x x
v d m dv mv
dt
dx
mdv dx
dt
dv
m dx ma dx F ma F
Integrando ambo i membri tra lascissa iniziale (zero) e lascissa generica X, ho:


X
X
V
x x
mv mV mv dx F
X
0
2
0
2
0
2
2
1
2
1
2
1
Nel momento in cui si realizza durante il moto la massima distanza dellauto dallorigine, in quel medesimo
istante la velocit lungo x deve essere nulla. Se l tale distanza, la relazione di sopra fornisce:


l
x
mv dx F
0
2
0
2
1
, daltra parte, assolutamente ovvio che, sia che il guidatore sterzi, sia che freni, la F
x
negativa e ponendo
allora F
x
= - f
x
, si ha:


l
x
mv dx f
0
2
0
2
1
143
Ora, il membro di destra un dato del problema, mentre a sinistra figura lintegrale di una funzione non
negativa , ma incognita: la f
x
e lintegrale fatto tra lo zero ed un valore l che si vuole il pi piccolo
possibile . Ma allora chiaro che lintegrando f
x
dovr essere preso il pi grande possibile. Ora, valendo per
sempre la (1), il valore pi grande possibile sar allora dato da f
x
=F
0
( e conseguentemente F
y
= 0). Segue
subito allora che la miglior manovra la frenata pura (senza sterzata, che farebbe sorgere una F
y
).
Laccelerazione dellauto allora: a
x
= F
0
/m e si arrester dopo aver percorso un tratto di strada pari a:
0
2
0
2F
v
m . Se tale quantit inferiore a L, non ci sar collisione, altrimenti impossibile evitarla.
Idee per un altra soluzione del problema:
Quale che sia la manovra intrapresa dal guidatore, chiaro che, nel momento di massimo avvicinamento
dellauto al muro, la v
x
di questa deve essere nulla, quindi, quale che sia la manovra, la v
x
decresce
costantemente dal valore v
0
a 0.
Supponiamo, per il momento, che la manovra migliore sia la frenata pura. Lauto subisce una
decelerazione lungo x il cuo valore : F
0
/m e perci, quando la sua v
x
si sar annullata, avr percorso un
tratto: l=v
0
2
m /(2F
0
). Se questo tratto inferiore ad L, la macchina salva in questa manovra, altrimenti no.
Mi chiedo ora: potr la macchina portare la v
x
a 0 eseguendo unaltra manovra spostandosi lungo lasse x
sempre del medesimo tratto l? Osservo che nel primo caso la a
x
costante coincide con laccelerazione
media lungo x tenuta nel tratto l. In unaltra manovra, per azzerare la v
x
nel medesimo tratto l, occorrer che
la a
x
sia a volte minore di F
0
/m ma a volte maggiore, ma questultima condizione non possibile!
NON LEGGERE SOTTO
Lauto dunque deve arrestarsi in un tratto L e quindi il valore assoluto dellaccelerazione che deve avere deve
essere:
L
v
a
2
2
0
Se dunque
L
v
m F
2
2
0
0
la macchina pu fermarsi alla parete semplicemente frenando
(ovviamente se
L
v
m F
2
2
0
0
> le cose vanno ancora meglio!). Il problema di vedere se, pur essendo
L
v
a
L
v
m F
x
2 2
2
0
2
0
0
< < , si riesce ad evitare lo scontro con unopportuna manovra combinata di freno e
sterzo. Ma la risposta chiaramente no perch
L
v
2
2
0
il valore della decelerazione che, applicata
costantemente impedisce lurto, cio porta la velocit lungo x dal valore v
0
a 0 in uno spazio L. Una
manovra combinata di freno e sterzo, dovendo pur sempre annullare la v
x
in uno spazio L, dovr comportare
una a
x
talvolta inferiore al valore trovato (
L
v
2
2
0
), ma talvolta superiore, e questo non permesso perch
allora la forza sarebbe maggiore di F
0
.
ESAME DI FISICA PER INGEGNERIA ENERGETICA 05-06-2008
Le risposte devono essere giustificate. Le giustificazioni devono essere chiare e stringate.Non riscrivete
quindi parti degli appunti o di testi: alla bisogna, datene solo lindicazione. Se il testo non chiaro, chiedete!
1) Che ora indicheranno le lancette di un orologio quando saranno sovrapposte per la prima volta dopo le ore
12?
Risposta:La prima volta saranno sovrapposte poco dopo luna, la seconda volta poco dopo le duela decima
volta poco prima delle undici e lundicesima volta nuovamente alle 12. Siccome lintervallo tra una generica
sovrapposizione e la segunete sempre il medesimo, langolo tra due sovrapposizionei sar di 360/11 =
72 . 32
. Ora, la lancetta dei minuti forma un angolo di (360/12) in 300 secondi, quindi un angolo di
72 . 32

verr coperto in
' '
72 . 332
= 27 27 5
' ' '
2) Unasta omogenea di lunghezza L e massa m poggia con un estremo sul pavimento (coefficiente dattrito
statica
S
) e con laltro ad una parete liscia. Determinare il valore della forza minima orizzontale da applicare
al suo centro affinch resti ferma formando un angolo con lorizzontale. (Consiglio: ricordate come si
trattato il caso in mancanza di ogni attrito!)
Risposta: La forza dattrito sar ovviamente diretta verso la parete ed il suo valore sar inferiore o al pi
uguale a mg
S
. Vediamo per quale angolo massimo pu esserci equilibrio con un valore della forza nullo.
144
Annullando il momento totale delle forze rsipetto al punto dincontro delle verticali al pavimento e parete
passanti per gli estremi della sbarra, si ha:
S
Max Max S Max
l mg mgl


1
) tan( 0 ) sin( ) ( ) cos(
Se langolo pi piccolo del valore trovato, la seconda equazione cardinale, riferita sempre allo stesso
polo, ora impone:
] 2 ) [cot( 0 ) sin( ) ( )] sin(
2
[ ) cos(
2
S S
mg f l mg
l
f
l
mg
3) Appendo ad un chiodo C in una parete un copertone di bicicletta e lo dispongo in modo che il suo centro
O sia alla stessa quota di C. Determinare la velocit di O e la forza che il chiodo esercita quando O raggiunge
la quota pi bassa, nellipotesi molto semplificatrice, anche se irrealistica che ogni attrito sia trascurabile.
Raggio del copertone: R; massa del copertone: m. Introdurre eventuali dati mancanti.(Consiglio: prima di
scrivere, ragionare e riflettere, poi riflettere e ragionare e quindi scrivere la soluzione!)
Risposta: Non essendoci attrito la forza esercitata dal chiodo perpendicolare alla tangente passante per il
chiodo o, detto in altri termini, la forza esercitata dal chiodo radiale, cio passante per il centro. Ma pure la
forza peso passante per il centro e quindi il momento delle forze esterne nullo rispetto al centro. Ne
consegue che il copertone non ruoter mai. Ma allora il suo moto sar una serie di traslazioni. Ma nelle
traslazioni di un corpo rigido tutti i punti hanno la medesima velocit, velocit che coincide dunque con
quella del centro di massa, sicch lenergia cinetica del copertone sar data da: m v
2
centro di massa
. Dalla
conservazione dellenergia meccanica ho: m v
2
centro di massa
+ mgh
centro di massa
= Costante e quindi v
massima
=
gR 2 . Il moto il medesimo di un pendolo di massa m e lunghezza R! Quanto poi alla forza del chiodo,
essa varr 3mg (vedi il caso del pendolo precedentemente trattato)
4) Tre cariche uguali, di valore Q si trovano ai vertici di un triangolo equilatero. Trovareil valore della carica
che posta al centro le mantenga in equilibrio.
Risposta: la forza elettrica su di una carica la somma vettoriale delle forze dovute alle altre due, somma che
diretta lungo la bisettrice e che vale: ) 30 cos(
4
1
2
2
2
0

l
Q

. Il centro del triangolo dista da un vertice
3
2 l
e
quindi la carica equilibratrice, di segno opposto alla Q deve avere valore assoluto Q
*
:
) 30 cos( 2
)
3
2
(
2
2
2
*

l
Q
l
Q Q
e quindi subito: 3 /
*
Q Q
5) Due sfere conduttrici cave e concentriche hanno raggio R
1
ed R
2
(>R
1
)e spessore trascurabile. Si deposita
su quella interna una carica Q
1
e su quella esterna una carica Q
2.
. Trovare le cariche sulle quattro facce delle
due sfere e la differenza di potenziale tra le due.
Risposta: Siano 1,2,3,4 le facce a cominciare dalla pi interna. Per la propriet dei conduttori, essendo la
cavit pi interna priva di cariche nel suo interno, si avr
1
=0 e quindi
2
= Q
1
/S
1
. Il teorema di Gauss
applicato ad una superficie chiusa passante tutta nello spessore della sfera pi grande, permette di dire subito
che :
3
= -Q
1
/S
2
. Ma allora, portando la sfera pi grande una carica Q
2
, deve essere Q
1
+
4
S
4
= Q
2 _
-->
4
=
(Q
2
+Q
1
)/S
4
.

Ovviamente S
1
= 4R
2
1
S
2
4R
2
1
S
3
= 4R
2
2
S
4
4R
2
2
6) Si vuole suddividere una sfera di raggio R
1
uniformemente riempita di una carica Q in due uguali, di
volume totale invariato, i cui centri distano tre volte il loro raggio. Calcolare il lavoro necessario.
Risposta: Si visto un esercizio a lezione ne quale veniva mostrato che per radunare della carica per formare
una sfera di raggio R era necessario un lavoro pari a:
R
Q
0
2
20
3

. Si operi allora cos:
1-Si disfi la nostra sfera di partenza, compiendo un lavoro
R
Q
L
20
3
2
1

145
2-Si formino due sfere a reciproca distanza molto grande. Per formarle occorre spendere il lavoro:
)
2
( 20
)
2
( 3
2
3
0
2
2
R
Q
L


, dato che il raggio della nuova sfera, r, quello della vecchia / radice cubica di 2
3- Si portino ora i centri delle due sfere a distanza 3r Ricordiamo che il campo elettrico allestermno di una
sfera omogenea lo stesso di quello che avremmo se tutta la carica fosse concentrata nel centro. Tale lavoro
allora:
r
Q
L
2
0
3
4
1

Il lavoro totale sar allora la somma algebrica dei tre lavori


7) I tre lati di un triangolo equilatero sono costituiti da tre resistenze: R
1
, R
2
, R
3
. Il triangolo scivola con
velocit V costante nel piano x-y, mantenendo il lato R
3
parallelo allasse y. Allistante t=0 inizia ad entrare
in un campo B uniforme, costante e diretto lungo lasse z che occupa il semispazio x>0. Determinare la
corrente nel circuito. Che approssimazioni fate per determinarla? Introducete voi eventuali dati mancanti.
Risposta: In un tempo t la punta del triangolo penetrata di un tratto Vt e quindi unarea pari a
3
) (
2
Vt

attraversata dal campo B. Quindi il suo flusso varia nel tempo attraverso la legge:
3
) (
) (
2
Vt
B t
B
. La
forza elettromotrice inserita nel circuito la derivata temporale di tale flusso (legge di Faraday). Se divido
tale forza elettromotrice per la resistenza elettrica totale ( pari alla somma delle tre resistenze), ottengo
lintensit della corrente. Ho dunque:
3 2 1
2
3 2 1
2
3
1
) (
R R R
t BV
R R R
t
dt
d
I
B
+ +

+ +


(il segno indica il verso). Naturalmente tale formula vale dallistante 0 fino a che tutta la spira non entrata
nel campo, poi la forza elettromotrice, e quindi la corrente, crolla a 0.Nel procedimento seguito si tascurata
la corrente di spostamento, approssimazione che si compie sempre quando si tratta di circuiti elettrici, ma
inoltre si trascurato il coefficiente di autoinduzione. Il tenerne conto impedirebbe alla corrente di crollare a
0 quando la spira tutta nel campo , ma farebbe decrescere con dolcezza nel tempo il valore di questa a 0
146
147
Esercizio da farsi dopo sulle unit di misura
Le equazioni di Maxwell secondo Landau sono:
148

'



t c c
t c
E
j H
H
H
E
E
1 4
0
1
4


e la forza di Lorentz data da:
1
]
1

+ H
v
E f
c
q
nel sistema MKSA invece si ha:

'



t
t
E
j B
B
B
E
E
0 0 0
0
0
/


e la forza di Lorentz data da: [ ] B v E f + q
Come si accordano i due sistemi? Teniamo presente che
0 0
2
1

c
Detto questo, osserviamo che la divisione della forza in prodotto di carica e campo soffre di una
indeterminazione: se si moltiplica la carica per e si divide il campo per , la forza non cambia. Detto
questo, sia:
MKS Landau

MKS
Landau
E
E

MKS Landau
c
B H

Le equazioni di Maxwell nella forma di Landau diventano:

'



MKS MKS MKS
MKS
MKS
MKS
Landau
Landau
MKS
MKS MKS
MKS MKS MKS
MKS
t c
t c t c c c
t
E j B
E
j
B E
j
H
B
B E
E
E
0 0
2
2
0 0 2 2 2
2
4
4 1 4
0
1
4 4

Mentre lespressione della forza diventa:


[ ] B v E f +
MKS MKS
q
Scegliendo:
0
4
1


Le equazioni coincidono
149

Das könnte Ihnen auch gefallen